You are on page 1of 605

See discussions, stats, and author profiles for this publication at: https://www.researchgate.

net/publication/354864088

MCQ general surgery book

Book · September 2021

CITATIONS READS

0 1,339

1 author:

Sherif Zaki Mohamed Kotb


Mansoura University
77 PUBLICATIONS 347 CITATIONS

SEE PROFILE

All content following this page was uploaded by Sherif Zaki Mohamed Kotb on 27 September 2021.

The user has requested enhancement of the downloaded file.


Index
-1-
1- General Principles of surgery :
• Wound healing and management
• Acutely injured patient
• Acute hemorrhage, blood transfusion and blood substitutes
• Hemostasis
• Shock
• Surgical infections, Cellulitis, Tetanus, Gas gangrene
• Surgical nutrition in Surgical patients
• Principles of organ Transplantation
• Principles of Laparoscopy
• Priciples of oncology
2- Burnand principles of plastic surgery:
• Types of burns and its complications
• Skin graft and flaps
• Cleft lip, Cleft palate
• Hemangioma and vascular malformation
3- Skin and soft tissue:
• Dermoid cyst , seabceous cyst, lipoma
• Skin tumors ( BCC and SCC )
• Melanoma
• Soft tissue Sarcoma
4- Vascular surgery:
• Acute and chronic limb ischemia
• Arterial aneurysm
• Varicose veins and chronic venous insufficiency
• DVT, Postphlebitic leg, Leg ulcers
• Lymphedema, Differential diagnosis of swollen limbs
5- Head and neck:
• Thyroid swelling including cancer
• Salivary glands, Anatomy, Sialadenitis, Parotid tumors
• Differential diagnosis of Jew swellings
• Differential Diagnosis of neck swelling (Medline and Lateral
• Cancer tongue and cancer lip
• Cleft lip and palate
• Cervical rib
6- Breast:
• Anatomy, embryology and congenital anomalies of the breast.
• Benign breast diseases , mastalgia , nipple discharge
• Breast cancer
7- Oesophagus:
• Esophagus: GERD and motility disorder
-2-
• Cancer esophagus and Dysphagia
8- Stomach and duodenum :
• Anatomy and congenital anomalies of the stomach
• Peptic ulcer disease and complications
• Cancer stomach.
9- Liver and portal vein, spleen:
• Segmental anatomy of the liver
• Cysts, abscesses and tumors of the liver
• Portal hypertension
• Upper gastrointestinal bleeding
• Splenomegaly
10- Biliary system :
• Gall stones, Cholecystitis
• Obstructive jaundice
11- Pancreas:
• Acute pancreatitis, Pancreatic tumors
12- Small and large intestine :
• Small Intestinal obstruction
• Inflammatory bowel disease
• Diverticular disease of the colon
• Colonic obstruction and Cancer colon
13- Vermiform Appendix :
• D.D of acut abdomen
14- Rectum and anal canal :
• Imperforate anus
• Anatomy, Anal fissure, Hemorrhoids, Rectalprolapse,
• Cancer rectum
• Anatomy, Anaorectal suppuration, Anal fistula
• Bleeding per rectum
15- Perineum, omentum and mesentery:
16- Hernia:
• Anatomy and surgical importance.
• Inguinal hernia.
• Femoral
• ventral hernias, Incisional hernia, recurrent hernia, burst abdomen.
17- Scrotum:
• Varicocele and hydrocele.
• Diseases of testis and epididymis.
• D.D of inguina-scrotal swelling
18- Pediatric surgery:
19- Surgical instruments and tubes:

-3-
20- X-rays:
21- Jars and surgical pathology:
22- Basic surgical skills (models) :
• Rectal examination
• Wound care and stitch
• Uretheral catheterization
• Nasogastric tube
• Examination of trauma patient
• Vascular acess
23- Cardiothoracic surgery:
• Bronchogenic carcinoma
• Mediastinum and Chest Tube
• Chest trauma
• Suppurative lung diseases
• I.H.D
• Valvular heart diseases
• Cong-H.D
• Mediastinum
• Diaphragm
24- Neurosurgery:
• Introduction and CNS examination
• Congenital CNS anomalies
• Head injuries
• Subarachniod hemorrhage and neurovascular diseases
• Brain tumours
• Spinal tumours
• Spinal fracture
• Peripheral nerve injuries
25- Urology:
• Urological Case taking
• Congenital anomalies
• Hematuria
• Hydronephrosis
• Stones
• urinary tract trauma
• Urinary tract infections
• Renal tumors
• Bladder cancer
• BPH, prostate cancer
• Anuria / Retention
• Testicular tumors
-4-
• Erectile dysfunction and Infertility
• Renal transplantation
26- Orthopedic surgery:
• General principles of fractures and dislocations
• Congenital orthopedic disorders and birth injuries
• Shoulder and upper limb fractures
• Pelvis and lower limb fractures
• Osteomyelitis and arthritis
• Bone tumors
27- Anaesthesia and intensive care :
• Preoperative assessment and premedication
• General anaesthesia (inhalational), (intravenous)
• Complications of GA
• Regional anaesthesia part
• Basic life support
28- Trauma
29- Head and neck
30- Oral cavity
31- ENT
32- EYE
33- Soft tissue
34- Gynae and obstatric
35- Lymphoma and LN
36- GIT
37- Psychiatry
38- Others

-5-
1- General Principles of surgery

Wound healing and management

1. About healing intentions:


a. Ulcers heal by first intention
b. Second intention gives better cosmoses than first
c. First intention occurs on clean cut edges without gaps or tissue loss
d. All of the above
Correct answer: C

2. All of the followings are factors affecting wound healing:


a. Age of the patient
b. General condition
c. Corticosteroid
d. Type of wound and type of closure
e. All of the above.
Correct answer: E

3. The most common site for the following diseases is correctly matched except:
a- Keloids: face, neck and skin over sternum
b- Hypertrophic scars: extensor surfaces
c- Hemangiomas: head and neck
d- Basal cell carcinoma: face
Correct answer: B

4. First-intention healing of clean incised wounds is characterized by the following


except:
a. An inflammatory reaction during the first few days.
b. Epithelialization within 48 hours.
c. Fibroblastic proliferation and capillary budding during the next week.
d. Region of normal tensile strength within 4 weeks.
e. Production of thin linear scar.
Correct answer: D

5. Wound healing is not impaired by


a. Anemia.
b. Hypoproteinemia.
c. Ascorbic acid deficiency.
d. Cortisone administration.
e. immunosuppressive therapy.
Correct answer: A

-6-
6 . Surgical wound of gastrojejunostomy is an example for -
A. Clean wound
B. Clean contaminated
C. Contaminated wound
D. Dirty wound
Correct answer: B

7. All of the followings’ ore factors affecting wound healing:


a. Age of the patient
b. General condition
c. Corticosteroid
d. Type of wound and type of closure
e. All the above.
Correct answer: E

8. All of the followings’ ore factors affecting wound healing:


a. Age of the patient
b. General condition
c. Corticosteroid
d. Type of wound and type of closure
e. All of the above.
Correct answer: E

9 . Which of the following is absolutely essential for wound healing?


A. Vitamin D
B. Carbohydrates
C. Vitamin C
D. Balanced diet
Correct answer: C

10. Which of the following is not a feature of a healthy healing ulcer


A. Sloping edge
B. Undermined edge
C. Serous discharge
D. Reddish base
Correct answer B

11. Keloids ore characterized by the following, except:


a. Consist of dense overgrowth of scar tissue.
b. Develop after wounds, burns and vaccination marks.
c. Are particularly common in negroes and pregnant females.
d. Occur most often on the face. neck and front of the chest.
e. May turn malignant.
Correct answer: E

-7-
Acutely injured patient

1. All but one of the following statements is true. Which is not true?
a. Successful clinical repair of injured veins had been affected by the turn of the
twentieth century.
b. Initial large experience in managing injured vein s came from the battlefields of
twentieth-century wars.
c. More than 50% of repaired injured veins thrombose.
d. Phlebography is useful in evaluating variable venous anatomy.
e. Repeated phlebography following attempted venous repair is useful in determining
the success rate.
Correct answer: C

2. The most urgent measure in the management of o severely injured patient in


the reception room is:
a. Control of active bleeding.
b. Taking blood sample for grouping and cross matching.
c. Establishing an intravenous line.
d. Securing clear airway and adequate pulmonary ventilation.
e. providing tetanus prophylaxis.
Correct answer: D

3 . In case of penetrating Trauma abdomen commonest organ injured is -


A. Liver
B. Spleen
C. Colon
D. Small bowel
Correct answer: D

4 . Which of the following organ is commonly injured in blast injury?


A. Spleen
B. Pancreas
C. Lung
D. Liver
Correct answer: C

5. About crush syndrome there will be


A. Alkaline urine
B. Small % of patient develops acute renal failure
C. Small % of the developed renal failure will need dialysis
D. Hypovolemic shock
Correct answer: D

-8-
6. A middle-aged worker reported with multiple injuries and fractures was found
to be in hypovolemic shock. He was resuscitated with IV fluids. Best indicator to
monitor his status of fluid requirement is:
A. Blood pressure and pulse rate
B. Skin turgor and dry tongue
C. State of consciousness
D. Central venous pressure recording
Correct answer: D

Acute hemorrhage, blood transfusion and blood substitutes

1. Bleeding in a case of obstructive jaundice is treated with


A. Fresh Frozen Plasma
B. Cryo precipitate
C. Whole blood
D. Buffy coat extract
Correct answer: A

2. Secondary hemorrhage is usually due to:


a) Trauma
b) Slipped ligature
c) Infection I
d) All of the above
Correct answer: C

3. A comatosed patient who has sustained multiple closed injures is admitted


with severe hypotension. The hypotension ls most probably due lo :
a) Intracranial hematoma.
b) Cerebral concussion.
c) Internal hemorrhage.
d) Neurogenic shock.
e) Cardiogenic shock.
Correct answer: C

4. The best guide for the required blood transfusion ln hemorrhagic shock ls :
a) Arterial B.P.
b) Pulse rate.
c) Hematocrits.
d) Central venous pressure (CVP)
e) Urine output per minute.
Correct answer: D

5. Secondary hemorrhage ls usually due to:


a. Trauma
b. Slipped ligature
c. infection
d. All of the above

-9-
Correct answer: C

6 . What is the best way to control external hemorrhage?


A. Direct pressure
B. Elevation
C. Proximal tourniquet
D. Artery forceps
Correct answer: A

7. The following disease may be transmitted during blood transfusion except


a. CMV
b. Malaria
c. Hepatitis A
d. Brucellosis
Correct answer: C

8. The best guide for the required blood transfusion ln hemorrhagic shock ls :
a. Arterial B.P.
b. Pulse rate.
c. hematocrit.
d. Central venous pressure (CVP)
e. Urine output per minute.
Correct answer: D

9. The most serious complication of blood transfusion is:


a. Pyrogenic reactions.
b. Thrombophlebitis of recipient vein.
c. Circulatory overloading.
d. incompatibility reactions.
e. Viral hepatitis.
Correct answer: D

10 . Which is the best fluid for resuscitation during shock state?


A. Crystalloids
B. Colloids
C. Plasma substitutes
D. 5% dextrose
Correct answer: A

11 . 5% dextrose in normal saline is -


A. Hypotonic
B. Isotonic
C. Hypertonic
D. A blood substitutes
Correct answer: C

12. Best treatment for external hemorrhage from an extremity is


A. elevation of the limb
B. proximal tourniquet
C. direct pressure

- 10 -
D. ligation of the bleeding vessel
Correct answer: C

13. The most serious complication of blood transfusion is:


a) Pyrogenic reactions.
b) Thrombophlebitis of recipient vein.
c) Circulatory overloading.
d) incompatibility reactions.
e) Viral hepatitis.
Correct answer: D

14. The earliest sign of hypocolcaemia is


a) Carpo-pedal spasms.
b) Positive Chovstek's sign.
c) Positive Trausseou's sign.
d) Tingling of fingers and circumoral region.
e) Defective blood coagulation,
Correct answer: D

15 . Transfusion of one unit of factor VIII concentrate will raise the level by
A. 1%
B. 2%
C. 10%
D. 25%
Correct answer: B

Hemostasis

1. Ringer lactate does not contain


A. Sodium
B. Potassium
C. Chloride
D. Bicarbonate
Correct answer: D

2. Which of the following is not an ECG sign of hypokalemia


A. Tall T waves
B. U waves
C. Flat ST segment
Correct answer: A

- 11 -
Shock

1. The most effective treatment of anaphylactic shock is :


a) lV antihistaminic.
b) lV corticosteroids.
c) lV antibiotics
d) Intracardiac adrenaline.
Correct answer: B

2. The following conditions may lead to shock except:


a) Penicillin injection.
b) Myocardial infarction.
c) Quinsy.
d) Loss of 8% blood volume.
e) None of the above.
Correct answer: D

3. As regard vasovagal shock all are true except:


a) Usual cause is trauma to trigger area.
b) Psychic trauma is o recognized cause.
c) Atropine is the gold standard treatment of the condition.
d) Usually there is peripheral pooling of blood.
Correct answer: C

4. Septic shock responds best to


a) Massive antibiotics.
b) Intravenous infusion.
c) Adrenocortical steroids.
d) Drainage of septic collections.
e) Vasopressors.
Correct answer: D

5. In an otherwise healthy male with previously normal pulmonary and


cardiac function, how much of the pulmonary vascular bed must usually be
occluded to produce an unstable cardiovascular state (shock)?
a. 10%.
b. 20%.
c. 40%.
d. More than 50%.
Correct answer: D

6. The following types of shock may complicate femoral fracture except:


a. Hypovolemic.
b. Neurogenic.
c. Carcinogenic.
d. Septic.

- 12 -
Correct answer: C

7. The following conditions may lead to shock except:


a. Penicillin injection.
b. Myocardial infarction.
c. Quinsy.
d. Loss of 8% blood volume.
e. None of the above.
Correct answer: D

8. As regard vasovagal shock all are true except:


a. Usual cause is trauma to trigger area.
b. Psychic trauma is a recognized cause.
c. Atropine is the gold standard treatment of the condition.
d. Usually there is peripheral pooling of blood.
Correct answer: C

9. Septic shock responds best to


a. Massive antibiotics.
b. intravenous infusion.
c. Adrenocortical steroids.
d. Drainage of septic collections.
e. Vasopressors.
Correct answer: D

10. The most effective treatment of anaphylactic shock is :


a. lV anti histaminic.
b. lV corticosteroids.
c. lV antibiotics
d. Intracardiac adrenaline.
Correct answer: B

11. A comatosed patient who has sustained multiple closed injures is admitted
with severe hypotension. The hypotension is most probably due to :
a. Intracranial hematoma.
b. Cerebral-concussion.
c. Internal hemorrhage.
d. Neurogenic shock.
e. Cardiogenic shock.
Correct answer: C

12 . After trauma, hypovolemic shock can be due to all except -


A. Pelvic fracture
B. Head injury
C. Blunt trauma to abdominal viscera
D. Hemothorax
Correct answer: B

- 13 -
13 . The recommended fluid for initial fluid challenge in a trauma victim in shock
is -
A. 5% dextrose
B. 10% dextrose
C. Dextrose normal saline
D. Hartman's solution
Correct answer: D

14 . A patient is brought to casualty with severe hypotension following a road


traffic accident. No external injury is evident. The cause of hypotension is:
A. Fracture rib
B. Intrathoracic and abdominal bleed
C. Iatrogenic shock
D. Intracranial bleed
Correct answer: B

15 . A 32yrs blunt trauma patient is brought to the emergency, in a state of shock;


he is not responding to IV crystalloids; Next step of management would be:
A. Albumin transfusion
B. Abdominal compression
C. An immediate laparotomy
D. Blood transfusion
Correct answer: C

16 . Postoperative shock, all are true except:


A. Hypotension is the 1st sign of hypovolemia
B. Acute tubular necrosis due to hypoperfusion is reversible.
C. It should be initially managed with normal saline.
D. Hypovolemia is the most common cause of postoperative kidney failure.
Correct answer: A

17 . A 35 year old Mona developed feature of septicemia Shock in form of


hypotension and low urine output. She was being the treatment for colonic
necrosis. What will be the m/n:
A. IV fluid + dopamine
B. IV fluid only
C. Only dopamine
D. Antibiotic in high dose
Correct answer: B

18 . Plasma expands are used in which shock ?


A. Septic shock
B. Vasovagal shock
C. Neurogenic shock
D. Cardiogenic shock
Correct answer: A

- 14 -
19 . A patient with multiple injuries, ribs, abdomen, spine. Has bradycardia and
hypotension - which type of shock is this?
A. Anaphylactic
B. Neurogenic
C. Hypovolemic and neurogenic
D. Hypovolemic and septic
Correct answer: C

20 . Which of the following is not an immediate cause of death?


A. Thromboembolism
B. Shock
C. Septicemia
D. Ventricular fibrillation
Correct answer: C

Surgical infections, Cellulitis, Tetanus, Gas gangrene

1. Most nosocomial infections involve the


a. Surgical wound.
b. intravenous sites.
c. Respiratory tract.
d. Urinary tract.
e. Deep veins of the leg.
Correct answer: D

2. True statements about Ludwig’s angina do not include that it:


a. is a virulent cellulitis of the floor of the mouth.
b. Usually results from infection with staphylococci.
c. Causes marked swelling in the submandibular region with severe edema of the tongue.
d. May cause suffocation.
e. May require urgent operation.
Correct answer: B

3. About Ludwig’s angina, all the following statements are true, except:
a. This is tuberculous infection of the deep neck spaces
b. Diabetics are more prone to develop this condition
c. It causes laryngeal oedema
d. Oedema of the floor of mouth pushes the tongue upwards
e. Urgent incision and drainage is needed
Correct answer: A

4 . Which of the following statements best represents Ludwig’s angina?


A. A type of coronary artery spasm
B. An infection of the cellular tissues around submandibular salivary gland
C. Esophageal spasm
D. Retropharyngeal infection
Correct answer: B

- 15 -
5. Which of the following is not a priority vaccine for the first year of life
A. Hib (Haemophiles influenzae type b )
B. Hepatitis B
C. DPT (diphtheria, whooping tetanus )
Correct answer: B

6. Localized tetanus is due to


A. large gaping wound
B. atypical strains of bacteria
C. abnormally long incubation period
D. partial immunity
Correct answer: A

7. The earliest finding in tetanus is:


a. Risus sardonicus.
b. Trismus.
c. Dysphagia.
d. Stridor.
Correct answer: C

8. The earliest finding in tetanus is:


a. Risus sardonicus.
b. Trismus.
c. Dysphagia.
d. Stridor.
Correct answer: C

9 . Clostridium difficile is the causative organism for -


A. Tetanus
B. Gas gangrene
C. Wound infection
D. Pseudo membranous enterocolitis
Correct answer: D

10 . In a case of tetanus what will be the most characteristic finding -


A. Incubation period of 2-3 days
B. Masseter spasm
C. Isolation of bacteria from wound
D. CSF examination shows high protein
Correct answer: B

11 . A 40yrs male presents with abrupt onset pain, weakness, loss of contour of
shoulder and muscle wasting on 5th day of tetanus toxoid immunization. Possible
cause is
A. Radial nerve entrapment
B. Hysterical
C. Thoracic outlet syndrome
D. Brachial plexus neuritis
Correct answer: D

- 16 -
12 . In case of missile injury which one is most important:
A. Immunization and nerve repair
B. Fixation of fractures and tetanus immunization
C. Debridement and cleaning
D. Immediate skin suturing
Correct answer: C

13 . Contaminated wound necrotized tissue - best management -


A. Tetanus Toxoid
B. Debridement
C. Antibiotics
D. Anti gas gangrene serum
Correct answer: B

14 . What will be the appropriate management of a patient with a clean wound


over forearm with slight loss of tissue. He has received, Tetanus toxoid 12 years
back:
A. Complete course of TT
B. Only one dose of TT
C. Full dose of Human tetanus Ig
D. No treatment needed
Correct answer: B

15 . Best way to prevent gas gangrene is -


A. Immunoglobulins
B. Hyperbaric oxygen
C. Proper wound debridement
D. Anti gas gangrene serum
Correct answer: C

16 . Gangrene can best be treated by:


A. Complete debridement
B. Hyperbaric oxygen chamber
C. Anti-gas gangrene serum
D. Third generation cephalosporin
Correct answer: A

17. A child presents with a history of scorpion sting. He is having increased


sweating. What is the best next step
A. lytic cocktail
B. atropine
C. antivenom
Correct answer: C

18. Sickness benefit in Employees' State Insurance (ESI) is for how many days
A. 51
B. 61
C. 91
D. 71
Correct answer: C

- 17 -
19. The drug of choice for type 2 lepra reaction is
A .Clofazimine
B. Chloroquin
C. Thalidomide
D. Steroids
Correct answer: C

20. Which of the following is most efficiently transmitted by needle stick injury
A. HIV
B. Hepatitis B
C. Hepatitis C
D. CMV
Correct answer: B

21. The minimum period of treatment for multibacillary leprosy is


A. 1yr
B. 2yrs
C. 6months
D. 9 months
Correct answer B

22 . Most common cause of cellulitis is ?


A. Streptococcus
B. Klebsiella
C. Proteus
D. Pseudomonas
Correct answer: A

23 . The most serious form of clostridial infection is


A. Simple contamination
B. Anaerobic cellulitis
C. Anaerobic myonecrosis
D. Putrefaction
Correct answer: C

- 18 -
Surgical nutrition in Surgical patients

1. Which of the following statements regarding nutritional requirements in


infants are true?
a. The total daily water requirement is estimated to be 100 ml/100 kcal ingested
b. The resting energy expenditure is approximately twice that of an adult
c. The highest rate of nitrogen retention with parenteral nutrition occurs in infants
given approximately 40% of the calories as carbohydrate and the remainder as fat
d. The protein requirement for a newborn infant is approximately 2.5 g/kg/day
Correct answer: a, b, c, d

2. Which of the following will not lead to decrease in cell mediated immunity
A. protein energy malnutrition
B. zinc deficiency
C. selenium deficiency
D. iron deficiency
Correct answer C

3. Parenteral nutrition is best given through


A. femoral vein
B. saphenous vein
C. subclavian vein
Correct answer C

4 . All nutrients are included in Total Parental Nutrition, except-


A. Proteins
B. Fibers
C. Lipids
D. Carbohydrates
Correct answer: B

5 . One is not the indication of total parenteral nutrition -


A. Acute pancreatitis
B. Enterocolic fistula
C. Chronic liver disease
D. Faecal fistula
Correct answer: C

6 . Most common metabolic complication following total parenteral nutrition is


A. Hepatic dysfunction
B. Metabolic acidosis
C. Hypophosphatemia
D. Essential fatty acid deficiency
Correct answer: A

- 19 -
7. All are components of Primary Health Care except
A. providing essential drugs
B. supplementary nutrition
C. treatment of locally endemic diseases and minor ailments
D. prevention of locally endemic diseases
Correct answer B

8 . A patient on total parenteral nutrition for 20 days presents with weakness,


vertigo and convulsions. Diagnosis is:
A. Hypercalcemia
B. Hyperkalemia
C. Hypomagnesemia
D. Hyperammonemia
Correct answer: C

9 . A patient undergoes a prolonged and complicated pancreatic surgery for


chronic pancreatitis. Most preferred route for supplementary nutrition in this
patient would be:
A. Feeding Gastrostomy
B. Total Parenteral Nutrition
C. Oral feeding
D. Feeding Jejunostomy
Correct answer: D

10 . Which of the following is not an indication for parenteral nutrition -?


A. Ileus
B. Severe pancreatitis
C. Colo cutaneous fistula
D. Massive bowel resection
Correct answer: A

11 . Which of the following is not a complication of total Parenteral Nutrition -?


A. Essential fatty acid deficiency
B. Metabolic bone disease
C. Hypophosphatemia
D. Congestive cardiac failure
Correct answer: D

12 . Most common complication of Total Parenteral Nutrition (TPN) is-


A. Acalculous Cholecystitis
B. Hypokalemia
C. Catheter Related complications
D. Acidosis
Correct answer: C

- 20 -
13 . 29yrs- old man, met an accident with a car and has been in 'deep coma' for
the last 15 days. Which of the following is the most suitable route for the
administration of protein and calories?
A. Nasogastric tube feeding
B. Central venous hyperalimentation
C. Jejunostomy tube feeding
D. Gastrostomy tube feeding
Correct answer: C

Principles of organ Transplantation

1. A jaundiced 6-week-old infant has biliary atresia. Which of the following


statements are true?
a. Portoenterostomy is the initial procedure of choice
b. Primary liver transplantation using either a reduced sized cadaveric graft or a living
related graft is now the procedure of choice
c. Approximately two-thirds of patients managed with portoenterostomy will develop
chronic liver disease sufficient to indicate liver transplantation
d. Because biliary atresia has pathogenic components of acute and chronic
inflammation, anti-inflammatory therapy is known to delay onset of liver failure
Correct answer: a, c

2 . In a patient with compensated liver cirrhosis presented with history of variceal


bleed. The treatment of choice in this patient is:
A. Propranolol
B. Liver transplantation
C. TIPS (Trans jugular intrahepatic portal shunt)
D. Endoscopic sclerotherapy
Correct answer: D

3 . Within 48 hours of transplantation, skin graft survives due to:


A. Amount of saline in graft
B. Plasma imbibition
C. New vessels growing from the donor tissue
D. Connection between donor and recipient capillaries
Correct answer: B

4 . According to Pugh's classification moderate to severe hepatic insufficiency is


managed by:
A. Sclerotherapy
B. Conservative
C. Orthotopic liver transplantation
D. Shunt surgery
Correct answer: C

- 21 -
5 . The advantage of Bladder drainage over Enteric drainage after Pancreatic
Transplantation is better monitoring of
A. Amylase levels
B. HBA1C levels
C. Electrolyte levels
D. Glucose levels
Correct answer: A

Principles of Laparoscopies

1 . Which gas is used in laparoscopy -?


A. CO2
B. N2O
C. O2
D. N2
Correct answer: A

2. Insufflation pressure during laparoscopy is ?


A. 5-10 mm Hg
B. 11-15 mm Hg
C. 15-20 mm Hg
D. 20-25 mm Hg
Correct answer: B

Priciples of oncology

1 . Which of the following is the Post - Chemotherapy based staging system in


Wilm's tumor
A. International Society of Pediatric Oncology (SIOP)
B. National Wilm's Tumor staging System (NWTSG)
C. Chadwick
D. AJCC TNM
Correct answer: A

2. Which of the following is not a tumor suppressor gene


A. N myc
B. Rb
C. p53
D. Mad-max
Correct answer A

- 22 -
3- A 75-year-old man with lung cancer was admitted to the hospital with
worsening dyspnea. He complains of progressive dyspnea for 2 months. On
physical examination, he has distended jugular veins, plethoric face, venous
collaterals on his chest wall, and distant heart sounds. What is the most likely
diagnosis?
A. Pericardial tamponade
B. Constrictive pericarditis
C. Congestive heart failure
D. Superior vena cava (SVC) syndrome
Correct answers: D

4- A 42-year-old man with a medical history significant for lung cancer is brought
to an emergency room for altered mental status. His family reports altered mental
status for 3 days. He has been lethargic and difficult to arouse. On examination,
he has orthostatic blood pressure and dry mucous membranes. Laboratory
evaluation reveals Ca 14.5 mg/dl and Cr 2.0 mg/dl. Which of the following is the
most appropriate initial management for this patient?
A. 0.9% saline bolus
B. Furosemide
C. Pamidronate
D. 0.45% saline bolus
Correct answers: A

5- A 25-year-old man was recently diagnosed with leukemia. He is admitted to the


hospital for further evaluation and chemotherapy initiation. He only complains of
fatigue and malaise. Laboratory evaluation reveals pancytopenia, hyperkalemia
(K 6.8), uric acid 13, hyperphosphatemia (12), and elevated lactate dehydrogenase
(LDH). What is the most likely cause of his electrolyte abnormalities?
A. Laboratory error
B. Tumor lysis syndrome
C. Hypercalcemia
D. Acute renal failure
Correct answers: B

6. The most urgent complication in a patient with Non-Hodgkin’s Lymphoma that


needs to be evaluated at diagnosis and followed closely is:
A. An elevated WBC > 50,000
B. A uric acid of 5.0
C. A mediastinal mass
D. Complaints of left flank pain
Correct answer: C

7. The untrue statement about desmoid tumors of the abdominal wall is that they :
a) Arise from the rectus muscle or its sheath.
b) Occur most often in multiparous women.
c) Are locally invasive.
d) May give rise to metastasis.
e) Should be treated by wide local excision.
Correct answer: D

- 23 -
8. The untrue statement about desmoid tumors of the abdominal wall is that they
:
a. Arise from the rectus muscle or its sheath.
b. Occur most often in multiparous women.
c. Are locally invasive.
d. May give rise to metastasis.
e. Should be treated by wide local excision.
Correct answer: D

9. The most frequent GIT complication of an extensive burn is :


a. Acute gastritis.
b. Acute dilatation of the stomach.
c. Curling's ulcer.
d. Paralytic ileus.
e. Diarrhea.
Correct answer: B

10. Which statement is incorrect concerning electrical burns?


a. Usually hove o small surface area.
b. Are always superficial.
c. Are often associated with massive muscle necrosis.
d. Moy cause reddish discoloration of the urine.
e. Are best treated by immediate excision and grafting.
Correct answer: B

11. Keloids are characterized by the following, except:


a. Consist of dense overgrowth of scar tissue.
b. Develop after wounds, burns and vaccination marks.
c. Are particularly common in negroes and pregnant females.
d. Occur most often on the face. neck and front of the chest.
e. May turn malignant.
Correct answer: E

12. ln the rule of nines:


a- Front of trunk 27%
b- Head and neck > 18%
c- Each lower limb I8%
d- All of the above
Correct answer: C

13 . Which of the following is the most common malignant tumor of adult males
in India?
A. Cola-rectal carcinoma
B. Lung cancer
C. Oropharyngeal carcinoma
D. Gastric carcinoma
Correct answer: C

- 24 -
2- Burn and Principles of plastic surgery

Types of Burns and its complications

1. Which burn can be treated with calcium gluconate?


a) hydrochloric acid
b) acetic acid
c) sulfuric acid
d) hydrofluoric acid
e) nitric acid
Correct answer: D

2. Which is not seen in hydrofluoric acid burns?


a) hyperkalemia
b) hypocalcemia
c) hypomagnesemia
d) hypernatremia
e) severe pain]
Correct answer: D

3. Which is false about chemical burns?


a) alkalis cause more damage than acids
b) irrigation is not the mainstay of treatment in elemental metals
c) eyes should be irrigated at least 30 mins with 1-2 L normal saline
d) calcium gluconate can be given by intraarterial infusion for hydrofluoric acid burns
e) alkali burns should be doused in sand prior to irrigation
Correct answer: E

4. What percentage of a 1-year-old child is burnt if their anterior torso and all of R
arm are burnt?
a) 10%
b) 15%
c) 24%
d) 32%
e) 49%
Correct answer: C

5. Which is not a feature of lightening strike?


a) death due to VF
b) cataracts
c) kerauno paralysis
d) feathering burns
e) perforated ear drum
Correct answer: A

- 25 -
6. Which is false about a lightening strike?
a) the patient should not be handled as they are charged and dangerous
b) flashover is potentially life saving
c) the likelihood of long-term impairment after recovery is low
d) being stuck directly is more dangerous than side flash and step voltage
e) if caught in a lightening strike you should keep your feet together and crouch down
Correct answer: A

7. Which is false of electrocution?


a) death is usually due to VF
b) delayed fatal arrythmias are extremely rare
c) blood vessels, nerves and muscles are the most conductive
d) dc current is worse than ac
e) if pregnant the fetus is usually OK
Correct answer: D

8. How much blood loss is necessary to first show hypotension?


a) 10%
b) 20%
c) 25%
d) 30%
e) 40%
Correct answer: D

9. Which is incorrect of cervical spine injuries?


a) The majority of spinal injuries occur in the cervical spine.
b) In the adult the spinal cord ends at L1.
c) The spinothalamic tract transmits pain and temperature from the opposite side of
the body.
d) The diaphragm is paralysed in spinal injury if the lesion is above C3.
e) The neurologic level of injury is the most caudal segment of the spinal cord with
abnormal sensory and motor function.
Correct answer: E

10. Which dermatome is INCORRECT?


a. C7 – middle finger
b. T4- nipple
c. T8 – umbilicus
d. S4-S5 – perianal region.
e. L4 – medial aspect of leg.
Correct answer: C

11. Above which level will the patient be tetraplegic rather than paraplegic?
a. C7
b. C8
c. T1
d. T2
e. T3
Correct answer: C

- 26 -
12. Which is incorrect of spinal cord syndromes?
a. Central cord syndrome has greater power loss of lower extremities than upper.
b. Central cord syndrome occurs with hyperextension injuries often with patients with
pre-existing cervical canal stenosis.
c. Anterior cord syndrome is characterized by paraplegia and loss of pain and
temperature sensation, with intact position sense, vibration.
d. Brown-Sequard syndrome results from hemi section of the cord.
e. Central cord syndrome and anterior cord syndrome are thought to result from
compromised anterior spinal artery blood supply.
Correct answer: A

13. Which is incorrect of cervical spine fractures?


a. Burst fracture of C1 (Jefferson), are best seen on open mouth view with lateral
displacement of lateral masses.
b. Rotatory subluxation of C1 will result in persistent rotation of head and on open
mouth view odontoid not equidistant from lateral masses.
c. Type 2 odontoid fractures occur through the base of the dens.
d. Hangman’s fracture involves the posterior elements of C1.
e. C5 is the most common level of cervical vertebral fracture.
Correct answer: D

14. Which is incorrect of injuries to the spine?


a. The 3-film series of C-spines with pick up approx. 92% of all injuries.
b. A bilateral facet dislocation results in forward movement of one vertebral body
over another by 50% or more.
c. Normal soft tissue width at C1-C4 in adults is 5 mm.
d. About 10% of patients with C-spine fracture will have a second fracture in the
vertebral column elsewhere.
e. The gap between the anterior surface of dens and posterior surface of C1 should be
less than 5 mm in adults and 3 mm in children.
Correct answer: E

15. Burns involving head face and trunk constitutes how much percentage
A. 45%
B. 55%
C. 60%
D. 65%
Correct answer A

16 . In excessive burns, least useful is -


A. Blood
B. Dextran
C. Ringer lactate
D. Nasogastric intubation
Correct answer: A

- 27 -
17. Parkland formula for burns is for
A. Ringer lactate
B. glucose saline
C. Normal saline
Correct answer: A

18 . All may be seen in deep burns except:


A. Fluid loss by evaporation
B. Vasodilatation
C. Hyperthermia
D. Increase vascular permeability
Correct answer: C

19. ln the rule of nines:


a- Front of trunk 27%
b- Head and neck > 18%
c- Each lower limb I8%
d- All of the above
Correct answer: C

20 . True regarding burns is:


A. Hyperglycemia is seen in early burns
B. Child with burns should have damp dressing
C. Chemical powder burn should be kept dry
D. 3rd degrees are painful
Correct answer: A

21 . Which of the following ulcer may develop in burn tissue?


A. Melanoma
B. Curlings
C. Marjolins
D. Rodent
Correct answer: C

22 . A patient suffered from 3rd degree burn of right upper limb, 2nd degree burns of
right lower limb and 1st degree of burn of whole of the back. Total percentage of burn
will be
A. 27 %
B. 36 %
C. 45 %
D. 54 %
Correct answer: A

23 . Myoglobinuria is seen in which type of burns ?


A. Flame burn
B. Scald burn
C. Electric burn
D. Contact burn
Correct answer: C

- 28 -
24 . A young man weighing 65 kg was admitted to the hospital with severe burns in a
severe catabolic state. An individual in this state requires 40 kcal per kg body weight
per day and 2 gms of protein/kg body weight/day. This young man was given a solution
containing 20% glucose and 4.25% protein. If 3000 ml of solution is infused per day:
A. The patient would not be getting sufficient protein
B. The calories supplied would be inadequate
C. Both protein and calories would be adequate
D. Too much protein is being infused
Correct answer: C

25 . A 16 years girl sustains a steam burn measuring 6 by 7 inches over the ulnar aspect
of her right forearm. Blisters develop over the entire area of the burn wound, and by the
time the patient is seen 6 hour after the injury, some of the blisters have ruptured
spontaneously. In addition to debridement of the necrotic epithelium, all therapeutic
regiments might be considered appropriate for him Except:
A. Application of mafenide acetate cream, but no daily washes or dressing
B. Heterograft application with sutures to secure it in place and daily washes, but no
dressing
C. Application of silver sulfadiazine and daily washes, but no dressing
D. Application of polyvinyl pyrrolidone foam, daily washes and a light occlusive
dressing changed daily
Correct answer: B

26 . Superficial burn true all except


A. Painful
B. Erythematous
C. Blister formation
D. White leathery skin
Correct answer: D

27 . Burns patient curling ulcer most common in which part of duodenum -


A. First part
B. Second part
C. Junction b/w second and third part
D. Third part
Correct answer: A

28 . Dermis involved in the -


A. First degree burns
B. Second degree burns
C. Third degree burns
D. Fourth degree burns
Correct answer: C

29 . Curlings ulcers are found in:


A. Extensive Burn Injury
B. CVA
C. Head injury
D. Diabetes mellitus
Correct answer: A

- 29 -
30 . Sun burns are -
A. First degree burns
B. Second degree burns
C. Third degree burns
D. Could be any of the above
Correct answer: A

31 . The best guide to adequate tissue perfusion in the fluid management of a patient
with burns, is to ensure a minimum hourly urine output of -
A. 10-30 ml
B. 30-50 ml
C. 50-70 ml
D. 70-100 ml
Correct answer: B

32 . In post burn patient, true is -


A. Hypokalemic alkalosis
B. Hyperkalemic alkalosis
C. Hyperkalemic acidosis
D. Hypokalemic acidosis
Correct answer: C

33 . Choice of fluid in case of burn in first 24 hours is ?


A. Normal saline
B. Ringer lactate
C. 5% dextrose
D. Whole blood
Correct answer: B

34 . 71 kg – men (154 pounds) is transferred to a burn centre 4 weeks after sustaining a


second and third-degree burn injury to 45% of his total body surface area. Before to
accident, the patients weight was 89 kg (198 pounds). The patient has not been given
anything by mouth since the injury except for antacids because of previous ulcer
history. On physical examination, the patients burn wounds are clean, but only minimal
healing is evident and thick adherent Escher is present. The patient's abdomen is soft
and nondistended, and active bowel sounds are heard. His stools are trace-positive for
blood, and he has a right inguinal hernia, which appears to be easily reducible. He has
poor range of motion of all involved joints and has developed early axillary and
popliteal fossae flexion contractures. For managing him, top priority must be given to
correcting:
A. The inguinal hernia treated by surgical repair using local anesthesia
B. The nutritional status by oral supplementation or parental hyperalimentation
C. The presence of blood in stools by the increasing the dose of antacids+H1 receptor
blocker
D. The open, poorly healing burn wounds treated by surgical excision and grafting
Correct answer: B

- 30 -
35. The main advantage of Lund and Browder charts over rule of nines is:
a- It is easier
b- More accurate to regarding sex of patient
c- More accurate regarding age of patient
d- All of the above
Correct answer: C

36. About 3rd degree (full thickness) burn:


a- Completely painless
b- No graft is needed
c- Healing occurs from skin appendages
d- Usually dry, white or black
Correct answer: D

37. ln the rule of nines:


a- Front of trunk = 27%
b- Head and neck = 18%
c- Each lower limb = I8%
d- All of the above
Correct answer: C

38. About management of burns:


a- Biological dressing decreases all complications of burn
b- Parkland's formula is a good rule for fluid replacement
c- Occlusive dressing is easier to perform
d- Exposure method is suitable for face and buttocks
e- All of the above
Correct answer: E

39. Which of the following is true about Langer's line of the skin?
a) These are in the direction of muscle fibers.
b) These are perpendicular to the long axis of underlying muscle fibers
c) Most favorable incision is circular over an exposed area
d) Most favorable incision is a rhomboid shaped one over an exposed area
Correct answer: B

40. The repair of cleft lip should be done at which age


a) 1 month
b) 3 months
c) 6 months
d) 1 year
Correct answer: B

41. Fastest promoter of epithelisation in wound healing occurs with


a) Tight approximation of wound edges
b) Occlusive dressing
c) Leaving the wound open
d) Steroid Application
Correct answer: B

- 31 -
42 . The malignancy in burns are:
A. Marjolins ulcer
B. Desmoid tumor
C. Basal cell ca
D. Squamous cell ca
Correct answer: A

43. The most frequent GIT complication of an extensive burn is :


a. Acute gastritis.
b. Acute dilatation of the stomach.
c. Curling's ulcer.
d. Paralytic ileus.
e. Diarrhea.
Correct answer: B

44. Which statement is incorrect concerning electrical burns?


a. Usually hove o small surface area.
b. Are always superficial.
c. Are often associated with massive muscle necrosis.
d. Moy cause reddish discoloration of the urine.
e. Are best treated by immediate excision and grafting.
Correct answer: B

45. The following are subtypes of capillary hemangioma except:


a- Strawberry hemangioma
b- Portwine hemangioma
c- Cirsoid aneurism
d- Salmon Patch
Correct answer: C

46.A subtype of capillary hemangioma raised above skin surface:


a- Strawberry hemangioma
b- Portwine hemangioma
c- Salmon patch
d- None of the above
Correct answer: A

47.Metabolic changes after burning are due to the following except:


a. The endocrine responses to injury.
b. Local fluid loss into the burnt area.
c. Reduced heat loss.
d. Increased insensible water loss.
e. Bacterial infection.
Correct answer: C

48. The most frequent gastrointestinal complication of an extensive burn is :


a. Acute gastritis.
b. Acute dilatation of the stomach.
c. Curling's ulcer.
d. Paralytic ileus.

- 32 -
e. Diarrhea
Correct answer: B

49. The most useful measure in preventing renal shut-down in post burn patients is:
a. Monitoring an hourly urine output between 30 and 50 ml.
b. Alkalinization of the urine.
c. Mannitol administration.
d. Administration of colloids.
e. Blood transfusion.
Correct answer: A

50. Which statement is incorrect concerning electrical burns?


a. Usually have a small surface area.
b. Are always superficial.
c. Are often associated with massive muscle necrosis.
d. May cause reddish discoloration of the urine.
e. Are best treated by immediate excision and grafting.
Correct answer: B

51. Keloids are characterized by the following, except:


a. Consist of dense overgrowth of scar tissue.
b. Develop after wounds, burns and vaccination marks.
c. Are particularly common in negroes and pregnant females.
d. Occur most often on the face. neck and front of the chest.
e. May turn malignant.
Correct answer: E

52. A paraplegic bed-ridden patient developed a large deep bed-sore over his sacrum.
The best regimen is by wide excision and:
a. Local applications.
b. Thiersch grafting.
c. Wolfe-grafting.
d. Local skin flops.
e. Myocutaneous flap.
Correct answer: E

53.ln unilateral hare-lip, the following statements are true except that it:
a. Affects the upper lip only.
b. May be partial or complete.
c. ls due to failure to fusion between the maxillary process with the frontal-nasal
process and the maxillary process of the opposite side.
d. ls always associated with nasal deformity.
e. Produces no serious speech defects.
Correct answer: E

54. Concerning complete cleft palate, the untrue statement is that it:
a. ls due to failure fusion of the palatal shelves of the maxillary processes with each
other and with the frontonasal process.
b. ls often associated with cleft-lip and broadening of the face.
c. interferes with nutrition and speech.

- 33 -
d. Predisposes to upper respiratory tract infections.
e. Requires surgical repair after the second year of life.
Correct answer: E

55. A 6-year-old boy has a 2 cm lesion on Right cheek. The lesion has been enhancing
since birth. On Auscultation lesion is pulsatile, it has a thrill, and it has a harsh sound
on auscultation. It has multiple feeding arteries. The best treatment option for this lesion
would be
a) External beam radiotherapy
b) Ligation of the feeding arteries
c) LASER therapy
d) Embolization followed by surgery
Correct answer: D

56. Which of the following is not true about keloids?


a) Keloid is excessive scar formation within and outside the boundaries of skin
incision
b) Recurrence rate of keloid after excision is 10-20%
c) Exact cause of keloid formation is not known but fibroblast dysfunction is
commonly attributed
d) Adjunctive treatment besides surgery gives the best results
Correct answer: B

Skin graft and flaps

1 . Graft Survival in first 48 hours depends on-


A. Plasma imbibition
B. Ingrowth of capillaries in donor tissue
C. Amount of Saline in graft
D. Connection between donor and recipient capillaries
Correct answer: A

2 . Full thickness graft can be obtained from A/E-


A. Supraclavicular area
B. Elbow
C. Axilla
D. Groin
Correct answer: C

3 . Ideal graft for leg injury with 10 x 10 cm. exposed bone:


A. Amniotic membrane graft
B. Pedicle graft
C. Full thickness graft
D. Split thickness skin graft
Correct answer: B

4 . Skin grafting is absolutely CI in which skin infection -


- 34 -
A. Staphylococcus
B. Pseudomonas
C. Streptococus
D. Proteus
Correct answer: C

5 . All can take split thickness graft except -


A. Fat
B. Muscle
C. Skull bone
D. Deep facia
Correct answer: C
6 . For bone deep wound the best cover is -
A. Split thickness skin graft
B. Full thickness skin graft
C. Pedicle graft
D. Muscle graft
Correct answer: C

7 . Which of the following is Dacron vascular graft?


A. Textile synthetic
B. Nontextile synthetic
C. Textile biologic
D. Nontextile biologic
Correct answer: A

8 . Best Material for below inguinal arterial graft is


A. Dacron
B. Teflon
C. Saphenous vein graft (upside - down)
D. PTFE
Correct answer: C

9 . Which of the following graft is known as wolf's graft -


A. Split thickness graft
B. Full thickness graft
C. Partial thickness graft
D. Myocutaneous graft
Correct answer: B

10 . Best cover for an exposed bone deep wound is -


A. Split-thickness skin graft
B. Full thickness skin graft
C. Pedicle graft
D. Muscle graft
Correct answer: C

- 35 -
Cleft lip, Cleft palate

1. Langenbeek's repair:
a- ls method of grafting in burns
b- ls method of repair after surgical excision of Sq.C.C.
c- ls method of repair of cleft lip
d- ls method of repair of cleft palate
Correct answer: D

2. About cleft upper lip all are correct except:


a- Median is the most common
b- Lateral is due to non-fusion of median nasal process (future frenulum) with
maxillary process
c- Lateral is more common on left side
d- Best time for repair is the earliest possible time (3 months)
e- Repair is mainly for cosmetic purposes
Correct answer: A

3. About cleft palate all are correct except:


a- Usually associated with other congenital anomalies
b- May range from cleft uvula up to tripartite deformity
c- Causes nasal regurgitation, nasal deformity
d- Best time for repair ls 5 years old
Correct answer: D

4. Langenbeek’s repair:
a. Is a method of grafting in burns
b. Is a method of repair after surgical excision of SCC
c. Is a method of repair of cleft lip
d. Is a method of repair of cleft palate
Correct answer : D

5. About cleft upper lip all are correct except:


a. Median is the most common
b. Lateral is due to non-fusion of median nasal process (future frenulum) with maxillary
process
c. Lateral is more common on left side
d. Best time for repair is the earliest possible time (3 months)
e. Repair is mainly for cosmetic purposes
Correct answer : A

6. About cleft palate all are correct except:


a- Usually associated with other congenital anomalies
b- May range from cleft uvula up to tripartite deformity
c- Causes nasal regurgitation, nasal deformity
d- Best time for repair is 5 years old
Correct answer : D

- 36 -
7 . The following are true of cleft lip except
A. It can be diagnosed inutero
B. Most of the cleft lip babies do not feed well
C. Most of them are associated with cleft palate
D. Surgery is usually performed before 6 months of age
Correct answer: B

8. Concerning complete cleft palate, the untrue statement is that it:


a. is due to failure fusion of the palatal shelves of the maxillary processes with each other
and with the frontonasal process.
b. is often associated with cleft-lip and broadening of the face.
c. interferes with nutrition and speech.
d. Predisposes to upper respiratory tract infections.
e. Requires surgical repair after the second year of life.
Correct answer : E

9 . Cleft palate is ideally repaired at:


A. 1 year of age
B. 5 month of age
C. 6-8 yrs of age
D. Before going to school
Correct answer: A

10. Abbey-Estlander flap is used in the reconstruction of which one?


A. Lip.
B. Buccal mucosa.
C. Palate.
D. Tongue.
Correct answer: A

11. All are true about submucosal cleft palate except -


A. Bifid uvula
B. Notched hard palate
C. Lip pots
D. Zona pellucida
Correct answer: C

12 . Cleft palate is best repaired at


A. Soon after birth
B. At 1 month
C. At 6-8 months
D. Between 1-1.5 years
Correct answer: D

- 37 -
Hemangioma and vascular malformation

1. A29-year-old female swimmer develops a pigmented lesion on the right thigh. With
reference to a pigmented lesion, there is an increased risk of developing melanoma if it
is identified with which of the following?
(A) Hutchinson freckle (lentigomaligna)
(B) Freckle involving basal layer of skin
(C) Congenital nevocellular nevi
(D) Hemangioma
(E) Tophi
Correct answer: C

2 . A port wine stain is a


A. Type of melanoma
B. Pre-malignant lesion of skin
C. Type of hemangioma
D. Type of bruising of skin
Correct answer: C

3 . Which of the following is true:


A. Cirsoid aneurysm is a hemangioma which heals spontaneously
B. Port wine stain disappears later in life
C. Strawberry angioma present at birth
D. Cirsoid aneurysm treated by ligation of feeding vessels
Correct answer: D

4 . Which type of haemangioma resolves spontaneously:


A. Strawberry angioma
B. Port wine stain
C. Plexiform angioma
D. Cavernous angioma
Correct answer: A

5. Haemangioma in ultrasound is:


a. lsoechoic
b. Hypo echoic
c. Hyper echoic
d. Any of the obove
Correct answer: C

- 38 -
3- Skin and soft tissue

Dermoid cyst , seabceous cyst, lipoma

1. Congenital dermoid cysts are characterized by the following features except


a. Occur of lines of embryological fusion.
b. Are commonest on the face.
c. Are often attached to the overlying skin.
d. May cause hollowing of subjacent bone.
e. Are lined by stratified squamous epithelium.
Correct answer: C

2. Sebaceous cyst is characterized by the following except that it:


a. ls due to obstruction of a sebaceous gland.
b. ls lined by stratified squamous epithelium.
c. Contains a yellow pultaceous greasy material known as sebum.
d. May occur on the palms and soles
e. ls always anchored to the overlying skin of the punctum of the obstructed gland.
Correct answer: D

3. All of the following statements are true as regard benign breast disease, except
A Benign breast disease is the most common cause of breast problems.
B Lipoma is a common condition of the breast.
C Traumatic fat necrosis can be mistaken for a carcinoma.
D 30 per cent of breast cysts recur after aspiration.
Correct answer: B

4- All of the following are true about lipoma except:


a) It is benign tumor of adipose tissues.
b) Pathologically, it is a yellowish lobulated aggregation of fat cells.
c) Attached to covering skin by puncture.
d) The most common types is the subcutaneous lipoma
Correct answer : C

5- If we have a case of multiple lipomas:


a) We will excise all of them.
b) Excise the skin covering most of them.
c) Do drainage and packing of biggest one.
d) Removing only the most complicated one
Correct answer : D

- 39 -
6- All the following can be considered as complication of lipoma is a patient
except:
a) Myxomatous transformation.
b) Sarcomatous transformation.
c) Infection, suppuration and discharge.
d) Calcification especially is axilla, groin and buttocks
Correct answer : C

7- The most serious type of lipoma is:


a) Retroperitoneal type.
b) Sub-periosteal type.
c) Intermuscular type.
d) Submucous type
Correct answer: A

8- The most common type of lipoma is:


a) Retroperitoneal lipoma.
b) subfacial lipoma.
c) Submucous lipoma
d) Subcutaneous lipoma
Correct answer: D

9- Attachment to the skin by fibrous tissues (= Dimpling and slippery edge), are
characters of:
a) Rodent ulcer.
b) Aneurysm.
c) Lipoma.
d) Sebaceous cyst.
Correct answer: C

10- Sebaceous cyst can appear in any place except:


a) Brain.
b) Palm and sole.
c) Scrotum.
d) Scalp.
Correct answer : B

11- The edge of the sebaceous cyst is:


a) Well defined.
b) Slippery.
c) Punched out.
d) Everted.
Correct answer: A

12- The following are example of swelling move in only direction except:
a) Ganglion.
b) Neurofibroma.
c) Lipoma.
d) Aneurysm.
Correct answer: C

- 40 -
13- Spot diagnosis to differentiated lipoma and sebaceous cyst is:
a) Consistency.
b) Attachment to deep muscle.
c) Slippery edge of the lipoma and the punctum of the sebaceous cyst.
d) Compressibility.
Correct answer: C

14 . Which of the following is Cock's peculiar tumor?


A. Squamous cell CA
B. Basal cell CA
C. Cylindroma
D. Ulcerated sebaceous cyst
Correct answer: D

15. Sebaceous cyst is characterized by the following except that it:


a) ls due to obstruction of o sebaceous gland.
b) ls lined by stratified squamous epithelium.
c) Contains a yellow pultaceous greasy material known as sebum.
d) Moy occur on the palms ond soles
e) ls always anchored to the overlying skin of the punctum of the obstructed gland.
Correct answer: D

Skin tumors ( BCC and SCC )

1. A 45-year-old soccer player presents with a 6-month history of an ulcerative


nodular lesion, 1.5 cm in diameter in the region of the right oral commissure.
Biopsy reveals basal cell carcinoma. The preferred treatment is:
(A) Mohs micrographic surgery and subsequent reconstruction
(B) Excision with a clinical margin and local flap repair
(C) Topical 5-FU
(D) Local radiation therapy
(E) Cryotherapy
Correct answer: A

2. A 67-year-old business executive and tennis player has a basal cell carcinoma
removed from the right cheek. What is TRUE of basal cell carcinoma (Fig. 2–3)?
(A) It may show a flat ulcer.
(B) It may metastasize to lymph nodes.
(C) It may metastasize to remote skin areas.
(D) It is found exclusively in the head and neck.
(E) It is best treated by topical 5-FU
Correct answer: A

- 41 -
3. List the layers of skin from the most superficial to the deepest layer adjacent to
the dermis (a) basal layer, (b) granular layer, (c) prickle layer, and (d) stratum
corneum.
(A) a b c d
(B) d b a c
(C) d c b a
(D) c a b d
(E) c a d b
Correct answer: C

4. A completely excised skin lesion of the face proved to be a basal cell carcinoma.
The further management of the case should be :
a. Lymph node dissection.
b. Radiotherapy.
c. Chemotherapy.
d. Regular follow up.
e. Reassurance of the patient.
Correct answer: D

5. A 35-year-old White male previously diagnosed with basal cell nevus syndrome
(Gorlin’s syndrome) presents with a new lesion for treatment. Apart from mulitple
basal cell lesions other features of this disorder may include:
(A) The disorder is genetically determined and transmitted as an autosomal dominant.
(B) Rib abnormalities such as splayed or bifid ribs.
(C) Skin ribs on the palms and soles.
(D) A benign clinical course before puberty.
(E) Normal mental development.
Correct answer: E

6. The following are (is) correctly matched about origin of skin tumors:
a- Squamous cell carcinoma: epidermis
b- Basal cell carcinoma: Basal areas or appendages
c- Melanoma: melanocytes
d- All of the above
Correct answer: D

7. The most characteristic histopathological finding ln basal cell carcinoma is:


a- Microscopic LNs metastasis
b- Cell nests
c- Pallisade appearance
d- Anaplastic collections
Correct answer: C

8- the most common type of basal cell carcinoma is:


a- Rodent ulcer
b- Turban type
c- Pigment type
d- Field fire type
Correct answer: :A

- 42 -
9. All of the following are signs of epitheliomatous transformation in basal cell
carcinoma except:
a- Rapid growth
b- lnvasion of basement membrane
c- Everted edges
d- Hard fixed LNs
Correct answer: B

10. A case presented by chronic ulcer resistant for healing, the following support
your diagnosis as basal cell carcinoma except:
a. Male sex
b. 50 years age
c. Face as primary site
d. The surgeon decision to conserve
Correct answer: D

11. A completely excised skin lesion of the face proved to be basal cell carcinoma.
The further management of the case should be:
a. Lymph node dissection.
b. Radiotherapy.
c. Chemotherapy.
d. Regular follow up.
e. Reassurance of the patient
Correct answer: D

12. Concerning basal-cell carcinomas, the false statement is that they are :
a. Much less common than squamous cell carcinomas.
b. Very rare in oriental and block races
c. Particularly common in tropical regions.
d. Characterized histologically by dork-staining solid mosses of cells arising from the
basal layer of the epidermis.
e. Commonest on the exposed skin of blonde subjects and outdoor workers
Correct answer: A

13. Which statement is incorrect concerning rodent ulcer?


a. ls a basal-cell carcinoma.
b. Have a red granular floor and a rolled-in beaded edge.
c. May be pigmented.
d. May spread to regional lymph nodes.
e. ls best treated by surgical excision.
Correct answer: D

14. A skin lesion which is ulcerated with rolled, smooth, pearly borders is most
likely:
a.Melanoma
b.Basal cell carcinoma
c.Seborrheic keratosis
d.Squamous cell carcinoma
Correct answer: B

- 43 -
15. Which of the following is NOT a pre-malignant (or does not have malignant
potential) skin condition/ lesion?
a. Actinic keratosis
b. Nevus sebaceous
c. Dysplastic nevus
d. Seborrheic keratosis
Correct answer: D

16. Which of the following skin lesions is NOT associated with sun exposure?
a. Actinic keratosis
b. Basal cell carcinoma
c. Seborrheic keratosis
d. Melanoma
Correct answer: D

17. This type of skin lesion is elevated, brown, and has a “stuck on” appearance:
a. Actinic keratosis
b. Basal cell carcinoma
c. Xanthoma
d. Seborrheic keratosis
e. Verruca
Correct answer: D

18.The following are all types of basal cell carcinoma EXCEPT:


a.Nodular
b.Pigmented
c.Marjolin’s ulcer
d.Morphea form
Correct answer: C
19.What is the most common basal cell carcinoma?
a.Sclerosing
b.Morphea form
c.Nodular
d.Pigmented
e.Superficial
Correct answer: C

20. Which of the following is NOT associated with Marjolin’s ulcer?


a.Basal cell carcinoma
b.Attenuated burn
c.Squamous cell carcinoma
d.Inflammation
e.Aged scar
Correct answer: A

- 44 -
21. The following are true of the skin lesions EXCEPT:
a.Actinic keratosis is completely benign with no malignant potential
b.Nodular ulcerative variant is a type of basal cell carcinoma
c.Squamous cell carcinoma occurs in primarily sun exposed area of skin
d.Basal cell carcinoma has a 95% cure rate if lesion is less than 2 cm in diameter
Correct answer:A

22 . Most common type of basal cell Ca is


A. Ulcerated
B. Morphea
C. Cystic
D. Pigmented
Correct answer: C

23. The following are (is) incorrectly matched about behavior of skin tumors:
a- Malignant melanoma: locally malignant tumor
b- Squamous cell carcinoma: malignant tumor
c- Basal cell carcinoma: locally malignant tumor
d- Hemangioma: Hamartoma
e- Nevi: benign tumor
Correct answer: A

24. The most common site for the following diseases is correctly matched except:
a- Keloids: face, neck and skin over sternum
b- Hypertrophic scars: extensor surfaces
c- Hemangiomas: head and neck
d- Basal cell carcinoma: face
Correct answer: B

25. The following about squamous cell carcinomas untrue:


a- Xeroderma pigmentosa is a recognized risk factor
b- SCC occurs in sun exposure areas
c- SSS is called marjolin's ulcer when it occurs in chronic ulcers
d- lt can give distant metastasis in contrary to BCC
e- All ore true
Correct answer: E

26. Contraindications to irradiation in skin malignancy include the following


except:
a- Recurrent tumors (after irradiation)
b- Deeply invasive tumors
c- Lesions in upper half of the face
d-old age group
Correct answer: D

- 45 -
27 . Characteristic of basal cell carcinoma is -
A. Keratin pearls
B. Foam cells
C. Nuclear palisading
D. Psammoma bodies
Correct answer: C

28 . Which of the following cutaneous malignancies do not metastasize through


the lymphatics
A. Squamous cell carcinoma
B. Basal cell carcinoma
C. Melanoma
D. Kaposi's sarcoma
Correct answer: B

29 . A 48 year old sports photographer has noticed a small nodule over the upper
lip from four months. The nodule is pearly white with central necrosis,
telangiectasia. The most likely diagnosis would be -
A. Atypical melanoma
B. Kaposi’s sarcoma
C. Basal cell carcinoma
D. Squamous cell carcinoma
Correct answer: C

30 . MOHs micrographic excision for basal cell carcinoma is used for all the
following except -
A. Recurrent tumor
B. Tumor less than 2 cm in diameter
C. Tumors with aggressive histology
D. Tumors with perineural invasion
Correct answer: B

31 . A rodent ulcer
A. Is a squamous cell carcinoma
B. Is a basal cell carcinoma
C. Is a venous ulcer
D. Contains epithelial pearls
Correct answer: B

32 . In Marjolin's ulcer, the most common malignancy is:


A. Squamous cell carcinoma
B. Basal cell carcinoma
C. Neurotrophic malignant melanoma
D. Malignant fibrous histiocytoma
Correct answer: A

- 46 -
33 . A patient presents with a small fast-growing lesion on the upper extremity
and comments that it has been present for past 2 weeks. The lesion most likely
represents -
A. Squamous cell cancer
B. Keratoacanthoma
C. Basal cell cancer
D. Junctional nevus
Correct answer: B

34. A 75-year-old farmer complained of a scaly, plaque like skin lesion on his
forearm with recent development of ulceration. Biopsy reveals invasive squamous
carcinomas within actinic keratosis negative examination of axillary nodes.
Definitive treatment is:
(A) Local wound care until the ulcer heals; then wide excision and repair
(B) Excision of the lesion with frozen section determined free margins and repair
(C) Wide excision; split-thickness skin graft and axillary node dissection
(D) Wide excision; split-thickness graft and radiation therapy
(E) Wide excision; split-thickness graft and Chemotherapy
Correct answer: B

35.A65-year-old light-complexioned male presents with a solitary scaly plaque like


lesion on his forearm present for many years. The lesion is 0.5 cm in diameter.
Shave biopsy reveals intraepithelial squamous cell carcinoma. (Bowen’s disease)
incompletely excised. Further treatment includes:
(A) Wide excision of the lesions and sentinel node biopsy
(B) Referral for local radiation therapy
(C) Excision and repair of this area, ensuring clear surgical margins
(D) No further treatment indicated
(E) Local application of 5-fluorouracil(5-FU) Cream
Correct answer: C

36- As an example of a malignant ulcer:


a) Tuberculous.
b) Marjoline ulcer.
c) Trophic ulcer.
d) Bed sores (recumbency ulcer)
Correct answer: B

37- The undermined edge of an ulcer, can be defined in:


a) Marjoline ulcer.
b) Neuropathic ulcer.
c) Rodent ulcer.
d) Tuberculous ulcer.
Correct answer: D

38- The rolled in edge of an ulcer, can be defined in:


a) Rodent ulcer.
b) Tuberculous ulcer.
c) Traumatic ulcer.
d) Syphilitic ulcer.

- 47 -
Correct answer: A

39- The punched out edge of an ulcer, can be defined in:


a) Healing ulcer.
b) Trophic ulcer.
c) Traumatic ulcer.
d) Basal cell carcinoma.
Correct answer: C

40- The commonest site of the venous ulcer is:


a) Dorsum of the foot.
b) Toes
c) Gaiter area.
d)metaphysis of bone
Correct answer: C

41- The neuropathic ulcer can appear in:


a) Dorsum of the foot.
b) Middle 1/3 of tibia.
c) Sole of the foot.
d) Face.
Correct answer: B

42- What is the commonest type of ulcers can appear at the neck:
a) Tuberculous ulcer.
b) Rodent ulcer.
c) Basal cell carcinoma.
d) Epithelioma .
Correct answer: A

43- 90% of the rodent ulcers can occur at:


a) Dorsum of the hand.
b) Skull.
c) The area above line between angle of mouth and lobule of the ear and below the
hair line.
d) Popliteal fossa
Correct answer: C

44- The most important predisposing factor of the rodent ulcer is:
a) Continuous friction.
b) Exposure to sunlight.
c) Hairy areas of the body.
d) Drugs.
Correct answer: B

45- The danger of the rodent ulcer is:


a) It is compressing on the surrounding structures.
b) Can interfere with the field of vision.
c) Drained directly to the cavernous sinus and can cause death.
d) Can increase in size.

- 48 -
Correct answer: C

46 . All of the following predispose to squamous cell carcinoma, except -


A. Lichen planus of mouth
B. Bowen's disease
C. Inverted papilloma of nose
D. Chronic irritation of oral mucosa by jagged teeth
Correct answer: A

47- Which of the following is accurate regarding risk factors for skin cancer?
A- Genetic predisposition and family history are more important risk factors for the
development of melanoma than are exposure to sunlight and radiation
B- Cutaneous squamous cell carcinoma (cSCC) related to immunosuppression
typically has a less aggressive course, with a lower rate of local recurrence,
metastasis, and death
C- Ultraviolet A (UV-A) radiation plays a greater role in the development of BCC
than does ultraviolet B (UV-B) radiation
28%
D- The presence of freckling and benign nevi indicate an increased risk for skin
cancer, with the number of nevi more important than the size in terms of melanoma
risk.
Correct answer : D

48- Which of the following is accurate about the types of cutaneous malignant
melanoma?
A- The most common type of head and neck melanoma is nodular.
B- Mucosal melanomas mostly arise in the nasal cavity and are associated with a poor
prognosis and recurrence after treatment.
C- Lentigo maligna (LM) is regarded as the most invasive form of melanoma.
D- Desmoplastic melanomas (DMs) are rarely associated with LM lesions, typically
appear on the extremities, are mildly infiltrative, and rarely recur
Correct answer : B

49- Which of the following is accurate regarding the clinical presentation and
physical examination of patients with nonmelanoma skin cancers?
A- Lip SCC is more common on the upper lip than on the lower lip
B- Nodular BCC is the most common type and usually presents with telangiectasis
C- The cornea is the most common location for conjunctival SCCs
D- Morphea form BCC is typically associated with ulceration, bleeding, and crusting
Correct answer : B

- 49 -
Melanoma

1. A 38-year-old female undergoes removal of a 2 x 1-cm skin lesion shown to be


a melanoma. It is reported as Clark level 1, which implies what?
(A) It is superficial to the basement membrane.
(B) It is 1 mm in thickness.
(C) It has nodal involvement.
(D) It involves the papillary layer.
(E) It involves the reticular dermis.
Correct answer: A

2: The best prognostic plastic factor in malignant melanoma is:


a- Clark's level
b- Age of the patient
c- Duration of the disease
d- Breslow’s staging (tumor thickness)
Correct answer: D

3.The following are(is) correct about incidence of malignant melanoma:


a-The most common type is the superficial spreading one
b- It is more common in males
c- Overall incidence is declining
d- Very rare to occur on top of benign melanoma
Correct answer:A

4. The following type of malignant melanoma has the best prognosis:


a- Superficial spreading type
b- Amelanotic melanoma
c- Acral type
d- Nodular melanoma
Correct answer: A

5. Amelanotic melanoma is differentiated from SCC by:


a- X-ray to detect bone affection
b- LNs biopsy
c- DOPA test
d- Can not be differentiated except after excision
Correct answer: C

6. Surgical excision in malignant melanoma:


a- ls performed only in inoperable case
b- ls advisable whatever the stage and type
c- Can be replaced by irradiation
d- There is no need for surgical excision of LNs
Correct answer: B

- 50 -
7. Risk factors for malignant transformation in naevi include the following
except:
a- Microscopic: junctional type
b- Macroscopic: lentigo
c- Incomplete removal
d- Chronic irritation
Correct answer: B

8. Which statement is untrue concerning malignant melanoma?


a. ls common in children, negroes and Asia
b. Usually occurs between the ages of 50 and 60 years.
c. May arise de novo or in a benign pigmented naevus.
d. Always carries a bad prognosis.
e. May undergo spontaneous regression.
Correct answer: A

9. Malignant melanoma is characterized by the following except:


a. Rarely arises from hair-bearing naevi.
b. Frequently arises from junctional naevi.
c. ls rare in the black races.
d. ls radiosensitive.
e. Carries the worst prognosis when arising in the head, neck or trunk.
Correct answer: D

10. The prognosis of patients with malignant melanoma depends on:


a. Depth of invasion.
b. Clinical stage of the disease.
c. Location of the tumor.
d. All of the above.
e. None of the above.
Correct answer: D

11.Which of the following is NOT a type of melanoma?


a.Superficial
b.Nodular
c.Acral lentiginous
d.Morphea form
Correct answer: D

12.What is the primary treatment of melanoma?


A.Excision
b.Radiation
c.Conservative treatment
d.Liquid nitrogen
Correct answer: A

- 51 -
13.Which of the following is TRUE about melanoma?
a.The most common skin cancer
b.The most common benign skin cancer
c.The most common cause of death from skin cancer
d.The most common skin cancer among Asians
Correct answer: C

14. A 65-year-old male presents to clinic with a small skin lesion (0.5 cm) on his
chest which is very suspicious for melanoma. Which of the following is the most
appropriate option for this patient?
a.Conservative treatment
b.Excisional biopsy with wide margins
c.Immediate chemotherapy and radiotherapy
d.Immediate CT scan
e.None of the above
Correct answer: B

15.You have excised an irregular pigmented lesion from chest of a 50-year-old


woman. The pathology report states that the lesion is melanoma, Clark’s level
III. This means that the lesion:
a.Has invaded into the reticular dermis
b.Has invaded into the subcutaneous fat
c.Has not crossed the basement membrane
d.Has invaded into the papillary-reticular junction
e.Has invaded into papillary dermis
Correct answer: D

16. A man has a suspected melanoma excised from his back. It appears to be less
than 1 mm deep. The smallest margin to plan for excision is:
a.5 cm
b.2 cm
c.1 cm
d.6 cm
e.2 mm
Correct answer: C

17.This subtype of melanoma is the most aggressive, arising in apparently normal


skin or in a nevus, and rapidly becoming a firm, elevated nodule of dense black or
brown-black colour. The nodule may have bluish hues, and in 5% of cases may be
amelanotic (flesh coloured). This subtype, which comprises 15% of all melanoma
cases, can arise at any site in the body. The name of this subtype is:
a.Superficial spreading melanoma
b.Nodular melanoma
c.Lentigo maligna melanoma
d.Acral lentiginous melanoma
Correct answer: B

- 52 -
18. A 40-year-old woman arrives at her family doctor’s office with concern about
a black, bleeding lesion on her left lower leg. She states that it has become enlarged
over the past several months and more irregular in appearance. Which of the
following is FALSE about melanoma?
a.Excision is the primary management
b.Nodular melanoma is the most common
c.Ulceration is a negative prognostic factor
d.Family history is a risk factor
e.The most common site on males is the back
Correct answer: B

19. A tumour descending into the reticular dermis, but not invading the
subcutaneous tissue, would be classified as:
a.Clarks level I
b.Clarks level III
c.Clarks level V
d.Clarks level IV
Correct answer: D

20. Which of the following regarding melanoma is TRUE?


a.Acral lentiginous is the most common subtype
b.Breslow’s depth of invasion is a more reliable indicator of prognosis than Clark’s
levels.
c. A Breslow depth of <0.76 mm carries a metastases rate of 25%
d.Complete excisional biopsy is usually not necessary
Correct answer: B

21.Which nevus would you most likely remove due to its increased malignant
potential?
a.Compound
b.Intradermal
c.Junctional
d.Large pigmented
e.None of the above
Correct answer: D

22 . Least common site for spread of melanoma is ?


A. GIT
B. Lungs
C. Liver
D. Renal
Correct answer: D

23 . Most common malignant melanoma is


A. Superficial spreading
B. Lentigo maligna melanoma
C. Nodular
D. Acral lentiginous
Correct answer: A

- 53 -
24 . Worst prognosis in melanoma is seen in subtype
A. Superficial spreading
B. Nodular melanoma
C. Lentigo maligna melanoma
D. Amelanotic melanoma
Correct answer: B

25 . Most common site of lentigo maligna melanoma is:


A. Face
B. Legs
C. Trunks
D. Soles
Correct answer: A

26 . Most common origin of melanoma is from


A. Junctional melanocytes
B. Epidermal cells
C. Basal cells
D. Follicular cells
Correct answer: A

27 . Biopsy from a mole on the left foot shows cytologic atypia of melanocytes
and diffuse epidermal infiltration by anaplastic cells, which are also present in
the papillary and reticular dermis. The most possible diagnosis is:
A. Dysplastic nevus
B. Melanoma, clark level III
C. Melanoma, clark level IV
D. Congenital melanocytic nevus
Correct answer: C

28 . All of the following statements about malignant melanoma are true except:
A. Acral lentiginous melanoma carrier a good prognosis
B. Prognosis is better in female than in male
C. Most common type is superficial spreading melanoma
D. Stage IIA shows satellite deposits
Correct answer: A

29 . Deep vain thrombosis is seen in all except one -


A. Prolonged immobilization
B. Subungual melanoma
C. Pregnancy
D. Major surgical operation
Correct answer: B

30 . Which one does not cause deep vein Thrombosis -


A. Oestrogen
B. Thrombocytosis
C. Sickle cell anemia
D. Paroxysmal nocturnal hemoglobinuria
Correct answer: C

- 54 -
31 . Deep vein thombosis occur most commonly after -
A. Total hip replacement
B. Gastrectomy
C. Prostatic operation
D. Brain surgery
Correct answer: A

Soft tissue Sarcoma

1 . Which of the following is best indicator of prognosis of soft tissue sarcoma:


A. Tumor size
B. Histological type
C. Nodal metastasis
D. Tumor grade
Correct answer: D

2 . In case of soft tissue sarcoma, prognosis is indicated by -


A. Size of tumour
B. Site of tumour
C. Grade of tumour
D. Vascularity
Correct answer: C

3 . Not true about bone metastasis -


A. Uncommon distal to elbow and knee
B. Breast secondary may be osteoblastic
C. Renal cell carcinoma secondary is expansile
D. Soft tissue sarcoma causes bony metastasis
Correct answer: D

4 . Lymphnode metastasis is a common feature with the following variant of soft


tissue sarcoma:
A. Angiosarcoma
B. Fibrosarcoma
C. Neurofibrosarcoma
D. Liposarcoma
Correct answer: A

5 . In soft tissue sarcoma the prognosis depends best upon


A. Grading of tumour
B. Site of tumour
C. Lymphadenopathy
D. Size of tumour
Correct answer: A

- 55 -
6 . Which one of the following soft tissue sarcomas frequently metastasizes to
lymph nodes?
A. Osteosarcoma.
B. Fibrosarcoma.
C. Alveolar soft part sarcoma.
D. Embryonal Rhabdomyosarcoma.
Correct answer: D

7 . All of the following soft tissue sarcoma have a propensity for lymphatic spread
except -
A. Neurofibrosarcoma
B. Synovial sarcoma
C. Rhabdomyosarcoma
D. Epitheloid sarcoma
Correct answer: A

8. A 12-year old has been referred to you following complete resection with clean
margins of a high-grade malignant peripheral nerve sheath tumor of the shoulder
region. The tumor measured approximately 3.5 cm in greatest dimension. A CT
scan of the chest and a bone scan were within normal limits. The patient does not
have evidence of NF1. Which of the following approaches would you recommend?
A. Chemotherapy with doxorubicin and ifosfamide
B. Radiotherapy
C. Chemotherapy with doxorubicin and ifosfamide plus radiotherapy
D. Observation
Correct answer is D.

9. Primary site represents an important prognostic factor for localized


rhabdomyosarcoma. Which of the following is the most unfavorable primary site?
A. Extremity
B. Prostate
C. Infratemporal fossa
D. Neck
E. Biliary tree
Correct answer is A.

10.A 6-year old with localized orbital rhabdomyosarcoma who underwent a


biopsy. You are considering the role of surgery and radiotherapy for local control.
Which of the following approaches to local control should you recommend?
A. Orbital exenteration before chemotherapy in order to eliminate the need for
radiotherapy
B. Debulking surgery, leaving the eye intact, before chemotherapy in order to reduce
the radiotherapy dose that will be administered beginning at week 12
C. No further surgery, instead begin chemotherapy and use radiotherapy beginning at
week 12 for local control of the tumor
D. No further surgery or radiotherapy, instead use chemotherapy for local control of the
tumor
Correct answer is C.

- 56 -
11.You are taking care of a 4-year old with a Stage 3, Group III embryonal
rhabdomyosarcoma of the bladder. After 12 weeks of therapy with vincristine,
dactinomycin, and cyclophosphamide (VAC), you obtain an MRI and the
radiologist tells you that the tumor has not changed in size. Which of the following
treatment approaches should you recommend?
A. Continue with the current chemotherapy regimen, which also calls for local
radiotherapy to begin shortly
B. Change to an alternate chemotherapy regimen and reassess in another 12 weeks
C. Proceed to extensive surgery followed by an alternate chemotherapy regimen
D. Change to an alternate chemotherapy regimen and initiate local radiotherapy
E. Biopsy the mass and base further decisions on the viability of the tumor
Correct answer is A.

12.You are seeing a 16-year old with a newly diagnosed grade 3 epithelioid
sarcoma of the right forearm. After the biopsy, you obtain an MRI of the forearm
that shows the mass to be > 5 cm in size. Your staging evaluations should include
which of the following?
A. Chest CT, bone scan
B. Chest CT, bone scan, bone marrow aspirate/biopsy
C. Chest CT, bone scan, lymph node biopsy
D. Bone scan, bone marrow aspirate/biopsy
E. Chest CT, lymph node biopsy
Correct answer is C.

13.A mass on the left arm of a 14-year old girl has been biopsied. The pathologist
tells you that the immunostaining for desmin is positive but wants to do an
additional immunostaining to confirm the diagnosis. Which of the following stains
would be most helpful to establish the diagnosis?
A. CD99
B. vimentin
C. synaptophysin
D. keratin
E. myogenin
Correct answer is E.

14. A mass on the left arm of a 14-year-old girl has been biopsied. The
pathologist tells you that the immunostaining for desmin is positive but wants
to do an additional immunostaining to confirm the diagnosis. Which of the
following stains would be most helpful to establish the diagnosis?
A. CD99
B. Vimentin
C. Synaptophysin
D. Keratin
E. Myogenin
Correct answer is E

- 57 -
15. A 3-month-old presents with paraspinal infantile fibrosarcoma. Molecular
studies show the presence of a characteristic chromosomal translocation. Based
on cytogenetic analysis, this tumor seems to share a genetic origin with which
of the following tumors?
A. Mesoblastic nephroma
B. Aggressive fibromatosis (Desmoid tumor)
C. Infantile hemangiopericytoma
D. Inflammatory myofibroblastic tumor
E. Malignant rhabdoid tumor
Correct answer is A

16. You are taking care of a 4-year-old with a stage 3, group III embryonal
rhabdomyosarcoma of the bladder. After 12 weeks of therapy with vincristine,
dactinomycin, and cyclophosphamide (VAC), you obtain an MRI and the
radiologist tells you that the tumor has not changed in size. Which of the
following treatment approaches should you recommend?
A. Continue with the current chemotherapy regimen, which also calls for local
radiotherapy to begin shortly.
B. Change to an alternate chemotherapy regimen and reassess in another 12 weeks.
C. Proceed to extensive surgery followed by an alternate chemotherapy regimen.
D. Change to an alternate chemotherapy regimen and initiate local radiotherapy.
E. Biopsy the mass and base further decisions on the viability of the tumor.
Correct answer is A

17. You are seeing a 16-year-old with a newly diagnosed grade 3 epithelioid
sarcoma of the right forearm. After the biopsy, you obtain an MRI of the
forearm that shows the mass to be >5 cm in size. Staging evaluations ideally
include which of the following?
A. Chest CT scan
B. Chest CT scan, bone marrow aspirate/biopsy
C. Chest CT scan, MRI of the regional lymph node bed
D. Chest CT scan, abdominal CT scan
Correct answer is C

18. You are treating a patient with localized osteosarcoma of the distal femur
with methotrexate, doxorubicin, and cisplatin (MAP) chemotherapy. At week
10 of treatment the patient undergoes complete resection of the tumor.
Pathology demonstrates 40% necrosis. Which of the following represents the
most appropriate further therapy?
A. Ifosfamide and etoposide (IE)
B. MAP + ifosfamide and etoposide (MAPIE)
C. Gemcitabine docetaxel
D. MAP
E. Sorafenib
Correct answer is D

- 58 -
19. You are discussing prognosis with the mother of a patient with stage 3,
group III rhabdomyosarcoma. Which of the following is the most unfavorable
primary site?
A. Extremity
B. Prostate
C. Infratemporal fossa
D. Neck
E. Biliary tree
Correct answer is A

20. A 12-year-old has been referred to you following complete resection with
clean margins of a high-grade malignant peripheral nerve sheath tumor of the
shoulder region. The tumor measured approximately 4 cm in greatest
dimension. A CT scan of the chest and a bone scan were within normal limits.
The patient does not have evidence of NF1. Which of the following treatment
approaches would you recommend?
A. Chemotherapy with doxorubicin and ifosfamide
B. Radiotherapy
C. Chemotherapy with doxorubicin and ifosfamide plus radiotherapy
D. Observation
Correct answer is D

21. A 9-year-old with a Para testicular rhabdomyosarcoma. The primary


tumor was completely resected. No other imaging studies had been done before
the surgery. You now obtain a CT scan that shows enlarged ipsilateral
retroperitoneal nodes that are obviously involved with tumor. No metastatic
disease was detected on bone scan, bone marrow, or chest CT. What is the
stage/group of this patient if you initiate chemotherapy at this point?
A. Stage 1, group I
B. Stage 1, group II
C. Stage 1, group III
D. Stage 3, group II
E. Stage 4, group IV
Correct answer is C

22. A 5-year-old presented with nasal congestion. A CT scan shows a 7-cm


tumor centered in the parapharyngeal region. A biopsy shows alveolar
rhabdomyosarcoma. The most appropriate evaluations to determine the extent
of the tumor are
A. MRI of the neck, chest CT, bone scan, and bone marrow aspirate/biopsy
B. MRI of the head and neck, chest CT, bone marrow aspirate/biopsy, and CSF
cytology
C. MRI of the head and neck, chest CT, bone scan, and bone marrow aspirate/biopsy
D. MRI of the head and neck, chest CT, bone scan, bone marrow aspirate/biopsy, and
CSF cytology
E. MRI of the neck, chest CT, bone scan, bone marrow aspirate/biopsy, and CSF
cytology
Correct answer is D

- 59 -
23. A patient with a 12-cm pelvic mass, a biopsy reveal high-grade
undifferentiated sarcoma.. Which of the following treatment approaches do you
recommend?
A. Upfront chemotherapy followed by either resection and/or radiotherapy, followed
by further chemotherapy
B. Upfront resection to avoid radiotherapy followed by chemotherapy
C. Resection alone
D. Radiotherapy alone
Correct answer is A

24. A 2-year-old girl is receiving chemotherapy for a stage 3, group III


rhabdomyosarcoma of the bladder. After her second course of chemotherapy,
she is admitted to the hospital with right upper quadrant pain, jaundice,
abdominal distension, and weight gain. Which of the following is the most
appropriate next step in the evaluation/treatment of this patient?
A. Echocardiogram
B. Nutrition consult
C. Doppler study of the liver
D. Serum hepatitis screen
E. Urinalysis
Correct answer is C

25. Biopsy of a bone tumor reveals small round blue cells. Immunostains show
that the tumor is CD99 positive. RT PCR did not identify the t(11;22)(q24;q12).
The pathologist asks you about the clinical and radiographic features of the
case. You state that the patient is an 11-year-old Caucasian girl and that the
tumor arises in the diaphysis of the femur and shows an onion-skin pattern on
imaging. You and the pathologist decide to do which of the following?
A. Initiate treatment for small round cell osteosarcoma
B. Repeat the biopsy
C. Do FISH using FLI1 break-apart probes
D. Do RT PCR for the t(1;13)(p36;q14)
E. Do FISH using EWS break-apart probes
Correct answer: E

- 60 -
4- Vascular surgery

Acute and chronic limb ischemia

1. About Claudication which of the following statements are true?


a. Intermittent claudication may be present at rest.
b. Intermittent claudication is commonly relieved by getting out of bed.
c. Intermittent claudication is most commonly felt in the calf.
d. Intermittent claudication distance is usually inconsistent on a day-ta-day basis for a
given patient.
e. Intermittent claudication is thought to be due to nerve compression in the leg muscle
compartments.
Correct answer: C

2. About Investigations of limb ischemia the following statements are true except?
a. Doppler ultrasound works on the basis of a frequency shift when sound waves hit
moving red blood cells.
b. ABPI means ‘ankle brachial pulsatility index.
c. An ABPI of greater than 0.9 is probably normal.
d. Duplex scans are a combination of wave form analysis and B-mode ultrasound.
e. Carotid surgery is often performed on the basis of duplex scans without further
imaging.
Correct answer: B

3. Which of the following drugs have been shown to improve claudication?


a. Beta-blockers
b. Aspirin
c. Oxpentifylline
d. Simvastatin
e. Prostacyclin.
Correct answer: C

4. Gangrene; which of the following is not a cause?


a. Buerger’s disease
b. Infection
c. Intra-arterial drug injection
d. Frostbite
e. Deep vein thrombosis insufficiency.
Correct answer: E

- 61 -
5. About amputation, which of the following is not an indication for major
amputation?
a. Clostridium infection of the lower leg.
b. Severe trauma
c. Neurofibroma
d. Knee flexion contracture
e. Severe rest pain without gangrene.
Correct answer: C

6. In acute ischemia pain is more prominent in:


a. At the site of obstruction
b. Most proximal part of the limb
c. Most peripheral part of the limb
d. All of the above
Correct answer: C

7. Indications of amputation include all the following except:


a. Fixed color changes
b. Absent pulsation
c. Tense calf
d. Bulging anterior leg compartment
Correct answer: B

8. about embolism, all the following are true except:


a. Young age
b. No collaterals
c. Trophic changes
d. The source of emboli may be undetectable
Correct answer: C

9. The most urgent aspect in treatment of arterial embolism is:


a. Digitalis
b. Heparin
c. Morphine
d. Diuretics
Answer: B

10. The action of heparin in arterial embolism is to:


a. Deal with cardiac problem
b. Prevent propagation of thrombosis
c. Prevent further embolization
d. All of the above
Answer: D

11. In a traumatized patient showing signs of fracture and acute ischemia:


a. Urgent repair of the affected vessel
b. Urgent ligation of the affected vessels
c. Reduction of the fracture and waiting for return of pulse is the I st step
d. None of the above
Answer: C

- 62 -
12. Arterial embolism may result from:
a. Atherosclerosis
b. Bone fracture
c. Parasites
d. All of the above
Answer: D

13. The most common cause of fat embolism is:


a. Weight gain
b. Weight loss
c. Bone fracture
d. None of the above
Answer: C

14. The most common site of arterial embolism


a. Common carotid artery
b. Brachial artery
c. Femoral artery
d. Popliteal artery
Answer: C

15. About acute arterial thrombosis all the following are true except:
a. Old age
b. History of chronic ischemia
c. No collateral
d. There may be history of diarrhea
Answer: C

16. Sure signs of arterial injury include all the following except:
a. Signs of ischemia
b. Evidence of adjacent nerve injury
c. Pulsating hematoma
d. Palpable thrill at the site of injury
Answer: B

17. In trauma causing arterial spasm treatment may include:


a. Pointing the artery with papaverine
b. Excision and grafting
c. Dilatation by Fogarty catheter
d. All of the above
Answer: D

18. Management of complete arterial tear may include all the following except
a. Dissection of the artery
b. Cut the minor branches
c. Suturing in transverse suture line
d. Saphenous grafting
Answer: C

- 63 -
19. A 28-year-old woman developed a painful thrombosis of a superficial varix in
the left upper calf 2 days previously. After spending the 2 days in bed with her leg
elevated, she felt better and the tenderness resolved; however, when out of bed she
developed a twinge of right-sided chest pain when walking and a feeling of
heaviness in the calf. Which treatment is most appropriate?
a. Check for leg swelling, tenderness, and Homan's sign, and obtain a Doppler
ultrasound study.
b. Begin antibiotics for a probable secondary bacterial infection.
c. Order emergency venography, and if it is abnormal, begin heparin administration.
d. Begin ambulation and discontinue bed rest that probably caused muscle pain by
hyperextension of the knee.
e. If there is no pain on dorsiflexion of the left foot reassure her, since a negative
Homan's sign precludes the diagnosis of DVT.
Answer: C

20. In a 55-year-old grocery store cashier with an 8-month history of leg edema
increasing over the course of a workday, associated with moderate to severe lower
leg bursting pain, the most appropriate investigative study or studies are:
a. Doppler duplex ultrasound.
b. Brodie-Trendelenburg test.
c. Ascending and descending phlebography.
d. Measurement of ambulatory and resting foot venous pressure.
e. Venous reflux plethysmography.
Answer: A

21. Lytic therapy in pulmonary embolism:


a. Should precede anticoagulation.
b. Can be considered for all patients.
c. Can be considered for hemodynamically unstable patients.
d. Is indicated for the majority of patients with documented pulmonary embolism.
Answer: C

22. The single most important indication for emergency pulmonary embolectomy
is:
a. The likelihood of another episode of embolism.
b. The inability to determine whether the problem is acute pulmonary embolism or
acute myocardial infarction.
c. The presence of persistent and in tractable hypotension.
d. Pulmonary emphysema.
Answer: C

23. In prevention of the fat emboli syndrome the primary therapy can be
accomplished by which of the following?
a. Systemic anticoagulation achieving a partial thromboplastin time greater than 50
seconds.
b. Intravenous administration of alcohol.
c. Prophylactic administration of methyl prednisolone.
d. Maintaining a serum albumin value greater than 3 gm. per 100 ml. in the days
immediately following injury.
Answer: D

- 64 -
24. Significant tachypnea and hypoxia follow development of fat emboli
syndrome, and the goal of ventilatory support should be:
a. Keeping the respiratory rate below 30.
b. Preventing respiratory alkalosis.
c. Reversing pulmonary shunting using positive end-expiratory pressure.
d. Maintaining an adequate total volume.
Answer: C

25. Carotid artery occlusive disease most often produces transient ischemic
attacks or stroke by which of the following mechanisms?
a. Reduction of flow to the affected area of the brain through stenotic or occluded
vessels.
b. Embolization of atheromatous debris and/or clot with occlusion of intracranial
branches of the carotid artery.
c. Thrombosis and propagation of the clot into the intracranial branches.
d. All of the above are equally common.
Answer: B

26. Which of the following does not describe intermittent claudication?


a. Is elicited by reproducible amount of exercise.
b. Abates promptly with rest.
c. Is often worse at night.
d. May be an indication for bypass surgery.
Answer: C

27. In terms of long-term graft patency, the best results in the femoral tibial
bypass position have been achieved with:
a. A modified human umbilical cord graft.
b. Polytetrafluoroethylene (PTFE [Gore-Tex]).
c. Saphenous vein allograft.
d. Segments of greater and lesser saphenous and cephalic veins spliced together.
Answer: D

28. Which of the following statements about femoral popliteal bypass grafting is
true?
a. Long-term graft surveillance by duplex scanning has no effect on graft patency
rates.
b. Graft failure and amputation occur in half the patients within 5 years.
c. If grafting is successful, long-term mortality is improved.
d. Patency rates of 80% to 90% at 1 year currently are expected.
Answer: D

29. Which of the following statements about percutaneous renal artery


transluminal angioplasty (PRTA) are true?
a. Patients with renovascular hypertension are usually cured after successful PRTA.
b. Patients with renovascular hypertension due to atherosclerosis are more likely to
benefit from PRTA than those in whom it is due to fibromuscular dysplasia.
c. PRTA of ostial atherosclerotic lesions is more successful than PRTA of nonostial
lesions.

- 65 -
d. PRTA is associated with a higher morbidity and mortality than angioplasty for
peripheral vascular disease.
Answer: D

30. Which of the following statements are true?


a. All arterial injuries are associated with absence of a palpable pulse.
b. Preoperative arteriography is required to diagnose an arterial injury.
c. The presence of Doppler signals indicates that an arterial in jury has not occurred.
d. Patients with critical limb ischemia have paralysis and paresthesia.
e. In all patients with multiple trauma, arterial in juries should be repaired before other
injuries are addressed.
Answer: D

31. A patient presents with a gunshot wound of the mid-neck. Although drunk, he
exhibits no lateralizing neurologic signs. After control of his airway is achieved, he
is taken directly to the operating room for control of hemorrhage. The common
carotid artery has a 2-cm. destroyed segment. There is also a major esophageal
injury. The best treatment for this carotid injury is:
a. Vein graft replacement of the common carotid artery.
b. Ligation of the common carotid artery.
c. Ligation of the common carotid artery proximally with a subclavian carotid bypass.
d. Ligation of the common carotid artery with sympathectomy.
e. Prosthetic graft replacement of the common carotid artery.
Answer: A

32. A 35-year-old man involved in a motor vehicle accident presents with


a knee dislocation that is easily reduced. Radiography of the knee shows no
fracture. Which of the following statements about this treatment are true?
a. If he has normal pulses he can be discharged.
b. If he has normal pulses, he requires either close observation or arteriography.
c. If he has absent distal pulses and severe ischemia he should undergo arteriography
in the radiology suite.
d. A popliteal vein injury is best treated with ligation.
e. A popliteal artery injury should be repaired with the ipsilateral saphenous vein if
available.
Answer: B

33. A 24-year-old man is involved in an industrial accident in which he sustains


a crushed pelvis. Diagnostic peritoneal lavage is positive. At exploration, a large
pelvic hematoma is found. What is the best treatment?
a. Explore all the major arteries and veins of the pelvis and surgically control the
bleeding if possible.
b. Do not explore the pelvic hematoma. Close the abdomen and apply a MAST suit.
c. Do not explore the pelvic hematoma. Apply a pelvic fixator and send the patient to
radiology for possible embolization of bleeding pelvic vessels.
d. Use sustained hypotensive anesthesia to try to control bleeding.
e. Open the pelvic hematoma and apply laparotomy pads with topical hemostatic
agents.
Answer: C

- 66 -
34. Which of the following statements about acute arterial occlusion today is not
true?
a. Most arterial emboli originate in the heart as a result of underlying cardiac disease.
b. It can be treated under local anesthesia.
c. It is usually due to atherosclerotic disease.
d. Surgical treatment can usually be avoided if the lesion is diagnosed early.
Answer: D

35. Which of the following statements about lytic agents is true?


a. They were first introduced well after the advent of balloon embolectomy.
b. Streptokinase is nonantigenic.
c. Systemic use is the most effective means of delivery.
d. The interval to reperfusion limits their utility in the treatment of advanced ischemia.
Answer: D

36. Which of the following is not an indication for postoperative heparinization ?


a. Suspected venous thrombosis.
b. Risk of embolism following acute myocardial infarction.
c. Advanced ischemia secondary to acute embolic occlusion.
d. Dissolution of residual thrombus after balloon thrombo embolectomy.
Answer: D

37. Which of the following is not true of the embolectomy catheter technique?
a. The balloon should be inflated by the same person who withdraws the catheter.
b. Distal exploration should be carried out in all major branches of the affected
extremity.
c. The balloon is designed to dilate as it traverses areas of luminal narrowing.
d. Removal of adherent thrombus requires alternate catheter-based therapy in addition
to balloon exploration.
Answer: C

38. Which of the following is the least reliable indicator of successful


thrombectomy?
a. Vigorous back-bleeding after removal of thrombotic material.
b. Arteriographically demonstrated patency of all runoff vessels.
c. Normal distal pulses.
d. Return of normal skin color and temperature.
Answer: A

39. A 21-year-old woman presents with digital color changes in response to cold
stimulation. Physical examination and laboratory data, including an autoimmune
disease screen, are normal. She should be advised that:
a. Her condition is characteristic of vasospastic Raynaud's syndrome and, while she
may be at a slightly higher risk for developing a connective tissue disease in the future,
there is no evidence of one currently.
b. Her problem with her fingers will get progressively worse and she will eventually
lose fingers.
c. She has scleroderma, which will manifest itself at a later date.
d. Her problem is “all in her head.”
Answer: A

- 67 -
40. Obstructive Raynaud's syndrome can be differentiated from vasospastic
Raynaud's syndrome by the:
a. Ice water test.
b. Digital hypothermic challenge test.
c. Antinuclear antibody levels.
d. Digital blood pressure measurement.
Answer: D

41. Which of the following statements about chronic mesenteric ischemia due to
atherosclerosis is correct?
a. Postprandial pain in these patients is due to gastric hyperacidity and in most cases is
relieved with H 2 blockers.
b. Men are more often affected than women.
c. Mesenteric endarterectomy is the surgical treatment of choice, since long-term
patency rates are superior to mesenteric bypass.
d. Arteriography is no longer necessary in these cases since noninvasive
diagnosis can be established using duplex ultrasound scanning.
e. Surgical treatment is indicated to prevent intestinal infarction in symptomatic
patients.
Answer: E

42. Which of the following statements about angiotensin II is correct?


a. It is a decapeptide.
b. It is an enzyme found in high concentration in the pulmonary circulation.
c. It is a direct vasoconstrictor and stimulates aldosterone production.
d. It is a vasoconstrictor and inhibits aldosterone secretion.
Answer: C

43. Which of the following statements about atherosclerotic obstruction of the


renal arteries is true?
a. Lesions are usually short.
b. These lesions are usually found in the distal renal arteries, particularly just beyond
branch points.
c. Ostial lesions are best treated with balloon angioplasty.
d. Lesion s of this type are the second most common cause of renal artery stenosis.
Answer: A

44. Which of the following statements about the treatment of renal artery stenosis
is true?
a. Though a significant cause of hypertension, renal artery stenosis seldom results in
loss of renal function.
b. In patients with medically controlled renovascular hypertension there is no need to
consider revascularization.
c. Balloon angioplasty is more effective in patients with atherosclerotic disease as
compared with those with fibromuscular disease.
d. In patients with severe atherosclerosis of the aorta, bypass from the splenic or
hepatic arteries should be considered.
Answer: D

- 68 -
45. Concerning in-vivo regulation of the anticoagulated state by endothelium, the
following is true:
a. Heparin-ATIII inactivates only thrombin
b. Thrombomodulin serves only to bind thrombin
c. Production of von Willebrand factor (VWF) inactivates platelets
d. Endothelial cells can secrete tissue factor
Answer: D

46. Ten years after irradiation of the neck for a tonsillar carcinoma, a 59-
year-old woman is found to have symptomatic carotid artery disease.
Arteriogram shows a 70% irregular stenotic lesion. The following is true:
a. Replacement of the artery should be planned due to radiation induced arterial injury
b. The pathology is most likely to be an inflammatory reaction with endothelial
sloughing and thrombosis
c. If atherosclerotic disease is found, the plaque will be no different than nonirradiated
plaques
d. The patient should be managed medically because of the radiation arterial in jury
Answer: C

47. A 23-year-old woman with fever, myalgia and anorexia presents with
hypertension and a cool, ischemic left arm. Angiography shows multiple stenoses
of the subclavian and renal arteries. The following is true:
a. Coronary angiography is indicated with high likelihood of finding coronary disease
b. Endarterectomy of the lesions would be preferred to transluminal angioplasty
c. The presentation is more suggestive of Behcet’s disease than Takayasu arteritis
d. Preferred management consists of corticosteroids
Answer: D

48. Concerning the fibrinolytic system the following is true:


a. Plasminogen is an a-globulin
b. Fibrin but not fibrinogen is lysed by plasmin
c. The main inhibitor of plasmin is a2-macroglobulin
d. TAP is activated during fibrin bonding to plasminogen
Answer: D

49. An 82-year-old man with a long history of coronary and peripheral


vascular disease presents with an acutely ischemic right lower extremity. The
following is true:
a. The first step in management should be an arteriogram
b. If intractable congestive heart failure is present, non-operative treatment with
heparin would be appropriate
c. If prolonged ischemia has occurred, reperfusion should be accompanied by sodium
bicarbonate
d. Regardless of the period of ischemia, fasciotomy should be based on the findings
postoperative
Answer: B

50. Two days following coronary angiography and angioplasty, a 47-year-old male
diabetic develops painful blue toes on both feet. The following is true:

- 69 -
a. It is very unlikely that there is any connection between the catheterization and the
extremity problem
b. The appropriate treatment is vasodilators and an antiplatelet agent
c. If both superficial femoral arteries are obstructed, the most likely etiology is in-situ
microvascular thrombosis
d. If renal failure or pancreatitis develops, the outlook for long term survival is very
poor
Answer: D

51. A 70-year-old man presents with sudden pain and ischemic changes in
his left leg. An arterial embolus is suspected. The following is true:
a. The most likely source of an arterial embolus is from intracardiac thrombus on a
previous MI
b. If atrial fibrillation (AF) is present, it is known that chronic AF is less likely to
produce embolism than paroxysmal AF
c. Currently, the most common cause of AF is ischemic rather than rheumatic heart
disease
d. Aspirin is more effective than Coumadin in AF for reducing risk of stroke and
cardiovascular mortality
Answer: C

52. Which of the following is appropriate candidates for exercise testing?


a. The patient with symptoms of intermittent claudication but normal resting ankle
brachial indices
b. The patient with rest pain, non-healing ulcers or gangrene
c. If the resting ankle pressure is below 30–40 mmHg
d. The patient with blue toe syndrome and readily palpable pedal pulses
Answer: A

53. Which of the following statements is true regarding the use of duplex
scanning as a means to follow and monitor bypass grafts?
a. Duplex scanning is accurate and cost effective
b. A localized increase in systolic velocity greater than 2 5% compared to adjacent
segments in the graft identifies a diameter reduction of at least 50%
c. Peak systolic velocities should be less than 40 cm/sec throughout the graft
d. Arterial venous fistulas associated with in situ bypass grafts are difficult to detect
with a duplex scanner
Answer: A

54. Which of the following is true with respect to hypertension?


a. There is a threshold effect of blood pressure on the risk of cardiovascular
complications
b. The risk of hypertension is essentially con fined to stroke
c. Common antihypertensive regimens may have adverse effects on a patient’s lipid
profile
d. Aggressive blood pressure reduction in patients with ischemic heart disease may
increase mortality and morbidity
Answer: C

- 70 -
55. Which of the following is true regarding treatment of diabetes?
a. Strict control with insulin but not oral hypoglycemic agents markedly reduces
the incidence of cardiovascular complications in diabetic persons
b. Vascular complications are directly proportion al to the degree of glycemic control
c. The effects of diabetes are most marked in individuals with other risk factors
d. The impact of diabetes and cardiovascular risk is relatively uniform
Answer: C

56. Pressure in superficial veins of the leg during standing:


a. 60 mmHg
b. 70 mmHg
c. 80 mmHg
d. 90 mmHg
Answer: C

57. Superficial thrombophlebitis may complicate:


a. Varicose veins
b. Trauma
c. Abscess
d. All of the above
Answer: D

58. About thrombophlebitis migrancies


a. It resolves spontaneously and reappear in another area
b. It is common with Burger's disease
c. It is common with visceral malignancy
d. All of the above
Answer: D

59. About phlegmasia cerulae dolens all the following are true except:
a. There is massive iliofemoral DVT
b. Severe congestion and cyanosis of lower limb
c. Treated by anticoagulants
d. May lead to venous gangrene
Answer: C
60. Interruption of Inferior Vena cava is not indicated in
A.multiple septic emboli
B.multiple small peripheral emboli
C.when anticoagulation is contraindicated
D.Ileo femoral thrombosis
Correct answer C

61. Graft used in infra inguinal bypass is


A. PTFE
B. Dacron
C. Autologous vein
Correct answer C

- 71 -
62 . Which of the following statements regarding arteriovenous fistulae is true?
A. Blood volume is decreased
B. Cardiac output is decreased
C. Heart rate is decreased
D. Peripheral vascular resistance is decreased
Correct answer: D

63 . All surgery can be done in occlusive arterial disease except:


A. Femoropopliteal bypass with saphenous graft
B. Aortofemoral bypass to avoid iliac lesion
C. Thromboendarterectomy
D. Amputation
Correct answer: D

64 . Lumber sympathectomy is of value in the management of:


A. Distal ischemia affecting the skin of the toes.
B. Intermittent Claudication.
C. Back pain
D. Arteriovenous Fistula
Correct answer: A

65. Following resection of the left colon, a 67- year-old obese woman develops left-
sided leg edema due to deep-vein thrombosis. She is placed on anticoagulants, but
after 2 weeks of warfarin (Coumadin), she develops a pulmonary embolus with
slight hypoxemia. What should the next step in management involve?
(A) Increasing the dose of anticoagulants
(B) Discontinuing anticoagulants
(C) Use of an inferior vena cava (IVC) filter
(D) CT scan of the leg and abdomen
(E) Femoral vein ligation
Correct answer: C

66. Which of the following is not a feature of Thrombo Angitis Obliterans


A.it usually occurs before the age of 50 years
B.lower limbs are affected before upper limbs
C.there is associated phlebitis
D.femoral artery is involved
Correct answer D

67 . Which is not true regarding thoracic inlet syndrome -


A. Most commonly radial nerve
B. Resection of 1st rib is effective the treatment
C. Physiotherapy and position exercises relieves symptom
D. Neurological signs and symptoms are common
Correct answer: A

- 72 -
68 . The most common cause of peripheral limb ischemia is -
A. Trauma
B. Altherosclerosis
C. Buerger's disease
D. Takayasty's disease
Correct answer: B

69. Which of the following statements correctly characterizes the healing of


prosthetic arterial grafts in humans?
a. Complete healing occurs within 3 months of graft implantation.
b. Complete healing occurs within 1 year of graft implantation.
c. Prosthetic grafts do not heal completely in human
d. Polytetrafluoroethylene (PTFE) grafts heal completely whereas Dacron grafts do
not.
e. Dacron grafts heal completely but PTFE grafts do not.
Answer: C

Arterial aneurysm

1. The most common risk associated with carotid artery aneurysm is:
a. Thrombosis of the aneurysm.
b. Embolization of mural thrombus.
c. Rupture of the aneurysm.
d. Compression of the hypopharynx.
Answer: B

2. The cause of subclavian arterial aneurysms is most often:


a. Sepsis.
b. A congenital defect.
c. Atherosclerosis.
d. Fibromuscular dysplasia.
Answer: C

3. Of the visceral aneurysms, which is the most common?


a. Celiac.
b. Superior mesenteric.
c. Hepatic.
d. Splenic.
Answer: D

4. Aneurysms of the renal artery are most common:


a. At its origin from the aorta.
b. In the main renal artery or the bifurcation into the primary branches.
c. Within the kidney.
Answer: B

- 73 -
5. An aortic abdominal aneurysm was first successfully resected by:
a. Matas.
b. Linton.
c. Dubost.
d. None of the above.
Answer: C

6. Evaluation of the natural history of abdominal aortic aneurysms in patients


who are followed without any surgical procedure indicates that:
a. Approximately 20% are alive at the end of 5 years.
b. Seventy-five per cent of patients succumb by the end of the first year.
c. Aortic rupture is quite common in this group, occurring in more than half by the
second year.
d. None of the above.
Answer: A

7. A 70-year-old man has surgery for an abdominal aortic aneurysm. About 1


month later the patient presents with a massive UGI bleeding. Which of the
following statements is TRUE?
(A) He should be given PPLs and observed in the intensive care unit.
(B) Most aortoenteric fistulas are primary.
(C) Most aortoenteric fistulas occur between the aorta and duodenum.
(D) It is not improtant to separate the aorta from the eosphagus after aortic surgery.
(E) This condition is always fatal.
Correct answer: C

8 . Which of the following is the most common cause of Abdominal Aortic


Aneurysm?
A. Syphilis
B. Vasculitis
C. Atherosclerosis
D. Trauma
Correct answer: C

9 . The hunterian Ligature operation is performed for -


A. A V fistuls
B. Acute arterial ischemia
C. Aneurysm
D. Varicose veins
Correct answer: C

10 . A female presented with severe headache of sudden onset. On CT scan a


diagnosis of subarachnoid heamorrhage is made. The most common site of
subarachnoid haemorrhage is:
A. Middle meningeal artery
B. Berry aneurysm rupture
C. Basilar artery
D. Subdural venous sinuses
Correct answer: B

- 74 -
11 . Peripheral aneurysm most commonly located at -
A. Femoral artery
B. Popliteal artery
C. Brachial artery
D. Axillary artery
Correct answer: B

12 . After doing graft repair of a thoracoabdominal aneurysm, the patient


developed weakness in both legs. Most probable cause for this is -
A. Decreased blood supply to the lower limbs
B. Thoracosplanchnic injury
C. Discontinuation of an arteria radicular is magna
D. Lumbosacral nerve injury
Correct answer: C

13 . Most common site of peripheral aneurysm -


A. Radial artery
B. Femoral artery
C. Brachial artery
D. Popliteal artery
Correct answer: D

14 . The most common site of rupture of Abdominal Aortic Aneurysm is


A. Posteriorly into the Posterior Retroperitoneum
B. Anteriorly into the Peritoneum (Intraperitoneal)
C. Laterally into the Left Retroperitoneum
D. Laterally into the Right Retroperitoneum
Correct answer: C

15 . All are common sites of berry aneurysm, except -


A. Posterior cerebral artery
B. Verterbral artery
C. Anterior cerebral artery
D. Middle cerebral artery
Correct answer: B

16 . Mortality rate of emergency Abdominal aortic aneurysm repair -


A. 0.1
B. 0.5
C. >50%
D. 0.2
Correct answer: C

- 75 -
17. A 67-year-old man with acute popliteal arterial embolism has a negative
cardiac echo for source of the thrombus. The following is true:
a. Most likely non -cardiac source is a thoracic aortic aneurysm
b. Embolism is more common from femoral than popliteal arterial aneurysms
c. Emboli from popliteal aneurysms are often clinically silent
d. Embolism is rare from subclavian artery aneurysms
Answer: C

18. The incidence of inflammatory aortic abdominal aneurysms with dense


periaortic adhesions and possible involvement of adjacent structures such as the
duodenum, renal vein, and ureter is approximately:
a. 2%.
b. 10%.
c. 25%.
Answer: B

19. In a patient with an abdominal aortic aneurysm and a history of several


previous abdominal procedures for release of dense peritoneal adhesions causing
episodes of intestinal obstruction, consideration should be given to which one
of the following at operation?
a. Cardiopulmonary bypass.
b. An incision from the xiphoid process to the symphysis pubis.
c. Incision in the left flank with a retroperitoneal approach.
d. An axillobifemoral graft.
Answer: C

20. Which of the following confirms the diagnosis of transection of the descending
thoracic aorta?
a. Widened mediastinum.
b. Fractured first rib.
c. Left pleural effusion.
d. Positive aortogram.
e. All of the above.
Answer: D

21. When complications occur after operating on a descending thoracic aorta,


perhaps the most devastating is:
a. Recurrent nerve injury.
b. Bleeding with hemothorax.
c. Paraplegia.
d. Renal insufficiency.
Answer: C

22. The appropriate treatment in most situations of an aortic abdominal graft that
has become infected is:
a. Intravenous antibiotics and observation for future complications.
b. Catheter drainage at the site of infection.
c. Replacement of the infected graft with another prosthetic graft.
d. Excision of the entire graft and insertion of ax illobifemoral grafts.
Answer: D

- 76 -
Varicose veins and chronic venous insufficiency

1. Which of the following regarding varicose veins are true?


a. Varicose veins may cause venous claudication.
b. Varicose veins may be associated with the fOXC2 gene.
c. The prevalence of varicose veins is twice as common in women than in men.
d. Right leg varicose veins are more common than left leg varicose veins.
e. Handheld Doppler provides an accurate assessment of saphenopopliteal competence.
Answer: B

2. A 55-year-old woman gives a history of tiredness, aching, and a feeling of


heaviness in the left lower leg for the past 3 months. These symptoms are relieved
by leg elevation. She is also awakened frequently by calf and foot cramping, which
is relieved by leg elevation, walking, or massage. On physical examination there
are superficial varicosities, non pitting edema, and a slightly painful, 2 cm.
diameter superficial ulcer 5 cm. above and behind the left medial malleolus. What
is the most appropriate diagnosis?
a. Isolated symptomatic varicose veins.
b. Superficial lymphatic obstruction.
c. Deep venous insufficiency.
d. Arterial insufficiency.
e. Incompetent perforating veins.
Answer: C

3. The best treatment plan for the patient described in the preceding question
should include:
a. Varicose vein ligation and stripping as soon as possible.
b. Ulcer debridement, vein stripping, and skin grafting.
c. Ligation of the medial perforating veins.
d. Transposition of saphenous vein valve.
e. Leg elevation, external support, and ambulation without standing.
Answer: E

4- Venous ulcer is a complication of:


a) Varicose veins
b) Acute ischemia.
c) T.B.
d) Diabetes
Correct answer: A

5 . Most commonly varicose veins are seen with:


A. Long saphenous vein
B. Short saphenous vein
C. Both
D. Popliteal and femoral vein
Correct answer: A

- 77 -
6 . Surgery in varicose veins in NOT attempted in presence of -
A. Deep vein thrombosis
B. Multiple incompetent perforators
C. Varicose veins with leg ulcer
D. All of the above
Correct answer: A

7 . Trendeleberg's operation is done for ?


A. Deep vain thrombosis
B. Varicose veins
C. Burger's disease
D. Obturator hernia
Correct answer: B

8 . Which of the following is first treatment of rupture of varicose veins at the


ankle?
A. Application of a tourniquet distally
B. Direct Pressure and Elevation
C. Rest in prone position of patient
D. Application of a tourniquet proximally
Correct answer: B

9 . A pt presented with pulsating varicose veins of the lower limb. Most probable
diagnosis is:
A. Klippel Trenaunay syndrome
B. Tricuspid regurgitation
C. DVT
D. Right ventricular failure
Correct answer: A

10 . Regarding varicose veins, which one of the following statements is true:


A. Over 20% are recurrent varicosities
B. The sural nerve is in danger during stripping of the long saphenous vein
C. The saphenous nerve is closely associated with the short saphenous vein
D. 5% oily phenol is an appropriate sclerosant for venous sclerotherapy
Correct answer: D

11 . An elderly patient is admitted with ruptured varicose vein with profuse


bleeding. Initial treatment given is:
A. Keeping the patient in sitting position
B. Pressure at the bleeding site
C. One tourniquet application proximal to ruptured site
D. Two tourniquet applications both proximal and distal to ruptured site
Correct answer: B

- 78 -
12 . A 55yrs male presents with varicose vein with saphena-femoral incompetence
and normal perforator management is:
A. Saphena-femoral flush ligation
B. saphenofemoral flush ligation with striping
C. endovascular striping
D. Sclera-therapy
Correct answer: B

13. Which of the following test is used to detect perforator incompetence in


varicose veins
A. Trendelenberg test
B. Fegan's test
C. Morissey's test
D. Homan's test
Correct answer B

DVT, Postphlebitic leg, Leg ulcers

1 . Which of the following is the best way of preventing development of deep vein
thrombosis(DVT) in post operative period ?
A. Physiotherapy
B. Early ambulation
C. Low dose aspirin
D. Prophylactic heparin
Correct answer: D

2 . Varicose vein surgery is contraindicated in-


A. Varicose ulcers
B. Deep vein Thrombosis
C. Pigmentation Over limb
D. Hemorrhoides
Correct answer: B

3 . A patient with critical lower limb ischemia presents with


A. Intermittent Claudication and gangrene
B. Intermittent Claudication.
C. Intermittent claudication and ischemic ulcers
D. Rest pain and ischemic ulcers
Correct answer: D

4 . Sympathectomy is indicated in all following conditions except:


A. Intermittent claudication
B. Ischemic ulcers
C. Acrocyanosis
D. Anhidrosis
Correct answer: A

- 79 -
5. In patients who develop a documented episode of deep venous thrombosis (DVT)
the most significant long-term sequala is:
a. Claudication.
b. Recurrent foot infections.
c. Development of stasis ulcer.
d. Pulmonary embolization.
e. Diminished arterial perfusion.
Answer: C

6. The most dangerous DVT which requires the longest period of treatment:
a. Calf vein DVT
b. Femoral vein DVT
c. Ileofemoral DVT
d. All of the above
Answer: C

7 . Treatment of choice for deep venous thrombosis is


A. Intravenous heparin
B. Fibrinolytic agents
C. Thrombectomy
D. Coumarin drug
Correct answer: A

8 . Deep venous thrombosis is best diagnosed by


A. Duplex ultrasonography
B. Plethysmography
C. Radiolabeled fibrinogen uptake
D. Doppler ultrasonography
Correct answer: A

Lymphedema, Differential diagnosis of swollen limbs

1. Lymph nodes are located in all of the following areas EXCEPT:


a) Armpit
b) Abdomen
c) Groin
d) Spleen
Correct answer : D

2. Primary lymphedema results from:


a) Congenital deformity
b) Breast cancer
c) Melanoma
d) Sarcoma
Correct answer : A

- 80 -
3. Second stage of lymphedema includes:
a) Soft tissue
b) Reversible swelling
c) Can be infected
d) 3-5 cm differences between limbs
Correct answer : D

4. __________ is the inability to pick up a fold skin on top of a digit:


a) Perometry
b) Gold standard
c) Stemmer’s sign
d) Puffiness
Correct answer : C

5. The disadvantage of the circumference test for lymph is:


a) Expensive
b) Time consuming
c) Measure cross sectional area
d) Pinching the skin
Correct answer : B

6. Which of the following NOT measures the limb volume?


a) Bioelectrical impedance
b) Water displacement
c) Perometry
d) Stemmer’s
Correct answer : D

7. The goals of combined decongestant therapy include all the following


EXCEPT:
a) Increase ROM
b) Increase edema volume
c) Decrease pain
d) Decrease limb size
Correct answer : B

8. When you examine a male patient with lymphedema, you discover pitting
edema, and it’s reversible with elevation. From your point of view, which stage of
lymph is?
a) Stage I
b) Stage II
c) Stage III
d) Stage IV
Correct answer : A

9. The principles of exercise program include all the following EXCEPT:


a) Don’t exercise with pain
b) Don’t exercise with short breath
c) Don’t exercise on wheel object
d) Don’t exercise slowly

- 81 -
Correct answer : D

10.One of lymphedema precautions is:


a) Avoid wearing compression garment
b) Avoid excessive heat
c) Avoid antibiotic cream on wound
d) Avoid extremely low exercise
Correct answer : D

11.Which of the following is NOT related to pressure garments?


a) Achieved by compression bandage
b) Maintain limb size
c) Prevent increase swelling
d) Correct muscle imbalance
Correct answer : D

12.Deep abdominal breathing exercises before and after exercise is effective for:
a) Stimulate lymph flow
b) Improve ROM
c) Increase lymph volume
d) Prevent infection
Correct answer : A

13. Grade 1 lymphedema means


A. pitting edema up to the ankle
B. pitting edema up to the knee
C. non-pitting edema
D. edema disappearing after overnight rest
Correct answer D

14. Milroys disease is lymphedema which


A. Follows filariasis
B. Is familial
C. Follows erysipelas
D. Is due to malignant disease
Correct answer: B

15 . Antibiotic of choice for lymphedema is ?


A. Penicillin
B. Amikacin
C. Metronidazole
D. Ceftazidine
Correct answer: A

16. Which of the following statements about upper extremity edema is true?
a. Lymphedema is more common than venous edema.
b. Signs and symptoms of venous obstruction include edema, distention of superficial
veins, tightness, aching, cyanosis and pain.
c. Distal venous obstructions are more likely than proximal venous obstructions to
cause symptoms in the upper extremity.

- 82 -
d. All patients with symptomatic upper extremity venous thrombosis should receive
fibrinolytic therapy.
Answer: B

17. Cancer en cuirasse is:


a- Skin nodule.
b- Lymphedema of breast skin.
c- Both.
d- None of the above.
Correct answer : C

18. The most frequent cause of primary lymphedema is:


A. A deficiency of transporting lymphatic channels.
B. Valvular incompetence in lymphatic channels.
C. Obstruction or removal of regional lymph nodes.
D. Thrombosis of lymphatic channels.
Correct answer: A

19. Most patients with lymphedema can be managed by:


A. Pedicle transfer of lymphatic bearing tissue into the affected area.
B. Elevation, elastic support garments, and massage therapy or mechanical pneumatic
compression.
C. Lymphatic bypass using an autogenous vein graft.
D. Excision of hypertrophic scarred fibrotic skin and subcutaneous tissue down
to muscle fascia and coverage with split-thickness skin grafts.
Correct answer: B

20. Which statements about lymphangiomas are true?


A. Most lesions appear during puberty.
B. These lesions frequently respond to small doses of radiation therapy.
C. The lesions usually grow slowly but may infiltrate local tissues.
D. Malignant transformation is frequent.
Correct answer: C

21 . Which of the following is not a risk factor for lymphoedema?


a. Limb surgery (e.g. varicose vein operation)
b. Obesity
c. Family history
d. A Baker’s cyst
e. Air travel.
Answer: D

22. Which of the following statements regarding lymphoedema are true?


a. Primary lymphoedema occurs in more than 5 per cent of the population.
b. Early treatment is usually successful.
c. Early treatment includes surgical drainage.
d. Fluid is relatively low in protein in lymphoedema.
e. Lymphoedema often involves the muscle compartments.
Answer: B

- 83 -
5- Head and neck

Thyroid swelling including cancer

1. All are true about deQuervain's thyroiditis except


A.Usually follows a viral infection
B.There is increased radio iodine uptake
C.Initial hyperthyroid state
D.Anti thyroids are of no use
Correct answer B

2 . Which of the following would be the best treatment for a 2 cm thyroid module
in a 45yrs man with FNAC revealing it to be a papillary CA?
A. Total thyroidectomy with left sided modified neck dissection
B. Hemithyroidectomy
C. Hemithyroidectomy with modified neck dissection
D. Near total thyroidectomy with radiotherapy
Correct answer: A

3. Abductor of the vocal cord is


A.lateral cricoarytenoid
B.cricothyroid
C.posterior cricoarytenoid
Correct answer C

4. Reversible dementia is a feature of


A.Alzheimer's disease
B.Pick's disease
C.Multi infarct dementia
D.Hypothyroidism
Correct answer D

5 . In Postoperative room after thyroid surgery Patient developed sudden


respiratory distress, dressing was removed, and it was found to be slightly blood
stained and wound was bulging. What will first thing to be done.
A. Tracheostomy
B. Cricothyroidotomy
C. Laryngoscopy and intubation
D. Remove the stitch and take the Patient to O.T.
Correct answer: D

- 84 -
6 . Which of the following is the commonest tumour of thyroid -
A. Anaplastic carcinoma
B. Follicular carcinoma
C. Papillary carcinoma
D. Medullary carcinoma
Correct answer: C

7 . A 32yrs pt presents with B/L proptosis, heat intolerance and palpitations;


most unlikely diagnosis would be-
A. Diffuse thyroid goitre
B. Reidel's thyroiditis
C. Hoshimoto's thyroiditis
D. Thyroid adenoma
Correct answer: B

8 . In an accident there was rupture of the pituitary stalk. This would lead to all
of the following except:
A. Hyperprolactinemia
B. Diabetes Insipidus
C. Hypothyroidism
D. Diabetes Mellitus
Correct answer: D

9 . A 38 year woman had recurrent episodes of headache and sweating. Her mother
had renal calculi and died of thyroid cancer. Physical observations revealed a
thyroid nodule and ipsilateral enlarged cervical lymph nodes. Surgeon should
order the woman before performing thyroid surgery:
A. Estimation of hydroxy indole acetic acid in urine
B. Thyroid scan
C. Estimation of TSH, and TRH levels in serum
D. Estimation of urinary metanephrines, VMA and catecholamines
Correct answer: D

10 . Which of the following would be the best treatment for a 2 cm thyroid nodule
in a 50 year old man with FNAC revealing it to be a papillary carcinoma?
A. Subtotal thyroidectomy with modified neck dissection
B. Hemithyroidectomy
C. Hemithyroidectomy with modified neck dissection
D. Near total thyroidectomy with modified neck dissection
Correct answer: D

11 . Recurrent laryngeal nerve is in close association with:


A. Superior thyroid artery
B. Inferior thyroid artery
C. Middle thyroid vein
D. Superior thyroid vein
Correct answer: B

- 85 -
12 . In prolactinoma most common symptom other than galactorrhea is ?
A. Bitemporal hemianopia
B. Amenorrhea
C. Thyroid dysfunction
D. Headache
Correct answer: B

13 . A 10 years boy presented with cervical lymphadenopathy. Needle biopsy


from the nodes revealed secondaries from papillary ca of thyroid. The child
underwent complete removal of tumor near total thyroidectomy and radical
neck dissection. The immediate next line of management is:
A. Bone scan to evaluate secondaries
B. CECT scan to assess any residual disease
C. Start thyroxine suppression therapy
D. Iodine 131 Whole body scan to assess extent of disease
Correct answer: C

14 . A 20 year old girl with 9 months history of neck and swelling with thyrotoxic
symptoms.On investigation increased T4 and decreased TSH with palpable 2 cm
nodule was found.Next investigation will be -
A. Thyroid scan
B. USG
C. CT scan
D. Radioactive iodine uptake
Correct answer: A

15 . All of the following are true about Hashimoto's thyroiditis, except-


A. Increase in lymphocytes
B. Follicular destruction
C. Oncocytic metaplasia
D. Oncocytic Annie eye nuclei
Correct answer: D

16 . In case of parathyroid nodule adenoma The treatment is -


A. Calcitonin and steroid
B. Removal of adenoma
C. Total parotidectomy and implantation in arm
D. Total parotidectomy
Correct answer: B

17 . In post operative ward a patient developed respiratory distress due to wound


hematoma after thyroidectomy. The treatment of choice is:
A. Insertion of an orotracheal tube
B. Determination of serum Ca conc.
C. Immediate opening of the wound
D. Administration of O2 by nasal canula
Correct answer: C

- 86 -
18 . Parathyroid adenoma most commonly involves which of the following sites:
A. Thyroid substance
B. Superior parathyroid lobe
C. Inferior parathyroid lobe
D. In the mediastinum
Correct answer: C

19 . Toxic nodular goitre in an elderly patient-the treatment of choice is-


A. Anti-thyroid drugs
B. Rdioiodine
C. Surgery
D. Lugol's iodine
Correct answer: C

20 . Most common solitary thyroid nodule is -


A. Follicular adenoma
B. Hurthle cell carcinoma
C. Papillary carcinoma
D. Solitary idiopathic goiter
Correct answer: A

21 . In Medullary Carcinoma thyroid Tumour marker is -


A. TSH
B. Calcitonin
C. T3, T4 and TSH
D. Alpha Feto protein
Correct answer: B
Correct answer: D

22 . Hypoparathyroidism following thyroid surgery commonly occurs within:


A. 24 hours
B. 2-5 days
C. 7-10 days
D. 2-3 weeks
Correct answer: B

23 . A 27 years lady presents with a palpable thyroid nodule, and needle biopsy
demonstrates amyloid in the stroma of the lesion. A cervical lymph node is
palpable on the same side as the lesion. The best treatment should be
A. Removal of the involved lobe, the isthmus, a portion of the opposite lobe, and he
enlarged lymph node
B. Removal of the involved node, the isthmus, and the enlarged lymph node.
C. Total thyroidectomy and irradiation of the cervical lymph nodes
D. Total thyroidectomy and modified neck dissection on the side of the enlarged
lymph node
Correct answer: D

- 87 -
24 . Cervical lymphadenopathy is seen most commonly in which variety of thyroid
carcinoma?
A. Papillary carcinoma
B. Follicular carcinoma
C. Anaplastic carcinoma
D. Medullary carcinoma
Correct answer: A

25 . Blow out carotid is characteristically seen with -


A. Thyroidectomy
B. Radical neek dissection
C. Flap necrosis
D. Cistrunk operation
Correct answer: B

26 . Not an embryogenic abnormality of thyroid gland:


A. Reidel's lobe
B. Substernal goiter
C. Thyroglossal cyst
D. Lingual thyroid
Correct answer: A

27 . A 40 year old lady after subtotal thyroidectomy developed stridor in post op


recovery room. There was some staining of the wound with blood. Least likely
cause is:
A. Tracheomalacia
B. Hematoma formation
C. Hypocalcemia
D. Bilateral recurrent nerve palsy
Correct answer: C

28 . In case of hypothyroidism which investigation is most informative and most


common used -
A. Serum TSH level
B. Serum Calcitonin assay
C. Serum T3, T4 level
D. Serum TRH assay
Correct answer: A

29 . Psammoma bodies seen in all of the following, except-


A. Serous cystadenocarcinoma of ovary
B. Meningioma
C. Follicular CA of thyroid
D. Papillary CA of thyroid
Correct answer: C

- 88 -
30 . MEN-II is seen with the following type of thyroid carcinoma:
A. Medullary
B. Papillary
C. Follicular
D. Anaplastic
Correct answer: A

31 . Thyroid carcinoma with pulsatile vascular skeletal metastasis is


A. Papillary
B. Follicular
C. Medullary
D. Anaplastic
Correct answer: B

32 . A 70 years woman presents with post myocardial infraction after 6 weeks with
mild CHF. There was past H/O neck surgery for parathyroid adenoma 5 years
ago. ECG shows slow atrial fibrillation. Serum Ca+2 13.0mg/L and urinary
Ca+2 is 300mg/24h.On examination their is small mass in the para tracheal
position behind the right clavicle. Appropriate management is-
A. Observation and repeat serum Ca+2 in two months
B. Ultrasound - guided alcohol injection of the mass
C. Repeat neck surgery
D. Treatment with technetium-99
Correct answer: B

33 . Neoadjuvant chemotherapy is not used in -


A. CA thyroid
B. CA breast
C. CA Oesophagus
D. CA lung
Correct answer: A

34 . A case of solitary thyroid nodule, investigation of choice is -


A. T3, T4 estimation
B. Thyroid scan
C. FNAC
D. Excision biopsy
Correct answer: C

35 . In chronic lymphocytic thyroiditis -


A. Surgery is indicated in a majority of cases
B. Thyroid antibody estimated is useful in a few cases
C. FNAC is the most appropriate investigation
D. There is no role for steroid therapy in the management
Correct answer: C

- 89 -
36 . Treatment of Medullary carcinoma thyroid -
A. Radiotherapy and Chemotherapy
B. Surgery and Radiotherapy
C. Radioiodine ablation
D. Surgery only
Correct answer: D

37 . A patient presents with swelling in the neck following a thyroidectomy;Most


likely complication is:
A. Hypovolumia
B. Hypocalcemia
C. Resp. obstruction
D. Recurrent laryngeal nerve palsy
Correct answer: C

38 . All can commonly occur in a patient who suffered decelerating injury in


which pituitary stalk was damaged. Except one:
A. Diabetes mellitus
B. Thyroid insufficiency
C. Adrenocortical insufficiency
D. Diabetes insipidus
Correct answer: A

39 . Treatment of choice for solitary thyroid nodule is:


A. Removal of the nodule
B. Hemithyroidectomy
C. Subtotal thyroidectomy
D. Total thyroidectomy
Correct answer: B

40 . Least malignant thyroid cancer is:


A. Papillary carcinoma
B. Follicular carcinoma
C. Medullary carcinoma
D. Anaplastic carcinoma
Correct answer: A

41 . RET proto oncogene mutation is a hallmark of which of the following


tumors ?
A. Paraganglionoma
B. Hurthle cell tumor thyroid
C. Medullary carcinoma thyroid
D. Astrocytoma
Correct answer: C

42 . Recurrent laryngeal nerve supplies all muscles of larynx except -


A. Stylopharyngeus
B. Cricothyroid
C. Intrinsic muscles of larynx
D. Extrinsic muscle of larynx

- 90 -
Correct answer: B

43 . Following are true about Hashimoto's thyroiditis, except-


A. Oncocytic metaplasia
B. Orphan Annie eye nuclei
C. Follicular destruction
D. Increase in lymphocytes
Correct answer: B

44 . All the following are true for carcinoma of thyroid except -


A. Lymph node involvement is rare
B. Vascular invasion is common
C. Younger patients have good prognosis
D. Diagnosis by fine needle aspiration
Correct answer: A

45 . Features to differentiate parathyroid adenoma from hyperplasia would


include which of the following:
A. Presence of excess chief cells
B. High levels of parathormone
C. Infiltration of capsule
D. Identifying hyperplasia of all 4 glands at surgery in parathyroid hyperplasia
Correct answer: D

46 . FNAC is useful in all the following types of thyroid carcinoma except


A. Papillary
B. Follicular
C. Anaplastic
D. Medullary
Correct answer: B

47 . Which of the following may be associated with Pancreatitis,pituitary tumor


and phaeochromocytoma ?
A. Anaplastic carcinoma of the thyroid
B. Follicular carcinoma of the thyroid
C. Medullary carcinoma of the thyroid
D. Papillary carcinoma of the thyroid
Correct answer: C

48 . The treatment of choice for Anaplastic carcinoma of thyroid infiltrating


trachea and sternum will be -
A. Radical excision
B. Chemotherapy
C. Radiotherapy
D. Palliative / Symptomatic treatment
Correct answer: D

- 91 -
49 . For solitary thyroid nodule. The treatment of choice is -
A. Radical thyroidectomy
B. Hemithyroidectomy
C. Total thyroidectomy
D. subtotal thyroidectomy
Correct answer: B

50 . A patient operated for thyroid surgery for a thyroid swelling later in the
evening developed difficulty in breathing. There was swelling in the neck. The
immediate management would be:
A. Epinephrine injection
B. Tracheostomy
C. IV calcium glucanate
D. Open the wound sutures in the ward
Correct answer: D

51 . Most common surgically repairable cause of hyperthyroidism ?


A. Adenoma
B. Carcinoma
C. Hyperplasia
D. Renal disease
Correct answer: A

52. A well differentiated follicular carcinoma of thyroid can be best


differentiated from a follicular adenoma by:
A. Lining of tall columnar and cuboidal cells
B. Hurthle cell change
C. Nuclear features
D. Vascular invasion
Correct answer: D

53 . Most common histological type of the thyroid carcinoma is:


A. Follicular type
B. Medullary type
C. Anaplastic type
D. Papillary type
Correct answer: D

54 . A 25 year male presents with ophthalmologic sings of Thyrotoxicosis. All are


possibilities, except:
A. Hashimoto's Thyroiditis
B. Diffused Thyroid goitre
C. Adenomatous Goitre
D. Riedel's Thyroditis
Correct answer: D

55 . All of the following are true about Lymphoma of the thyroid except:
A. More common in females
B. Slow growing
C. Clinically confused with undifferentiated tumors

- 92 -
D. May present with respiratory distress and dysphagia
Correct answer: B

56 . Which of the following is not a histological variant of thyroid neoplasm?


A. Follicular
B. Merkel cell
C. Insular
D. Anaplastic
Correct answer: B

57 . The most appropriate operation for a solitary nodule in one lobe of thyroid is
A. Hemithyroidectomy
B. Lobectomy
C. Partial lobectectomy with 1 cm margin around nodule
D. Nodule removal
Correct answer: A

58 . Patient presents with neck swelling and respiratory distress few hours after a
thyroidectomy .Next management would be:
A. Wait and watch
B. Oxygen by mask
C. Open immediately
D. Tracheostomy
Correct answer: C

59 . A well differentiated follicular carcinoma of thyroid can be best


differentiated from a follicular adenoma by:
A. Lining of tall columnar and cuboidal cells
B. Hurthle cell change
C. Nuclear features
D. Vascular invasion
Correct answer: D

60 . A 50 year old male is suffering from severe dyspnea after thyroid surgery-
the treatment of choice is -
A. Tracheostomy
B. Open in operative site
C. Wait and watch
D. Cricothyroidotomy
Correct answer: B

61 . A patient on the same evening following thyroidectomy presents with a


swelling in the neck and difficult breathing. The next management is:
A. Wait and watch
B. Administer oxygen by mask
C. Open immediately
D. Intubate oro-tracheally
Correct answer: C

- 93 -
62 . Which of the bellow is true?
A. Excess calcium intake can lead to hyperthyroidism
B. Goitre more than 5% of population is endemic goitre
C. Colloid goitre mostly presents as hyperthyroidism
D. Thyroid storm, the clinical features are primarily due to increased thyroxine
Correct answer: B

63 . A Post-Thyroidectomy patient develops signs and symptoms of Tetany. The


management is:
A. Calcitonin
B. Vitamin D
C. I.V. Calcium gluconate
D. Bicarbonate
Correct answer: C

64 . Most common cause of solitary thyroid nodule is -


A. Papillary carcinoma
B. Adenomatous goiter
C. Follicular goiter
D. Papillary adenoma
Correct answer: B

65 . Serum marker used after surgery for differentiated thyroid cancer is -


A. T4
B. Thyroglobulin
C. Calcitonin
D. PTH
Correct answer: B

66 . A patient presented with headache and flushing. He has a family history of


his relative having died of a thyroid tumour. The investigation that would be
required for this patient would be:
A. Chest X-ray
B. Measurement of 5 HIAA
C. Measurement of catecholamine
D. Intravenous pyelography
Correct answer: C

67 . A 45 year old patient presents with thyroid swelling and lymph node in neck,
on aspiration amyloid material was found, management is:
A. Total thyroidectomy with neck dissection
B. Hemithyroidectomy
C. Hemithyroidectomy with neck dissection
D. Total thyroidectomy with neck irradiation
Correct answer: A

- 94 -
68 . In a patient presenting with a swelling of the thyroid, the radionuclide scan
showed a cold nodule and the ultrasound showed a non cystic solid mass. The
management of this patient would be:
A. Lobectomy
B. Hemithyroidectomy
C. Eltroxin
D. Radio Iodine therapy
Correct answer: B

69 . Which of the following is not used in the management of thyroid storm ?


A. Potassium iodide
B. Reserpine
C. Propranolol
D. Calcium channel blockers
Correct answer: B

70 . After thyroidectomy, patient developed stridor within 2 hrs. All are likely
cause of stridor except:
A. Hypocalcemia
B. Recurrent laryngeal nerve palsy
C. Larynogomalacia
D. Wound hematoma
Correct answer: A

71 . Investigation of choice to differentiate between multinodular goitre and


thyroid tumor:
A. FNAC
B. CT scan
C. USG
D. X-ray
Correct answer: A

72 . Bone Metastasis is common in which thyroid tumor:


A. Follicular
B. Papillary
C. Hurthle cell tumour
D. Anaplastic
Correct answer: C

73 . Thyroid nodule of 4 cm size, mobile but causing compressive symptoms, all


are true except -
A. FNAC is investigation of choice
B. FNAC can not distinguish follicular adenoma from carcinoma
C. Managed by Subtotal thyroidectomy
D. Cold nodules are diagnosed of malignancy
Correct answer: D

- 95 -
74 . Which thyroid malignancy can present with diarrhea?
A. Papillary carcinoma
B. Follicular carcinoma
C. Medullary carcinoma
D. Hurthle cell carcinoma
Correct answer: C

75 . Investigation of choice in case of a patient with episodic hypertension,


headache and thyroid nodule -
A. Urinary HIAA
B. Urinary catecholamine and aspiration of module
C. Thyroid function test only
D. Urinary basic amino acid metabolite
Correct answer: B

76 . Patient after thyroid surgery presents with perioral paresthesia. Serum Ca


level is 7 mg/dl. What will be the best m/n (management):
A. Oral vit D3
B. Oral vit D3 with Ca
C. I.V.Ca. gluconate
D. Oral calcium
Correct answer: D

77 . Hashimoto's disease is -
A. An autoimmune thyroiditis
B. A granulomatous thyroiditis
C. Focal thyroiditis
D. A parathyroid tumor
Correct answer: A

78 . Most dangerous complication in a patient who had undergone thyroid


surgery and develop hematoma at the operative site:
A. Respiratory obstruction
B. Recurrent laryngeal nerve palsy
C. Dysphagia
D. Shock
Correct answer: A

79 . In which of the following is medullary thyroid cancer in most aggressive


from?
A. MEN type I
B. MEN type II a
C. MEN type II b
D. Sporadic cases
Correct answer: C

- 96 -
80 . A young patient has a midline, tender swelling in neck occurring after an
attack of score throat. The diagnosis is -
A. Acute thyroiditis
B. Thyroglossal cyst
C. Subacute thyroiditis
D. Toxic goiter
Correct answer: A

81 . Complications of Hemithyroidectomy include all of the following Except?


A. Recurrent laryngeal nerve palsy
B. External branch of superior laryngeal nerve palsy
C. Hypocalcemia
D. Wound hematoma
Correct answer: C

82 .During thyroidectomy the vessel first ligated is:


A. Superior thyroid vein
B. Middle thyroid vein
C. Inferior thyroid vein
D. Anterior jugular vein
Correct answer: B

83 .'Hurthle cells' are seen in


A. Agranulomatous Thyroiditis
B. Hashimoto's Thyroiditis
C. Papillary carcinoma of the thyroid
D. Thyroglossal cyst
Correct answer: B

84 . Commonest cause of hyperparathyroidism is -


A. Single adenoma
B. Multiple adenoma
C. Single gland hyperplasia
D. Multiple gland hyperplasia
Correct answer: A

85 .Medullary carcinoma thyroid arises from -


A. Parafollicular cells
B. Cells lining the acini
C. Capsule of thyroid
D. Stroma of the gland
Correct answer: A

86 .Which type of thyroid carcinoma has the best prognosis


A. Papillary Ca
B. Anaplastic Ca
C. Follicular Ca
D. Medullary Ca
Correct answer: A

- 97 -
87 . A patient presents with respiratory distress and neck swelling few hours
after thyroid surgery. The next immediate management would be:
A. Open stiches immediately
B. Wait and watch
C. Tracheostomy
D. Oxygen by mask
Correct answer: A

88 . A well differentiated follicular carcinoma of thyroid can be best


differentiated from a follicular adenoma by which of the following:
A. Lining of tall columnar and cuboidal cells
B. Hurthle cell change
C. Nuclear features
D. Vascular invasion
Correct answer: D

89 . Amyloid stroma is seen in which Ca thyroid


A. Papillary Ca
B. Medullary Ca
C. Anaplastic Ca
D. Follicular Ca
Correct answer: B

90 . Lateral aberrant thyroid refers to


A. Congenital thyroid abnormality
B. Matestatic foci from primary in thyroid
C. Struma ovarii
D. Lingual thyroid
Correct answer: B

91 . A 52 year female patient presents with symptoms of pheochromocytoma. She


also has a thyroid carcinoma. Her thyroid Ca is of which type:
A. Anaplastic
B. Medullary
C. Follicular
D. Papillary
Correct answer: B

92 . Treatment for parathyroid hyperplasia is:


A. Removal of all four glands
B. Calcitonin
C. Removal of 3.5 glands
D. Enlarged glands to be removed
Correct answer: C

93 . Screening method for medullary carcinoma thyroid is:


A. S.alkaline phosphate
B. S. acid phosphatase
C. Serum calcitonin
D. S.calcium

- 98 -
Correct answer: C

94 . All the following are true of DeQuervan's Thyroiditis except


A. Pain
B. Increased ESR
C. Increased radioactive iodine uptake
D. Fever
Correct answer: C

95 . Hypoparathyroidism develops after thyroid surgery.


A. 1day
B. 2-5 days
C. 7 days
D. 1 month
Correct answer: B

96 . Serum marker used after surgery for differentiated thyroid cancer is -


A. T4
B. Thyroglobulin
C. Calcitonin
D. PTH
Correct answer: B

97 . Occult thyroid malignancy with nodal metastasis is seen -


A. Medullary carcinoma
B. Follicular Ca
C. Papillary Ca
D. Anaplastic Ca
Correct answer: C

98 . Which of the following is the treatment of Medullary Carcinoma thyroid ?


A. Radiotherapy and Chemotherapy
B. Surgery and radiotherapy
C. Radioiodine ablation
D. Surgery only
Correct answer: D

99 . Which of the following will not lead to respiratory distress after Thyroid
Surgery
A. Laryngomalacia
B. Bilateral recurrent laryngeal nerve injury
C. Hypocalcemia
D. None of the above
Correct answer: D

- 99 -
100 . For a 21 years lady, needle biopsy of solitary thyroid nodule with palpable
cervical lymph nodes on the same sides demonstrates amyloid in stroma of
lesion. Possible diagnosis is
A. Thyroid adenoma
B. Multi nodular goitre
C. Medullary carcinoma thyroid
D. Follicular carcinoma thyroid
Correct answer: C

101 . In patients with papillary carcinoma of thyroid, the following factors are
associated with poor prognosis EXCEPT -
A. Age more than 40 years
B. Size more than 3 cms
C. Female gender
D. Tall cell variant
Correct answer: C

102 . Most common subtype of thyroid cancer is ?


A. Medyllary carcinoma
B. Papillary carcinoma
C. Follicular carcinoma
D. Anaplastic carcinoma
Correct answer: B

103. As regard embryology of thyroid:


a) Develops from I st branchial Arch.
b) Develops from 4th branchial Arch.
c) Both A And Above.
d) Develops from 2nd branchial pouch
e) Non of the above
Correct answer: C

104 . A patient undergoes thyroid surgery, following which he develops perioral


tingling. Blood Ca is 8.9 meq; Next step of management is:
A. Oral Ca and vit D
B. Vit D orally
C. Wait for Ca to decrease to <7.0 before taking further action
D. Intravenous calcium gluconate and serial monitoring
Correct answer: D

105 . Carcinoma thyroid with blood borne metastasis is -


A. Follicular
B. Papillary
C. Mixed
D. Medullary
Correct answer: A

- 100 -
106 . One of the following is not correct in papillary carcinoma of thyroid -
A. Always unifocal
B. Can be reliably diagnosed using fine needle aspiration cytology
C. requires a total thyroidectomy for large tumours
D. Typically spreads to the cervical lymph nodes
Correct answer: A

107 . A 26 years old woman presents with a palpable thyroid nodule, and needle
biopsy demonstrates amyloid in the stroma of lesion. A cervical lymph node is
palpable on the involved side as the lesion, the preferred treatment should be
A. Removal of the enlarged node, the isthmus, and the enlarged lymph node.
B. Removal of the involved lobe, the isthmus portion of the opposite lobe, and the
enlarged lymph node.
C. Total thyroidectomy and modified neck dissection on the side of the enlarged
lymph node.
D. Total thyroidectomy and irradiation of the cervical lymph nodes.
Correct answer: C

108 . Lymph node metastasis is least commonly seen with


A. Papillary Ca Thyroid
B. Medullary Ca Thyroid
C. Follicular Ca Thyroid
D. Anaplastic Ca Thyroid
Correct answer: C

109 . The most common presentation of endemic goiter is


A. Hypothyroid
B. Diffuse goiter
C. Hyperthyroid
D. Solitary nodule
Correct answer: B

110 . Most common cause of Thyroiditis is:


A. Subacute Thyroiditis
B. Reidl's Thyroiditis
C. Viral Thyroiditis
D. Hashimoto's Thyroiditis
Correct answer: D

111 . Treatment of choice for medullary carcinoma of thyroid to -


A. Total thyroidectomy
B. Partial thyroidectomy
C. I131 ablation
D. Hemithyroidectomy
Correct answer: A

112 . Associated with Thyroid storm, except:


A. Stressful illness in thyrotoxicosis
B. I131 therapy for thyrotoxicosis
C. Surgery for thyroiditis

- 101 -
D. Surgery for thyrotoxicosis
Correct answer: C

113 . All the following are characteristics of hypoparathyroidism occurring after


accidental removal of parathyroid during thyroid surgery EXCEPT -
A. Decreased serum phosphate levels
B. Carpopedal spasm
C. Seizures
D. Stridor
Correct answer: A

114 . A patient has pituitary tumour and pheochromocytoma and a thyroid


module. Which Ca is most likely to occur:
A. Follicular Ca
B. Medullary Ca
C. Papillary Ca
D. Anaplastic Ca
Correct answer: B

115 . Complications of total thyroidectomy include all except -


A. Hoarseness
B. Airway obstruction
C. Hemorrhage
D. Hypercalcaemia
Correct answer: D

116 . A 45 year old male presents with 4 x 4 cm, mobile right solitary thyroid
nodule of 5 months duration. The patient is euthyroid. The following statements
about his management are true except -
A. Cold nodule on thyroid scan is diagnostic of malignancy
B. FNAC is the investigation of choice
C. The patient should undergo hemithyroidectomy if FNAC report is inconclusive
D. Indirect laryngoscopy should be done in the preoperative period to asses mobility
of vocal cords
Correct answer: A

117. Reversible dementia is a feature of


A.Alzheimer's disease
B.Pick's disease
C.Multiinfarct dementia
D.Hypothyroidism
Correct answer D

118.C-cells of thyroid gland:


a) Develop from ultimobranchial body.
b) Produce calcitonin.
c) Are the origin of medullary carcinoma.
d) All of the above.
Correct answer: D

- 102 -
119. As regard to T3:
a) Less concentration thon circulating T4.
b) More potent thon T4.
c) Mostly bound to TBG.
d) All of the above.
Correct answer: D

120.The correct sequence of events for the metabolism of iodine and synthesis
thyroid hormone is: E
a) Trapping, organification, coupling, release,oxidation.
b) Oxidation, trapping, coupling, organification, release.
c) Coupling, organification, trapping, oxidation, release.
d) Trapping, coupling, oxidation. release, organification.
e) Trapping., oxidation, organification, coupling. release
Correct answer:

121.The daily requirement of iodine is:


a) 50-20 pg.
b) 80-90 ug.
c) 100-125 prg.
d) 125-l65 ug.
e) None of the above
Correct answer: C

122.Most diagnostic single investigation for toxic adenoma is:


a) T3-T4.
b) US.
c) Thyroid scan.
d) FNABC.
Correct answer: C

123. the following statements regarding TSH measuring are true except:
a) lt is increased after total thyroidectomy.
b) Normal TSH is about 5 micro units/liter.
c) lt is the most sensitive test for mild cases.
d) All of the above
Correct answer: B

124. Worm nodule meCorrect answer


a) Inactive nodule.
b) Usually cancerous.
c) Active nodule.
d) Toxic adenoma.
Correct answer: C

125.AS regards FNABC all cancel except:


a) Outpatient procedure.
b) Cheap and safe.
c) Requires general anesthesia.
d) Can not differentiate follicular adenoma from carcinoma.

- 103 -
Correct answer: C

126.Presentations of ectopic thyroid include:


a) Dysarthria.
b) Midline neck swelling.
c) Myxedema if removed by mistake.
d) Any of the above.
Correct answer: D

127.The most common site of thyroglossal cyst is:


a) Subhyoid.
b) Suprahyoid.
c) Sublingual.
d) At thyroid cartilage.
Correct answer: A

128. The following ore true regarding thyroglossal cyst EXCEPT:


a) Usually presents as midline neck swelling.
b) Moy be confused with ectopic thyroid.
c) Best treatment is follow up.
d) Should be excised completely for fear of complications.
Correct answer: C

129. Preparation of retrosternal goiter for surgery include:


a) Neomercazole.
b) Lugol's iodine.
c) Propranolol.
d) A ond C.
Correct answer: C

130. Sporadic goiter may occur due to the following EXCEPT:


a) Cabbage.
b) Water pollution by excreta.
c) iodine deficiency.
d) Percolates.
Correct answer: C

131. Autoimmune manifestations of Grove's disease include the following


EXCEPT:
a) Palmar erythema.
b) Clubbing.
c) Pretibial myxedema.
d) Exophthalmos.
Correct answer: A

132.All of the following ore recognized complications of neomercazole EXCEPT:


a) Goiter.
b) Agranulocytosis.
c) Renal failure.
d) Hepatotoxicity.

- 104 -
Correct answer: C

133.Thyrotoxicosis in children all correct except:


a) Usually goes into spontaneous remission.
b) Medical treatment alone con control the disease.
c) Radioactive iodine is the ideal treatment.
d) Thyroidectomy should be near total to avoid recurrence.
Correct answer: C

134. Which of the following treatment schedule for diffuse toxic goiter is True:
a) Over 45 years: radioactive iodine.
b) Under 45 years: with large goiter: surgery.
c) Under 45 years: with small goiter: antithyroid drugs.
d) None of the above.
e) All of the above.
Correct answer: E

135.Toxic goiter hos the following signs except:


a) Flapping tremors of the hand
b) Exophthalmos
c) Diarrhea
d) Menstrual irregularities
Correct answer: A

136. 3 hours post-thyroidectomy, 30 years old woman developed agitation and


difficulty breathing, tachycardia and dry dressing but inferior cervical swelling.
The most appropriate immediate step is :
a) Insertion of an oro-tracheal tube
b) Reopening of the cervical wound
c) Estimation of serum calcium level
d) IV morphine
Correct answer: B

137. On the 1 St postoperative day after total thyroidectomy, patient complains


of tingling of the finger ,lips and the serum calcium level of
5.5m9/d L. The next step should be:
a) Observation only
b) Administration of vitamin D2 or D3 50000-100000 units/ day
c) Administration of vitamin D3 1-2pg/day
d) Administration of calcium gluconate 3-5g /day, by slow IV drip
Correct answer: D

138. Hashimoto disease may present by:


a) Thyrotoxicosis.
b) Myxedema.
c) Goiter.
d) Any of the above.
Correct answer: D

- 105 -
139. The following type of thyroiditis mimics malignancy:
a) Hashimoto thyroiditis.
b) Riedle thyroiditis.
c) De Queryoin's thyroiditis.
d) All of the above.
Correct answer: B

140. the most common cause of goitrous hypothyroidism in adults is:


a) Graves' disease.
b) Riedel’s thyroiditis.
c) Hoshimoto's disease.
d) De Queryoin's thyroiditis.
Correct answer: C

141. Clinical picture of thyroid carcinoma include:


a) Dyspnea and dysphagia.
b) Referred otology.
c) Horsiness.
d) Lateral aberrant thyroid.
e) All of the above
Correct answer: E

142. A familial form of medullary thyroid carcinoma (MTC) should be


suspected whenever:
a) The tumor is multifocal.
b) The tumor is bilateral (foci of tumor ore present in both thyroid lobes.
c) Pathologic examination of the resected thyroid gland reveals the presence of C-
cell hyperplasia in areas of the gland adjacent to foci of MTC.
d) All of the above.
Correct answer: D

143. A thyroid nodule could be malignant if it shows:


a) Rapid growth
b) Pain referred to the ear
c) Hardness
d) Associated hoarseness of voice
e) All of the above
Correct answer: E

144.Which of the following statements regarding follicular adenoma is True:


a) It presents clinically as solitary nodule.
b) Distinction between follicular adenoma and carcinoma can only be Made by
histological examination.
c) In adenoma there is no invasion of the capsule or of pericapsular blood vessels.
d) Preferable treatment is lobectomy
e) All of the above.
Correct answer: E

- 106 -
145. In thyroid carcinoma, mediastinal node involvement is o feature of which
type:
a) Follicular.
b) Anaplastic.
c) Papillary.
d) Medullary.
Correct answer: D

146.The term lateral aberrant thyroid implies


a) congenital aberrant thyroid tissue lateral to the thyroid
b) metastasis in cervical lymph node from on occult thyroid carcinoma
c) a metastasis from carcinoma of the larynx
d) a type of bronchial cyst
e) that a loose piece of thyroid has become implanted in a thyroidectomy scar
Correct answer: B

147.Hoarseness of voice denote :


a) Compression of the superior laryngeal nerve
b) Infiltration of the recurrent laryngeal nerve
c) Infiltration of the superior laryngeal nerve
d) Tracheal compression
Correct answer: B

148. All ore causes of hypercalcemia, Except:


a) Metastatic cancer
b) Sarcoidosis
c) Multiple myeloma
d) Vitamin D intoxication
e) Medullary carcinoma of the thyroid
Correct answer: E

149. A 17 year old girl presented with o 2.5 cm nodule in the right lobe of the
thyroid gland and enlarged three cervical LNs confirmed by US. FNA cytology
revealed malignant cells with vesicular nuclei, the most probable diagnosis is:
a) Lymphoma
b) b. Anaplastic carcinoma
c) Follicular carcinoma
d) Papillary carcinoma
e) Medullary carcinoma
Correct answer: D

150. A 3A-year old female presents for evolution of a palpable thyroid nodule
Tc99 scan demonstrated a single cold nodule it may be the following EXCEPT:
a) Carcinoma
b) Non-functioning adenoma
c) Thyroid cyst
d) Colloid nodule
e) Autonomous nodule
Correct answer: E

- 107 -
151. The most frequent variety of thyroid cancer is:
a) Follicular carcinoma
b) Papillary carcinoma
c) Anaplastic carcinoma
d) Medullary carcinoma
Correct answer: B

152. Thyroid disease treatment :


a) Lymphoma ----( irradiation and chemotherapy)
b) Follicular adenoma ----( lobectomy)
c) Follicular carcinoma---- (total thyroidectomy and radioiodine)
d) Follicular carcinoma----(by histopothological surprise completion thyroidectomy
and radioiodine)
e) All of the above
Correct answer: E

153. Thyroid carcinoma during pregnancy, appropriate treatment is:


a) Rodioiodine l3l
b) Chemotherapy
c) Surgery
d) Wait for delivery
Correct answer: C

154. Recurrent goiter may be due to:


a) Inadequate initial removal.
b) Persistence of etiology.
c) Foreign body reaction.
d) Any of the above
Correct answer: D

155. Excision of o thyroglosssal cyst should include removal of


a) Thyroid isthmus.
b) Pyramidal lobe.
c) Body of the hyoid bone.
d) Foramen cecum.
e) Remnants of the thyroglossal duct.
Correct answer: C

156. ln physiological goiter the following statements are true except that it:
a) Affects males more often thon females.
b) Presents as fullness of the neck (Venus neck).
c) ls characterized by uniform smooth enlargement with fleshy or firm consistency.
d) Moy be associated with toxic or pressure symptoms.
e) Usually resolves spontaneously.
Correct answer: D

- 108 -
157. The best routine management for multinodular goitre is by:
a) Hemithyroidectomy.
b) Partial thyroidectomy.
c) Bilateral wedge resection.
d) Subtotal thyroidectomy.
e) Thyroxin administration.
Correct answer: D

158. Among the following statements about retrosternal goiter, the false one is
that it:
a) Usually arises in aberrant intra-thoracic thyroid tissue.
b) is particularly common in males.
c) Moy present with symptoms of mediastinal compression (syndrome).
d) is often associated with palpable enlargement of the thyroid.
e) is best removed through a cervical incision.
Correct answer: A

159.A middle-aged female presented with on asymptomatic nodule in the right


lobe of the thyroid. She gave a history of irradiation in childhood. The nodule
was cold on radioactive iodine scanning and the sonogram reveled that it was a
solid mass. The appropriate management of this case is:
a) Aspiration biopsy.
b) Treatment with thyroxin.
c) Right lobectomy.
d) Subtotal thyroidectomy.
e) Total thyroidectomy.
Correct answer: C

160. voice fatigue after thyroidectomy is due to injury to which of the following
nerves ? :
a) Superior laryngeal.
b) External laryngeal.
c) internal laryngeal.
d) Recurrent laryngeal.
e) Vagus.
Correct answer: B

161. Medical treatment of thyrotoxicosis is least useful in:


a) Cases with true exophthalmos.
b) Pregnant females.
c) Secondary thyrotoxicosis.
d) Uncomplicated thyrocardic potients.
e) Post-operative recurrence.
Correct answer: C

162.Thyrotoxicosis during pregnancy is best treated by:


a) Subtotal thyroidectomy.
b) Corbimazole.
c) Beta blockers.
d) Lugol's iodine.

- 109 -
e) Rodioiodine.
Correct answer: B

163. the following statements about treatment of thyrotoxicosis by radioactive


iodine are fine except that:
a) is contraindicated in patients below the age of 40.
b) is particularly useful in elderly and thyrocardiac patients.
c) Produces its beneficial effects within o few days.
d) Moy be followed by myxedema.
e) Carriers the risk of occurrence of thyroid cancer.
Correct answer: C

164. Hypothyroidism is most often due to :


a) Multinodulor goitre.
b) Solitary adenoma,
c) Thyroid. cancer.
d) Chronic thyroiditis.
e) iotrogenic courses.
Correct answer: E

165. True statements about papillary carcinoma of the thyroid do not include
that it :
a) Often affects adolescents.
b) ls a slow-growing tumor.
c) May be hormone-dependent.
d) Metastasize early by the blood stream.
e) ls rodioresistant.
Correct answer: D

166. ln subacute thyroiditis (de Quervoin's diseose), it is untrue that it:


a) ls a virus infection related to influenza or mumps.
b) Has a sudden onset with fever and painful swelling of the gland.
c) Moy cause radiating pain in the ear.
d) Never resolves spontaneously.
e) Responds well to prednisone.
Correct answer: D

167. in hoshimoto's disease (lymphodenoid goitre), the false statement that it :


a) Usually affects menopausal women.
b) is characterized by uniform smooth enlargement of the gland with hard
consistency.
c) May be associated with hypothyroidism.
d) Has a characteristic histological picture.
e) Is best treated by radiotherapy.
Correct answer: E

168. The most frequent cause of primary hyperparathyroidism is:


a) Parathyroid adenoma.
b) idiopathic parathyroid hyperplasia.
c) Primary parathyroid carcinoma.

- 110 -
d) Familial hyperparathyroidism.
e) Ectopic production of parathormone.
Correct answer: A

169. Hypertension in a patient with a family history of medullary thyroid


carcinoma is most often due To:
a) Renal artery stenosis.
b) Glomerulonephritis.
c) Cushing's syndrome.
d) Hyperparathyroidism.
e) All of the above.
Correct answer: E

170- All the following are True about the thyroglossal cyst EXCEPT:
a) Painless swelling.
b) Cystic in consistency.
c) Moves up and Down with swallowing but not with protrusion of the tongue.
d) If infected, can form a thyroglossal fistula.
Correct answer: C

171- Regarding the Thyroiditis, all are true EXCEPT:


a) Inflammatory conditions of the thyroid gland.
b) It's rare.
c) Painless.
d) Dramatic response to prednisone and anti-inflammatories.
Correct answer: C

172- the simple Goiter has the following featuring EXCEPT:


a) More common on females.
b) Presented by diffuse, soft enlargement of the gland in cases if diffuse simple goiter.
c) Shows toxic manifestations.
d) Can be treated surgically.
Correct answer: C

173- The simple nodular goiter can be complicated by one of the following
EXCEPT:
a) 2ry thyrotoxicosis.
b) Haemorrhage.
c) Turn malignant in 30% of cases
d) Stridor.
Correct answer: D

174- Retrosternal goiter has the following features EXCEPT:


a) Dullness on direct sterna percussion.
b) Not an indication for surgery.
c) Diagnosis by X-ray.
d) More common in male.
Correct answer: B

- 111 -
175- All are features of Grave's disease EXCEPT:
a) Affects mainly elderly males more than females.
b) Shows clinical picture of hyperthyroidism.
c) Thyroid profile has a "low" TSH value.
d) Can be accompanied by exophthalmos.
Correct answer: A

176- Exophthalmos can be detected by all of the following EXCEPT:


a) Elevated free T3 and T4.
b) Clinically by starring look and loss of forehead wrinkling on looking upwards.
c) Exophthalometer.
d) Autoimmune survey to detect the TSA (Thyroid Stimulating Antibodies)
Correct answer: A

177- As differentiation between primary and 2ry toxic goiter all are true
EXECPT:
a) The 2ry is more common in elderly.
b) The 2ry appears with gradual onset.
c) the 2ry is presented by diffuse enlargement of the gland.
d) Nervous manifestation, metabolic disorders and eye signs are more prominent in
the 1ry form.
Correct answer:C

178- All are indications of surgery in Goiter EXECPT:


a) Retrosternal goiter.
b) 2ry toxic goiter.
c) Thyroiditis.
d) Failure of medical treatment
Correct answer: C

179- Pre-operative preparation for the patient include all of the following
EXECPT:
a) CBC, coagulation profile, liver and kidney profile.
b) CXR and indirect laryngoscopy.
c) Thyroid profile and neck ultrasound.
d) Recurrent laryngeal nerve conduction.
Correct answer: D

180- Regarding the recurrent laryngeal nerve, all are true EXECPT:
a) If injured bilaterally, completely, the patient will be aphonic.
b) Stridor will occur in unilateral complete injury.
c) Can be injured during devascularization of the inferior lobe.
d) Can be non-recurrent only on the right side.
Correct answer:B

181- Most common type of the thyroid malignancy is:


a) Papillary carcinoma.
b) Anaplastic carcinoma.
c) Follicular carcinoma.
d) Metastatic deposits.

- 112 -
Correct answer: A

182-Anaplastic carcinoma, has the following characters:


a) More common in elderly, bad prognosis, early spread and aggressive.
b) Main way of spread is blood.
c) Can occur in children.
d) characterized pathologically by Pasmmoma Bodies.
Correct answer:A

183- The following is not one of the characters of the malignant thyroid cyst:
a) Its aspirate is haemorrhagic.
b) Cytology shows malignant cells.
c) Rapid re-accumulation after aspiration.
d) The cyst will be completely disappeared after aspiration.
Correct answer: D

184- Malignant goiter, if painful the pain will be referred to the ear through:
a) Phrenic nerve.
b) Sympathetic plexus.
c) Bronchial plexus.
d) Arnold's nerve ( branch of vagus nerve)
Correct answer:D

185- The following is a cause of painful goiter:


a) Acute thyroiditis.
b) Simple nodular goiter.
c) Retrosternal goiter.
d) Goiter with pregnancy.
Correct answer: A

186- In a case of goiter, the following may be cause of un-equal pulse during
examination:
a) Diffuse goiter.
b) Cancer thyroid.
c) Retrosternal goiter.
d) Hashimoto's disease.
Correct answer:C

187- Hard goiter can be felt during examination of the gland in the following
cases:
a) Malignancy.
b) Calcified simple nodular goiter.
c) Riedle's thyroiditis.
d) Tense thyroid cyst.
Correct answer: A

188-All of the following are causes of dullness over the manubrium stain
EXECPT:
a) Ectopic thyroid.
b) Pneumothorax.

- 113 -
c) Pre-tracheal lymph nodes.
d) Retrosternal goiter.
Correct answer: B

Salivary glands, Anatomy, Sialadenitis, Parotid tumors

1. Pain in salivary calculi is referred to the ear through:


a. The 5th nerve.
b. The 9th nerve.
c. The 10th nerve.
d. 2nd and 3rd cervical nerve.
Correct answer: A

2. The following salivary gland secretes viscid secretion rich in Ca:


a. Parotid.
b. Sublingual.
c. Accessory glands.
d. All of the above.
Correct answer : B

3. Serous salivary secretion is produced by:


a. Parotid gland.
b. Submandibular gland.
c. Sublingual gland
d. None of the above.
Correct answer : A

4. Parotid gland is the most common salivary gland affected by the following
pathologies except:
a. Acute bacterial sialadenitis.
b. Salivary stones. ,
c. Pleomorphic adenoma.
d. Adenolymphoma (Warthin'stumor)
e. Mumps.
Correct answer : B

5. Operoble carcinoma of parotid is best treated by:


a. Radiotherapy alone.
b. Chemotherapy alone.
c. Superficial conservative parotidectomy.
d. Radical parotidectomy with block dissection of LNs.
Correct answer : D

6. Which statement about salivary calculi is untrue?


a. Are commonest in the parotid gland.
b. May lie in the gland or its duct.
c. Produce recurrent pain and swelling during meals.
d. Contain a high proportion of calcium.

- 114 -
e. May require excision of the affected gland.
Correct answer : A

7. Concerning pleomorphic adenoma (mixed salivary tumor), which of following


statements is incorrect:
a. is the commonest salivary tumor.
b. Has a very heterogeneous histological structure.
c. is well-encapsulated.
d. Presents as a slow-growing firm swelling just below the lobule of the ear.
e. Tends to recur after inadequate excision.
Correct answer : C

8. Which of the following structures is not an anatomical relation to the


submandibular salivary gland?
a. The anterior facial vein
b. The facial artery
c. The inferior alveolar nerve
d. The lingual nerve
e. The hypoglossal nerve.
Correct answer : C

9. Which structure marks the posterior boundary of the submandibular duct which
can safely be accessed via an intra oral approach?
a. The third molar tooth
b.The body of the submandibular duct
c. The lingual nerve
d. The posterior edge of the mylohyoid
e. The marginal mandibular nerve.
Correct answer : C

10. Which structure attaches the deep lobe of the submandibular gland to the lingual
nerve?
a. The hypoglossal nerve
b.The submandibular ganglion
c. The deep cervical fascia
d. The tendon of digastric
e.The mylohyoid.
Correct answer : B

11. Which of the following is not a complication of submandibular gland excision?


a. Frey’s syndrome
b. Anaesthesia of the ipsilateral tongue
c. Weakness of the corner of the mouth
d. Anaesthesia of submental skin
e. Paralysis of the ipsilateral tongue.
Correct answer : A

12. What percentage of submandibular tumors is malignant?


a. 20 per cent
b. 30 per cent

- 115 -
c. 40 per cent
d. 50 per cent
e. 60 per cent.
Correct answer : D

13. Which of the following is not a feature of salivary malignancy?


a. Facial nerve weakness
b. Rapid enlargement
c. Induration of the overlying skin
d. Cervical node enlargement
e. Rubbery consistency.
Correct answer : E
14. Which is the most appropriate form of biopsy for a major salivary gland tumour?
a. Open surgical biopsy to allow histology
b. Salivary washings
c. Frozen section during formal excision
d. Fine-needle aspiration cytology (FNAC)
e. Biopsy is contraindicated.
Correct answer : D

15. Which of the following structures does not lie in the parotid gland?
a. The facial nerve
b. Terminal branches of the external carotid
c. The glossopharyngeal nerve
d. The retromandibular vein
e. Lymph nodes.
Correct answer : C

16. What is the most common site for a parotid tumour?


a. At the anterior border of the masseter
b. Inferior to the angle of the mandible
c. As a parapharyngeal mass
d. Anterior to the ear
e. Behind the angle of the mandible.
Correct answer : E

17. Which nerve must be transected as part of a superficial parotidectomy?


a. The facial nerve
b. The hypoglossal nerve
c. The greater auricular nerve
d. The accessory nerve
e. The auriculotemporal nerve.
Correct answer :C

18. Which of the following landmarks is used to locate the facial nerve trunk?
a. The insertion of sternomastoid
b. The greater horn of the hyoid
c. The superior-most portion of the cartilaginous ear canal
d. The insertion of digastric
e. The insertion of masseter.

- 116 -
Correct answer : D

19. Which of the following is not a branch of the facial nerve?


a. Temporal
b. Oribtal
c. Zygomatic
d. Buccal
e. Cervical.
Correct answer : B

20. Which of the following branches of the facial nerve can be divided without the
need for immediate cable graft repair?
a. Temporal
b. Oribtal
c. Zygomatic
d. Buccal
e. Cervical.
Correct answer : D

21. What is Frey’s syndrome following parotidectomy?


a. Gustatory sweating
b. Dry mouth due to reduction in salivary flow
c. Development of a sialocele over the parotid bed
d. Cosmetic deformity due to loss of parotid bulk
e. Hyperplasia of the contralateral parotid gland.
Correct answer : A

22. With which malignancy is Sjögren’s syndrome associated?


a. Acinic cell carcinoma
b. Adenoid cystic carcinoma
c. Carcinoma ex pleomorphic adenoma
d. Salivary sarcoma
e. Lymphoma.
Correct answer : E

23. The commonest discrete lump of the parotid gland is:


a.Pleomorphic adenoma
b.Mucoepidermoid carcinoma
c. Mumps
d.Parotid gland stone
e.Tuberculosis of the parotid lymph node
Correct answer : A

24. About submandibular salivary stones, all the following statements are true, except:
a.The majority are radia-opaque
b.A stone can be detected by CT scan
c.A stone may present by a submandibular swelling that increases in size by eating
d.A submandibular salivary swelling can be rolled over the angle of the mandible
e.A stone in the gland is treated by submandibular sialadenectomy
Correct answer : D

- 117 -
25. About pleomorphic adenoma of the parotid gland, all the following statements are
true, except:
a.It has epithelial and mesenchymal components
b.The tumour is painless
c.It has an incomplete capsule
d.The tumour is usually present deep to the facial nerve
e. It may turn malignant
Correct answer : D

26. About adenolymphoma, all the following statements are true, except:
a.The tumour is related to smoking
b.It may be bilateral
c.It has epithelial and lymphoid components
e.The tumour is cystic
e.Facial nerve palsy is characteristic of this tumour
Correct answer : E

27. All the following are possible complications of parotidectomy, except:


a.Loss of sensation on one side of the face
b.Inability to close the eyelids
c.salivary fistula
d.sweating on the side of the when eating(gustatory sweating)
e.Immobility of one angle of the mouth when patient smiles
Correct answer : A

28. Salivary gland stones are commonest in………..glands:


a) Submandibular.
b) Parotid.
c) Sublingual.
d) Minor.
Correct answer : A

29. All of the following is correct about salivary glands’ stones except….:
a) Neck US may be a useful investigation.
b) 90% of them are radiopaque.
c) In bidigital examination, they are better felt from inside.
d) Pain may refer to tongue or teeth.
Correct answer : A

30.All of the following is true about salivary glands’ tumors except:


a) 80% of them occur in parotid gland.
b) 80% of parotid gland tumors are benign.
c) 80% of parotid gland tumors occur in the deep lobe.
d) 80% of benign parotid tumors are pleomorphic adenomas.
Correct answer : C

31- Highest percent of salivary stone occurr in


a) Parotid
b) Submandibular

- 118 -
c) Submaxillary
d) Minor salivary gland
Correct answer : B

32-The most common salivary tumor


a) Pleomorphic adenoma
b) Adenolymphoma
c) Epidermoid carcinoma
d) Adenocarcinoma
Correct answer : A
33-Spot the wrong statement
a) 80% of salivary neoplasms arise in parotid
b) 80 % of parotid tumorsare begnin
c) 80%of begnin parotid tumours are pleomorphic adenoma
d) All are correct
Correct answer : D

34-warthins tumor refers to


a) Pleomorphic adenoma
b) Adenolymphoma
c) Mucoepidermoid tumour
d) Adenoid cystic carcinoma
Correct answer : B

35-Malignant parotid tumour is characterized by


a) Pain
b) Hard and fixed mass
c) Paralysis of facial muscles
d) All of the above
Correct answer : D

36- Salivary carcinoma with poor prognosis is


a) Adenoid cystic carcinoma
b) Adenocarcinoma
c) Squamous carcinoma
d) Mucoepidermoid carcinoma
Correct answer : C

37-salivary carcinoma with perineural spread to brain is


a) Adenocarcinoma
b) Adenoid cystic carcinoma
c) Squamous carcinoma
d) Acinic cell carcinoma
Correct answer : B

38-adenolymphomas are common to


a) Parotid gland
b) Submandibular gland
c) Minor salivary glands
d) All of the above

- 119 -
Correct answer : A

39-the most common indication for removal of sublingual salivary gland is


a) Sialadenosis
b) Neoplasm
c) Ranula
d) Lymphoma
Correct answer : C

40- Parotid duct opens into mouth opposite


a) Upper third molar
b) Upper second molar
c) Behind third molar
d) Near to midline of hard palate
Correct answer : B

41-the treatment of submandibular calculus lying within the duct is to


a) Dilate the duct
b) Remove the stone by making an opening in the duct
c) Slit open the duct at the papilla
d) Remove the gland
Correct answer : B

42. Concerning pleomorphic adenoma (mixed salivary tumor), which of following


statements is incorrect:
a. is the commonest salivary tumor.
b. Has a very heterogenous histological structure.
c. is well-encapsulated.
d. Presents as a slow-growing firm swelling just below the lobule of the ear.
e. Tends to recur after inadequate excision.
Correct answer: C

43. Which of the following is not a feature of salivary malignancy?


a. Facial nerve weakness
b. Rapid enlargement
c. Induration of the overlying skin
d. Cervical node enlargement
e. Rubbery consistency.
Correct answer: E

44. What percentage of submandibular tumours are malignant?


a. 20 per cent
b. 30 per cent
c. 40 per cent
d. 50 per cent
e. 60 per cent.
Correct answer: D

- 120 -
45. Which is the most appropriate form of biopsy for a major salivary gland
tumour?
a. Open surgical biopsy to allow histology
b. Salivary washings
c. Frozen section during formal excision
d. Fine-needle aspiration cytology (FNAC)
e. Biopsy is contraindicated.
Correct answer: D

46. What is the most common site for a parotid tumour?


a. At the anterior border of the masseter
b. Inferior to the angle of the mandible
c. As a parapharyngeal mass
d. Anterior to the ear
e. Behind the angle of the mandible.
Correct answer: E

47. Which nerve must be transacted as part of a superficial parotidectomy?


a. The facial nerve
b. The hypoglossal nerve
c. The greater auricular nerve
d. The accessory nerve e. The auriculotemporal nerve.
Correct answer: C

48. What is Frey’s syndrome following parotidectomy?


a. Gustatory sweating
b. Dry mouth due to reduction in salivary flow
c. Development of a sialocele over the parotid bed
d. Cosmetic deformity due to loss of parotid bulk
e. Hyperplasia of the contralateral parotid gland.
Correct answer: A

49. With which malignancy is Sjögren’s syndrome associated?


a. Acinic cell carcinoma
b. Adenoid cystic carcinoma
c. Carcinoma ex pleomorphic adenoma
d. Salivary sarcoma
e. Lymphoma.
Correct answer: E

50.The commonest discrete lump of the parotid gland is:


a.Pleomorphic adenoma
b.Mucoepidermoid carcinoma
c. Mumps
d.Parotid gland stone
e.Tuberculosis of the parotid lymph node
Correct answer: A

- 121 -
51.All of the following is true about salivary glands’ tumors except:
a) 80% of them occur in parotid gland.
b) 80% of parotid gland tumors are benign.
c) 80% of parotid gland tumors occur in the deep lobe.
d) 80% of benign parotid tumors are pleomorphic adenomas.
Correct answer: C

52.A 62-year-old man undergoes excision of a cylindroma of the submandibular gland.


He is most likely to have an injury to which of the following?
(A) Maxillary branch of the trigeminal nerve
(B) Lingual nerve
(C) Vagus nerve
(D) Floor of the maxilla
(E) Frontozygomatic branch of the facialnerve
Correct answer: B

53. Salivary carcinoma with poor prognosis is:


(A)Adenoid cystic carcinoma
(B)Adenocarcinoma
(C)Squamous carcinoma
(D) Mucoepidermoid carcinoma
Correct answer: C

54. Salivary carcinoma with perineural spread to brain is:


(A)Adenocarcinoma
(B)Adenoid cystic carcinoma
(C)Squamous carcinoma
(D)Acinic cell carcinoma
Correct answer: B

55. Adenolymphomas are common to:


(A) parotid gland
(B) Submandibular gland
(C) Minor salivary glands
(D)All of the above
Correct answer: A

56. Which of the following salivary gland tumors shows perineural spread
A.Warthin's tumor
B. Adenoid cystic carcinoma
C. Carcinoma in pleomorphic adenoma
D. Mucoepidermoid carcinoma
Correct answer B

57 . The most common site of ectopic salivary tumor is :


A. Neck
B. Tongue
C. Palate
D. Cheek
Correct answer: A

- 122 -
58 . Which of the following is most common neoplasm of salivary gland :
A. Pleomorphic adenoma
B. Adenocystic Carcinoma
C. Mucoepidermoid
D. Mixed tumour
Correct answer: A

59 . Calculus is most commonly seen in which salivary gland:


A. Sublingual
B. Palatal
C. Parotid
D. Submandibular
Correct answer: D

60 . Commonest salivary gland to get stones is:


A. Parotid
B. Submandibular
C. Minor salivary gland
D. Sublingual
Correct answer: B

61 . Salivary gland calculus is seen more commonly in the duct of


A. Parotid gland
B. Submandibular gland
C. Sublingual gland
D. Minor salivary gland
Correct answer: B

62 . The salivary gland neoplasm which characteristically spreads perineurally


along the cranial nerve is -
A. Pleomorphic adenoma
B. Acinic cell carcinoma
C. Mucoepidermoid carcinoma
D. Adenoid cystic carcinoma
Correct answer: D

63 . Most common site of ectopic salivary tumor-


A. Neck
B. Palate
C. Tongue
D. Cheek
Correct answer: A

64 . Most common site of salivary gland calculi is ?


A. Parotid
B. Submandibular
C. Sublingual
D. Minor salivary glands
Correct answer: B

- 123 -
65 . Which of the following is the most common tumor of the salivary gland?
A. Acinic cell tumor
B. Pleomorphic adenoma
C. Mucoepidermoid tumor
D. Warthin's tumor
Correct answer: B

66 . The salivary gland neoplasm which characteristically spreads perineurally


along the cranial nerve is -
A. Pleomorphic adenoma
B. Acinic cell carcinoma
C. Mucoepidermoid carcinoma
D. Adenoid cystic carcinoma
Correct answer: D

67 . Pleomorphic adenoma of the salivary glands are :


A. Usually malignant
B. Most common in submandibular gland
C. Associated with calculi
D. Most common in parotid gland
Correct answer: D

58 . Sub-mandibular salivary gland is divided into superficial and deep lobes by :


A. Digastric tendon.
B. Hypoglosus muscle.
C. Mandible.
D. Mylohyoid muscle.
Correct answer: D

69 . Commonest benign tumor of the parotid salivary gland is -


A. Hemangioma
B. Lymphoma
C. Mixed parotid tumor (Pleomorphic adenoma )
D. Warthin's tumor
Correct answer: C

70 . A Warthin's tumour is -
A. An adenolymphoma of parotid gland
B. A pleomorphic adenoma of parotid
C. A carcinoma of the parotid
D. A carcinoma of sunmandibular salivary gland
Correct answer: A

71 . Bilateral multicentric/multifocal parotid tumor:


A. Warthin's tumour
B. Pleomorphic adenoma
C. Mucoepidermoid Carcinoma
D. Adenoid cystic carcinoma
Correct answer: A

- 124 -
72 . Surgery for Ranula involves -
A. Incision and drainage
B. Excision of cyst
C. Excision of affected sublingual salivary gland
D. Excision of cyst and affected sublingual salivary gland
Correct answer: D

73 . A Warthin's tumour is
A. An adenolymphoma of parotid gland
B. A pleomorphic adenoma of the parotid
C. A carcinoma of the parotid
D. A carcinoma of submandibular salivary gland
Correct answer: A

74 . Warthin's tumor is found in


A. Salivary gland
B. Thyroid gland
C. Brunner's gland
D. Adrenal gland
Correct answer: A

75 . Sialosis refers to -
A. Bilateral parotitis
B. Sjogren's syndrome
C. Noninflammatory parotid enlargement
D. Bilateral salivary duct ectasia
Correct answer: C

76-The false statement about ranula is that it:


a. is a bluish cyst in the angle between the tongue and the floor of mouth.
b. Usually lies to one side of the middle line.
c. May extend into the submandibular region.
d. May assume an hour-gloss appearance.
e. is best treated by complete excision.
Correct answer : E

77. The term plunging ranula refers to which clinical entity?


a. A malignant congenital salivary mass arising from the submandibular gland
b. A benign salivary mass involving the parotid and submandibular glands
c. A mucous retention cyst originating from the sublingual glands, limited by the
mylohyoid muscle
d. A mucous retention cyst originating from the submandibular and sublingual glands
which perforates the mylohyoid muscle to enter the neck
e. A midline neck mass which moves on tongue protrusion.
Correct answer : D

78-Ranula is a:
a) Cystic swelling in the floor of the month
b) Forked uvula
c) Sublingual thyroid

- 125 -
d) Thyroglossal cyst
Correct answer : A

79- Which of the following best represents 'ranula' ?


A. A type of epulis
B. A thyroglossal cyst
C. Cystic swelling in the floor of mouth
D. Forked uvula
Correct answer: C

80 . Most correct about 'Ranula' is :


A. It is a cystic swelling in the floor of mouth
B. It is type of Epulis
C. It is a type of mucus retention cyst
D. It is a type of thyroglossal cyst
Correct answer: A

81. Which statement most accurately describes the anatomy of the sublingual glands?
a. They drain through the sublingual duct which opens into the floor of mouth.
b. They drain either directly on to the floor of mouth or into the submandibular duct.
c. They consist of two lobes separated by the mylohyoid muscle.
d. They are embedded in the intrinsic muscles of the ventral surface of the tongue.
e. They lie in the space between the mandible and the two bellies of digastric.
Correct answer : B

82 . In case of benign mixed parotid tumours The treatment of choice is -


A. Superficial Parotidectomy
B. Total Parotidectomy
C. Leave facial nerve and remove all gland
D. Radical Parotidectomy
Correct answer: A

83.About adenolymphoma ,all the following statements are true , except:


a.The tumour is related to smoking
b.It may be bilateral
c.It has epithelial and lymphoid components
d.The tumour is cystic
e.Facial nerve palsy is characteristic of this tumour
Correct answer: E

Differential diagnosis of Jaw swellings

1. Correct statements about dentigerous cyst include the following except that it:
a. Occurs in children and adolscents in relation to a missing tooth.
b. is more common in the upper than in the lower jaw.
c. Presents as a globular swelling expanding the jaw.
d. Is lined with squamous epithelium.
Correct answer : B
- 126 -
2.Correct statements about dentigerous cyst include the following except
that it:
a. Occurs in children and adolscents in relation to a missing tooth.
b. is more common in the upper than in the lower jaw.
c. Presents as a globular swelling expanding the jaw.
d. is lined with squamous epithelium.
Correct answer: B

3. All true except dentigerous cyst:


a) An odontome
b) Contain ateeth inside
c) Treatment removal of the teeth
d) affects upper jaw more than lower jaw
Correct answer: D

4 . A 82 years patient presents with a midline tumor of the lower jaw, involving the
alveolar margin. He is edentulous. Treatment of choice is:
A. Commando operation
B. Hemimandibulectomy
C. Marginal mandibulectomy
D. Segmental mandiblectomy
Correct answer: D

5. All true regarding dentigerous cyst except:


a) An odontome
b) Contain a teeth inside
c) Treatment removal of the tooth only
d) Affect mandible more than upper jaw
Correct answer : C

6. Fissural cyst of the mandible:


a) Nasopalatine cyst is an example
b) Acquired by staph infection
c) Treatment by removal of the wall completely
d) Occur in fusion line of face
Correct answer : B

7 . Which jaw cyst is pre-malignant ?


A. Nasopalatine cyst
B. Radicular cyst
C. Odontogenic keratocyst
D. Dentigerous cyst
Correct answer: C

8. The most frequent fracture of the face involves the:


a. Zygoma.
b. Maxilla.
c. Orbital floor.
d. Nasal bones.

- 127 -
e. Mandible.
Correct answer: E

9 . A 65years man who is edentulous developed squamous cell Ca in buccal mucosa


that has infiltrated to the alveolus.Which of the following is not indicated for
treatment?
A. Segmental mandibulectomy
B. Radiotherapy
C. Marginal mandibulectomy involving removal of upper half of mandible
D. Marginal mandibulectomy involving removal of the outer table only
Correct answer: D

10.The most frequent fracture of the face involves the:


a. Zygoma.
b. Maxilla.
c. Orbital floor.
d. Nasal bones.
e. Mandible.
Correct answer: E

11. The most frequent fracture of the face involves the:


a. Zygoma.
b. Maxilla.
c. Orbital floor.
d. Nasal bones.
e. Mandible.
Correct answer : E

12 . Treatment of stage T3N1 of carcinoma maxilla is -


A. Radiation therapy only
B. Chemotherapy only
C. Surgery and radiation
D. Chemotherapy and radiation
Correct answer: C

13-Adenocarcinoma is the predominant malignant lesion in which of the following?


A. Hard palate
B. Lip
C. Anterior two-thirds of the tongue
D. Larynx
E. Esophagus
Correct answer : A

14 . In which of the following head and neck cancers, is lymph node metastasis least
common -
A. Buccal mucosa
B. Tongue
C. Lower alveolus
D. Hard palate
Correct answer: D

- 128 -
15 . All are true about carcinoma palate, except -
A. Slow growing
B. Bilateral lymphatic spread
C. Adenocarcinoma
D. Presents in pain
Correct answer: D

16. Adenocarcinoma is the predominant malignant lesion in which of the following?


(A) Hard palate
(B) Lip
(C) Anterior two-thirds of the tongue
(D) Larynx
(E) Esophagus
Correct answer: A

17 . All are feature of carcinoma palate, except -


A. Slow growing
B. Lymphatic spread
C. Adenocarcinoma
D. Painful
Correct answer: D

18 . What are Rushton bodies ?


A. Hyaline bodies of odotogenic cyst
B. Refractile bodies of radicular cysts
C. Bodies seen in amelobastomas
D. Hyaline bodies seen in dentigerous cysts
Correct answer: A

Differential Diagnosis of neck swelling (Medline and Lateral)

1.A 24 years old female presented with a neck swelling on the left side, on clinical
examination it was located in the carotid triangle deep to the sternomastoid muscle
,mobile from side to side ,it was transopaque on transillumination, Aspiration revealed
mucoid fluid rich in cholesterol crystals, your provisional diagnosis is………..:
a) Cervical lymph node level III.
b) Carotid body tumor.
c) Cystic hygroma.
d) Branchial cyst.
Correct answer : D

- 129 -
2 . A central midline neck swelling is noted in a 4year old girl posted for
tonsillectomy.The swelling is, painless, mobile and cystic, just below the hyoid
bone of size 2 X 1.1 X 1 cm. U/S showed a thick walled cystic lesion. Management
would include -
A. Percutaneous aspiration
B. Chest X-ray
C. Surgical removal
D. Antiboitics
Correct answer: C

3. Midline cystic swelling in the neck, moves up anddown with deglutition and with
protrusion of the tongue:
a) Thyroid cancer.
b) Branchial cyst.
c) Ranula.
d) Thyroglossal cyst.
Correct answer : D

Cancer tongue and cancer lip

1. Swollen cherry red lip with strawberry tongue is a feature of


A.Kawasaki disease
B.Scarlet fever
Correct answer A

2. Presentations of cancer tongue:


a- Asymptomatic
b- Referred otalgia
c- Fissured tongue
d- Halitosis
e- Al of the above.
Correct answer : E

3. All the followings are predisposing factors for cancer tongue except:
a- Cigarette smoking
b- Spicy food
c- Septic tooth
d- Sjogren syndrome
e- Syphilis
Correct answer : D

4. In unilateral hare-lip the following statements are true except that it:
a. Affects the upper lip only.
b. May be partial or complete.
c. Is due to failure to fusion between the maxillary process with the frontonasal process
and the maxillary process of the opposite side.

- 130 -
d. Is always associated with nasal deformity.
e. Produces no serious speech defects.
Correct answer : E

5. The wrong statement about carcinoma of the lip is that it:


a. Affects males much more often than females.
b. Usually occurs on the upper lip.
c. May produce a "Kissing cancer on the other lip,
d. is most often squamous cell carcinoma.
e. Spreads to the regional glands by lymphatic permeation.
Correct answer : B

6. Dental ulcer is characterized by the following features except that it:


a. Occurs in relation to a carious or irregular tooth.
b. Is usually painful.
c. Has an elongated shallow floor and a sloping edge.
d. Never causes enlargement of the regional lymph nodes.
e. May closely simulate malignant ulcer.
Correct answer : D

7. Carcinoma of the tongue infiltrating the mandible is best treated by:


a. Radiotherapy of both primary and regional gland.
b. Twa-stage excision of primary and regional glands.
c. On block excision of primary and whole lymphatic area (Commando operation).
d. Radiotherapy for primary followed by radical neck dissection.
e. Excision of primary and radiotherapy to cervical lymph nodes.
Correct answer : C

8. The anterior 2/3 of the tongue receives common sensation through ………nerve:
a) Glossopharyngeal.
b) Chorda tympani of facial.
c) Lingual.
d) Hypoglossal.
Correct answer : C

9. A 62-year-old alcoholic presents with an indurated ulcer, 1.5 cm in length, in the left
lateral aspect of her tongue (not fixed to the alveolar ridge). There are no clinically
abnormal glands palpable in the neck, and a biopsy of the tongue lesion reveals
squamous cell carcinoma. What should she undergo?
(A) Chemotherapy
(B) Local excision of the ulcer
(C) Wide excision and left prophylactic neck dissection
(D) Antibiotic therapy and should be encouraged to stop smoking
(E) Wide excision of ulcer and radiotherapy
Correct answer : C

- 131 -
10. A 59-year-old woman has discomfort in the posterior part of her tongue. Abiopsy
confirms that the lesion is a carcinoma. What is true in carcinoma of the posterior
third of the tongue?
(A) Lymphoid tissue is absent.
(B) Lymph gland spread is often encountered.
(C) There is an excellent prognosis.
(D) The tissue is well differentiated.
(E) The recurrent laryngeal nerve is infiltrated.
Correct answer : B

11-A 5-year-old girl presents with difficulty breathing. On examination, of the oral
cavity a 3-cm mass is found in the midline on the posterior aspect of the tongue. The
most likely diagnosis is:
(A) Lingual tonsil
(B) Lingual thyroid
(C) Foreign body stuck to the tongue
(D) Dermoid
(E) Angioneurotic edema
Correct answer : B

12. All the followings are presentations of cancer tongue:


a- Asymptomatic
b- Referred otalgia
c- Fissured tongue
d- Halitosis
e- All of the above.
Correct answer: E

13. All the followings are predisposing factors for cancer tongue except:
a- Cigarette smoking
b- Spicy food
c- Septic tooth
d- Sjogren syndrome
Correct answer: D

14. A 59-year-old woman has discomfort in theposterior part of her tongue. Abiopsy
confirms that the lesion is a carcinoma. What is true incarcinoma of the posterior
third of the tongue?
(A) Lymphoid tissue is absent.
(B) Lymph gland spread is often encountered.
(C) There is an excellent prognosis.
(D) The tissue is well differentiated.
(E) The recurrent laryngeal nerve isinfiltrated.
Correct answer: B

15 . Multiple painful ulcers on tongue are seen in all except


A. Aphthous ulcers
B. Tuberculous ulcers
C. Herpes ulcers
D. Carcinomatous ulcers

- 132 -
Correct answer: D

16.A 5-year-old girl presents with difficulty breathing. On examination, of the oral
cavity a3-cm mass is found in the midline on the posterior aspect of the tongue. The
most likelydiagnosis is:
(A) Lingual tonsil
(B) Lingual thyroid
(C) Foreign body stuck to the tongue
(D) Dermoid
(E) Angioneuroticedema
Correct answer: B

17. Carcinoma of the tongue infiltrating the mandible is best treated by:
a. Radiotherapy of both primary and regional gland.
b. Twa-stage excision of primary and regional glands.
c. on block excision of primary and whole lymphatic area (Commando operation).
d. Radiotherapy for primary followed by radical neck dissection.
e. Excision of primary and radiotherapy to cervical lymph nodes.
Correct answer: C

18 . A patient has carcinoma of tongue in the right lateral aspect with lymph node of 4
cm size in level 3 on the left side of neck, what is the stage ?
A. N1
B. N0
C. N3
D. N2
Correct answer: D

19 . A patient presented with a 1 X 1.5 cms growth on the lateral border of the
tongue.The treatment indicated would be
A. Laser ablation
B. Interstitial brachytherapy
C. External beam radiotherapy
D. Chemotherapy
Correct answer: B

20 . A patient with Ca Tongue is found to have lymph nodes in the lower neck.The the
treatment of choice for the lymph nodes is:
A. Lower cervical neck dissection
B. Suprahyoid neck dissection
C. Tele radiotherapy
D. Radical neck dissection
Correct answer: D

21 . Which carcinoma most commonly metastasizes to cervical lymph nodes -


A. Maxillary sinus
B. Posterior tongue
C. Cheek
D. Hard palate
Correct answer: B

- 133 -
22 . Abbe-Estlander Flap is used for:
A. Eyelid
B. Ears
C. Lip
D. Tongue
Correct answer: C

23 . Which Ca. has best prognosis -


A. Ca. Lip
B. Ca. Cheek
C. Ca. Tongue
D. Ca. Palate
Correct answer: A

24 . Which of the following is the commonest site of oral cancer among Indian
population?
A. Floor of mouth
B. Tongue
C. Lip
D. Alveobuccal complex
Correct answer: D

Cervical rib

1-cervical rib may present by which of the followings


a) Numbness in fingers
b) Bruit around clavicle
c) Lump in lower neck
d) All of the above
Correct answer : D

2-pain in arm due to cervical rib is caused by


a) Compression of t1
b) Compression of c7
c) Muscle ischemia
d) All of the above
Correct answer : C

3 . Adsons test is done for:


A. Cervical rib symptoms
B. Testing patency of ulnar artery
C. Testing meniscus injury
D. Anterior dislocation of the shoulder
Correct answer: A

- 134 -
4 . Allen's test is useful in evaluating which of the following?
A. Presence of cervical rib
B. Thoracic outlet compression
C. Digital blood flow
D. Integrity of palmar arch
Correct answer: D

5 . Adson's test is positive in:


A. Raynaud's disease
B. Cervical rib
C. Subclavian steal syndrome
D. Anterior compartment syndrome
Correct answer: B

6. About thoracic outlet syndrome, all statements are true, except:


a.In all cases a cervical rib is present
b.The commonest manifestations are caused by compression of the lower trunk of the
brachial plexus
c. There may be hyposthesia of medial side of forearm
d.It is one cause of Reynaud’s phenomenon
e.There may be chronic heaviness and oedema of the upper limb
Correct answer : A

6- Breast

Anatomy, embryology and congenital anomalies of the breast.

1. After undergoing modified radical mastectomy for cancer of the right breast, a
52-year-old female teacher becomes aware that the medial end of her scapula
becomes prominent in protraction movements at the shoulder. She also complains
of some weakness in complete abduction of the same shoulder. What nerve was
injured?
(A) Long thoracic
(B) Thoracodorsal
(C) Ulnar
(D) Intercostobrachial
Correct answer: A

2. The following statements about lymphatic drainage of the breast are all true,
except
a. Lymph from medial half of the breast may drain in internal mammary nodes.
b. From anywhere in the breast lymphatics drain mostly to axillary nodes

- 135 -
c. Division of axillary nodes into levels I, II and III depends upon the anatomical
relationship with the pectoralis minor.
d. Level one axillary nodes is the highest in the axilla.
e. The sentinel node is the earliest axillary node to be involved with cancer spread.
Correct answer: D

3. The axillary lymph nodes are divided into three levels I,II, III by
a. The axillary artery
b. The axillary vein
c. The pectoralis minor
d. The clavicle
e. The nerve to latissimus dorsi
Correct answer: C

4. How many lactiferous ducts drain into the nipple of the mature female breast
a. 5-10
b. 15-20
c. 25-30
d. 35-40
Correct answer: B

5. The medial mammary artery is a tributary of the


a. 2nd, 3rd, 4th intercostal arteries
b. Internal mammary artery
c. Thoraca-acromial artery
d. Posterior intercostal arteries
Correct answer: A

Benign breast diseases , mastalgia , nipple discharge

1- A blood stained discharge from the nipple indicates which of the bellow:
A. Fibroadenoma.
B. Breast abscess.
C. Fat Necrosis of Breast.
D. Duct Papilloma.
Correct answer: D

2. Most common cause of bloody discharge from nipple is


A.Duct ectasia
B.Duct carcinoma
C.Duct papilloma
Correct answer C

- 136 -
3 . A blood stained discharge from the nipple indicates:
A. Breast abscess
B. Fibroadenoma
C. Duct papilloma
D. Fat necrosis of breast
Correct answer: C

4 . A women noticed mass in her left breast with bloody discharge. Histopathology
revealed duct ectasia. Treatment is -
A. Lobectomy
B. Hadfield operation
C. Simple mastectomy
D. Microdochotomy
Correct answer: B

5 . Thrombophlebitis of the superficial veins of the breast are called -


A. Mondor's disease
B. Milroy's disease
C. Montgomery's nodule
D. Metzenbaum disease
Correct answer: A

6 . Treatment of choice in duct papilloma of breast is:


A. Microdochectomy
B. Simple mastectomy
C. Chemotherapy
D. Local wide excision
Correct answer: A

7 . Regarding cystic disease of breast which is true -


A. Common in 25 years of age
B. Excision is the treatment
C. May turn into malignant
D. Aspiration is the treatment
Correct answer: D

8 . A 14 years old healthy girl of normal height and weight for age, complains that
her right breast has developed twice the size of her left breast since the onset of
puberty at the age of 12 Both breasts have a similar consistency on palpation with
normal nipples areolae. The most likely cause for these findings is:
A. Cystosarcoma phyllodes
B. Virginal hypertrophy
C. Fibrocystic disease
D. Early state of carcinoma
Correct answer: B

9 . The treatment of choice in Duct papilloma of the breast is


A. Simple mastectomy
B. Microdochectomy

- 137 -
C. Local wide excision
D. Chemotherapy
Correct answer: B

10 . A 40-year-old woman comes to her physician after noticing a lump in her


breast. Physical examination reveals a 3 cm, firm irregular mass on the lateral
aspect of the right breast with dimpling of the overlying skin. A subsequent biopsy
of this mass reveals chronic inflammation, necrotic adipose tissue with
saponification, and areas of calcification. These histologic findings are consistent
with diagnosis of -
A. Comedocarcinoma
B. Fat necrosis
C. Duct ectasia
D. Granulomatous mastitis
Correct answer: B

11 . Cystosarcoma phyllodes is:


A. Large,soft fibroadenoma of breast
B. Sarcomatous change in fibrocystic disease of breast
C. Never metastasized via the bloodstream
D. Treated by radical mastectomy
Correct answer: A

12 . True about bacterial mastitis is -


A. Exclusively occurs in lactating breasts
B. Cleaning baby's mouth offers protection
C. Most commonly caused by staphylococcus aureus
D. Fluctuation is an early sign of breast abscess
Correct answer: C

13 . Common presentation of duct papilloma of breast is ?


A. Bloody nipple discharge
B. Mass in breast
C. Breast eczema
D. Paget's disease
Correct answer: A

14 . Mondor's disease is superficial thrombophlebitis of -


A. The lower limbs
B. The upper limbs
C. The breast and anterior chest wall
D. The abdominal wall
Correct answer: C

15 . A 25 year old female complains of discharge of blood from a single duct in her
breast.The most appropriate treatment is :
A. Microdochectomy
B. Radical excision
C. Biopsy to rule out carcinoma
D. Radical mastectomy

- 138 -
Correct answer: A

16 . Retromammary abscess is found in:


A. Paget's disease of breast
B. Empyema nessecitans
C. Ductular carcinoma of breast
D. Lobular carcinoma of breast
Correct answer: B

17 . All are true regarding Fibrocystic disease of Breast, except:


A. FNAC is helpful in the diagnosis
B. Excision biopsy may be done
C. There is 10-12 times increased risk of Ca breast
D. Most common age is 30-40 years.
Correct answer: C

18 . Which of the following is not a treatment for Phyllodes tumor ?


A. Quadrantectomy
B. Enucleation
C. Wide local excision
D. Simple mastectomy
Correct answer: A

19. An 18-year-old presents with a well circumscribed 2-cm mass in her right
breast. The mass is painless and has a rubbery consistency and discrete borders.
It appears to move freely through the breast tissue. What is the likeliest diagnosis?
(A) Carcinoma
(B) Cyst
(C) Fibroadenoma
(D) Cystosarcomaphyllodes
(E) Intramammary lymph node
Correct answer: C

20. Galactorrhea, a milky discharge from the nipple in nonpregnant women, is


most likely to be associated with which of the following?
(A) Fibroadenoma
(B) Tubular adenoma
(C) Pituitary adenoma
(D) Hyperparathyroidism
(E) Breast abscess
Correct answer: C

21. A 36-year-old woman complains of a 3-month history of bloody discharge from


the nipple. At examination, a small nodule is found, deep to the areola. Careful
palpation of the nipple areola complex results in blood appearing at the 3 O’clock
position. Mammogram findings are normal. What is the likeliest diagnosis?
(A) Intraductal papilloma
(B) Breast cyst
(C) Intraductal carcinoma
(D) Carcinoma in situ

- 139 -
(E) Fat necrosis
Correct answer: A

22. A 36-year-old woman presents with a substantial unilateral breast


enlargement. She had presumed that this was normal, but on examination, a large,
firm tumor is palpated by the attending physician. There is early erosion on the
skin. A favorable outlook can be anticipated if the lesion is which of the following?
(A) Sarcoma
(B) Cystosarcomaphyllodes
(C) Colloid carcinoma
(D) Infiltrating carcinoma
Correct answer: B

23. The following statements about post-mastectomy arm edema, all are true,
except
a. Is always transient
b. Adding radiotherapy to the axilla after clearance of lymph nodes increases the risk
of edema
c. Rough dissection of the axillary vein at surgery is one of the causes
d. Insertion of an IV line in the ipsilateral upper limb is a predisposing factor
e. May be caused by recurrence in the axilla.
Correct answer: A

24. The following statements about post-mastectomy arm edema, all are true,
except
a. Is always transient
b. Adding radiotherapy to the axilla after clearance of lymph nodes increases the risk
of edema
c. Rough dissection of the axillary vein at surgery is one of the causes
d. Insertion of an IV line in the ipsilateral upper limb is a predisposing factor
e. May be caused by recurrence in the axilla.
Correct answer: A

25. About acute lactational breast abscess, one statement only is true
a. Is commonly multilocular
b. Weaning the baby is mandatory
c. Aspiration pus by a wide bore needle is superior to incisional drainage as it avoids
injury to lactiferous ducts
d. Mammography is indicated if an abscess in suspected
e. Treatment is essentially conservative by antibiotics
Correct answer: A

26. The following about fibrocystic breast are all true, except
a. May be asymptomatic
b. May cause nipple retraction
c. Is a cause of breast pain
d. Produces breast nodularity
e. May cause discharge per nipple
Correct answer: C

- 140 -
27. During pregnancy, alveolar epithelium develops in the breast which is
responsible after delivery for the production of milk. Which portion of alveolar
epithelial cell is responsible for production of the fat present in human milk
a. Endoplasmic reticulum
b. Mitochondria
c. Cell membrane
d. Cytoplasm
Correct answer:D

28. Which of the following hormones is primarily responsible for differentiation


of breast ductal epithelium
a. Estrogen
b. Testosterone
c. Progestrone
d. Prolactin
Correct answer: C

29. Absence of the breast (amastia) is associated with


a. Turner’s syndrome
b. Kleinfelter’s syndrome
c. Poland syndrome
d. Fleischer’s syndrome
Correct answer: C

30. The treatment of choice of Zuska’s disease


a. Observation and NSAIDs
b. Antibiotics, incision and drainage
c. Wide resection of the affected areas
d. Mastectomy
Correct answer: B

31. Which of the following conditions leads to gynecomastia due to an increased


production of estrogen
a. Klinefelter’s syndrome
b. Hepatocellular carcinoma
c. Aging
d. Renal failure
Correct answer: B

32. The treatment of choice of mondor’s disease


a. Observation and NSAIDs
b. Antibiotics, incision and drainage
c. Wide resection of the affected area
d. Mastectomy
Correct answer: A

- 141 -
33. In the ANDI (aberrations of normal development and involution)
classification, a 2 cm fibroadenoma is considered
a. Normal
b. A disorder
c. A disease
d. A premalignant disease
Correct answer: B

34. Moderate ductal hyperplasia of the breast is characterized by the microscopic


finding of
a. 3-4 cell layer above the basement membrane
b. 5 or more cell layers above the basement membrane
c. Obstruction of >50% of the ductal lumen by hyperplastic cells
d. Obstruction of >70% of the ductal lumen by hyperplastic cells.
Correct answer: B

35. Which of the following is appropriate treatment for 3 cm fibroadenoma


a. Resection
b. Crya-ablation
c. Observation
d. All of the above.
Correct answer: D

36. Massive swellings of the breast include all the following except:
a- Cystosarcomaphylloides
b- Atrophic scirrhous carcinoma.
c- Diffuse hypertrophy.
d- Giant fibroadenoma.
Correct answer: B

37. The most useful investigation for breast with prosthesis is:
a. Mammography.
b. US.
c. FNAB.
d. MRI.
e. Hormonal receptors.
Correct answer : D

38. Triple assessment includes the following except:


a. Clinical evaluation.
b. lmaging.
c. Laboratory investigations.
d. Cytology.
Correct answer : C

39. Signs of pus formation are the following except:


a- Hectic fever.
b- Edema of overlying skin.
c- No response to medical treatment for 48 hours.
d- Fluctuation occurs early in breast abscess.

- 142 -
Correct answer : D

40.Best treatment of chronic breast abscess is:


a- Prolonged antibiotic courses.
b- Repeated aspiration.
c- lncision ond drainage.
d- Excision of the whole abscess.
Correct answer : D

41.Clinical findings of fibroadenosis include:


a- Cyclic pain.
b- Cyclic swelling.
c- Cyclic discharge.
d- All of the above.
Correct answer : D

42. Cyst of BIood Good is:


a- Retention cyst of duct papilloma.
b- Precancerous lesion of breast.
c- Giant fibroadenomo.
d- Hoemorrhagic cyst containing altered blood.
Correct answer : D

43. Treatment of fibroadenosis all true except:


a- Psychotherapy has a main role.
b- Primrose improves the condition.
c- Initial treatment should include antiestrogen.
d- Surgery is reserved to complicated cases.
Correct answer : C

44. The commonest route of infection of breast abscess is:


a- Along blood vessels
b- Retrograde infection along lymphatic vessels
c- Along natural passages (nipple)
d- Along artificial passages like fissures or crocks of nipple ond areola
e- Local extension from infection of the chest wall muscles or ribs
Correct answer : C

45. Mondor’s disease is:


a- An obscure type of thrombophlebitis particularly affecting veins of the breast.
b- Lymphoedema of the arm.
c- Chondritis of a costal cartilage.
d- Pectus excavatum.
Correct answer : A

46 Bleeding on zonal pressure is highly suggestive of:


a- Breast abscess.
b- Duct papilloma.
c- Cancer of breast.
d- phylloides tumor.

- 143 -
Correct answer : B

47. Best treatment for duct papilloma is:


a- Follow up.
b- Local excision.
c- Local excision with safety margin.
d- Chemotherapy and/or radiation.
Correct answer : C

48.The following is treatment of choice in peri canalicular fibroadenoma:


a- Irradiation.
b- Enucleation.
c- Removal en block.
d- Simple mastectomy.
Correct answer : B

49. Probe test can differentiate phylloides from:


a- Fibroadenosis.
b- Breast mouse.
c- Cancer breast.
d- Chronic breast abscess.
Correct answer : C

50. The cause in skin dimpling in any fibrotic affection of:


a- Milk duct.
b- Milk acini.
c- Cooper's ligament.
d- All of the above.
Correct answer : C

51. fibrocystic disease of the breast , all true except:


a.very common in female breast
b. may be asymptomatic
c. common on lt side
d. nipple retraction may be the only clinical criteria
e. easily confused with cancer
Correct answer : D

52. Treatment of cyclic mastalgia


a. exclusion of malignancy and assurance
b. avoid breast support
c.giving up caffeine consumption
d.bromocriptine 2.5 BID
e. Danazole 100-200 BID
Correct answer : B

- 144 -
53. breast cyst all true except
a.commonest is galactocele
b.malignancy is suspected if the cyst is tense
c. malignancy is suspected if the lump doesnot totally disappear after complete
aspiration
d. multiplicity of cyst raise suspiscion of malignancy
e.cyst excision is necessary in most cases
Correct answer : D

54. Mammary duct ectasia , all are true except


a. may produce mass similar to carcinoma in axillary tail
b. secretion is viscid thick
c. mammographic finding may mimic cancer
d. anerobic infection are common
e. may produce non lactitional mastitis and abscess
Correct answer : A

55. About mammography all are true except


a. expose the patient for some radiation
b. is safe and can be repeated at any time
c.malignancy can be proved my microcalcification
d. thickened skin and retracted nipple can be noted in cancer patient
e. irregular outline of the mass is another criterion of malignancy
Correct answer : B

56. Most common site of actinomycosis:


a. GlT.
b. Head ond neck.
c. Breast.
d. Lungs.
Correct answer: B

57. A 35-year-old professional dancer presents with a well-defined, tense, smooth


mass in the upper outer quadrant of the left breast. She states that the mass
becomes larger just before onset of her periods. Aspiration yields a clear yellow
fluid, and the mass disappears. The most likely diagnosis is:
(A) Fibroadenoma is a cyst.
(B) Fibrocystic disease of the breast.
(C) Carcinoma in a cyst.
(D) Lipoma.
(E) Galactocele
Correct answer: B

58 . Popcorn calcification in mammography is seen in


A. Fibroadenoma
B. Fat necrosis
C. Cystosarcoma phylloides
D. Ca Breast
Correct answer: A

- 145 -
59. Most common site of actinomycosis:
a. GlT.
b. Head and neck.
c. Breast.
d. Lungs.
Correct answer: B

60 . Mondor's disease is
A. Thrombophlebitis of the Superficial veins of Breast
B. Carcinoma of the breast
C. Premalignant condition of the breast
D. Filariasis of the breast
Correct answer: A

61 . When galactorrhea occurs in a high school student, a diagnostic associated


finding would be -
A. Gonadal atrophy
B. Bitemporal hemianopia
C. Exophthalmos and lid Sag
D. Episodic hypertension
Correct answer: B

Breast cancer

1 . Which the following tumor may occur in the residual breast or overlying skin
following wide local excision and radiotherapy for mammary carcinoma?
A. Basal cell carcinoma
B. Angiosarcoma
C. Leiomyosarcoma
D. Squamous cell carcinoma
Correct answer: B

2 . Peau d' orange appearance of breast is due to ?


A. Infiltration of cooper ligament
B. Infiltration of lactiferous ducts
C. Obstruction of dermal lymphatics
D. Involvement of skin
Correct answer: C

3 . A 45 year female patient with a family history of breast carcinoma. Showed


diffuse microcalcification on mammography. Intraductal carcinoma in situ was
seen on biopsy. Most appropriate management is:
A. quadrantectomy
B. Radical mastectomy
C. Simple mastectomy
D. Chemotherapy

- 146 -
Correct answer: C

4 . Triple assessment for CA Breast includes


A. USG, Mammogram and FNAC
B. Clinical Examination , Mammogram and FNAC
C. History, Clinical examination and mammogram
D. History, Clinical examination and FNAC
Correct answer: B

5 . Commonest type of ca breast is -


A. Papillary ca
B. Pagets disease
C. Fibrosarcoma
D. Infiltrative ductal ca
Correct answer: D

6 . Ipsilateral supraclavicular lymphnodes are positive in a patient of Ca


Breast.Stage is -
A. III B
B. II
C. IV
D. III C
Correct answer: D

7 . In case of CA breast most prognostic factor is -


A. Size of tumour
B. Lymph node status
C. Presence of estrogen receptor
D. Age. of menopause
Correct answer: B

8 . Sentinel lymph node biopsy is an important part of the management of


which?
A. Carcinoma breast
B. Carcinoma prostate
C. Carcinoma nasopharynx
D. Carcinoma lung
Correct answer: A

9 . Best diagnostic method for breast lump is -


A. USG
B. Mammogram
C. Biopsy
D. FNAC
Correct answer: C

10 . All of the following are true regarding schirrhous carcinoma of breast


except:
A. Grating sensation in cutting
B. Gray in color

- 147 -
C. Surface is convex
D. Capsulated
Correct answer: D

11 . In patients with breast cancer, chest wall involvement means involvement of


any of the following structures except -
A. Serratus anterior
B. Pectoralis Major
C. Intercostal Muscles
D. Ribs
Correct answer: B

12 . For Breast carcinoma which of the following is best indicator of prognosis -


A. Estrogen receptor status
B. Lymph node status
C. Metastasis
D. Size of tumour
Correct answer: B

13 . Neoadjunvant chemotherapy is not useful in -


A. Pnet of chest wall
B. Ca ovary stage III
C. Ca breast stage I
D. Osteosarcoma
Correct answer: C

14 . All are risk factor for Ca breast EXCEPT -


A. Ovarian malignancy
B. Family h/o breast Ca
C. Fibroadenosis
D. Multiparity
Correct answer: D

15 . Best prognosis amongst the following histological variants of breast


carcinoma is seen with:
A. Lobular
B. Ductal
C. Intraductal
D. Colloid
Correct answer: C

16 . False-positive mammography occurs in 11% cases due to:


A. Dense breast
B. Small lesion
C. Faulty technique
D. After aspiration
Correct answer: A

17 . Most common site metastasis for breast carcinoma ?


A. Thoracic vertebra

- 148 -
B. Pelvis
C. Femur
D. Lumbar vertebra
Correct answer: D

18 . In which of the following types of breast carcinoma, would you consider


biopsy of opposite breast?
A. Medullary carcinoma
B. Adenocarcinoma-poorly differentiated
C. Comedo carcinoma
D. Lobular carcinoma
Correct answer: D

19 . Which is a screening process for breast carcinoma -


A. Mammography breast
B. CA125-
C. MRI breast
D. CT scan
Correct answer: A

20 . A 44 years female presents with a hard and mobile lump in the breast.Next
investigation is:
A. Mammography
B. Excision biopsy
C. FNAC
D. USG
Correct answer: C

21 . Breast conservation surgery for breast cancer is indicated in one of the


following conditions:
A. T1 breast tumor
B. Multicentric tumor
C. Extensive in situ cancer
D. T4b breast tumor
Correct answer: A

22 . 35 year old female with painful breast lump in a breast feeding woman.
Investigation of choice is -
A. Mammography
B. US
C. MRI
D. CT
Correct answer: B

23 . True about treatment of early breast carcinoma -


A. Postmastectomy radiation therapy is given when 4 or more lymph nodes are
positive
B. Aromatase inhibitors are replacing tamoxifen in premenopausal women
C. In premenopausal women, multidrug chemotherapy in selected patients
D. Tamoxifen is not useful in post-menopausal women

- 149 -
Correct answer: A

24 . Breast conservation surgery NOT advise in -


A. Lobular Ca.
B. Ductal Ca. in situ
C. Early breast Ca.
D. Screening detected Ca.
Correct answer: D

25 . In the early detection of breast cancer, the following screening programs


have been recommended EXCEPT -
A. All women in age group 20 to 40 years should have complete breast examination
once in 6 months
B. All women over 20 years should be advised to examined their breasts monthly
C. All women over 40 years should have yearly breast examination
D. Mammography every year for women who are at high-risk for developing cancer
breast, for example, family history, endometrial cancer, nulliparous, etc
Correct answer: A

26 . Most common presentation of Lobular carcinoma is ?


A. Nipple discharge
B. Breast mass
C. Mammographic calcification
D. Nipple retraction
Correct answer: B

27 . Carcinoma breast with high incidence of involving opposite breast is -


A. Lobular carcinoma
B. Medullary carcinoma
C. Scirrhous adenocarcinoma
D. Atrophic scirrhous carcinoma
Correct answer: A

28 . Which of the following is the most important prognostic factor in breast


carcinoma ?
A. Stage of the tumor at the time of diagnosis
B. Histological grade of the tumor
C. Over expression of p-53 tumour suppressor gene
D. Status of estrogen and progesterone receptors
Correct answer: A

29 . Most common type of breast carcinoma is:


A. Infiltrative duct ca
B. Invasive lobular ca
C. Papillary ca
D. Paget's disease of breast
Correct answer: A

30 . Least risk of Ca breast is seen -


A. Li-Fraumeni syndrome

- 150 -
B. Ataxia Telangiectasia
C. BRCA 1
D. BRCA 2
Correct answer: B

31 . In which of the following types of Carcinoma Breast, Comedo growth


pattern in seen ?
A. Ductal carcinoma in situ
B. Medullary carcinoma
C. Lobular carcinoma in situ
D. Infiltrating lobular carcinoma
Correct answer: A

32 . All are true about CA breast, except -


A. Affected sibling is a risk factor
B. Paget's disease of nipple is intraductal type of CA
C. Common in age nulliparous
D. Increased incidence with prolonged breast feeding
Correct answer: D

33 . Which of the following carcinomas is familial?


A. Cervix
B. Vaginal
C. Breast
D. Prostate
Correct answer: C

34 . Breast carcinoma which is bilateral is:


A. Ductal
B. Lobular
C. Mucoid
D. Colloid
Correct answer: B

35 . The risk factor for increased incidence relapse in stage I, carcinoma breast
includes all except:
A. Aneuploidy
B. Decreased Her-2 / neu oncogene
C. -ve estrogne / progesterone receptor status
D. High 's' phase
Correct answer: B

36 . Which of the following breast malignancy occurs bilaterally?


A. Lobular carcinoma
B. Infiltrating ductal carcinoma
C. Medullary carcinoma
D. Papillary cercinoma
Correct answer: A

- 151 -
37 . Sentinel lymph node biopsy is useful in case of:
A. Breast carcinoma
B. Prostrate ca
C. Thyroid ca
D. Colon carcinoma
Correct answer: A

38 . Features, which are evaluated for histological grading of breast carcinoma,


include all of the following except -
A. Tumour necrosis
B. Mitotic count
C. Tubule formation
D. Nuclear pleomorphism
Correct answer: A

39 . Breast Ca which is multicentric and bilateral -


A. Ductal
B. Lobular
C. Mucoid
D. Colloid
Correct answer: B

40 .Earliest mammographic finding of breast carcinoma:


A. Microcalcification
B. Stellate density
C. Irregular margin
D. Skin thickening
Correct answer: A

41 .Which is ca breast is B/L -


A. Lobular
B. Paget's ds
C. Medullary
D. None
Correct answer: A

42 . A 30 years female presented with unilateral breast cancer associated with


axillary lymph node enlargement. Modified radical mastectomy was done,
further treatment plan will be -
A. Adriamycin based chemotherapy followed by tamoxifen depending on
estrogen/progesterone receptor status
B. Observation and follow up
C. Tamoxifen only
D. Adriamycin based chemotherapy only
Correct answer: A

43. Paget's disease of the nipple is -


A. Superficial manifestation of underlying breast Ca
B. Galactocele
C. Eczema

- 152 -
D. All of the above
Correct answer: A

44 . A 55 year old post-menopausal woman, on hormone replacement


therapy(HRT),presents with heaviness in both breasts. A screening mammogram
reveals a high density speculated mass with cluster of pleomorphic
microcalcification and ipsilateral large axillary lymph nodes. The mass described
here most likely represents:
A. Cystosarcoma phylloides
B. Lymphoma
C. Fibroadenoma
D. Carcinoma
Correct answer: D

45 . Moderately increased risk of invasive Breast carcinoma is associated with


which of the following benign lesions of the breast:
A. Atypical lobular hyperplasia
B. Sclerosing Adenosis
C. Squamous metaplasia
D. Apocrine metaplasia
Correct answer: A

46 . For CA breast best chemotherapeutic regimen -


A. Cyclophosamide, methotrexate, 5-fluorouracil
B. Methotrexate, cisplatin
C. Cisplatin, Adriamycin, steroid
D. Methotrexate, Adriamycin, steroid
Correct answer: A

47 . A Score of BIRDS-6 on Mammography indicates that:


A. Radiographically confirmed breast ca.
B. Histopathologically confirmed breast ca.
C. Highly suspicious of malignancy
D. No evidence of malignancy
Correct answer: B

48 . Use of tamoxifen in carcinoma of breast patients does not lead to the following
side effects
A. Thromboembolic events
B. Endometrial Carcinoma
C. Cataract
D. Cancer in opposite breast
Correct answer: D
49 . Histological variety of Breast carcinoma with breast prognosis is ?
A. Medullary
B. Colloid
C. Lobular
D. Tubular
Correct answer: D

- 153 -
50 . Ca breast Stage T4b involves all except -
A. Nipple retraction
B. Skin ulcer over the swelling
C. Dermal edema
D. Satellite nodule
Correct answer: A

51 . Drug of choice for estrogen receptor positive breast cancer is ?


A. Transtuzumab
B. Tamoxifen
C. Cyproterone acetate
D. Anastrozole
Correct answer: B

52 . Nodule in upper outer segment of breast with cyclical pain, the most likely
diagnosis is:
A. Phylloides tumor
B. Fibroadenoma
C. Fibrocystic disease of breast
D. None
Correct answer: C

53 . Sentinel node biopsy-done in-


A. Breast carcinoma
B. Anaplastic carcinoma of lung
C. Sq. cell carcinoma of skin
D. All of the above
Correct answer: A

54 . A 35-year-old woman was diagnosed to be links medullary carcinoma of the


right breast with TNM staging of T2N1M0. The current recommended treatment
is -
A. Multimodality therapy consisting of surgery
B. Modified radical mastectomy
C. Simple mastectomy with radiotherapy
D. Combination chemotherapy
Correct answer: A

55 . In breast carcinoma metastasis, prognosis depends best upon:


A. Axillary lymph node status
B. Extrogen receptors status
C. Site of tumour
D. Size of tumour
Correct answer: A

56 . Large breast is Not seen in -


A. Filariasis
B. Giant fibroadenoma
C. Cystosarcoma phylloides
D. Schirrhous carcinoma

- 154 -
Correct answer: D

57 . On which of the following does prognosis in male breast Ca depend-


A. Duration of disease
B. Nipple discharge
C. Ulceration of nipple
D. Lymph node status
Correct answer: D

58 . In breast conservative surgery the healthy margin excised is typically?


A. 1 cm
B. 2 cm
C. 3 cm
D. 5 cm
Correct answer: A

59 . Best diagnostic test for breast lump:


A. FNAC
B. Bidirectional mammography
C. USG
D. MRI
Correct answer: A

60 . A female patient present with a hard mobile lump in her right. breast. Which
investigation would be most helpful in making on diagnosis:
A. FNAC
B. Needle biopsy
C. Excision biopsy
D. Mammography
Correct answer: C

61 . Drug used in estrogen dependent breast cancer -


A. Tamoxifen
B. Clomiphene citrate
C. Estrogen
D. Adriamycin
Correct answer: A

62 . Which of the following stage of Breast Ca corresponds with following


feature-->Breast mass of 6x3cm. size with hard mobile ipsilateral axillary lymph
node and ipsilateral supraclavicular lymph node
A. T4 N2M0
B. T3 N1M1
C. T4 N1M1
D. None
Correct answer: D

63 . All of the following are used for reconstruction of breast except:


A. Transverse rectus abdominus myocutaneous flap
B. Latissimus dorsi myocutaneous flap

- 155 -
C. Pectoralis major myocutoneous flap
D. Transversus rectus abdominus free flap
Correct answer: C

64 . Which of the following is the next investigation for painful breast lump in a
lactating woman?
A. USG
B. Mammography
C. X-ray
D. MRI
Correct answer: A

65 . Not true about CA breast in India -


A. Incidence is 20/1,00,000
B. Average age 42 years
C. Positive family history in a risk factor
D. More common in Muslims
Correct answer: D

66 . The most important prognostic factor for breast carcinoma is:


A. Histological grade of the tumor
B. Stage of the tumor at the time of diagnosis
C. Status of estrogen and progesterone receptors
D. Status of Her 2/neu oncogene by overexpression of p53 tumor suppressor gene
Correct answer: B

67. One of the following statements as regard investigations for breast cancer is
true
A Fifty per cent of breast cancers are missed by population-based mammographic
screening.
B Ultrasound can also be used as a screening tool.
C Magnetic resonance imaging (MRI) can be a useful imaging tool.
D Fine-needle aspiration cytology (FNAC) and core biopsy are equally useful
diagnostically.
Correct answer: C

68. Which of the following conditions have an increased risk of breast carcinoma
A Breast cyst
B Duct ectasia
C Atypical ductal or lobular hyperplasia
D Fibroadenoma.
Correct answer: C

69. In breast carcinoma, which one of the following statements is false?


A Ductal carcinoma is the most common variant.
B Lobular carcinoma occurs in 15 per cent.
C There may be a combination of lobular and ductal features.
D Colloid, medullary and tubular carcinomas carry a poor prognosis.
Correct answer: D

- 156 -
70. In the treatment of breast cancer, which of the following statements are false?
A There is a higher rate of local recurrence after conservative surgery and radiotherapy.
B After mastectomy, radiotherapy to the chest wall is not indicated.
C Sentinel lymph node biopsy should be done in clinically node-negative disease.
D Besides treating the patient, the role of axillary surgery is to stage the patient
accurately.
Correct answer: B

71. A 35-year-old premenopausal woman whose mother had breast cancer comes
into your office and has been told that she has fibrocystic breasts. On examination
she has multiple areas of thickening but no discrete mass. Of the following
diagnostic tests, which should be performed?
(A) Re-examination in 6 months
(B) Bilateral breast ultrasound
(C) Thermography
(D) Bilateral breast magnetic resonance imaging (MRI) with gadolinium
(E) Spot compression views if an area of discrete asymmetry or concerning
calcifications is seen
Correct answer: D

72. During a routine screening mammography, a 62-year-old teacher is informed


that she has changes on her mammography, and she should consult her physician.
She can be reassured that the findings that indicate a benign condition are which
of the following?
(A) Discrete, stellate mass
(B) Fine, clustered calcifications
(C) Coarse calcifications
(D) Solid, clearly defined mass with irregular edges
(E) Discrete, non palpable mass that has enlarged when compared with a mass shown
on a mammogram taken 1 year previously.
Correct answer: C

73. A50-year-old patient has recently undergone mastectomy for a 2.5-cm


multicentric breast cancer with three positive axillary lymph nodes(stage IIB). A
metastatic survey is done, and is negative, and she receives adjuvant
chemotherapy. The most common site for distant metastasis would be:
(A) Brain
(B) Bone
(C) Lung
(D) Liver
Correct answer: B
74. The following statements about pulmonary metastases of breast cancer are all
true, except:
a. May be asymptomatic
b. May produce persistent cough
c. May produce dyspnea
d. May cause pleural effusion
e. If localized to one lung while the other lung shows good function, pneumonectomy
is indicated.
Correct answer:E

- 157 -
75. Hematogenous spread of breast cancer may produce any of the following,
except:
a. Brain metastases.
b. Fracture of the spine
c. Pleural effusion
d. Jaundice
e. Hypocalcemia
Correct answer:E

76. Concerning sampling of a breast lump, all the following statement are true,
except
a. The most accurate is excisional biopsy
b. Fine need aspiration cytology does not require general anaethesia
c. Fine need aspiration cytology shows tissue architecture.
d. Frozen section biopsy is sometimes inconclusive.
e. Trucut biopsy obtains a core of tissue.
Correct answer: C

77. Regarding duct carcinoma insitu, one statement only is true


a. May be discovered on screening mammography for asymptomatic women.
b. It’s not a true malignancy
c. Close observation is all that is needed
d. Is always bilateral
Correct answer: A

78. Early breast cancer


a. Means T1N1 tumor or less
b. Means that cure is impossible
c. Means that microscopic metastasis are not present
d. Primary treatment is by surgery and/or radiotherapy
e. Adjuvant chemotherapy is not indicated.
Correct answer: D

79. For Modified radical mastectomy, all the following are true, except
a. The areola and nipple are removed
b. The whole breast tissue is removed
c. Block dissection of axillary nodes requires excision of axillary vein
d. Nerve to serratus anterior should be spared
e. The pectoralis major is spared.
Correct answer: C
80. When screening females for breast carcinoma, what if the most significant risk
a. Three previous breast biopsies in premenopausal females.
b. More than 2 first degree relatives with ovarian or breast carcinoma
c. Hyperplasia in breast biopsy
d. None of the above.
Correct answer: B

- 158 -
81. What is the hereditary factor percentage in female breast cancers
a. 20.30%
b. 45-50%
c. 6-10%
d. 75-80%
e. None of the above.
Correct answer: C

82. A clinically positive sub clavicular lymph nodes is a


a. Level I
b. Level II
c. Level III
d. Level IV
Correct answer: C

83. The appropriate therapy of Paget’s disease of the nipple


a. Topical steroid cream
b. Topical antifungal medication
c. Intralesional steroid injection
d. Resection
Correct answer: D

84. Which of the following conditions increases a woman’s risk of breast cancer
a. Sclerosing adenosis
b. Fibroadenoma
c. Atypical lobular hyperplasia
d. Intraductal papilloma
Correct answer: C

85. A 35 year old woman with BRAC-1 gene mutation seeks your advice about her
known increased risk of breast cancer. You should recommend
a. Mammograms and physical examination every 6 months until she is 50, then
bilateral prophylactic mastectomy
b. Mammograms and physical examination every 6 months + tamoxifen
c. Prophylactic bilateral mastectomy and if she has completed childbearing,
prophylactic bilateral oophorectomy.
d. None of the above.
Correct answer: C

86. Which of the following about lobular carcinoma in situ is true


a. In general, LCIS occurs at an older age than ductal carcinoma in situ
b. The majority of women with LCIS are premenopausal
c. LCIS is bilateral in 10-20% of women
d. Invasive ductal carcinoma can be expected to occur an average of 5-10 years later
in approximately 75% of women with LCIS.
Correct answer: B

- 159 -
87. All the following are recognized side effects of radiotherapy except:
a- Local burn.
b- Pulmonary fibrosis.
c- End arteritis.
d- None of the above.
Correct answer: D

88. Adjuvant chemotherapy is indicated in:


a- +ve LN biopsy.
b- -ve hormonal receptors.
c- Poor prognosis of the case.
d- All of the above.
Correct answer: D

89. Chemotherapy is the primary palliative treatment in the following conditions:


a- Visceral metastasis.
b- Advanced case in premenopausal women.
c- -ve hormonal receptors with distant metastasis.
d- All of the above.
Correct answer: D

90. The main presentation of Paget disease is:


a- Mass under the nipple
b- Bleeding per nipple
c- Unilateral red scally nipple
d- Bilateral itchy red vesicles.
Correct answer: C

91. Lobular carcinoma of breast All true except:


a- Is more common than ductal carcinoma.
b- In situ state is considered risk factor for malignancy.
c- With in situ carcinoma mammography of both breasts is mandatory.
d- Usually bilateral and multicentric.
Correct answer: A

92. As regard incidence of cancer breast all correct except:


A- Most common female malignancy.
B- Rare in nulliparous females.
C- Very rare below 20 years.
D- Males are rare to be affected but with grave prognosis.
Correct answer: B

93. Tamoxifen use in breast cancer causes all, Except:


a- Decreases recurrence in affected breast.
b- Decreases incidence in contralateral breast.
c- lncreased incidence of endometrial cancer.
d- lncreased incidence of myocardial infarction.
Correct answer: A

- 160 -
94. In curable breast cancer in premenopausal females may be treated by
the following except:
a- Local mastectomy
b- Radiotherapy
c- Estrogen administration
d- Chemotherapy
e- Tamoxifen
Correct answer: C

95. Etiology of breast abscess includes:


a- Mostly in lactating women.
b- Due to Staph from baby mouth.
c- Bad hygiene.
d- All of the above.
Correct answer : D

96. Risk factors of cancer breast include all the following except:
a- Mutation in suppressor genes (BRCA l, Il).
b- Early menarche.
c- Carcinoma in situ.
d- Prolonged lactation.
e- White races.
Correct answer : D

97. Increased occurrence of cancer breast in upper loteral quadrant is due to:
a- increased oestrogen receptors.
b- increased breast mass.
c- Both of the above.
d- None of the above.
Correct answer : D

98. Peu d'orange is due to:


a- Skin metastasis.
b- Lymphatic obstruction.
c- Thrombophlebitis.
d- Any of the above.
Correct answer : B

99. Contraindications to conservative surgery include:


a- Large tumour.
b- Paget’s disease of nipple.
c- Previous irradiation.
d- Distant metastasis.
e- All of the above.
Correct answer : E

100. The following is contraindication to conservative breast surgery:


a- Palpable mobile axillary LNs
b- Tumour 3 cm
c- Mastitis carcinomatosis

- 161 -
d- Patient aged 50 years
Correct answer : C

101. Commonest histologic type of breast cancer is


a.duct carcinoma in situ
b.lobular carcinoma in situ
c.invasive lobular carcinoma
d. medullary carcinoma
e. infilterating duct carcinoma not otherwise specified
Correct answer : E

102. a women has 3 cm breast mass non palpable LNs and no clinical evidence of
distant metastasis, which is true?
a. stage II breast cancer
b.grade I breast cancer
c. pathological evidence of cancer is necessary
d. consider fibroadenoma and give treatment
e.excision of the mass only is enough
Correct answer : C

103. As regard prognosis of breast cancer All are true except


a. worsen by detection of positive axillary LNs
b. better when strong positive estrogen receptor
c. better in lower inner quadrant
d.worse for invasive duct carcinoma than phylloids tumor
e.affected by tumor grading
Correct answer : C

104. Paget disease of the breast all are true except


a. Is an eczema of the skin
b. Sometimes there is palpable mass
c. Mastectomy is indicated
d. The disease is not related to Paget disease of bone
e. Is a duct carcinoma arising from main lactiferous ducts
Correct answer : E

105. Axillary LNs all are true except


a. Anterior, posterior and lateral group are level II
b. Central l/n are related to intercostobrachial nerve
c. Apical group of l/n are level III
d. Positive axillary l/n affect the prognosis of breast cancer
Correct answer : A

106. Most virus associated malignancy is


A. Carcinoma cervix
B. Burkitt's lymphoma
C. Carcinoma Breast
Correct answer C

- 162 -
107- An 82-year-old woman with a history of breast cancer presents to clinic with
complaints of headache for the past 2 weeks. She describes daily retra-orbital
headaches that are worse in the morning. In addition, she notes severe nausea and
vomiting. MRI of the brain reveals multiple metastatic lesions. Which of the
following is the most appropriate initial management for this patient?
1. Biopsy the lesions
2. Surgical resection
3. Dexamethasone
4. Repeat MRI in 6 months
Correct answers: (3)

108. Which of the following is accurate regarding the use of mammography in


women who have had previous surgery for breast cancer?
A- Mammography is contraindicated in women who have had subcutaneous
or nipple-sparing mastectomy.
B- The earliest that mammography of a surgically treated breast should be
performed is 1 year after the operation.
C- The use of MRI as an adjunct to standard mammography is
contraindicated in patients with silicone breast implants.
D- After surgery for breast cancer, mammography for follow-up and screening
should be performed every 6-12 months.

7- Oesophagus

Esophagus: GERD and motility disorder

1 . Corkscrew esophagus is seen in which?


A. Achalasia cardia
B. Diffuse esophagus spasm
C. Carcinoma esophagus
D. Scleroderma
Correct answer: B

2 . A female patient has dyspepsia, intermittent epigastric pain.On endoscopy,


esophagus was dilated above the narrow at the bottom.Treatment is -
A. Proton Pumb Inhibitors
B. Esophagectomy
C. Dilatation
D. Heller's Cardiomyotomy
Correct answer: D

- 163 -
3 . Boerhaave syndrome is ?
A. Spontaneous Rupture of esophagus
B. Traumatic Rupture of esophagus
C. Tracheo esophageal fistula
D. Foreign body Esophagus
Correct answer: A

4 . Most common site of esophageal perforation is ?


A. Cervical region
B. Cardiac region
C. Mid esophagus
D. Lower esophagus
Correct answer: A

5 . Most common site of tear in Mallory-Weiss syndrome is in -


A. Upper esophagus
B. Mid esophagus
C. Lower esophagus
D. Cardia of stomach
Correct answer: D

6 . Which of the following is non progressive contraction of Esophagus ?


A. Secondary
B. Primary
C. Quaternary
D. Tertiary
Correct answer: D

7 . A 40 year old female patient presents with dysphagia to both liquids and
solids and regurgitation for 3 months. The dysphagia was non progressive. What
is the most likely diagnosis?
A. Lower esophagus mucosal ring
B. Carcinoma of the esophagus
C. Reflux esophagitis with esophageal stricture
D. Achalasia cardia
Correct answer: D

8 . Boerhaave's syndrome is due to


A. Reflux esophagitis
B. Spontaneous perforation of esophagus
C. Corrosive ingestion
D. Aspiration pneumonia
Correct answer: B

9 . Corkscrew esophagus is seen in


A. Achalasia cardia
B. Diffuse esophageal spasm
C. Corrosive stricture of the esophagus
D. Carcinoma esophagus
Correct answer: B

- 164 -
10 . Non progressive contraction of Esophagus are:
A. Secondary
B. Primary
C. Quaternary
D. Tertiary
Correct answer: D

11 . Which of the following investigation is diagnostic of motility disorder of the


oesophagus ?
A. Acid Ph. studies
B. Endoscopy
C. Barium swallow
D. Oesophageal manometry
Correct answer: D

12 . A 60 year old female presents with recurrent pneumonia, regurgitation of


food and feeling of fullness. Most probable diagnosis is ?
A. Hiatus hernia
B. Carcinoma esophagus
C. Tracheoesophageal fistula
D. Achlasia cardia
Correct answer: A

13 . Regarding Barret's esophagus which of the following is incorrect?


A. Is due to chronic esophageal reflux
B. Is lined by columnar epithelium
C. Is associated with elevated lower esophageal sphincter
D. Is associated with increased incidence of adenocarcinoma of esophagus
Correct answer: C

14 . 24 hour pH manometery is gold standard investigation for ?


A. Hiatal hernia.
B. Barrett's esophagus
C. Reflux esophagitis
D. Esophageal ulcers
Correct answer: C

15 . Regarding Mallory-Weiss syndrome which of the following statements is


incorrect
A. Is due to tear of gastric mucosa near incisura angularis
B. Presents with upper GI bleed
C. In majority of cases bleeding stops spontaneously
D. Usually seen in alcoholics
Correct answer: A

16 . Heller's operation is done for ?


A. Gastric outlet obstruction
B. Achalasia cardia
C. Hiatal hernia
D. Esophageal spasms

- 165 -
Correct answer: B

17 . Which of the following mechanism can not prevent Gastroesophageal reflux .


A. Looping fibers of rt crus of diaphragm
B. Mucosal folds at gastroesophageal junction
C. Circular muscle fibers of GE sphincter
D. Angle made by the oesophagus with stomach
Correct answer: C

18. A known HIV positive patient complains of severe odynophagia. He avoids eating
and drinking because of the intense pain, and he has lost a significant amount of
weight. Which of the following is TRUE?
(A) Esophagectomy is the treatment of choice.
(B) Cancer is the only condition that can explain these findings.
(C) UGI series is not useful.
(D) Candida is the most common cause of infectious esophagitis.
(E) Esophageal candidiasis is almost certain if the patient has oral thrush.
Correct answer: D

19. A 54-year-old man presents with a massive UGI bleed. After resuscitation,
endoscopy is performed. No esophageal varices, gastritis, or gastric ulcers are seen.
After copious irrigation, a pinpoint lesion is seen near the GE junction.
What can be said about this lesion?
(A) It is a carcinoid.
(B) It is related to alcohol use.
(C) It is exclusively a mucosal lesion.
(D) Surgery if first-line therapy.
(E) Bleeding is from a submucosal vessel
Correct answer: E

20. A patient has compressive symptoms of the esophagus. He has a barium


esophagram that shows posterior extrinsic compression of the esophagus. Which of the
following is true?
(A) Vascular rings are acquired atherosclerotic lesions.
(B) Both the trachea and esophagus can be affected by vascular rings.
(C) The two most common types of complete vascular rings are double aortic arch
and left aortic arch.
(D) There is no role for Echo and Doppler.
(E) Surgery involves division of the esophagus.
Correct answer: B

21. A 50-year-old gynecologist complains of dysphagia, regurgitation, and weight loss.


She also states that she feels as if food is stuck at the level of the xiphoid. An upright
chest x-ray shows a dilated esophagus with an air-fluid level. Which of the following
is FALSE?
(A) A barium swallow will show a “bird’s peak” deformity
(B) Manometry will demonstrate that the LES fails to relax during swallowing.
(C) Upper endoscopy should be avoided because of the risk of complications.
(D) Medical treatment includes nitrates and calcium channel blockers.
(E) Intersphincteric injection of botulinum toxin can be therapeutic.

- 166 -
Correct answer: C

22. A patient has been diagnosed with achalasia. He refused surgery initially, preferring
to try nonoperative therapy. He tried life style modification, calcium channel blockers,
botulin toxin injection, and endoscopic pneumatic dilatation. None of the treatments
alleviated his symptoms. What are his surgical options?
(A) Esophagectomy
(B) Surgical esophagomyotomy proximal to the LES
(C) Modified Heller myotomy and partial fundoplication
(D) Repeat pneumatic dilation using pressures of loops
(E) Nissen fundoplication
Correct answer: C

23. A 50-year-old man presents to the emergency department with chest pain. The
patient is evaluated for a myocardiac infarction. The workup is negative. On further
questioning, his symptoms include dysphagia (with both liquids and solids). Which of
the following is TRUE?
(A) A barium swallow will always show a corkscrew esophagus.
(B) Manometry shows simultaneous high-amplitude contractions.
(C) Initial evaluation should exclude coronary artery disease.
(D) A pulsion diverticulum may be present.
Correct answer: A

24. A 79-year-old retired opera singer presents with dysphagia, which has become
progressively worse during the last 5 years. He states that he is sometimes aware of a
lump on the left side of his neck and that he hears gurgling sounds during swallowing.
He sometimes regurgitates food during eating. What is the likely diagnosis?
(A) Carcinoma of the esophagus
(B) Foreign body in the esophagus
(C) Plummer-Vinson (Kelly-Patterson) syndrome
(D) Zenker’s (pharyngoesophageal) diverticulum
(E) Scleroderma
Correct answer: D

25. A symptomatic patient has a barium swallow that reveals a 3-cm Zenker’s
diverticulum. The next step in management is?
(A) H2 blockers
(B) Anticholinergic drugs
(C) Elemental diet
(D) Bougienage
(E) Surgery (cricopharyngealmyotomy and diverticulectomy)
Correct answer: E

26 . Which of the following is TRUE about Zenker's diverticulum


A. It is asymptomatic
B. Occurs in the mid-esophagus
C. Treatment is simple excision
D. It occurs in children
Correct answer: C

- 167 -
27 . All of the following statements about Zenker's diverticulum are true Except:
A. False diverticulum
B. Out pouching of the anterior pharyngeal wall,just above the cricopharyngeus
muscle
C. Acquired diverticulum
D. Lateral X-rays on Barium swallow are often diagnostic
Correct answer: B

28 . Investigation of choice for Zenker's diverticulum is -


A. CECT
B. Endoscopy
C. Esophageal manometry
D. Barium Swallow
Correct answer: D

29 . Commonest complication of Zenker's diverticulum is -


A. Dysphonia
B. Gastroesophageal reflux
C. Lung abscess
D. Perforation
Correct answer: C

30. A30-year-old psychiatric patient has a barium swallow after removal of a foreign
body to rule out a small perforation of the esophagus. No perforation is seen, but an
epiphrenic diverticulum is visualized. An epiphrenic diverticulum may be associated
with which of the following?
(A) Duodenal ulcer
(B) Gastric ulcer
(C) Cancer of the tongue
(D) Cancer of the lung
(E) Hiatal hernia
Correct answer: E

27. A 53-year-old moderately obese woman presents with heartburn aggravated mainly
by eating and lying down in the horizontal position. Her symptoms are suggestive of
gastroesophangeal reflux disease (GERD). Which of the following statements is
TRUE?
(A) It is best diagnosed by an anteroposterior (AP) and lateral film of the chest.
(B) It may be alleviated by certain drugs, especially theophylline, diazepam, and
calcium channel blockers.
(C) It is not relieved by cessation of smoking.
(D) If it is associated with dysphagia, it suggests a stricture or motility disorder.
(E) It should be immediately treated with Surgery
Correct answer: D

28. A 64-year-old man has symptoms of reflux esophagitis for 20 years. The barium
study shown (Fig. 5–2) demonstrates a sliding hiatal hernia. Which is TRUE in sliding
hiatal hernia?
(A) A hernia sac is absent.
(B) The cardiais displaced into the posterior mediatstinum.

- 168 -
(C) Reflux esophagitis always occur.
(D) A stricture does not develop.
(E) Surgery should always be avoided.
Correct answer: B

29. A 45-year-old man presents with a long history of heartburn, especially at night. He
uses three pillows to sleep and has medicated himself with a variety of antacids over
the past 15 years. Recently he has been complaining of dysphagia that he localized to
the precordial area. Which is the most likely diagnosis?
(A) Adenocarcinoma of the esophagus
(B) Angina pectoris
(C) Benign peptic stricture of the esophagus
(D) Achalasia of the esophagus
(E) Lower esophageal ring (Schatzki’s ring)
Correct answer: C

30. A 60-year-old man presents with excruciating chest pain. The pain follows an
episode of violent vomiting that occurred after a heavy meal. Subcutaneous emphysema
was noted in the neck. X-rays shows air in the mediastinum and neck, and a fluid level
in the left pleural cavity. What is the most likely diagnosis?
(A) Perforated duodenal ulcer
(B) Spontaneous rupture of the esophagus
(C) Spontaneous pneumothorax
(D) Inferior wall myocardial infarction
(E) Dissecting aortic aneurysm
Correct answer: B

31. A 45-year-old pilot has retrosternal burning; especially when he eats and lies down
to go to sleep. He has self-medicated himself with over the counter heartburn
medications. Upper endoscopy reveals an erythematous and inflamed distal esophagus.
In severe reflux esophagitis, the resting pressure of the LES is decreased. This may be
physiologically increased by which of the following?
(A) Pregnancy
(B) Glucagon
(C) Gastrin
(D) Secretin
(E) Glucagon
Correct answer: C

32. A 25-year-old man arrives in the emergency department in respiratory distress


following a motor vehicle collision. A chest x-ray shows abdominal viscera in the left
thorax. What is the most likely diagnosis?
(A) Traumatic rupture of the diaphragm
(B) Sliding esophageal hernia
(C) Short esophagus with intrathoracic stomach
(D) Rupture of the esophagus
(E) Bochdalek hernia
Correct answer: A

- 169 -
33 . All of the following operations are done for gastro esophageal reflux disease
except -
A. Heller's cardiomyotomy
B. Belsey operation
C. Nissen fundoplication
D. Hill procedure
Correct answer: A

34 . Which of the following is the best test to diagnose gastroesophageal reflux


disease and quantify acid output ?
A. Manometry
B. 24hr pH monitoring
C. Esophagogram
D. Endoscopy
Correct answer: B

35 . Best to diagnose gastroesophageal reflux disease and quantify acid output is -


A. Manometry
B. 24 hour pH monitoring
C. Esohagogram
D. Endoscopy
Correct answer: B

36 . Which of the following is the most common cause of esophagitis?


A. Spicy and hot food
B. Esophageal reflux
C. Alcohol
D. Smoking
Correct answer: B

Cancer esophagus and Dysphagia

1- Barrett's esophagus is diagnosed by -


A. Squamous metaplasia
B. Intestinal metaplasia
C. Squamous dysplasia
D. Intestinal dysplasia
Correct answer: B

2 . Upper GI endoscopy and biopsy from lower esophagus in a 48 years old lady
with chronic heart burn shows presence of columnar epithelium with goblet cells.
The feature is most likely consistent with:
A. Dysplasia
B. Hyperplasia
C. Carcinoma in - situ

- 170 -
D. Metaplasia
Correct answer: D

3 . All of the following condition causing death in advanced carcinoma of


esophagus except-
A. Progressive cachexia and dehydration
B. Aspiration pneumonia due to TO fistula
C. Perforation leading to mediastinitis
D. Massive bleeding due to erosion of abdominal aorta
Correct answer: D

4 . Best substitute of esophagus after esophagectomy is


A. Stomach
B. Jejunum
C. Left colon
D. Right colon
Correct answer: A

5 . Most common site of CA oesophagus is -


A. Middle 1/3 rd
B. Upper 1/3 rd
C. Lower 1/3 rd
D. Lower end of esophagus
Correct answer: A

6 . The commonest site of carcinoma esophagus in India is:


A. Upper 1/3rd
B. Middle 1/3rd
C. Lower 1/3rd
D. Gastro Eosophageal junction
Correct answer: B
7 . Adenocarcinoma in oesophagous occurs in
A. Middle esophagus
B. Upper esophagus
C. Barret's ulcer
D. None of the above
Correct answer: C

8 . All except one are true regarding Barret's oesophagus -


A. More incidence of sq. cell Ca
B. Metaplasia
C. Columnar epithelium
D. Involves lower oesophagus
Correct answer: A

9 . True regarding Barret's oesophagus is -


A. Benign course
B. Premalignant condition
C. Sq. metaplasia of lower oesophagus
D. Medical The treatment is not useful

- 171 -
Correct answer: B

10 . Commonest benign tumor of oesophagus is:


A. Leiomyoma
B. Hemangioma
C. Papilloma
D. Lipoma
Correct answer: A

11 . Regarding Barret's esophagus all are true except:


A. Columnar metaplasia
B. Causes peptic ulcer
C. Causes squamous cell carcinoma
D. Causes reflux esophagitis
Correct answer: C

12 . Barret's esophagus can lead to -


A. Stricture
B. Reflux esophagitis
C. Peptic ulcer
D. Achalasia
Correct answer: A

13 . Which of the following is called Barrett's oesophagus?


A. Lower oesophagus lined by ciliated epithelium
B. Lower oesophagus lined by pseudostretifide epithelium
C. Lower oesophagus lined by columnar epithelium
D. Upper oesophagus lined by columnar epithelium
Correct answer: C

14 . Which of the following is the most common site for squamous cell ca
esophagus ?
A. Middle third
B. Upper third
C. Gastra-esophageal junction
D. Lower third
Correct answer: A

15 . After oesophagectomy, best substitute of oesophagus is -


A. Stomach
B. Jejunum
C. Left colon
D. Rt colon
Correct answer: A

16 . Site for oesophageal adenocarcinoma is -


A. Upper oesophagus
B. Middle oesophagus
C. Barret's oesophagus
D. None of the above

- 172 -
Correct answer: C

17 . In Barrett's esophagus lining is -


A. Squamous cell epithelium
B. Transitional cell epithelium
C. Secreting columnar cell epithelium
D. None of the above
Correct answer: C

18 . What is true regarding Barrett's esophagus -


A. Seen in females
B. Premalignant condition
C. Responds to conservative management
D. Squamous metaplasia is seen
Correct answer: B

19 . Which of the following is shown by Barrett's esophagus ?


A. Intestinal metaplasia
B. Intestinal dysplasia
C. Columnar cell metaplasia
D. Squamous cell metaplasia
Correct answer: A

20 . Laproscopic operation for gastrointestinal reflux should be considered in a


patient with any of the following situations except -
A. Barrett's esophagus
B. Daily proton pump inhibitors for symptomatic relief
C. Esophageal stricture
D. Significant Esophageal shortening
Correct answer: D

21 . Trans hiatal esophagectomy was planned for adenocarcinoma of lower end


of esophagus. The approach would be in the following order -
A. Neck - Thorax - Abdomen
B. Abdomen - Thorax
C. Abdomen - neck
D. Abdomen - Thorax - neck
Correct answer: C

22 . Which of the following is not a predisposing factor for Ca esophagus?


A. Human papilloma virus
B. Diverticula
C. Caustic ingestion
D. Mediastinal fibrosis
Correct answer: D

23 . Which of the following is not associated with increased incidence of


carcinoma esophagus
A. Congenital tylosis of esophagus
B. Achalasia cardia

- 173 -
C. Barret' s esophagus
D. All of the above
Correct answer: D

24 . Barret's esophagus is a type of -


A. Squamous cell carcinoma
B. Adenocarcinoma
C. Gastric epithelium in upper esophagus
D. Gastric type of epithelium in lower esophagus
Correct answer: D

25 . Adenocarcinoma of esophagus-developed in which of the bellow?


A. Corrosive structure
B. Alcohol abuse
C. Barret's esophagus
D. Long standing achalasia
Correct answer: C

26 . Barrett's esophagus is diagnosed by -


A. Intestinal metaplasia
B. Squamous metaplasia
C. Intestinal dysplasia
D. Squamous dysplasia
Correct answer: A

27 . Adenocarcinoma of esophagus is commonly found in:


A. Barret's oesophagus
B. Achlasia acardia
C. Chronic smoking
D. Plummer vinson syndrome
Correct answer: A

28 . Barrets esophagus is commonly associated with one of the following -


A. Sarcoma
B. Gastrointestinal stromal tumor
C. Adenocarcinoma
D. Squamous cell carcinoma
Correct answer: C

29. A 52-year-old gastroenterologist suffers from intermittent dysphagia attributed to


the presence of a lower esophageal stricture. The doctor’s condition is characterized
by which of the following?
(A) A full thickness scar in the upper esophagus
(B) Symptoms of mild-ta-moderate dysphagia
(C) A low incidence in men
(D) The absence of a sliding hiatal hernia in most case
(E) The need for antireflux surgery at an early stage
Correct answer: B

- 174 -
30. A46-year-old man has a long history of heartburn (GERD). His barium study
shows an irregular, ulcerated area in the lower third of his esophagus. There is marked
mucosal disruption and overhanging edges. What is the most likely diagnosis?
(A) Sliding hiatal hernia with GERD
(B) Paraesophageal hernia
(C) Benign esophageal stricture
(D) Squamous carcinoma of the esophagus
(E) Adenocarcinoma arising in a Barrett’s Esophagus
Correct answer: E

31. A 46-year-old man present with dysphagia of recent onset. His esophogram shows
a lesion in the lower third of his esophagus. Biopsy of the lesion shows
adenocarcinoma. His general medical condition is excellent, and his metastatic
workup is negative. What should his management involve?
(A) Chemotherapy
(B) Radiation therapy
(C) Insertion of a stent to improve swallowing
(D) Surgical resection of the esophagus
(E) Combination of chemotherapy and radiation therapy
Correct answer: D

8- Stomach and duodenum

Anatomy and congenital anomalies of the stomach

1 . Major blood supply of stomach -


A. Left gastric artery
B. Short gastric Artery
C. Right gastroepiploic Artery
D. Left inferior phrenic artery
Correct answer: C

2. A 64-year-old woman with arthritis is a chronic NSAID user. She develops severe
epigastric pain and undergoes an upper endoscopy. She is told that she has an ulcer
adjacent to the pylorus. Which of the following is TRUE about the pylorus?
(A) It cannot be palpated at laparotomy.
(B) It is not covered completely by omentum.
(C) It is a distinct anatomic entity that can be distinguished during laparotomy.
(D) It is a true physiologic sphincter.
(E) It is a site where cancer is rarely found
Correct answer: C

- 175 -
Peptic ulcer disease and complications

1- A 42 years old business executive presents with sudden upper GI bleed (5


liters ) of bright red blood, with no significant previous history. The diagnosis is:
A. Duodenal ulcer
B. Oesophageal varices
C. Gastric erosion
D. Gastritis
Correct answer: A

2 . What is the most common benign mesenchymal tumor of the stomach?


A. Polypoid adenoma
B. Leiomyoma
C. Glomus tumor
D. Lipoma
Correct answer: B

3 . What is probable diagnosis of a patient with spinal POP cast presenting with
bilious vomiting:
A. Acute dilation of stomach
B. Duodenal obstruction
C. Peritonitis
D. Acute pancreatitis
Correct answer: B

4 . Which of the following statements related to gastric injury is not true?


A. Mostly related to penetrating trauma
B. Treatment is simple debridement and suturing
C. Blood in stomach is always related to gastric injury
D. Heals well and fast
Correct answer: C

5 . Acute dilatation of stomach is not managed by -


A. N/G tube aspiration
B. Stop oral feeds
C. Fluid and electrolyte balance
D. Surgery
Correct answer: C

6 . Commonest site of peptic ulcer is:


A. Distal 1/3 of stomach
B. Pylorus of the stomach
C. Ist part of Duodenum
D. 2nd part of duodenum
Correct answer: C

7 . Electrical pacemaker of stomach is situated in


A. Fundus
B. Body
- 176 -
C. Incisura angularis
D. Gastroesophageal junction
Correct answer: A

8 . The following have strong casual associated with H. pylori infection except -
A. Chronic gastritis
B. Peptic ulcer disease
C. Gastric carcinoma
D. Gastric adenoma
Correct answer: D

9 . True statement regarding rolling hernia are all except:


A. Air fluid level behind the cardiac shadow on X ray in upright position
B. A barium meal is the best method of diagnosis
C. Endoscopy reveals gastric pouch lined with rugal folds
D. More common in women
Correct answer: C

10 . Which of the following is a side effect of vagotomy ?


A. Delayed gastric emptying
B. Gastric ulcer
C. Diarrhea
D. All of the above
Correct answer: D

11 . A 33 years male presents to the Emergency Department with symptoms of


epigastric pain radiating to back that wakes him up at night and is relieved by
consuming food. He gives history of similar pain in the past which was diagnosed
as perforated duodenal ulcer and treated with omental patch surgery on two
occasions. Pain before and after surgery has been controlled with proton pump
inhibitors and analgesics. The possible diagnosis is:
A. Atrophic Gastritis
B. Chronic pancreatitis
C. Duodenal Ulcer
D. Gastric Ulcer
Correct answer: C

12 . In gastric outlet obstruction in a peptic ulcer patient, which of the following is


the possible site of obstruction?
A. Duodenum
B. Antrum
C. Pyloric canal
D. Pylorus
Correct answer: A

13 . Bariatric surgical procedures include -


A. Biliopancreatic diversion
B. Ileal Transposition
C. Gastric Banding
D. Gastric Bypass

- 177 -
Correct answer: B

14 . Metabolic abnormality seen in gastric outlet obstruction is -


A. Hypochloremic, hypokalemic alkalosis
B. Hypochloremic, hypokalemic acidosis
C. Hypochloremic, hyperkalemic alkalosis
D. Hyperchloremic, hypokalemic acidosis
Correct answer: A

15 . A 33 years patient presented with massive hematemesis. A 2x2 cm ulcer was


visualized on upper GI endoscopy on the posterior aspect of first part of
duodenum. The Bleeding vessel was visualized but bleeding could not be
controlled endoscopically. Blood transfusion was done, and patient was planned
for surgery. His BP was recorded as 90/70 and pulse rate was 110 with a Hb of 9
gm% at the time of surgery. The best surgical management is:
A. Duodenotomy, with ligation of bleeding vessels with postoperative PPI
B. Antrectomy with ligation of left gastric artery
C. Duodenotomy, ligation of bleeding vessels, highly selective vagotomy
D. Duodenotomy, ligation of bleeding vessels, truncal vagotomy and pyloroplasty
Correct answer: A

16 . Which is not true regarding gastric outlet obstruction -?


A. Hypokalemia
B. Hypochloremia
C. Alkaline urine
D. Metabolic alkalosis
Correct answer: C

17 . Primarily restrictive operations for morbid obesity are A/E-


A. Switch duodenal operation
B. Vertical band gastroplasty
C. Laparoscopic adjustable gastric banding
D. Roux en Y operation
Correct answer: A

18 . The lowest recurrence of peptic ulcer is associated with which of the


following?
A. Vagotomy+drainage
B. Gastric resection
C. Highly selective vagotomy
D. Vagotomy+Antrectomy
Correct answer: D

19 . Most common complication of chronic gastric ulcer is -


A. Tea pot stomach
B. Scirrhous carcinoma (adenocarcinoma)
C. Perforation
D. Massive hematemesis
Correct answer: D

- 178 -
20 . Which of the following management procedures of acute upper
gastrointestinal bleed should possibly be avoided:
A. Intravenous vasopressin
B. Intravenous beta-blockers
C. Endoscopic sclerotherapy
D. Balloon tamponade
Correct answer: B

21 . True regarding congenital hypertrophic pyloric stenosis is-


A. Hypochloremic alkalosis
B. More common in girls
C. Most often manifests at birth
D. Hellers myotomy is the procedure of choice.
Correct answer: A

22. A 44-year-old woman is scheduled for gastric surgery. She has no comorbid disease.
The anesthesiologist has difficulty inserting the orotracheal tube. In between intubation
attempts he uses an ambu-bag to oxygenate the patient. The patient’s abdomen gets
distended and tympany is noted in the left upper quadrant. Suddenly the patient
becomes hypotensive. Which of the following can cause a vosogvagal response during
anesthesia?
(A) The gastric remnant following a distal gastrectomy
(B) Corrosive gastritis
(C) Pernicious anemia
(D) Gastric volvulus
(E) Acute gastric dilatation
Correct answer: E

23. A 40-year-old woman complains of heartburn located in the epigastric and


retrosternal areas. She also has symptoms of regurgitation. Endoscopy shows erythema
of the esophagus consistent with reflux esophagitis. The patient has tried conservative
measures, including PPls with no improvement in symptoms. Which of the following
is TRUE?
(A) Manometry does not add any additional information.
(B) The 24-hour pH test is no longer used.
(C) If endoscopy has been done, an esophagogram is unnecessary.
(D) Nissen fundoplication is the surgical treatment of choice.
(E) Toupet fundoplication is 360 curve
Correct answer: D

24. A50-year-old man is involved in a major motor vehicle collision and suffers
multiple traumas. He is admitted to the intensive care unit. After 2 days of hospital
admission he bleeds massively from the stomach. What is the probable cause?
(A) Gastric ulcer
(B) Duodenal ulcer
(C) Hiatal hernia
(D) Mallory-Weiss tear
(E) Erosive gastritis
Correct answer: E

- 179 -
25. A 65-year-old lawyer has an elective colon resection. On postoperative day number
five, the patient develops fever, leukocytosis, and increasing abdominal pain and
distension. An anastomotic leak is suspected. During the preparation for a CAT scan,
fresh blood and coffee grounds are seen in the nasogastric tube.
Acute stress gastritis is best diagnosed by?
(A) CAT scan
(B) UGI series
(C) Angiogram
(D) Capsule endoscopy
(E) Upper endoscopy
Correct answer: E

26. A73-year-old woman is admitted to the hospital with a mild UGI hemorrhage that
stopped spontaneously. She did not require transfusion. She had ingested large
amounts of aspirin in the past 4 months to relieve the pain caused by severe
rheumatoid arthritis. Endoscopy confirms the presence of a duodenal ulcer. A biopsy
is done. What is the next step in the management of a duodenal ulcer associated with a
positive biopsy for H. pylori?
(A) H2 blockers
(B) Bipolar electrocautery of the ulcer
(C) Triple therapy
(D) Photocoagulation
(E) Elective surgery
Correct answer: C

27. A52-year-old artist develops epigastric pain that is relieved by antacids. She also
complains that her stool has changed color and is black and tarry. What is the most
important cause of the entity presenting above other than H. pylori?
(A) Submucosal islet cells
(B) Hyperglycemia
(C) Diet
(D) Acid secretion
(E) Acute erosive gastritis
Correct answer: D

28. An elderly patient delayed seeking medical attention for his early satiety and
weight loss because he attributed these changes to aging. When he underwent upper
endoscopy a large mass was seen in the stomach. Which statement is TRUE regarding
gastric carcinoma?
(A) During resection, it is safe to leave cancer at the cut edges.
(B) The incidence is increased in patients with gastric ulcer disease.
(C) Draining lymph nodes should not be removed.
(D) It is caused by diverticulitis.
(E) It is associated with hyperchlorhydria.
Correct answer: B

29. A 45-year-old man complains of burning epigastric pain that wakes him up at
night. The pain is relieved by eating or using over-the counter antacids and H2
blockers. Diagnosis is best confirmed by which of the following?
(A) Urea breath test

- 180 -
(B) Serum gastrin levels
(C) Barium meal examination
(D) Upper endoscopy
(E) Upper endoscopy and biopsy
Correct answer: E

30. A44-year-old dentist was admitted to the hospital with a 1-day history of
hematemesis caused by a recurrent duodenal ulcer. He has shown considerable
improvement following operative treatment by a truncal vagotomy and pyloroplasty,
10 years prior to this incident. Which is TRUE of truncal vagotomy?
(A) It is performed exclusively via the thorax.
(B) It can be performed in the neck.
(C) If complete, it will result in increased acid secretion.
(D) It requires a gastric drainage procedure
(E) It has been abandoned as a method to treat ulcer disease
Correct answer: D

31. A42-year-old executive has refractory chronic duodenal ulcer disease. His
physician has suggested several surgical options. The patient has chosen a parietal
(highly selective) vagotomy instead of a truncal vagotomy and antrectomy because?
(A) It results in a lower incidence of ulcer recurrence.
(B) It benefits patients with antral ulcers the most.
(C) It reduces acid secretion to a greater extent.
(D) The complication rate is lower.
(E) It includes removal of the ulcer.
Correct answer: D

32. A63-year-old woman is admitted to the hospital with severe abdominal pain of 3-
hour duration. Abdominal examination reveals board-like rigidity, guarding, and
rebound tenderness. Her blood pressure is 90/50 mm Hg, pluse 110 bpm (beats per
minute), and respiratory rate is 30 breaths per minute. After a thorough history and
physical, and initiation of fluid resuscitation, what diagnostic study should be
performed?
(A) Supine abdominal x-rays
(B) Upright chest x-ray
(C) Gastrograffin swallow
(D) Computerized axial tomography (CAT) scan of the abdomen
(E) Abdominal sonogram
Correct answer: B

33. A frail elderly patient is found to have an anterior perforation of a duodenal ulcer.
He has a recent history of nonsteroidal anti-inflammatory drug (NSAID) use and no
previous history of peptic ulcer disease. A large amount of bilious fluid is found in the
abdomen. What should be the next step?
(A) Lavage of the peritoneal cavity alone
(B) Lavage and omental patch closure of the ulcer
(C) Total gastrectomy
(D) Lavage, vagotomy, and gastroenterostomy
(E) Laser of the ulcer
Correct answer: B

- 181 -
34. Three months after recovery from an operation to treat peptic ulcer disease, a
patient complains that she has difficulty eating a large meal. A 99m Tc-labeled
chicken scintigraphy test confirms a marked delay in gastric emptying. A delay in
gastric emptying may be due to which of the following?
(A) Zollinger-Ellison syndrome (ZES)
(B) Steatorrhea
(C) Massive small-bowel resection
(D) Previous vagotomy
(E) Hiatal hernia
Correct answer: D
35. A64-year-old supermarket manager had an elective operation for duodenal ulcer
disease. He has not returned to work because he has diarrhea with more than 20 bowel
movements per day. Medication has been ineffective. The exact details of his
operation cannot be ascertained. What operation was most likely performed?
(A) Antrectomy and Billroth I anastomosis
(B) Gastric surgery combined with choleystectomy
(C) Truncal vagotomy
(D) Highly selective vagotomy
(E) Selective vagotomy
Correct answer: C

36. A40-year-old man has had recurrent symptoms suggestive of peptic ulcer disease
for 4 years. Endoscopy reveals an ulcer located on the greater curvature of the
stomach. A mucosal biopsy reveals Helicobacter. pylori. What is TRUE
About H. pylori?
(A) Active organisms can be discerned by serology.
(B) It is protective against gastric carcinoma.
(C) It is associated with chronic gastritis.
(D) It causes gastric ulcer but not duodenal ulcer.
(E) It can be detected by the urea breath test in <60% of cases.
Correct answer: C

37. A 63-year-old man has an upper gastrointestinal (UGI) study as part of his workup
for abdominal pain. The only abnormal finding was in the antrum, where the mucosa
prolapsed into the duodenum. There were no abnormal findings on endoscopy. What
should he do?
(A) Sleep with his head elevated.
(B) Be placed on an H2 antagonist.
(C) Undergo surgical resection of the antrum.
(D) Be observed and treated for pain accordingly.
(E) Have laser treatment of the antral mucosa.
Correct answer: D

38. A63-year-old man underwent gastric resection for severe peptic ulcer disease. He
had complete relief of his symptoms but developed “dumping syndrome.” This patient
is most likely to complain of which of the following?
(A) Gastric intussusception
(B) Repeated vomiting
(C) Severe diarrhea

- 182 -
(D) Severe vasomotor symptoms after eating
(E) Intestinal obstruction
Correct answer: D

39. A 65-year-old man was admitted to the hospital for severe bilious vomiting
following gastric surgery. This occurs in which circumstance?
(A) Following ingestion of gaseous fluids
(B) Spontaneously
(C) Following ingestion of fatty foods
(D) Following ingestion of bulky meals
(E) In the evening
Correct answer: B

40. A 64-year-old man has had intermittent abdominal pain as a result of duodenal
ulcer disease for the past 6 years. Symptoms recurred 6 weeks before admission. He is
most likely to belong to which group?
(A) A and secretor (blood group antigen in body fluid)
(B) B and Lewis antigen
(C) AB
(D) O and nonsecretor
(E) O and secretor
Correct answer: D

41. A64-year-old man was evaluated for moderate protein deficiency. He underwent a
gastrectomy 20 years earlier. He is more likely to show which of the following?
(A) Porphyria
(B) Hemosiderosis
(C) Aplastic anemia
(D) Hemolytic anemia
(E) Iron deficiency anemia
Correct answer: E

42. A68-year-old woman has been diagnosed with a benign ulcer on the greater
curvature of her stomach, 5 cm proximal to the antrum. After 3 months of standard
medical therapy, she continues to have guaiac positive stool, anemia, and abdominal
pain with failure of the ulcer to heal. Biopsies of the gastric ulcer have not identified a
malignancy. The next step in management is which of the following?
(A) Treatment of the anemia and repeat all studies in 6 weeks
(B) Endoscopy and bipolar electrocautery or laser photocoagulation of the gastric
ulcer
(C) Admission of the patient for total parenteral nutrition (TPN), treatment of anemia,
and endoscopic therapy
(D) Surgical intervention, including partial gastric resection
(E) Surgical intervention, including total Gastrectomy
Correct answer: D

43. Over the past 6 months, a 60-year-old woman with long standing duodenal ulcer
disease has been complaining of anorexia, nausea, weight loss and repeated vomiting.
She recognizes undigested food in the vomitus. Examination and workup reveal

- 183 -
dehydration, hypokalemia, and hypochloremic alkalosis. What is the most likely
diagnosis?
(A) Carcinoma of the fundus
(B) Penetrating ulcer
(C) Pyloric obstruction due to cicatricial stenosis of the lumen of the duodenum
(D) ZES (Zollinger Ellison Syndrome)
(E) Anorexia nervosa
Correct answer: C

44. A 50-year-old woman presents with duodenal ulcer disease and high basal acid
secretory outputs. Secretin stimulated serum gastrin levels are in excess of 1000
pg/mL. She has a long history of ulcer disease that has not responded to intense
medical therapy. What is the most likely diagnosis?
(A) Hyperparathyroidism
(B) Pernicious anemia
(C) Renal failure
(D) ZES
(E) Multiple endocrine neoplasia
Correct answer: D

45. A50-year-old man presents with vague gastric complaints. Findings on physical
examination are unremarkable. The serum albumin level is markedly reduced (1.8
g/100 mL). A barium study of the stomach shows massive gastric folds within the
proximal stomach. These findings are confirmed by endoscopy. What is the correct
diagnosis?
(A) Hypertrophic pyloric stenosis
(B) Gallstone ileus
(C) Mallory-Weiss tear
(D) Hypertrophic gastritis
(E) Crohn’s disease
Correct answer: D

46. A 2-cm ulcer on the greater curvature of the stomach is diagnosed in a 70-year-old
woman by a barium study. Gastric analysis to maximal acid stimulation shows
achlorhydria. What is the next step in management?
(A) Antacids, H2 blockers, and repeat barium study in 6 to 8 weeks
(B) Proton pump inhibitor (PPI) (e.g., omeprazole) and repeat barium study in 6 to 8
weeks
(C) Prostoglandin E (misoprostol) and repeat barium study in 6 to 8 weeks
(D) Immediate elective surgery
(E) Upper endoscopy with multiple biopsies (at least 8 or 9) for the ulcer
Correct answer: E

47. A55-year-old school bus driver was diagnosed 3 months ago with an antral ulcer.
He was treated for H. pylori and continues to take a PPI. Repeat endoscopy
demonstrates that the ulcer has not healed. What is the next treatment option?
(A) Treatment with H2 blockers
(B) Vagotomy alone without additional surgery
(C) Endoscopy and laser treatment of the ulcer
(D) Distal gastrectomy with gastroduodenal anastomosis (Billroth I)

- 184 -
(E) Elevating the head of the bed when asleep
Correct answer: D

48. A 55-year-old man complains of anorexia, weight loss, and fatigue. AUGI study
demonstrates an ulcerated lesion at the incisura. Where is the incisura?
(A) Cardia
(B) Fundus
(C) Greater curvature
(D) Lesser curvature
(E) Gastrocolic ligament
Correct answer: D

49. A 64-year-old woman presents with severe upper abdominal pain and retching of
1-day duration. Attempts to pass a nasogastric tube are unsuccessful. X-rays show an
air-fluid level in the left side of the chest in the posterior mediastinum. An
incarcerated paraesophageal hernia and gastric volvulus is diagnosed. What is the next
step in management?
(A) Insertion of a weighted bougie to untwist the volvulus
(B) Elevation of the head of the bed
(C) Placing the patient in the Trendelenburg position with the head of the bed lowered
(D) Laparotomy and vagotomy
(E) Surgery, reduction of the gastric volvulus, and repair of the hernia
Correct answer: E

50. A 48-year-old man undergoes surgery for a chronic duodenal ulcer. The procedure
is a truncal vagotomy and which of the following?
(A) Gastroenterostomy
(B) Removal of the duodenum
(C) Closure of the esophageal hiatus
(D) Incidental appendectomy
(E) No further procedure
Correct answer: A

51. A healthy 75-year-old man bleeds from a duodenal ulcer. Medical management
and endoscopic measures fail to stop the bleeding. What is the next step in
management?
(A) Continued transfusion of 8 U of blood
(B) Administration of norepinephrine
(C) Oversewing of the bleeding point
(D) Oversewing of the bleeding point, vagotomy and pyloroplasty
(E) Hepatic artery ligation
Correct answer: D

52 . The commonest cause of massive upper gastrointestinal bleeding is -


A. Esophagial varices
B. Erosive gastritis
C. Gastric carcinoma
D. Peptic ulcer
Correct answer: D

- 185 -
Cancer stomach

1 . For early diagnosis of Ca stomach which method is used -


A. Endoscopy
B. Staining with endoscopy biopsy
C. Physical examination
D. Ultrasound abdomen
Correct answer: B

2 . Which of the following is the most common cause of gastric outlet obstruction
in India?
A. Cancer of stomach
B. Tuberculosis
C. Peptic ulcer disease
D. Duodenal lymphoma
Correct answer: A

3 . A 52 years male presents with obstructive symptoms. Biopsy of stomach


reveals Gastrointestinal stromal tumor (GIST). Appropriate marker for GIST is
A. CD 117
B. CD 34
C. CD 10
D. CD 30
Correct answer: A

4 . In which of the following, Sister Mary Joseph nodule is commonly seen?


A. Stomach cancer
B. Ovarian cancer
C. Pancreatic cancer
D. Colon cancer
Correct answer: A

5 . Most common site for carcinoma stomach is -


A. Antrum
B. Fundus
C. Lesser curvature
D. Greater curvature
Correct answer: A

6. A 55 year male presents with obstructive symptoms. Biopsy of stomach reveals


Gastrointestinal stromal tumor (GIST). Appropriate maker is:
A. CD 117
B. CD 34
C. CD 10
D. CD 30
Correct answer: A

- 186 -
7 . A 60 year old male diagnosed to have carcinoma stomach. CT scan of abdomen
showed a mass measuring 4 X 4 cm in the antrum with involvement of celiac nodes
and right gastric nodes. Management of choice is:
A. Total gastrectomy
B. Subtotal gastrectomy
C. Palliative
D. Chemotherapy
Correct answer: B

8 . The best prognosis in carcinoma stomach is with


A. Superficial spreading type
B. Ulcerative type
C. Linitis plastica type
D. Polypoidal fungating type
Correct answer: A

9 . In which of the following, Sister Mary Joseph nodule is most commonly seen?
A. Stomach cancer
B. Ovarian cancer
C. Pancreatic cancer
D. Colon cancer
Correct answer: A

10 . Commonest site for lymphoma in GIT is -


A. Stomach
B. Duodenum
C. Ileum
D. Rectum
Correct answer: A

11 . Zollinger Ellison syndrome is characterized by -


A. Fulminating ulcer in stomach and duodenum
B. Recurrent ulceration despite adequate therapy
C. Non-beta islet cell tumors of the pancreas
D. All of the above
Correct answer: D

12 . Acidosis with hypokalemia present in


A. Stomach
B. Ileum
C. Duodenum
D. Colon
Correct answer: D

13 . Which vitamin deficiency is found in gastric cancer patients ?


A. Vitamin C
B. Vitamin B12
C. Vitamin A
D. Vitamin D
Correct answer: B

- 187 -
14 . All of the following are indications for surgery in gastric lymphomas except
A. Bleeding
B. Perforation
C. Residual disease following chemotherapy
D. Intractable pain
Correct answer: D

15 . All are features of early gastric carcinoma except -


A. Mucosal involvement
B. Submucosal involvement
C. Muscularis propria involvement
D. Dysplasia with Ca in situ
Correct answer: C

16 . Best Diagnosis for Ectopic gastric mucosa of meckels diverticulum:


A. Fluoroscopy
B. Occult blood test in stool
C. Ultrasound abdomen
D. Radionuclide scan
Correct answer: D

17 . True statement about Gastric carcinoma is:


A. Often associated with Hypochlorhydria/Achlorhydria
B. Squamous cell carcinoma is the most common histological subtype
C. Highly Radiosensitive tumor
D. Occult blood in stool is not seen
Correct answer: A

18 . Gastric carcinoma is associated with all except


A. Gastric volvulus may be associated
B. Migratory thrombophlebitis
C. Trousseau's sign
D. Linitis plastica
Correct answer: A

19 . For inoperable gastric neoplasms involving the cardia, the best palliation is
by -
A. Intubation
B. Exclusion
C. Presternal bypass
D. Gastroenterostomy
Correct answer: A

20 . Which of the following is a risk factor for development of gastric Ca?


A. Intestinal hyperplasia
B. Intestinal metaplasia type III
C. Blood group O
D. Duodenal ulcer
Correct answer: B

- 188 -
21 . Investigation of choice for liver metastasis in gastric carcinoma is ?
A. US
B. CT
C. MRI
D. HIDA scan
Correct answer: B

22 . All of the following are indications for surgery in gastric lymphoma except -
A. Bleeding
B. Perforation
C. Residual disease following chemotherapy
D. Intractable pain
Correct answer: D

23 . Most common site for gastric cancer is:


A. Upper 1/3rd
B. Antrum
C. Pylorus
D. Body
Correct answer: B

24 . False about gastric lymphoma is -


A. Associate with H. pylori infection
B. Stomach is the most common site
C. 5 years survival rate after treatment is 60%
D. Total gastrectomy with adjuvant chemotherapy is the treatment of choice
Correct answer: D

25 . Indication of early gastric cancer is/are ….


A. Involvement of mucosa and submucosa
B. Involvement of mucosa
C. Involvement of mucosa, submucosa and adjacent lymph nodes
D. Involvement of mucosa, submucosa and muscularis
Correct answer: D

26. A 60-year-old man has been having vague symptoms of upper abdominal
discomfort, early satiety, and fatigue. He is referred to a gastroenterologist, who
performs an upper endoscopy. Although a discrete mass is not visualized, the stomach
looks abnormal. It does not distend easily with insufflation. A biopsy shows signet ring
cells. Which of the following is TRUE?
(A) Signet ring cells are typically found in intestinal type gastric adenocarcinoma.
(B) Signet ring cell cancer is the most common type of gastric cancer.
(C) “Leather bottle stomach” is a term used to describe a nondistensible stomach
infiltrated by cancer.
(D) The gross appearance of the stomach always shows classic findings of linitus
plastica.
(E) Linitus plastica has an excellent prognosis.
Correct answer: C

- 189 -
27. A 26-year-old man is diagnosed with adenocarcinoma of the stomach. He wants to
know what could have caused him to develop this condition. He does an internet search.
Which of the following is a risk factor for developing gastric cancer?
(A) Exposure to ionizing radiation
(B) Blood group B
(C) A diet high in fiber
(D) H. pylori infection
(E) North American descent
Correct answer: D

28. A 30-year-old executive learns that he has a duodenal ulcer. His gastroenterologist
prescribes and outlines medical therapy. The patient worries that if medical therapy
fails he may need surgery. Which of the following is the best indication for elective
surgical therapy for duodenal ulcer disease?
(A) An episode of melena
(B) Repeated episodes of pain
(C) Pyloric outlet obstruction due to scar formation from an ulcer
(D) Frequent recurrences of ulcer disease
(E) Referral of pain to the back, suggestive of pancreatic penetration
Correct answer: C

29. A 36-year-old man presents with weight loss and a large palpable tumor in the
upper abdomen. Endoscopy reveals an intact gastric mucosa without signs of
carcinoma. Multiple biopsies show normal gastric mucosa. A UGI study shows a
mass in the stomach. At surgery, a 3-kg mass is removed. It is necessary to remove
the left side of the transverse colon. What is the most likely diagnosis?
(A) Gastric cancer
(B) Gastrointestinal stromal tumor (GIST)
(C) Choledochoduodenal fistula
(D) Eosinophilic gastroenteritis
(E) Linitis plastica
Correct answer: B

30. A 74-year-old man presents with anorexia and self-limited hematemesis. During
endoscopy a mass is discovered and a biopsy is done. A hematopathologist diagnoses
non-Hodgkin’s lymphoma. What is the recommended therapy?
(A) Chemotherapy alone
(B) Immunotherapy
(C) Radiation and chemotherapy
(D) Surgery, radiation, and chemotherapy
(E) Surgery alone
Correct answer: C

31. A63-year-old woman is admitted to the hospital with a UGI bleeding that subsides
spontaneously within a short time after admission. A barium study shows a gastric
ulceration that is described by the radiologist as having a “doughnut sign.” What is the
most likely diagnosis?
(A) Lipoma
(B) Gastric ulcer
(C) Ectopic pancreas

- 190 -
(D) GIST
(E) Carcinoma
Correct answer: D

32. A50-year-old woman is diagnosed with multiple hyperplastic polyps in the


stomach during endoscopy and biopsy. How are these best treated?
(A) Total gastrectomy
(B) Partial gastrectomy
(C) Staged endoscopic removal after brushing for cytologic examination
(D) Ablation by laser
(E) No treatment other than repeated endoscopy and multiple brush biopsies
Correct answer: C

33. A 60-year-old woman complains of early satiety and undergoes an upper


endoscopy. A small mass is seen in the antrum with sparing of the mucosa. GIST is
suspected. a CT scan of the chest, abdomen, and pelvis is performed.
What does she require next?
(A) Fulguration of the tumor
(B) Distal gastrectomy
(C) Laser therapy followed by radiation therapy
(D) Chemotherapy alone
(E) Total gastrectomy
Correct answer: B

34. A 67-year-old woman complains of paresthesias in the limbs. Examination shows


loss of vibratory sense, positional sense, and sense of light touch in the lower limbs.
She is found to have pernicious anemia. Endoscopy reveals an ulcer in the body of the
stomach. What does she most likely have?
(A) Excess of vitamin B12
(B) Deficiency of vitamin K
(C) Cancer of the stomach
(D) Gastric sarcoma
(E) Esophageal varices
Correct answer: C

35 . All of the following are true about GIST, except


A. PET is used to assess response to therapy
B. Well circumscribed
C. Most common in duodenum
D. Necrosis and ulceration present
Correct answer: C

36 . Which of the following is the Gold standard test for recurrent gastrointestinal
stromal tumor ?
A. USG
B. PET CT
C. MRI
D. MIBG
Correct answer: B

- 191 -
9- Liver and portal vein, spleen:

Segmental anatomy of the liver

1. Following an emergency operation for hepatic and splenic trauma, the surgeon
inserts a finger into the foramen of Winslow in an attempt to stop the bleeding. Which
is TRUE of the hepatic artery?
(A) It is called the common hepatic artery at this level.
(B) It is medial to the common bile duct and anterior to the portal vein.
(C) It is posterior to the portal vein.
(D) It is posterior to the inferior vena cava.
(E) It forms the superior margin of the epiploic foramen.
Correct answer: B

2 . Most common ' Nodule' of liver is -


A. Hepatic adenoma
B. Cholangioadenoma
C. Hemangioma
D. Hamartoma
Correct answer: C

3 . Which of the following is not a liver capsular plate ?


A. Portal plate
B. Hilar plate
C. Umbilical plate
D. Cystic plate
Correct answer: A

4 . Vascular inflow occlusion of the liver is by -


A. Clamping the hepatic artery
B. Occluding the portal vein
C. Clamping the hepatic veins
D. The Pringle maneuver
Correct answer: D

5 . Extended Criteria for Liver Donation include A/E:


A. HBsAg Positive donors
B. Donor age > 70 years
C. Donor after cardiac Death
D. Mild Hepatic steatosis
Correct answer: A

- 192 -
6 . True about fibrolamellar carcinoma of the liver,except:
A. Better prognosis than HCC
B. More common in females
C. Occur in younger individuals
D. AFP levels always greater than >1000
Correct answer: D

7 . In orthotropic liver transplantation, which is the best way to get bile drainage in
donor liver?
A. Donor bile duct with jejunum of recipient
B. External drainage for few days followed by choledochojejunostomy
C. Donor bile duct with recipient bile duct or Roux en Y choledochojejunostomy
D. Donor bile duct with duodenum of recipient
Correct answer: C

Cysts, abscesses and tumors of the liver

1. The most common cause of pyogenic liver abscess in children today is which
of the following?
a. Perforated appendicitis
b. Blunt liver injury
c. Immunocompromised host
d. Percutaneous liver biopsy
e. Omphalitis
Correct answer: c

2. Which of the following is the most common liver tumor of childhood?


a. Hemangioma and hemangioendothelioma
b. Hepatoblastoma
c. Hepatocellular carcinoma
d. Mesenchymal hamaratoma
Correct answer: b

3. Hepatoblastomas are childhood liver tumors characterized by which of the


following features?
a. Multicentricity
b. Cirrhosis in the uninvolved liver
c. Unresectable tumors subjected to cytoreductive chemotherapy may be resected with
long-term survival
d. Jaundice
Correct answer: c

4 . Expanded Criteria for Liver donation include A/E-


A. Age of donor may be more than 70 years
B. Can be taken from individual with mild hepatic steatosis
C. Taken from deceased individual after brain death

- 193 -
D. Hepatitis B serology positive
Correct answer: C

5. A 55-year-old man has had previous hemicolectomy for a carcinoma of the right
colon. At this time, 3 years after the primary resection, a CT scan shows a solitary
lesion in the right lobe of the liver. What is the next step in management?
(A) Laser cauterization
(B) Radiotherapy
(C) Hepatic artery catheterization and local chemotherapy
(D) Symptomatic treatment with analgesics, because the colon disease is now stage IV
(E) Exploratory laparotomy and resection of the tumor.
Correct answer: E

6 . After undergoing surgery, for carcinoma of colon a patients developed single


liver metastasis of 2cm. Next line of management-
A. Resection
B. Chemoradiation
C. Acetic acid injection
D. Radiofrequency ablation
Correct answer: A

7 . Surgeon excises a portion of liver to the left of the attachment of the falciform
ligament. Which of the following segments may be resected?
A. Segment 1 and 4b
B. Segment 1a and 4
C. Segment 1 and 3
D. Segment 2 and 3
Correct answer: D

8 . All the following cancers commonly metastasize to the liver except -


A. Breast
B. Prostate
C. Colon
D. Pancreas
Correct answer: B

9 . Commonest cause for mortality in Liver Levis operations -


A. Pulmonary atelectasis
B. Anastomotic leak
C. Thoracic duct fistula
D. Subdiaphragmatic collection
Correct answer: B

10. Indications of alpha feta-protein measuring include:


a. Sudden deterioration of hepatic patient.
b. Focal lesion by abdominal US.
c. Follow up after surgical removal of tumour.
d. All of the above.
Correct answer: D

- 194 -
11. Mortality from rupture liver is about:
a. 5%.
b. 20%.
c. 4O%.
d. 50%.
Correct answer: B

12.Treatment of traumatic injury to the liver includes all except:


a. Resection debridement of devitalized tissues
b. Ligation of exposed blood vessels and bile ducts
c. Pocking with gauze.
d. Pringle's manoeuvre
e. Partial hepatectomy
Correct answer: C

13 .The following is not a common complication(s) of hydatidosis :


a. Anaphylactic shock.
b. Obstructive jaundice.
c. Malignant transformation of surrounding tissue
d. Multi-system spread.
Correct answer: C

14.The most common presentation of hydatids is :


a. Asymptomatic.
b. Obstructive jaundice.
c. Anaphylactic shock.
d. Cough and haemoptysis.
Correct answer: A

15 .As regard treatment of hydatids the best is:


a. Follow up.
b. Enucleation.
c. Removal en block.
d. Segmentectomy.
Correct answer: B

16. High fever in amoebic hepatitis indicates:


a. Amoebic liver abscess.
b. Systemic spread.
c. Secondary infection in amoebic liver abscess.
d. All of the above.
Correct answer: C

17. The most common type of bilharzia periportal fibrosis is:


a. Fine.
b. Coarse.
c. Mixed.
Correct answer: C

- 195 -
18. Stage Ill of hepatic bilharziasis means:
a. Hepatomegaly.
b. Hepatosplenomegaly.
c. Splenomegaly with shrunken liver.
d. None of the above.
Correct answer: C

19. The most common complication of hepatic hydatid disease is :


a. Jaundice
b. Rupture into peritoneal cavity
c. Suppuration
d. Rupture into biliary channel
Correct answer: D

20. Rupture of amoebic liver abscess occurs most often ln :


a. Peritoneal cavity
b. Pleural cavity
c. Pericardial cavity
d. Duodenum
e. Colon
Correct answer: B

21. Premalignant syndrome may present by the following except:


a. Polycythaemia.
b. Fever of unknown aetiology.
c. Hypocalcaemia.
d. Hypertension and diabetes.
e. Haemoptysis.
Correct answer: C

22. Angiography for detection of hepatoma should be done through:


a. Portal vein.
b. Hepatic artery.
c. Retrograde through hepatic veins.
d. Any of the above.
Correct answer: B

23. Most common predisposing factor for hepatoma is :


a.Hepatitis B .
b. Hepatitis C.
c. Chronic irritation.
d. Portal hypertension.
Correct answer: C

24. Tumour marker in fibrolamellar hepatoma is :


a. Alpha feta-protein.
b. Carcina-embryonic Ag.
c. CA 19-9 .
d. Carboxythrombin .
e. None of the above.

- 196 -
Correct answer: D

25. Which of the following statements(s) is/are true about benign lesions of the
liver:
a. Adenomas are true neoplasms with o predisposition for complications
and should usually be resected.
b. Focal nodular hyperplasia (FNH) is a neoplasm related to oral
contraceptive pills and usually requires resection.
c. Hemangiomas are the most common benign lesions of the liver that
come to the surgeon's attention.
d. Nodular regenerative hyperplasia does not usually accompany cirrhosis.
Correct answer: A

26. Rupture of on amoebic liver abscess occurs most often into the:
a. Peritoneal cavity.
b. Pleural cavity.
c. Pericardial cavity.
d. Duodenum.
e. Colon.
Correct answer:B

27. The most common malignant tumours of the Iiver are:


a. Hepatomas.
b. Cholangiomas.
c. Angiosarcomas.
d. Metastatic deposits.
e. Lymphomas.
Correct answer: D

28. The most appropriate treatment for amoebic Iiver abscess is by:
a. Emetine hydrochloride.
b. Metronidazole.
c. Aspiration.
d. Open drainage.
e. Excision.
Correct answer:.B

29. A serum bilirubin of 2-3 mg % conforms to Child's criteria of chronic liver


disease :
a. Class A.
b. class B.
c. class C.
d. All of the above.
Correct answer: B

30. Which of the following procedures is associated with least risk of hepatic
encephalopathy:
a. Mesocaval shunt .
b. Proximol splenorenal shunt.
c. Distal splenorenol shunt' (Warren shunt).

- 197 -
d. Side to side portocaval shunt.
Correct answer: C

31. A small cirrhotic liver with grossly enlarged caudate lobe demands
exclusion of:
a. Portal vein thrombosis.
b. Budd-Chiari syndrome.
c. Hepatoma .
d. Primary sclerosing cholangitis
Correct answer: B

32 .The most feared complication of Denver shunt is:


a. infection.
b. Malfunction.
c. DC.
d. Rupture.
Correct answer: A

33 . Alpha fetoproteins are a marker of -


A. Secondaries in liver
B. Cholangiocarcinoma
C. Hepatoma
D. Both B and C
Correct answer: C

34 . The commonest cause of death in patients’ alcoholic cirrhosis following


portosystemic shunting is -
A. Bleeding esophageal varices
B. Hepatic failure with encephalopathy
C. Malnutrition
D. Hepatocellular carcinoma
Correct answer: B

35 . General dot sign is seen in -


A. Liver Hamartoma
B. Primary sclerosing cholangitis
C. Polycystic liver disease
D. Caroli's disease
Correct answer: D

36 . The most common benign tumour of the liver is


A. Adenoma
B. Hemangioma
C. Leiomyoma
D. Lymphangioma
Correct answer: B

- 198 -
Portal hypertension

1. Regarding portal vein thrombosis, the untrue statement is:


a. Occur in patient with thrombocytopenia
b. Occurs after severe appendicitis
c. Is a cause of mesenteric bowel ischemia
d. Cause splenomegaly
e. Cause portal hypertension
Correct answer: A

2 . A 40 year male presents with hematemesis. On examination his BP was 90/60


mmHg and Heart rate was 120/min. Splenomegaly was also present. The most
probable cause of his bleeding is –
A. Portal Hypertension
B. Gastric ulcer
C. Duodenal ulcer
D. Drug-induced GI injury
Correct answer: A

3. Which of the following complications of portal hypertension often require


surgical intervention (for more than 25% of patients)?
a. Hypersplenism.
b. Varceal haemorrhage.
c. Ascites.
d. Encephalopathy.
Correct answer: B

4. Normal portal venous pressure is:


a. 5-7 mmHg.
b. 8-12 mmHg.
c. l0-15 mmHg.
d. 15-20 mmHg.
Correct answer: B

5. Which of the following treatments most effectively preserves hepatic


portal perfusion?
a. Distal splenorenal shunt.
b. Conventional splenorenal shunt.
c. Endoscopic sclerotherapy.
d. Side-ta-side portocaval shunt.
Correct answer: A

6 . On barium swallow the grade IV esophageal varices appear as:


A. Mucosal folds above the carina
B. Mucosal folds below the carina
C. Mucosal folds at the carina
D. A thick band
Correct answer: B

- 199 -
7 . Which of the following statements is not true regarding chylous ascites
A. Can be seen in abdominal lymphoma
B. Can occur following abdominal surgery
C. Low fat diet is useful in controlling chylous ascites
D. Ascitic fluid contains no lymphocytes
Correct answer: D

Upper gastrointestinal bleeding

1 . A patient comes with hematemesis and melena. On the upper GI endoscopy


there was no significant finding. 2-days after the patient rebleeds. Next line of
investigation is -
A. Enteroscopy
B. Laprotomy
C. Emergency angiography
D. Repeat upper GI endoscopy
Correct answer: B

2. An appropriate initial therapy for diagnosed bleeding oesophageal varices is:


a. lV vasopressin
b. Endoscopic sclerotherapy .
c. Emergency portocaval shunt .
d. Emergency esophogeol transection
e. Esophogeal balloon tamponed
Correct answer: A

3. The wrong statement about gastro-esophogeol bleeding is:


a. Bleeding may often be severe to cause collapse.
b. Endoscopic sclerotherapy con arrest bleeding.
c. Prophylactic sclerotherapy in GE varices obviates chance of bleeding and
prolongs survival.
d. Tamponed by sungestaken tube is effective.
Correct answer: C

4. Varceal bleeding not responding to drug and sclerotherapy is treated by:


a.Embolisation .
b.Surgical ligation.
c.TIPSS.
d.Liver transplant.
Correct answer: C

- 200 -
Splenomegaly

1- Which of the following statements regarding splenic function in humans are


true?
a. The specific immune function of the spleen is principally related to its antigen
processing role
b. The spleen is the major site of synthesis of complement pathway proteins
c. The spleen is more efficient than the liver at removing bacteria with a high density
of surface opsonin.
d. The spleen serves as a principal source of nonspecific opsonin.
Correct answer: a, d

2 . Splenectomy is most useful in


A. Sickle cell anemia
B. Thalassemia
C. Hereditary spherocytosis
D. Acquired auto ammonia hemolytic anemia
Correct answer: C

3 . In splenic injury conservative m/n is done in:


A. Hemodynamically unstable
B. Young patient
C. Shattered spleen
D. Extreme pallor and hypotension
Correct answer: B

4 . Kehr sign seen in splenic injury is


A. Pain over left shoulder
B. Pain over right shoulder
C. Pain over renal angle
D. Pain over pera-umbilical region
Correct answer: A

5 . Kehr's sign is present in -


A. Diaphragmatic rupture
B. Splenic injury
C. Liver abscess
D. Cholelithiasis
Correct answer: B

4. Which of the following is the best initial screening test for thrombocytapenia
A.bleeding time
B.clotting time
C.platelet count
Correct answer A

- 201 -
5. A 12-year-old boy who underwent a previous splenectomy for thalassemia
presents to the emergency room with fever, chills, and septic shock. The parents
give a history of seemingly minor sore throat, which started only a few hours
previously. The child is hypotensive and appears moribund. A diagnosis of
overwhelming post splenectomy infection (OPSI) is made.
Which of the following statements about OPSIis TRUE?
(A) The condition is more common in children.
(B) The condition is more common after splenectomy for trauma.
(C) Prophylactic antibiotics have not been shown to improve outcome in children.
(D) Prophylactic vaccination against Enterococcus should be performed.
(E) The condition is very common after splenectomy.
Correct answer A.

6. A28-year-old woman is diagnosed with TTP. In addition to purpura and


thrombocytopenia, studies will show which of the following?
(A) Normal arterioles on biopsy of the spleen
(B) Absence of infarction on biopsy of the spleen
(C) Leukopenia
(D) Elevated urea and creatinine levels
(E) Suppression of reticulocytes
Correct answer D.

7. Splenectomy is often indicated in the management of which of the following?


(A) Hereditary spherocytosis
(B) Hereditary neurofibromatosis
(C) Aplastic anemia
(D) Pheochromocytoma
(E) Hashimoto’s disease
Correct answer A.

8. As the functional anatomy of the spleen is divided into red pulp, white
pulp, and marginal zone, what function is incorporated into the anatomy of
the cortical zone that relates to infection control?
A. Filtration of red cells, encapsulated bacteria, and other foreign material.
B. Red pulp for formation of red cells.
C. White pulp for its role in formation of granulocytes.
D. Gray areas, so formed because of the production of platelets.
E. Fibrous trabeculae.
Correct answer: A

9. During the evolution of the understanding of hematologic diseases, the


indications for splenectomy have changed. The most common indications for
splenectomy are, in descending order of frequency:
A. Traumatic injury, immune thrombocytopenia, hypersplenism.
B. Immune thrombocytopenic purpura, traumatic injury, hypersplenism.
C. Hypersplenism, traumatic injury, immune thrombocytopenia.
D. Immune thrombocytopenia, hypersplenism, traumatic injury.
E. Non e of the above.
Correct answer: A

- 202 -
10. Useful methods for detection of splenic in jury, in descending order of
sensitivity, are:
A. Diagnostic peritoneal lavage.
B. CT.
C. Ultrasonography.
D. Isotope scan.
E. Magnetic resonance imaging (MRI).
Correct answer: B

11. The following comments about immune thrombocytopenic purpura (ITP) are
accurate:
A. Platelet count is low.
B. Circulating antiplatelet factor is present.
C. Antiplatelet factor is immunoglobulin G (IgG) antibody.
D. Purpura is directed against a platelet-associated antigen.
E. May be fatal.
F. All of the above.
Correct answer: A

12. ITP:
A. Is most common in men in their 20s.
B. Is frequently cured in adults by corticosteroid administration.
C. Usually requires splenectomy in children.
D. Is most common in the sixth decade of life.
E. Is in remission in more than 80% of patients with splenectomy.
Correct answer: E

13. Splenectomy and perioperative therapy for ITP:


A. Follow successful steroid therapy.
B. Respond permanently to high-dose intravenous gamma globulin.
C. Are best preceded by polyvalent vaccines for Pneumococcus, Haemophilus
influenzae, and Neisseria meningitidis.
D. Cannot be done laparoscopically.
E. Are associated with splenomegaly.
Correct answer: C

14. Thrombotic thrombocytopenic purpura (TTP) is a syndrome characterized


by all of the following except:
A. Thrombocytopenia.
B. Microangiopathic hemolytic anemia.
C. Deposition of platelet microthrombi.
D. Fluctuating neurologic abnormalities.
E. Renal failure.
F. Afebrile.
Correct answer: F

15. Which of the following comments does not describe hypersplenism?


A. It may occur without underlying disease identification.
B. It may be secondary to man y hematologic illnesses.
C. It is associated with work hypertrophy from immune response.

- 203 -
D. It requires evaluation of the myelo proliferation.
E. It is associated with antibodies against platelets.
Correct answer: E

16. Hyposplenism is a potentially lethal syndrome. Which of the following


statements is incorrect?
A. It is confirmed by isotope scan.
B. It is always associated with an atrophic spleen.
C. It may be associated with overwhelming post-splenectomy sepsis syndrome
(OPSS).
D. It is associated with thyrotoxicosis, corticosteroid administration, and some
contrast agents.
E. It may be associated with ulcerative colitis or sickle cell anemia.
Correct answer: B

17. Hodgkin's disease is a malignant lymphoma with four histologic


subtypes. Which of the following is not one of the subtypes?
A. Lymphocyte predominance.
B. Nodular sclerosis.
C. Mixed cellularity.
D. Lymphocyte depletion.
E. Leukocyte-lymphocyte dominance.
Correct answer: E

18. Which of the following statements about lymphatic capillaries are true?
A. These vessels have delicate tricuspid valves every 2 to 3 mm.
B. Lymphatic capillaries are more permeable than blood capillaries.
C. Lymphatic capillaries are less permeable than blood capillaries.
D. Lymphatic capillaries contain gaps large enough to admit particles as large as
lymphocytes.
Correct answer: C,D

19. Which of the following forces do not promote the formation of interstitial
fluid?
A. Increased venous pressure.
B. Constrictive pericarditis.
C. Hypernatremia.
D. Hypoproteinemia.
Correct answer: C

20. The two primary causes of death from sickle cell disease in the first decade of
life are which of the following?
a. Sepsis
b. Splenic sequestration crisis
c. Acute chest syndrome
d. Heart failure
Correct answer: a, b

- 204 -
21. The best therapy for a patient with thrombotic thrombocytopenic purpura is
which of the following:
a. Plasmapheresis
b. Corticosteroids
c. Splenectomy
d. Intravenous immune globulin
Correct answer: a

22. Which of the following statements regarding post splenectomy sepsis are
true, except?
a. The incidence in children is generally reported as less than 5%
b. Hemophilus influenzae, Streptococcus pneumoniae and Neisseria meningitidis
are the most common causative organisms.
c. Auto transplantation techniques eliminate this risk
d. The mortality rate is now approximately 50%
e. The incidence in adults in approximately 1%
Correct answer: c.

23. A 50-year old male with Laennec’s cirrhosis, portal hypertension and
hypersplenism. He has no history of gastrointestinal bleeding. You would
recommend which of the following?
a. Splenectomy
b. Prophylactic sclerotherapy for esophageal varices
c. Portosystemic shunt
d. Observation
Correct answer: d

24. Glucose-6-phosphate dehydrogenase (G6PD) deficiency is an abnormality


of erythrocyte metabolism associated with hemolytic anemia. Pharmocologic
agents which induce hemolysis include which of the following?
a. Acetylsalicylic acid
b. Vitamin E
c. Sulfamethoxazole
d. Desferrioxamine
e. Nitrofurantoin
Correct answer: a, c, e

25. Hypersplenism is associated with which of the following diseases?


a. Portal hypertension
b. Lymphoma
c. Mononucleosis
d. Systemic lupus erythematosus
e. Gaucher disease
f. All of the above.
Correct answer: f.

- 205 -
26. A 40-year old woman with chronic immune thrombocytopenic purpura
(ITP) is refractory to corticosteroids. The approximate likelihood she will
benefit from a splenectomy is approximately which of the following?
a. Less than 2 0%
b. 40%
c. 60%
d. 80%
Correct answer: c

27 . A 50-year-old male presented with history of hematemesis - 500 ml of blood


and on examination shows BP - 90/60, PR - 110/min and splenomegaly 5 cm
below lower costal margin. Most probable diagnosis is -
A. Gastritis
B. Portal hypertension
C. Mallory Weiss tear
D. Duodenal ulcer
Correct answer: B

28 . Splenectomy is performed mainly for -


A. Trauma
B. Idiopathic thrombocytic purpura
C. Hemolytic anemias
D. All of the above
Correct answer: A

10- Biliary system

Gall stones, Cholecystitis

1. A patient presents to the emergency department with obstructive jaundice. A


percutaneous transhepatic cholangiogram and biliary drainage is performed. Shortly
afterward, the patient develops a UGI bleeding . What is the most likely cause?
(A) The patient has developed stress gastritis.
(B) The patient has ingested NSAIDs after the procedure.
(C) The patient has developed haemobilia.
(D) The patient is bleeding from esophageal varices.
(E) The catheter has migrated from the biliary tree into the stomach
Correct answer: C

2. A 74-year-old woman complains of vomiting and intermittent colicky abdominal


pain. X-rays reveal fluid levels and air in the biliary tree. What is the likely cause?
(A) Abdominal adhesions
(B) Gallstone ileus
(C) Carcinoma of the right colon
- 206 -
(D) Abdominal lympho sarcoma
(E) Previous choledochoduodenostomy
Correct answer: B

3 . Which of the following is not a component of Saint's triad ?


A. Oesophageal diverticula
B. Hiatus hernia
C. Gall stones
D. Colonic diverticula
Correct answer: A

4 . Precancerous lesion of gall bladder is -


A. Porcelain gall bladder
B. Mirrizi syndrome
C. Cholesterosis
D. Acalculous Cholecystitis
Correct answer: A

5 . In a 45 years male after laparoscopic cholecystectomy, specimen is for


histopathology which shows carcinoma gallbladder stage Ib. Which of the
following is the appropriate management?
A. Extended cholecystectomy
B. Conservative and follow up
C. Radiotherapy
D. Excision of all port sites
Correct answer: D

6 . Laproscopic cholecystectomy technique was first described by-


A. Kurt Semm
B. Eddie Reddick
C. Erich Muhe
D. Phillip Moure
Correct answer: C

7.The most accurate investigation to diagnose cholecystitis is:


a- CT scan
b- lV cholangiography
c- HIDA scan
d- US
e- MRI
Correct answer: C

8.The best of the following investigations of gall stones is :


a- Oral cholangiography
b- US
c- Plain X-ray
d- ERCP
Correct answer: B

- 207 -
9. With intraoperative T-tube cholangiography, incidence of missed stones is:
a-0%
b- 5%
c- 20%
d- 30%
Correct answer: E

10. Without intraoperative T-tube cholangiography incidence of missed stone


is:
a- 1A-2O%
b- 40-50%
c- 70%
d- 80%
Correct answer: A

11. Which percentage of gall stones is radio-opaque:


a- 10%
b- 40%
c- 70%
d- 90%
Correct answer: A

12. The commonest presentation of gall stones is:


a- Asymptomatic
b- Biliary colic
c- Charcot’s triad
d- Jaundice
Correct answer: A

13. The best treatment of asymptomatic gall stone in diabetics is:


a- Prophylactic cholecystectomy
b- ESWL
c- Follow up
d- None of the above
Correct answer: C

14. Which of the following statements(s) about gallstone ileus is/are not true?
a- The condition is seen most frequently in women older than 70.
b- Concomitant with the bowel obstruction, air is seen in the biliary tree.
c- The usual fistula underlying the problem is between the gallbladder and the
ileum.
d- When possible, relief of small bowel obstruction should be accompanied by
definitive repair of the fistula since there is of significant incidence of
Recurrence if the fistula is left in place.
e- Ultrasound studies may be of help in identifying gallstone of the
obstructing agent.
Correct answer: C

- 208 -
15. Gas in the biliary system in a plain X-ray is diagnostic of
a. Choledochoduodenal fistula
b. intestinal obstruction
c. Emphysematous Cholecystitis
d. Viral hepatitis
e. A and C
Correct answer: E

16. Characters of pain in acute cholangitis are the following except:


a- Starts as colicky pain
b- Becomes dull aching with time
c- Throbbing pain means empyema formation
d- Referred to groin and inner side of thigh
Correct answer: D

17. As regards empyema of gall bladder all are correct except:


a- is a complication of gall stones .
b- Leads to gall bladder mass .
c- ls an indication for cholecystectomy .
d- Should be treated by cholecystectomy .
Correct answer: C

19. Which statement about acute non calcular cholecystitis is correct?


a- The disease is often accompanied by or associated with other conditions.
b- The diagnosis is often difficult.
c- The mortality rote is higher than that for acute calculous cholecystitis .
d- The disease has been treated successfully by percutaneous
cholecystectomy.
e- All of the above
Correct answer: E

20. which of the following statements about cholangitis is incorrect:


a- Charcot’s triad is always present.
b- Associated biliary tract disease is always present.
c- Chills and fever are due to the presence of bacteria in the bile duct system.
d- The most common cause of cholangitis is choledocholithaisis .
Correct answer: A

21.T-tube should be removed:


a- Once cholangiography is done to ovoid fibrosis
b- After 4 days of operation if there is no missed stone
c- After of least 10 days of operation whatever the result of cholangiography
d- None of the above
Correct answer: C

22.As regards treatment of acute cholecystitis all correct except:


a- Cholecystectomy is the best treatment in early cases
b- Morphine is given to decrease pain of anxiety of patient
c- Modified Fowler's position is preferred
d- Failure of conservative treatment indicates cholecystectomy

- 209 -
Correct answer: B

23.Saint's triad includes all the following except:


a- Achalasia
b- Hiatus hernia
c- Chronic calcular cholecystitis
d- Diverticular disease of the colon
Correct answer: A

24. Which of the following lesions are believed to be associated with the
development of carcinoma of the gallbladder?
a- Cholecystoenteric fistula.
b- A calcified gallbladder.
c- Adenoma of the gallbladder.
d- Xanthogranulomatous cholecystitis.
e- All of the above.
Correct answer: E

25. The preferred treatment for carcinoma of the gallbladder is:


a- Radical resection that includes gallbladder in continuity with the right
hepatic lobe and regional lymph node dissection.
b- Radiation therapy.
c- Chemotherapy.
d- Combined treatment involving surgical therapy, chemotherapy, and radiation.
e- Palliative treatment .
Correct answer: E

26. Which of the following is true about post cholecystectomy syndrome:


a- Postoperative stricture is the most common cause of the syndrome .
b- Dyskinesia may be the cause
c- ERCP is the investigation of choice
d- Usually resolves on medical treatment
e- All of the above
Correct answer: E

27 . About carcinoma gall bladder which statement is not correct?


A. More common in the west
B. Etiology in 90% cases is gall stone
C. Late presentation
D. Cholecystectomy with hepatectomy is done in stage-III disease
Correct answer: A

28 . The treatment of choice for silent stones in Gall bladder is:


A. Cholecytectomy
B. Lithotripsy
C. Observation
D. Chenodeoxy cholic acid
Correct answer: C

- 210 -
29 . True statement about gall stones are All/Except:
A. Lithogenic bile is required for stone formation
B. May be associated with carcinoma gall bladder
C. Associated with diabetes mellitus
D. More common in males between 30-40 years of age
Correct answer: D

30 . A 24 years female presents with three day history of epigastric pain


radiating to back. Serum amylase levels were observed to be normal while USG
abdomen reveals gall bladder stones and an enlarged pancreas. CT abdomen was
done which clinched the diagnosis. The most possible diagnosis is-
A. Acute Pancreatitis
B. Acute Cholecystitis
C. Acute Peritonitis
D. Acute Appendicitis
Correct answer: A

31 . A 70-year-old man presents with chest pain on ovulation, he was diagnosed


from coronary disease. On doing ultrasonography Gall stone was detected. What
should be the most appropriate management of gall stones in this case?
A. Open cholecystectomy
B. Laparoscopic cholecystectomy
C. Oral dissolution agents
D. Wait and watch
Correct answer: D

32 . Medical treatment for gall bladder stone is amenable for:


A. Calcium billirubinate oxalate stone
B. Nonfunctioning stones associated with gall bladder
C. Stones less than 15 mm in size
D. Radiopaque stone
Correct answer: C

33 . All of the following are predisposing factors for gallstone formation except:
A. Vagotomy
B. Jejunal resection
C. Ileal resection
D. Right hemicolectomy
Correct answer: D

35 . Complications of gallstones include the following except for -


A. Acute gastritis
B. Intestinal obstruction
C. Obstructive jaundice
D. Acute pancreatitis
Correct answer: A

36 . Stone formation in Gall bladder is enhanced by all except


A. Clofibrate therapy
B. Ilial resection

- 211 -
C. Cholestyramine therapy
D. Vagal stimulation
Correct answer: D

37 . Contraindication for laparoscopic Cholecystectomy is ?


A. Coagulopathy
B. Obstructive pulmonary disease
C. End-stage liver disease
D. All of the above
Correct answer: D

38 . The commonest site of obstruction in Gall stone ileus is


A. Proximal ileum
B. Distal ileum
C. Ileocecal junction
D. Transverse colon
Correct answer: C

39 . Which of the following statements is incorrect in regard to stones in the


gallbladder?
A. 10% of gallstones are radiopaque
B. A mucocele of the gallbladder is caused by a stone impacted in Hartmann's pouch
C. Pigment stones are due to increase excretion of conjugated bilirubin
D. Are considered a risk factor for the development of gallbladder carcinoma
Correct answer: B

40 . Risk factors for malignant change in an asymptomatic patient with a gall


bladder polyp on ultrasound include all of the following, Except:
A. Rapid increase in size of polyp
B. Age > 60 years
C. Associated Gall stones
D. Size of polyp > 5 mm
Correct answer: D

41 . Incidence of gall stone is high in -


A. Partial hepatectomy
B. Ileal resection
C. Jejunal resection
D. Subtotal gastrectomy
Correct answer: B

42 . Mirizzi's syndrome is ?
A. Gall bladder stone compressing hepatic duct
B. Gall bladder carcinoma invading IVC
C. Gall bladder stone causing cholecystitis
D. Pancreatic carcinoma
Correct answer: A

43 . Commonest type of ca gallbladder with all stone is -


A. Adenocarcinoma

- 212 -
B. Anaplastic carcinoma
C. Sq. cell carcinoma
D. Transitional cell ca
Correct answer: A

44 . A 72 years male patient presented with history of chest pain and was
diagnosed to have coronary artery disease. During routine evaluation, an
ultrasound of the abdomen showed presence of gallbladder stones. There was no
past history of biliary colic or jaundice. The best treatment advice for this
patient for his gallbladder stones is
A. Laparoscopic cholecystectomy
B. Open cholecystectomy
C. ERCP and removal of gallbladder stones
D. No surgery for gallbladder stones
Correct answer: D

45 . A 69 years old male patient having coronary artery disease was found to
have gall bladder stones while undergoing a routine ultrasound of the abdomen.
There was no history of biliary colic or jaundice at any time. What is the best
treatment advice for such a patient for his gallbladder stones?
A. Open cholecystectomy
B. Laparoscopic cholecystectomy
C. No surgery for gallbladder stones
D. ERCP and removal of gallbladder stones
Correct answer: C

46 . Which of the following organism is associated with fish consumption and


also causes carcinoma gallbladder?
A. Anglostrongyloidosis cantonensis
B. Gnathostoma
C. H. dimunata
D. Clonorchis sinensis
Correct answer: D

47 . Which of the following is the most common site of intestinal obstruction in


gallstone ileus?
A. Ileum
B. Jejunum
C. Sigmoid colon
D. Transverse colon
Correct answer: A

48 . Which of the bellow is associated with fish consumption and also cause ca
gallbladder ?
A. Anglostronglyoidosis cantonensis
B. H. dimunata
C. Clonorchis sinensis
D. Gnathisomia
Correct answer: C

- 213 -
49 . Survival in unresectable Gall Bladder CA is?
A. 1year.
B. 12-24 months
C. 4-6 months
D. 8-10 months
Correct answer: C

50 . Which is not required for visualization of gall bladder in oral


cholecystography -
A. Functioning liver
B. Motor mechanism of gall bladder
C. Patency of cystic duct
D. Ability to absorb water
Correct answer: B

51 . A 40 year old patient has undergone an open cholecystectomy’s procedure


was reported as uneventful by the operating surgeon. She has 100 ml of bile
output from the drain kept in the gallbladder bed on the first post operative day.
On examination she is afebrile and anicteric. The abdomen is soft and bowel
sounds are normally heard. As an attending physician. What should be your best
possible advice?
A. Order an urgent endoscopic retrograde cholangiography and biliary stenting
B. Urgent laparotomy
C. Order an urgent hepatic iminodiacetic acid scintigraphy (HIDA)
D. Clinical observation
Correct answer: D

52 . Medical treatment of gallstone is contraindicated in -


A. Radia-opaque stone
B. Functioning gall bladder
C. Small stone
D. Radiolucent stone
Correct answer: A

53 . Most common site for impaction of gall stones in gall stone ileus is:
A. IIst part of duodenum
B. Ist part of duodenum
C. Colon
D. Terminal ileum
Correct answer: D

54 . Association of carcinoma gall bladder with gall stones is about -


A. 50%
B. 70%
C. 90%
D. 20%
Correct answer: C

- 214 -
55 . Regarding primary sclerosing cholangitis, all of the following are true
EXCEPT -
A. Ultrasound examination is often normal
B. ERCP is the preferred approach
C. Cholangiographic appearance may mimic diffuse form of cholangiocarcinoma
D. Affects only intrahepatic ducts
Correct answer: D

56 . All of the following extraintestinal manifestations of ulcerative colitis


respond to colectomy except -
A. Primary sclerosing cholangitis
B. Pyoderma gangrenosum
C. Episcleritis
D. Peripheral arthralgia
Correct answer: A

57 . True for patients of ulcerative colitis associated with primary sclerosing


cholangitis (PSC), except:
A. Increased risk of hilar cholangiocarcinoma.
B. PSC reverts after a total colectomy.
C. They may develop biliary cirrhosis.
D. May have raised alkaline phosphatase.
Correct answer: B

58 . All of the following are predisposing factors for cholangiocarcinoma except:


A. Common bile duct stone
B. Ulcerative colitis
C. Sclerosing cholangitis
D. Clonorchia sinensis
Correct answer: A

Obstructive jaundice

1. The risk of biliary tract adenocarcinoma developing in a patient with a


choledochal cyst left in situ is approximately which of the following?
a. Less than 1%
b. 3% to 5%
c. 10% to 15%
d. Greater than 25%
Correct answer: b

2- The gold standard for the definitive diagnosis of the extrahepatic biliary
atresia is:
A. Peroperative cholangiography
B. Hepatobiliary scintigraphy
C. Alkaline phosphate level
D. Liver biopsy

- 215 -
Correct answer: A

3. Not true regarding choledochal cyst -


A. Epigastric mass
B. Jaundice
C. Pain in abdomen
D. Cystojejunostomy is the treatment of choice
Correct answer: D

4. Gall bladder stone formation in influenced by A/E


A. Primary biliary cirrhosis
B. Hypercholesterolemia
C. Clofibrate therapy
D. Hyperalimentation
Correct answer: B

5 . True colour of cholesterol stone is ?


A. Black
B. Brown
C. Dark yellow
D. Pale yellow
Correct answer: D

6 . Gall stone can be which type of stone ?


A. Triple phosphate stone
B. Oxalate stone
C. Cholesterol stone
D. Urate stones
Correct answer: C

7 . Most common gall bladder stone is ?


A. Pigment
B. Cholesterol
C. Oxalate
D. Mixed
Correct answer: B

8 . Lithogenic bile has the following properties


A. Increase Bile and Cholesterol ratio
B. Decrease Bile and cholesterol ratio
C. Equal bile and cholesterol ratio
D. Decrease Cholesterol only
Correct answer: B

9 . All of the following are required for visualization of gall bladder in an oral
cholecystogram except:
A. Ability to absorb water
B. Functioning hepatocytes
C. Motor mechanism of gall bladder
D. Patency of cystic duct

- 216 -
Correct answer: C

10 . The type of obstruction in budd-chiari syndrome is -


A. Extra hepatic pre sinusoidal
B. Extra hepatic post sinusoidal
C. Sinusoidal
D. Intra hepatic post sinusoidal
Correct answer: B

11. Kasai operation is for treatment of:


a- lntrahepatic biliary atresia
b- Extrahepatic biliary atresia
c- Caroli's disease
d- Choledochal cyst
Correct answer: B

12. Primary unconjugated hyperbilirubinemia is a feature of


A. Dubin Jhonson syndrome
B. Rotor syndrome
C. Gilbert's syndrome
D. Primary biliary cirrhosis
Correct answer C

13 . Commonest cause of surgical jaundice is -


A. CA head of pancreas
B. CBD stone
C. CA gall bladder
D. Gall stone ileus
Correct answer: B

14 . 5 days after CBD surgery there is a small leak. What will be the best The
treatment -
A. Ultrasound guided aspiration
B. ERCP and stenting
C. Re-exploration and hepatojejunostomy
D. Re-exploration and primary repair
Correct answer: B

15 . The procedure of choice for elective removal of CBD stones for most patients
is -
A. Open choledocholithotomy
B. Endoscopic choledocholithotomy
C. Laparoscopic choledocholithotomy
D. Percutaneous choledocholithotomy
Correct answer: C

16 . Small gall bladder with dilated intrahepatic biliary channels found in:
A. GB stone
B. CBD stone
C. Carcinoma head of pancreas

- 217 -
D. GB cancer
Correct answer: B

17 . A surgeon with less experience of laparoscopic cholecystectomy while doing


lap surgery found some stone in common bile duct. What should be ideally do ?
A. Open cholecystectomy with choledocodudenostomy
B. Lap exploration of CBD and removal of stone
C. Lap CBD extraction through the cystic duct
D. Convert it open cholecystectomy and remove CBD stone
Correct answer: D

18 . In cholangiography CBD stone appears as -


A. Meniscus sign
B. Cut off sign
C. Slight flow of dye from the sides of stone
D. Chain of lake appearance
Correct answer: A

19 . A patient presents 1 year after cholecystectomy with a CBD stone of 2.5 cm


in size. Treatment of choice is -
A. Preduodenal choledochotomy with exploration
B. Trans duodenal choledochojejuostomy
C. Trans duodenal sphincterotomy
D. Endoscopic sphincterotomy with stone extraction
Correct answer: A

20 . Commonest cause of surgical obstructive jaundice is:


A. Carcinoma of liver
B. Chronic cholecystitis
C. CBD stone
D. Sclerosing cholangitis
Correct answer: C

21 . 1 year After cholecystectomy, a patient presented with 2.5 cm size stone in


CBD,
Treatment of choice is:
A. Trans duodenal choledochojejunostony with sphincturoplasty
B. Endoscopic sphincterotomy with stone extraction
C. Choledocholithotomy with T-tube drainage
D. Supraduodenal choledochotomy with exploration
Correct answer: B

22 . Most common Surgical cause of Obstructive Jaundice is:


A. Alcoholism
B. Malignancy
C. CBD stone
D. Stricture in CBD
Correct answer: C

- 218 -
23 . The treatment of choice for an 8 mm retained common bile duct (CBD) stone
is -
A. Laparoscopic CBD exploration
B. Percutaneous stone extraction
C. Endoscopic stone extraction
D. Extracorporeal shock wave lithotripsy
Correct answer: C

24 . Most common surgical cause of obstructive jaundice -


A. Periampullary carcinoma
B. Carcinoma gall bladder
C. Carcinoma head of pancreas
D. CBD stones
Correct answer: D

11- Pancreas;

Acute pancreatitis, Pancreatic tumors

l. As regard presentations of acute pancreatitis all are correct, except:


a. Symptoms are more evident than signs.
b. Pain usually radiate to hypogastrium.
c. Hematemesis ond/or melena may occur.
d. Broad like rigidity of abdomen may occur.
Correct answer: B

2. Indications for laparotomy in acute pancreatitis include the following except:


a. Doubtful diagnosis.
b. Complicated cases.
c. Pseudocyst.
d. Detoriating cases for debridement.
Correct answer: C

3. The most common cause of acute pancreatitis:


a. Bile duct stone.
b. Alcoholism.
c. latrogenic
d. ldiopathic.
Correct answer: A

4. The most common cause of chronic pancreatitis:


a. Bile duct stone.
b. Alcoholism.
c. lotrogenic.
d. ldiopothic.
- 219 -
Correct answer: B

5. The best treatment of acute pancreatitis is:


a. Conservative (ryle, line, lV fluids and antibiotic).
b. CT guided aspiration.
c. Laparotomy and debridement.
d. None of the above.
Correct answer: A

6. Raised amylase level:


a. ls diagnostic for chronic pancreatitis.
b. ls a screening test for pancreatic cancer.
c. More than I000 units is bad prognostic criteria in acute pancreatitis.
d. May occur in non pancreatic diseases as dissecting aortic aneurism.
Correct answer: D

7. The following are complications of acute pancreatitis. The most common cause of
death is:
a. Neurogenic shock.
b. Multiorgan failure.
c. Tetany.
d. Pancreatic abscess and septicemia.
Correct answer: B

8. ERCP finding suggestive of chronic pancreatitis include:


a. Dilated main duct with stones and strictures.
b. Pancreatic cysts.
c. Chain of lakes appearance.
d. All of the above.
Correct answer: D

9. The most sensitive test for diagnosis of acute pancreatitis is:


a. Serum amylase level.
b. Serum lipase level.
c. Serum calcium level.
d. Amylase creatinine clearance ratio.
e. Urinary amylase level.
Correct answer:B

10. As regards surgical drainage of pseudo pancreatic cyst all are correct except:
a. lndicated in all cases.
b. lndicated in infection.
c. lndicated if the cyst >6cm or persistent >5weeks.
d. Done by cystojejunostomy or more common cystogastrostomy.
Correct answer: A

11. What is not true of pancreatic pseudocyst:


a. Presents in epigastrium as a fixed swelling.
b. Mimics aneurysm of aorta if very tense.
c. Pseudocysts less than 6 cm invariably hove spontaneous resolution.

- 220 -
d. All are true.
Correct answer: D

12. Presentations of Zollinger Ellison syndrome include the following except:


a. Peptic ulcer resistant for treatment and recurrent after treatment.
b. Peptic ulcer in ectopic sites.
c. Peptic ulcer complicated by bleeding and perforation in short time.
d. constipation.
Correct answer:D

13-The most important investigation for the extent of cancer pancreas is:
a. ERCP.
b. US.
c. CT.
d. Barium meal.
Correct answer: C

14. As regard treatment of cancer pancreas:


a. Whipple's operation for operable cases.
b. Triple anastomosis for inoperable cases.
c. Endoscopic stenting in patients unfit for surgery.
d. All of the above.
Correct answer: D

15. Carcinoma of the pancreas:


a. Has a peak incidence in young age.
b. Had a good prognosis
c. Commonly presents with asthenia.
d. ls more common in smokers.
Correct answer: C

16. Concerning pancreatic pseudocysts, the following statements are correct


except that:
a. Trauma is the most common cause.
b. They often present as midline pulsating swellings.
c. They may resolve spontaneously.
d. Ultrasound is helpful in establishing the diagnosis.
e. The best surgical treatment is by cysto-gastrostomy or cysto- jejunostomy.
Correct answer: A

17. Which of the following statements about Pancreatic Carcinoma is not true
A. Hereditary Pancreatitis significantly increases the risk
B. Mutation in P53 gene is associated in 75% of cases
C. Five years survival after curative pancreaticoduodenectomy is 15 - 20%
D. Median survival in locally advanced (stage III) disease is 3-6 months
Correct answer: D

- 221 -
18. A 42-year-old man who consumed more than 3 bottles of vodka weekly over
the past 20 years is admitted with upper abdominal pain radiating to the back,
nausea, and vomiting. Serum amylase and lipase are elevated, and a diagnosis of
pancreatitis is made. In determining his prognosis, which of the following factors
would cause the greatest concern?
(A) Hypercalcemia (Ca>12 mg/dL)
(B) Age over 40 years
(C) Hypoxemia
(D) Hyperamylasemia (>600 U)
(E) Elevated lipase.
Correct answer: C

19-A 24-year-old college student recovers from about of severe pancreatitis. He


has mild epigastric discomfort, sensation of bloating, and loss of appetite.
Examination reveals an epigastric fullness that on ultrasound is confirmed to be
a pseudocyst. The swelling increases in size over a 3-week period of observation.
What should be the next step in management?
(A) Percutaneous drainage of the cyst
(B) Laparotomy and internal drainage ofthe cyst
(C) Excision of pseudocyst
(D) Total pancreatectomy
(E) Administration of pancreatic enzymes
Correct answer: A

20- A 40-year-old alcoholic male is admitted with severe epigastric pain radiating
to the back. Serum amylase level is reported as normal, but serum lipase is
elevated. The serum is noted to be milky in appearance. A diagnosis of
pancreatitis is made. The serum amylase is normal because
(A) The patient has chronic renal failure.
(B) The patient has hyperlipidemia.
(C) The patient has alcoholic cirrhosis.
(D) The patient has alcoholic hepatitis.
(E) The diagnosis of pancreatitis is incorrect.
Correct answer: B

21- A 52-year-old woman is admitted to the hospital with abdominal pain. She
reports that she drinks alcohol only at social occasions. The amylase is elevated
to 340 U. Which following x-ray finding would support a diagnosis of idiopathic
pancreatitis?
(A) Hepatic lesion on CT scan
(B) Choledocholithiasis on ultrasound
(C) Anterior displacement of the stomach on barium upper GI series
(D) Large loop of colon in the RUQ
(E) Irregular cutoff of the CBD on cholangiogram
Correct answer: C

- 222 -
22- A 40-year-old man with a history of alcohol consumption of 25-year duration
is admitted with a history of a 6-lb weight loss and upper abdominal pain of 3-
weeks duration. Examination reveals fullness in the epigastrium. His
temperatureis 99F, and his WBC count is 10,000.
Which is the most likely diagnosis?
(A) Pancreatic pseudocyst
(B) Subhepatic abscess
(C) Biliary pancreatitis
(D) Cirrhosis
(E) Splenic vein thrombosis
Correct answer: A.

23- A 48-year-old woman is admitted with acute cholecystitis. The bilirubin level
is elevated, as are the serum and urinary amylase levels. Which radiologic sign
indicates biliary obstruction in pancreatitis?
(A) Pancreatic intraductal calcification
(B) Smooth narrowing of the distal CBD
(C) Stomach displaced anteriorly
(D) Calcified gallstone
(E) Air in the biliary tree
Correct answer: B.

24- A 42-year-old woman with a history of chronic alcoholism is admitted to the


hospital because of acute pancreatitis. The bilirubin and amylase levels are in the
normal range. An ultrasound reveals cholelithiasis. The symptoms abate on the
fifth day after admission. What should she be advised?
(A) To start on a low-fat diet.
(B) To increase the fat content of her diet.
(C) To undergo immediate cholecystectomy.
(D) To undergo cholecystectomy during the same hospital, stay as well as an
assessment of her bile ducts.
(E) That she will be discharged and now should undergo elective cholecystectomy
after 3 months.
Correct answer: D.

25- Following a motor vehicle accident a truckdriver complains of severe


abdominal pain. Serum amylase level is markedly increased to 800 U. Grey
Turner’s sign is seen in the flanks. Pancreatic trauma is suspected. Which
statements true of pancreatic trauma?
(A) It is mainly caused by blunt injuries.
(B) It is usually an isolated single-organ injury.
(C) It often requires a total pancreatectomy.
(D) It may easily be overlooked at operation.
(E) It is proved by the elevated amylase level.
Correct answer: D.

- 223 -
26- A 40-year-old woman with severe chronic pancreatitis is scheduled to
undergo an operation, because other forms of treatment have failed. The
ultrasound shows no evidence of pseudocyst formation or cholelithiasis and
endoscopic retrograde cholangiopancreatogram (ERCP) demonstrates dilated
pancreatic ducts with multiple stricture formation. Which operations suitable to
treat this condition?
(A) Pancreaticojejunostomy (Puestow procedure)
(B) Gastrojejunostomy
(C) Cholecystectomy
(D) Splenectomy
(E) Subtotal pancreatectomy
Correct answer: A.

27- A 26-year-old woman with known history of chronic alcoholism is admitted


to the hospital with severe abdominal pain due to acute pancreatitis. The serum
and urinary amylase levels are normal. On the day following admission to the
hospital, there is no improvement, and she has a mild cough and slight dyspnea.
What is the most likely complication?
(A) Pulmonary atelectasis
(B) Bronchitis
(C) Pulmonary embolus
(D) Afferent loop syndrome
(E) Pneumonia
Correct answer: A.

28- A 66-year-old man with obstructive jaundice is found on ERCP to have


periampullary carcinoma. He is otherwise in excellent physical shape and there
is no evidence of metastasis. What is the most appropriate treatment?
(A) Radical excision (Whipple procedure) where possible
(B) Local excision and radiotherapy
(C) External radiotherapy
(D) Internal radiation seeds via catheter
(E) Stent and chemotherapy
Correct answer: A.

29- A 74-year-old man complains of epigastric discomfort. There is no jaundice


evident, but an enlarged gallbladder is palpated. The bilirubin level is 13 mg/dL,
the alkaline phosphatase level is 410 U, and the hematocrit is 35%. CT scan and
MRI findings are shown in Fig. 7–1. What is the most likely malignant tumor
causing extrahepatic obstructive jaundice?
(A) Gallbladder
(B) Common hepatic duct
(C) Cystic duct
(D) Periampullary area
(E) Head of the pancreas
Correct answer: E.

- 224 -
30- Insulinoma is most commonly located in which part of the pancreas:
A. Head
B. Body
C. Tail
D. Equally distributed
Correct answer: D

31- Treatment of choice in a pseudocyst of pancreas of <5 cm in size and <3


weeks of duration is-
A. Serial ultrasonic measurement and follow up
B. Excision of the cyst
C. Internal drainage
D. External drainage
Correct answer: A

32 . Patient with chronic pancreatitis gives chain of lakes appearance in ERCP


examination. Management is:
A. Total pancreatectomy
B. Sphincteroplasty
C. Side to side pancreatico jejunostomy
D. Resecting the tail of pancreas and performing a pancreatojejunostomy
Correct answer: C

33 . Frey's procedure is done for -


A. Meckel's diverticula
B. Auriculotemporal neuropathy
C. Ulcerative colitis
D. Chronic pancreatitis
Correct answer: D

34 . A 55 years lady presents with history of recurrent abdominal pain. Imaging


shows multiple small cystic lesions like a bunch of grapes in the head of pancreas
with a grossly dilated main pancreatic duct. The most possible diagnosis is:
A. Mucinous Cystadenoma (MCN)
B. Serous Cystadenoma (SCN)
C. Pancreatic Pseudocyst
D. Intraductal Papillary Mucinous Neoplasm (IPMN)
Correct answer: D

35 . Most common cause of pseudo pancreatic cyst in children is:


A. Drug induced pancreatitis
B. Traumatic pancreatitis
C. Chloledochal cyst
D. Annular pancreas
Correct answer: B

36 . About pseudocyst pancreas false statement is ?


A. Fibrous coat
B. Has mucous lining epithelium
C. Not a true cyst

- 225 -
D. Presents as epigastric mass
Correct answer: B

37 . Commonest endocrine tumor of pancreas -


A. A cells
B. B cells
C. Delta cells
D. VIPoma
Correct answer: B

38 . All of the following are removed in Whipple's operation except


A. Duodenum
B. Head of pancreas
C. Portal vein
D. Common bile duct
Correct answer: C

39 .A chronic Alcoholic presents with repeated episodes of severe intractable


abdominal pain. At evaluation the pancreatic duct was found to be dilated and
stones were noted in the tail of pancreas. The most appropriate management is:
A. Pacreatico Jeunostomy
B. Pancreatic Tail Resection
C. Medical management
D. Percutaneous removal of stone
Correct answer: A

40 . False about annular pancreas is -


A. Prevalent in Down's syndrome
B. Bilious vomiting
C. Double bubble sign on X ray
D. Gastrojejunostomy offers best remedy
Correct answer: D

41 . A 45 year female presented with recurrent attacks of giddiness and


abdominal pain since three months. Endoscopy was normal. Her fasting blood
glucose was 40 mg% and insulin levels were elevated. CT abdomen showed a
well-defined 8 mm enhancing lesion in the head of pancreas, with no other
abnormal findings. The treatment plan for this patient is-
A. Enucleation
B. Whipple's operation
C. Administration of streptozotocin
D. Enucleation with radiotherapy
Correct answer: A

42 . All are true about pseudo pancreatic cyst of pancreas except:


A. Serum amylase is increased
B. Most common site is in head of pancreas
C. Common after acute pancreatitis
D. Presents as an abdominal mass
Correct answer: B

- 226 -
43 . Whipples triad is seen in -
A. Insulinoma
B. Somatostatinoma
C. Glucagonoma
D. CA pancreas
Correct answer: A

44 . Treatment of choice for annular pancreas is -


A. Gastro jejunostomy
B. Pancreatectomy
C. Whipple's procedure
D. Duodeno duodenostomy
Correct answer: D

45 . Treatment of annular pancreas is -


A. Conservative
B. Gastrojejunostomy
C. Duodeno- duodenostomy
D. Removal of compressing part of pancreas
Correct answer: C

46 . Regarding injury to pancreas, what is not true -


A. Majority of postoperative complications are due to missed duct injury
B. Fracture is common at the junction of head and body
C. Commonly associated with vascular injury
D. Peritoneal lavage is good for making the diagnosis
Correct answer: B

47 . All of the following statements about Pseudopancreatic cysts are true except:
A. Serum amylase levels are increased
B. Presents as an epigastric mass
C. Percutaneous aspiration is treatment of choice
D. Cystojejunostomy is treatment of choice
Correct answer: C

12- Small and large intestine

Small Intestinal obstruction

1. A 30-year-old women is accidently found to have a wide mouthed Meckel's-


diverticulum during laparotomy. What is the best method of treatment?
A. Diverticulectomy
B. Leave it alone
C. Resection of diverticulum and invagination of stump

- 227 -
D. Resection of diverticulum with a short segment of ileum and anastomosis
Correct answer B

2 . Which of the following is an early post operative complication of ileostomy in


the post operative period?
A. Necrosis
B. Obstruction
C. Prolapse
D. Diarrhea
Correct answer: A

3 . Nonpropulsive peristalsis is feature of -


A. Paralytic ileus
B. Impacted faeces
C. Mesenteric vascular occlusion
D. Incarceration
Correct answer: C

4 . Short bowel syndrome is characterized by all of the following except:


A. Steatorrhoea
B. Hypogastrinemia
C. Diarrhea
D. Weight loss
Correct answer: B

5. What is the most common complication of Meckel’s diverticulum among adults?


(A) Bleeding
(B) Perforation
(C) Intestinal obstruction
(D) Ulceration
(E) Carcinoma
Correct answer: C

6. A 68-year-old male musician presents to the emergency department with a sudden


onset of colicky abdominal pain and massive vomiting of 4-hour duration. Examination
shows an elevated WBC of 13,200 with a HCT of 45%. Electrolytes and blood urea
nitrogen (BUN) are normal. An erect film of the abdomen reveals dilatation of the
stomach with distended loops of bowel. What is his clinical diagnosis?
(A) Complete proximal intestinal obstruction
(B) Incomplete proximal intestinal obstruction
(C) Complete ileal obstruction
(D) Incomplete ileal obstruction
(E) Small-bowel perforation
Correct answer: A

7. A 55-year-old woman presents with vague RLQ abdominal pain. A palpable mass
is noted on abdominal examination. The mass is painless, well defined, mobile, and
nonpulsatile. What is the most likely diagnosis?
(A) A mesenteric cyst
(B) Appendix mass

- 228 -
(C) Perforated tuba-ovarian abscess
(D) Cholecystitis
(E) Meckel’s diverticulum
Correct answer: A

8. A 68-year-old man presents with crampy abdominal pain and distention with
vomiting. Findings on physical examination are positive for healed abdominal scars.
X-rays reveal multiple gas fluid levels. The WBC count is 12,000. What is the most
likely diagnosis?
(A) Small-bowel intestinal obstruction due to adhesions
(B) Hernia
(C) Appendicitis
(D) Inflammatory bowel disease
(E) Gallstones and ascites
Correct answer: A

9- ln acute intestinal obstruction, it is untrue that :


a. Abdomen should always the examined for laparotomy scars and
external hernias.
b. Signs of peritonitis indicate strangulation.
c. The serum amylase is often raised.
d. A rectal examination is essential.
e. Plain X-ray examination is not helpful in the diagnosis.
Correct answer: E

10- ln intestinal strangulation, it is untrue that


a- it is difficult to differentiate from simple occlusion.
b- Moy complicate closed-loop obstruction.
c- Causes bleeding into the affected bowel.
d- Frequently courses peritonitis.
e- Requires urgent laparotomy.
Correct answer: A

11- ln children and adolescents, the commonest cause of intestinal obstruction is:
a. Bonds and adhesions.
b. lntussusception.
c. Strangulated hernia.
d. Neoplasm.
e. Benign stricture.
Correct answer: C

12- The commonest cause of pyloric obstruction in adults is:


a. Prepyloric gastric ulcer.
b. Benign gastric tumor.
c. Carcinoma of the stomach.
d. Hypertrophic pyloric stenosis.
e. Chronic duodenal ulcer.
- 229 -
Correct answer: E

13- Chronic pyloric obstruction produces all of the following except:


a. Alkalosis.
b. Hypokalemia.
c. Hypochloremia.
d. Alkaline urine.
e. Dehydration.
Correct answer: D

14- A 40-year-old female presented with colicky abdominal pain, vomiting and
constipation of 48 hours duration. She gave a history of previous laparotomy and
examination revealed abdominal distension with loud bowel sounds. The most
probable diagnosis is:
a. Adhesive intestinal obstruction.
b. Gall-stone ileus.
c. internal hernia.
d. Paralytic ileus.
e. Mesenteric vascular occlusion.
Correct answer: A

15 . What is not seen in massive resection of small bowel -


A. Hypogastrinemia
B. B12 Deficiency
C. Malabsorption
D. Oxalate Stone
Correct answer: A

16 . A female patient presents with pigmentation of the lips and oral mucosa and
intestinal polyps. Her sister also gives the same history. Most probable diagnosis is:
A. Carcinoid tumor
B. Melanoma
C. Villous adenoma
D. Peutz-Jegher syndrome
Correct answer: D

17- Peutz-Jegher syndrome can present by the followings:


a- Bleeding per rectum
b- Colics and vomiting
c- Anemia of unknown etiology
d- Oral pigmentation
e- Any of the above
Correct answer: E

- 230 -
Inflammatory bowel disease

1 . Ileocecal TB is associated with which of the bellow?


A. Sideroblastic anemia
B. Normocytic normochromic anemia
C. Megaloblastic anemia
D. Iron deficiency anemia
Correct answer: C

2 . A 40 year old male patient presented with mild abdominal pain, mild
constipation with a feeling of incomplete evacuation and mucus in stools for the
past 4 years. On examination, tenderness is present in left iliac fossa. The most
likely diagnosis is -
A. Ulcerative colitis
B. Diverticular disease of the colon
C. Irritable bowel syndrome
D. Ca colon
Correct answer: C

3- Toxic megacolon is a known complication of -


A. Pseudomembranous colitis
B. Ulcerative colitis
C. Amebic colitis
D. Hirschprung's disease
Correct answer: B

4. Compared with Crohn's disease Ulcerative colitis does not have more
A.Malignant change
B.Fistula formation
C.Bleeding per rectum
D.Cholangio carcinoma
Correct answer B

5 . All are true for ulcerative colitis except -


A. Usual age of presentation is > 60 years
B. Back wash ileitis
C. Pancolitis
D. Cobblestoning
Correct answer: D

- 231 -
Diverticular disease of the colon

1- The following statements about diverticular disease of the colon are true except that
it :
a. increases in incidence with advancing age.
b. Does not involve the rectum.
c. ls essentially due to o high-residue diet.
d. Moy be asymptomatic.
e. Moy cause massive rectal bleeding.
Correct answer: C

2-The best investigation of diverticular disease is:


a- Endoscopy
b- Barium enema
c- Angiography
d- Abdominal US
Correct answer: B

Colonic obstruction and Cancer colon

1- All of the following are true about Familial adenomatosis polyposis (FAP),
except-
A. Polyps develop in late adulthood
B. Epidermal cysts and osteomas may occur
C. Autosomal recessive inheritance
D. Screening done by sigmoidoscopy
Correct answer: C

2 . Gene responsible for FAP is located at -


A. Chromosome 5
B. Chromosome 8
C. Chromosome 15
D. Chromosome X
Correct answer: A

3- Treatment of choice for squamous cell carcinoma of anus is ?


A. Chemotherapy
B. Chemoradiation
C. Sphincter sparing surgery
D. Monoclonal antibodies
Correct answer: B

- 232 -
4. The X-ray finding which helps to distinguish Hirschsprung's disease from
congenital megacolon is
A.Delayed evacuation of barium
B.Air fluid levels
C.Presence of a transition zone
D.Dilated bowel loops
Correct answer C

5 . The fascia separating rectum from coccyx is called -


A. Scarpa's fascia
B. Waldeyer's fascia
C. Denovillier's fascia
D. Colle's fascia
Correct answer: B

6 . True about solitary rectal ulcer syndrome is all/except -


A. Crypt distortion
B. Increased muscle layer proliferation
C. Subepithelial fibrosis
D. Lamina propria infiltration with lymphocyte
Correct answer: D

7 . Which of the following is true regarding 'Fistula is ano' ?


A. High and low divisions are made in relation to the pelvic floor
B. Intersphincteric is the most common type
C. Posterior fistulae have straight tracks
D. High fistulae can be operated with no fear of incontinence
Correct answer: B

8 . Genes may be involved in development of carcinoma of colon Except -


A. Beta - Catenin
B. APC
C. Mismatch Repair Genes
D. K - ras
Correct answer: A

9. AII of the following are high anal anomalies except:


a. Rectal atresia.
b. Anorectal agenesis.
c. Ectopic anus.
d. Persistent cloaca.
Correct answer: C

10. lnvertogram should be done:


a. 6 hours after birth.
b. 12 hours after birth.
c. 24 hours after birth.
d. 48 hours after birth.
Correct answer: C

- 233 -
11. Possible sites for pilonidal sinus include the following except:
a. Anal cleft.
b. Axilla.
c. Umbilicus.
d. None of the above.
Correct answer: D

12. The following findings are present in chronic fissure except:


a. Sentinel piles.
b. Anal papillae.
c. Severe pain.
d. Induration.
Correct answer: C

13. Causes of perianal pain include the following except:


a. Fissure.
b. Anal carcinoma.
c. Perianal abscess.
d. Piles.
e. Proctalgia fugax.
Correct answer: D

14. Regarding treatment of piles all are true except:


a. 1st degree - life style change.
b. 2nd degree -hemorrhoidectomy.
c. 3rd and 4th degree - hemorrhoidectomy.
d. Abscess -drainage ondantibiotics.
Correct answer: B

15. Usually piles are present at:


a. l, 5, ond 9 o'clock positions.
b. 3,7ond I 'lo'clock positions.
c. 3 ond 9 o'clock positions.
d. None of the obove.
Correct answer: B

16. The most common complication after hemorrhoidectomy is:


a. Urinary retention
b. Rectal bleeding
c. Incontinence
d. Wound infection
Correct answer: A

17. The most common form of anorectal abscess is:


a. Perianal
b. Ischiorectal
c. Submucous.
Correct answer: A

- 234 -
18. An elderly male with history of habitual constipation presented because of
progressive abdominal distension with diarrhea and tenesmus. He should be
suspected lo suffer from:
a. Carcinoma of the rectum
b. None-specific ulcerative colitis
c. Amoebic colitis
d. Crohn's diseose
e. Proctocolitis
Correct answer: A

19. Which statement is untrue concerning pilonidal sinus:


a. ls a skin-lined track over the sacrococcygeal joint.
b. Often contains a tuft of hair.
c. Frequently presents by suppuration.
d. ls particularly common in dork hairy people.
e. ls easily treated by simple excision.
Correct answer: E

20. Concerning internal piles, the following statements are correct except that
they:
a. Are due to varicosity of the internal hemorrhoidal plexus.
b. May consist of mother or daughter piles.
c. Present clinically by bleeding and prolapse.
d. Are usually associated with severe pain.
e. May resolve under conservative treatment in the early stages.
Correct answer: D

21. Which of the following statements about anorectal fistulas is untrue?


a. Commonly result from failure of healing of anorectal abscess.
b. Rarely originate in on anal cry patient
c. Are classified according to relation of track to sphincteric musculature.
d. Hove curved tracks when arising in the posterior midline of anal canal.
e. Moy be associated with Crohn's disease or non-specific ulcerative colitis.
Correct answer: B

22. Which of the following is false regarding anal canal anatomy?


a. The anorectal ring marks the junction between the rectum and the anal canal.
b. The puborectalis muscle is concerned with the continence mechanism.
c. The intersphincteric plane is not of much significance.
d. The internal sphincter is the thickened distal continuation of the circular muscle
layer of the rectum.
e. The dentate line is an important landmark representing the site of fusion of the
proctodeal and postallantoic gut.
Correct answer: C

23. Which of the following is false regarding anal glands?


a. These are found in the anal mucosa.
b. They drain via ducts into anal sinuses at the level of the dentate line.
c. They are widely considered as the potential source of anal sepsis.
d. They secrete mucus, which helps to ease defecation.

- 235 -
Correct answer: A

24. Which of the following statements regarding pilonidal sinus is false?


a. It is usually acquired.
b. The primary sinuses are always in the midline between the sacrococcygeal joint and
the tip of coccyx.
c. Bascom’s procedure involves a midline incision directly over the sinus cavity.
d. It is more common in women.
Correct answer: D

25. Which of the following statements about anal incontinence is false?


a. The most common cause of anal sphincter disruption is obstetric damage.
b. Anal incontinence cannot occur in the absence of sphincter disruption.
c. Double overlap repair is the standard technique to repair discrete muscle disruption.
d. Sacral nerve stimulation works by neurophysiological modulation of the hindgut by
stimulation of the sacral nerve roots.
Correct answer: B

26. Which of the following statements about hemorrhoids is true?


a. Most hemorrhoids are congenital.
b. Pain is rare in uncomplicated grade 1 hemorrhoids.
c. Piles which remain permanently prolapsed are called grade 3 hemorrhoids.
d. Profuse haemorrhage is never seen.
Correct answer: B

27. Which of the following statements regarding management of piles is false?


a. Banding of piles involves application of tight elastic bands onto the base of the
pedicle which cause ischemic necrosis of the piles.
b. Cryotherapy and infrared coagulation are also methods of treating piles.
c. Milligan–Morgan hemorrhoidectomy is a closed technique where the piles are
excised, and the mucosa completely closed.
d. Stapled hemorrhoidectomy is a method where a special stapling gun is used to
excise a strip of mucosa and submucosa.
Correct answer: C

28. Which of the following statements regarding pruritus ani is false?


a. It is a common problem.
b. Threadworms need to be excluded, especially in young patients.
c. Skin biopsies may sometimes be needed to confirm diagnosis.
d. Washing the area with soap can be helpful.
Correct answer: D

29. Which of the following statements regarding perianal abscesses is true?


a. They present as a painful, throbbing swelling in the perianal area.
b. They are always associated with an underlying anal fistula.
c. Fistulotomy is advised if a fistula is found at the time of draining the abscess.
d. Finding Gram-positive organisms on pus culture is associated with an underlying
anal fistula.
Correct answer: A

- 236 -
30. Which of the following statements regarding anal warts is false?
a. They are associated with HPV infection.
b. The incidence has decreased over the last three decades.
c. Acetic acid is helpful in diagnosis.
d. Treatment options include 25 per cent podophyllin and surgical excision
Correct answer: B

31. The risk factors for anal squamous cell carcinoma (SCC) include the
following except:
a. Human papillomavirus (HPV) infection
b. HIV infection
c. Renal transplant
d. Other genital cancers
e. Rectal cancer.
Correct answer: E

32. Which of the following statements regarding anal cancers is false?


a. They constitute less than 2 per cent of all bowel cancers.
b. They are usually SCCs.
c. The primary groups of lymph nodes are the iliac nodes in the pelvis.
d. Pain and bleeding are the most common symptoms.
Correct answer: C

33- Correct statements about Hirschsprung's disease include the following except
that it
a. ls due to congenital absence of ganglion cells from the rectum and port
of the colon.
b. Presents with constipation since birth.
c. ls characterized by o tight empty rectum on rectal examination.
d. Rarely needs rectal biopsy for definitive diagnosis.
e. Always requires surgical treatment.
Correct answer: D

34- Sigmoid volvulus is characterized by the following features except:


a. Occurring typically in elderly men.
b. Acute onset of severe cramping abdominal pain.
c. Absence of abdominal distension.
d. Absolute constipation with distressing tenesmus.
e. Characteristic signs in the barium enema.
Correct answer: C

35- Which statement among the following is untrue about cancer of right colon ?
a. ls commonest in the coecum.
b. Rarely presents with intestinal obstruction.
c. Moy present with anemia of obscure origin.
d. ls rarely palpable on abdominal examination.
e. ls best treated by extended right colectomy.

- 237 -
Correct answer: D

36- Carcinoma of the left colon differs from that of the right colon in the following
except that it:
a. ls usually a stenosing scirrhous lesion.
b. Frequently presents with intestinal obstruction.
c. Rarely causes diarrhea.
d. ls usually impalpable on abdominal examination.
e. Carriers a better prognosis after radical resection.
Correct answer: C

37- An elderly male with history of habitual constipation presented because of


progressive abdominal distension with diahrea and tenesmus. He should be
suspected to suffer from:
a. Carcinoma of the rectum.
b. None-specific ulcerative colitis.
c. Amoebic colitis.
d. Crohn's disease.
e. Proctocolitis.
Correct answer: A

38- The best screening investigation for cancer of the colon in the general
population is:
a. Abdominal ultrasound.
b. Barium enema.
c. Stool occult blood test.
d. Carcinoembryonic antigen (CEA) assay.
e. Colonoscopy.
Correct answer: C

39- Concerning infantile intussusception the following statements are true except
that it:
a. Usually occurs during the first year of life.
b. Causes recurrent attacks of severe colicky abdominal pain.
c. ls characterized by the passage of "red-current jelly" stool.
d. Can be diagnosed without radiological examination.
e. Always requires urgent operation.
Correct answer: E

40- As regard treatment of congenital megacolon:


a- Mild cases should be prepared for elective surgery
b- Emergency cases need urgent decompression for fear of Obstructive
enterocolitis.
c- Duhamel's operation preserves stretch receptors and rectal capacity
d- All ore true
Correct answer: D

- 238 -
41- Complications of congenital megacolon include the following except:
a- Failure to thrive
b- Bad chest
c- Cancer colon
d- Obstructive enterocolitis.
Correct answer: C

42- Hirschsprung disease may presents by the following except:


a- No meconium for 24 hours
b- Persistent non-bile stained vomiting since birth
c- Chronic constipation relieved only by glycerin suppositories or mother
finger
d- Acute intestinal obstruction
Correct answer: B

43- The commonest cause of acquired megacolon is:


a- Bad bowel habits
b- Anal fissure
c- Amoebiasis
d- Bilharziasis
Correct answer: A

44- The usual presentation of uncomplicated diverticulosis is:


a- Asymptomatic
b- Dull aching right iliac fossa pain
c- Fever and throbbing pain
d- Bleeding per rectum
Correct answer: A

45- Which of the following is not true of diverticular disease:


a- lt is more common in the United States and Western Europe than in
Asia and Africa.
b- A low-fiber diet may predispose to development of diverticulosis.
c- lt involves sigmoid colon in more than 90% of patients.
d- sixty percent develop diverticulitis sometime during their lifetime.
e- lt is the most common cause of massive lower gastrointestinal
hemorrhage.
Correct answer: D

46-Which of the following statements regarding the risk of cancer in the context
of ulcerative colitis is/are correct?
a- After 10 years of active disease, the risk of cancer approximates 20%
to 30%

- 239 -
b- After 10 years of active disease, the risk of cancer approximates 27" to
3%
c- The risk of colon cancer in ulcerative colitis is identical to controls
d- After 20 years of disease activity, the risk of colon cancer
approximates 80%
Correct answer: B

47- Psudopolyps in ulcerative colitis:


a- ls highly precancerous
b- ls on indication for total proctoclectomy
c- Consists of edematous mucosa surrounded by ulcers
d- All of the above
Correct answer: D

48-Barium enema findings suggestive of ulcerative colitis are all Except:


a- Loss of haustrations.
b- Granular mucosa.
c- Sow tooth appearance.
d- Pseudo polyps.
Correct answer: C

49- As regard crohn's disease All are correct except:


a- Associated with gall stones in many cases
b- Most acceptable etiological theory is autoimmune
c- ls has a transmural affection
d- lt does not increase incidence of colonic carcinoma
Correct answer: D

50- Gardner's syndrome includes all the following except:


a- Fomilial polyposis coli
b- Osteoma of the mandible
c- Desmoid tumors
d- Hyperparathyroidism
e- Subcutaneous cysts
Correct answer: D

51-The following inherited disorder is autosomal recessive:


a- Peutz-jeger's syndrome
b- Familial polyposis coli
c- Turcot's syndrome
d- Gardner syndrome
Correct answer: C

- 240 -
52- The most common site for cancer colon is:
a- Coecum
b- Ascending colon
c- Transverse colon
d- Descending colon
e- Sigmoid and rectum
Correct answer: E

53- Duke's classification:


a- Helps to decide operability of the case
b- Duke's B means tumor limited to colon
c- Duke's C means nodal involvement
d- lt is a clinica-pathological classification
Correct answer: C

54- What has been found to be an acceptable screening technique for detecting
recurrent colon cancer:
a- Screening sigmoidoscopy.
b- Screening the stool for occult blood.
c- Stool cytology.
d- Measurement of carcinoembryonic antigen (CEA) levels.
e- Colonoscopy.
Correct answer: D

55 True statements regarding intestinal carcinoma that arises following ulcerative


colitis include the following except:
a- lt is more malignant thon the carcinoma that occurs in otherwise
normal adults.
b- The incidence of carcinoma increases with the duration of active
ulcerative colitis.
c- The carcinoma occurs only in the rectum.
d- The carcinoma is frequently multicentric.
Correct answer: A

56- Which of the following is a precancerous state in the large bowel:


a- Diverticular disease
b- Bilharzial Colitis
c- Peutz-Jegher's syndrome
d- Gardner's syndrome
Correct answer: C

- 241 -
57. A 25-year-old male develops diarrhea and colicky abdominal pain. Ulcertive
colitis is diagnosed on colonoscopy. Which of the following findings is consistent
with the diagnosis?
(A) The rectum is not involved.
(B) The disease is confluent, there are no skip areas in the colon and the rectum is
involved.
(C) The full thickness of the bowel wall is involved.
(D) Microscopic examination of the mucosa reveals normal cells without evidence of
dysplasia.
(E) The incidence of colorectal cancer is equal to that of the general population
Correct answer: B

58. A 35-year-old man has known ulcerative colitis. Which of the following is an
indication for total proctocolectomy?
(A) Occasional bouts of colic and diarrhea
(B) Sclerosing cholangitis
(C) Toxic megacolon
(D) Arthritis.
(E) Iron deficiency anemia
Correct answer: C

59.A 55-year-old man presents with left lower quadrant (LLQ) abdominal pain of
2-day duration, associated with constipation. On physical examination, he has
tenderness localized to the LLQ with fullness in that area leukocyte count is 22,000
and temperature is 101.5°F. Which would be the best diagnostic study to evaluate
this man?
(A) Diagnostic laparoscopy
(B) Barium enema
(C) Plain abdominal roentgenogram
(D) Computed tomography (CT) of the abdomen/pelvis with orally (PO) and
intravenous (IV) contrast
(E) Colonoscopy
Correct answer: D

60.A patients CT scan reveals diverticulitis confined to the sigmoid colon. There is
no associated pericolic abscess. What is best course of treatment?
(A) Bowel rest, nasogastric suction, IV fluids, and broad spectrum antibiotics
(B) Urgent surgical resection
(C) Steroids
(D) Diverting colostomy
(E) Ileostomy
Correct answer: A

61. The standard initial therapy for acute sigmoid volvulus is:
(A) Laparotomy to reduce the volvulus and replace the sigmoid colon to its normal
position
(B) IV neostigmine
(C) Colonoscopy
(D) Ileostomy
(E) Rigid sigmoidoscopy

- 242 -
Correct answer: E

62. A 30-year-old male is diagnosed with Peutz- Jeghers syndrome. What findings are
consistent with the diagnosis?
(A) Adenomas
(B) Hamartomas
(C) Adenomatous polyps
(D) Villoglandular polyps
(E) Villotubular polyps
Correct answer: B

63. A 12-year-old boy complains of pain in the lower abdomen (mainly on the right
side). Symptoms commenced 12 hours before admission. He had noted anorexia
during this period. Examination revealed tenderness in the right iliac fossa, which was
maximal 1 cm below Mc Burney’s point. In appendicitis, where does the pain
frequently commence?
(A) In the right iliac fossa and remains there
(B) In the back and moves to the right iliac fossa
(C) In the rectal region and moves to the right iliac fossa
(D) In the umbilical region and then moves to the right iliac fossa
(E) In the right flank
Correct answer: D

64. On examination, patients presenting with appendicitis typically show maximal


tenderness over which of the following?
(A) Inguinal region
(B) Immediately above the umbilicus
(C) At a point between the outer one-third and inner two-thirds of a line between the
umbilicus and the anterior superior iliac spine
(D) At a point between the outer two-thirds and inner one-third of a line between the
umbilicus and the anterior superior iliac spine
(E) At the midpoint of a line between the umbilicus and the anterior superior iliac
Spine
Correct answer: C

65. A 29-year-old woman presents to her physician’s office with pain in the right iliac
fossa. Examination reveals tenderness in this region. Her last menstrual cycle was 2
weeks previously and findings on gynecologic examination and leukocyte count are
normal. A provisional diagnosis of acute appendicitis is made. She should be
informed that operations to treat this condition reveal acute appendicitis in what
percentage of cases?
(A) A small percentage of cases
(B) 50–89% of cases
(C) 90–99% of cases
(D) More than 99% of cases
(E) No reliable statistics are available
Correct answer: C

- 243 -
66. A 28-year-old man is admitted to the emergency department complaining of pain
in the umbilical region that moves to the right iliac fossa. Which is a corroborative
sign of acute appendicitis?
(A) Referred pain in the right side with pressure on the left (Rovsing) sign
(B) Increase of pain with testicular elevation
(C) Relief of pain in lower abdomen with extension of thigh
(D) Relief of pain in lower abdomen with internal rotation of right thigh
(E) Hyperanesthesia in the right lower abdomen
Correct answer: A

67. A 54-year-old man with diarrhea is found to have ulcerative colitis. Colectomy
should be advised in patients with ulcerative colitis who have symptoms that persist
for more than which of the following?
(A) 1 month
(B) 6 months
(C) 1–5 years
(D) 10–20 years
(E) More than 25 years
Correct answer: D

68. A 48-year-old woman develops colon cancer. She is known to have a long history
of ulcerative colitis. In ulcerative colitis, which of the following is a characteristic of
colon cancer?
(A) Occurs less frequently than in the rest of the population.
(B) Is more likely to occur when the disease is confined to the left colon.
(C) Occurs equally in the right and left side.
(D) Has a synchronous carcinoma in 4–5% of cases.
(E) Has an excellent prognosis because of physician awareness
Correct answer: C

69. A72-year-old woman is scheduled to undergo right hemicolectomy for cancer of


the cecum. In this condition, she can anticipate subsequent recurrence
(A) Of 20–30% if confined to the mucosa
(B) Close to 100% if there is lymph node involvement
(C) Which will not result in small-bowel obstruction
(D) Which will not result in hydronephrosis
(E) Which with microscopic lymph node metastasis would have a lower rate than that
with macroscopic spread
Correct answer: E

70. An 83-year-old man is diagnosed on colonoscopy to have cancer of the colon. He


refuses surgical intervention and after a 3-month follow up period is admitted to the
emergency department with large-bowel obstruction. Carcinoma of the colon is most
likely to obstruct if found in the
(A) Cecum
(B) Ascending colon
(C) Descending colon
(D) Rectum
(E) Transverse colon

- 244 -
Correct answer: C

71. A43-year-old man is seen in his physician’s office for severe pain in the perineum.
Examination reveals exquisite tenderness in the area to the right side of the anal verge
due to a perianal abscess. Rectal examination is refused. What should be the next step
in management?
(A) Drainage of the abscess in the office under local anesthesia.
(B) Excision of the vertical fold of Morgagni.
(C) Drainage under general anesthesia and immediate colonoscopy.
(D) CT scan of the abdomen.
(E) Insertion of a rectal tube.
Correct answer: A

72.A 64-year-old man undergoes CEA surveillance for cancer, because his brother
and father both had colon cancer. What information should he be provided?
(A) CEA is highly sensitive for diagnosis.
(B) If CEA is elevated preoperatively, it implies unresectable disease.
(C) Increases in CEA after resection may indicate tumor recurrence.
(D) CEA is highly specific for the presence of colon cancer.
(E) CEA is present in normal adult colonic mucosa
Correct answer: C

73. A 70-year-old man presents with pallor and breathlessness on exertion. He does
not complain of abdominal pain. He has microcytic, hypochromic anemia. What is the
most probable cause?
(A) Diverticulosis of the colon
(B) Peptic ulcer disease
(C) Crohn’s disease
(D) Ulcerative colitis
(E) Carcinoma of the right colon
Correct answer: E

74. A 25-year-old man has recurrent, indolent fistula in Ano. He also complains of
weight loss, recurrent attacks of diarrhea with blood mixed in the stool, and tenesmus.
Proctoscopy revealed a healthy, normal-appearing rectum. What is the most likely
diagnosis?
(A) Crohn’s colitis
(B) Ulcerative colitis
(C) Amoebic colitis
(D) Ischemic colitis
(E) Colitis associated with acquired immunodeficiency syndrome (AIDS)
Correct answer: A

75. A 65-year-old woman with a history of chronic constipation is transferred from a


nursing home because of abdominal pain and marked abdominal distention. On
examination, her abdomen is found to be distended and tender in the LLQ. What is the
most likely diagnosis?
(A) Appendicitis
(B) Carcinoma of the colon
(C) Volvulus of the sigmoid colon

- 245 -
(D) Volvulus of the cecum
(E) Small-bowel obstruction
Correct answer: C

76. A 40-year-old man with a long history of bloody diarrhea presents with increased
abdominal pain, vomiting, and fever. On examination, he is found to be dehydrated
and shows tachycardia and hypotension. The abdomen is markedly tender with
guarding and rigidity. What is the most likely cause?
(A) Toxic megacolon in ulcerative colitis
(B) Small-bowel perforation from regional enteritis
(C) Perforated carcinoma of the sigmoid colon
(D) Volvulus of the sigmoid colon
(E) Acute perforated diverticulitis
Correct answer: A

77. A 55-year-old woman presents with pain in the LLQ of the abdomen and fever of
102°F. On examination, she is found to be dehydrated and has tenderness in the LLQ.
A CT scan shows a mass in the LLQ involving the sigmoid colon. There is a minimal
amount of free fluid and no free air. What should the initial treatment of this patient
include?
(A) IV fluids, penicillin, and steroids
(B) IV fluids, cefoxitin, and nasogastric drainage
(C) IV fluids, blood transfusion, and laparotomy
(D) Immediate laparotomy
(E) Bowel preparation followed by Laparotomy
Correct answer: B

78. A 60-year-old man complains of recurrent attacks of painless rectal bleeding.


Colonoscopy reveals normal mucosa between the cecum and the anal verge. What is
the most helpful test to determine the cause of bleeding?
(A) Angiography to look for angiodysplasia
(B) Technetium scan for Meckel’s diverticulum
(C) Upper GI endoscopy for peptic ulcer
(D) Small-bowel series for tumor
(E) Ultrasound for abdominal aortic Aneurysm
Correct answer: A

79. A 64-year-old man has a benign lesion of the colon. He is informed that the lesion
does not predispose to colon cancer. What is the lesion he has?
(A) Ulcerative colitis
(B) Villous adenoma
(C) Hyperplastic polyp
(D) Adenoma in familial polyposis
(E) Colon mucosa in a patient with colon Carcinoma
Correct answer: C

80- The least common site for cancer in the alimentary tract is:
a. Pharynx.
b. Esophagus.
c. Stomach.
- 246 -
d. Small bowel.
e. Colon and rectum.
Correct answer: D

81- The following is/are risk factors for volvulus of pelvic colon except:
a. Old age.
b. Chronic constipation.
c. Short sigmoid.
d. Adhesions.
Correct answer: C

82 . Colonic Pseudo-obstruction occurs in all, except -


A. Diabetes mellitus
B. Dermatomyositis
C. Scleroderma
D. Hyperthyroidism
Correct answer: D

83 . Transmural inflammation with skip lesions in colon are characteristic of -


A. Regional ileitis (Crohn's disease)
B. Ischemic colitis
C. Ulcerative colitis
D. Non specific colitis
Correct answer: A

84 . An elderly woman is admitted with weakness, weight loss, anemia and a


palpable abdominal mass. She has a colon carcinoma. The most likely anatomic
site is -
A. Rectum
B. Sigmoid colon
C. Left colon
D. Transverse colon
Correct answer: D

85 . 60 year old male with CA descending colon presents with acute intestinal
obstruction. In emergency department the treatment choice is -
A. Defunctioning colostomy
B. Hartman's procedure
C. Total colectomy
D. Lt hemicolectomy
Correct answer: B

86 . Not true about Familial polyposis coli is:


A. Loss of APC gene
B. Results from defect in colonic mucosa leading to abnormal proliferative pattern
C. Associated with abnormal mucins
D. Decreased expression of c-myc gene
Correct answer: D

- 247 -
87 . Carcinoma right colon is most commonly of which type -
A. Stenosing
B. Ulcerative
C. Tubular
D. Fungating
Correct answer: D

88 . What is the most commonly involved in carcinoma colon -


A. Caecum
B. Rectosigmoid colon
C. Transverse colon
D. Ascending colon
Correct answer: B

89 . Most common cause of a cola-vesical fistula in an adult male would be:


A. TB
B. Cancer colon
C. Crohns disease
D. Ulcerative colitis
Correct answer: C

90 . In which case Anterior resection is the method of treatment -


A. Ca sigmoid colon
B. Ca rectum
C. Ca colon
D. CA anal canal
Correct answer: B

91 . Aganglionic segment is encountered in which part of colon in case of


Hirchsprung's disease:
A. Distal to dilated segment
B. In whole colon
C. Proximal to dilated segment
D. In dilated segment
Correct answer: A

92 . A report of 'lead-pipe' appearance of the colon barium enema would be


diagnostic of -
A. Amebiasis
B. Ulcerative colitis
C. Tuberculosis of the colon
D. Agranulomatous polyposis
Correct answer: B

93 . Which of the following colonic polyps is not premalignant -


A. Villous adenomas
B. Tubular adenomas
C. Juvenile polyps
D. Hamartomatous potyps associated with Peutz-Jegher's syndrome
Correct answer: C

- 248 -
94 . Based on Epidemiological studies, which of the following has been found to
be most protective against Carcinoma Colon:
A. Low selenium diet
B. Low protein diet
C. High fiber diet
D. Low fat diet
Correct answer: C

95 . In Emergency department, a 70 year old male presents with acute


obstruction. He has Ca. descending colon, M/n of choice:
A. Defunctioning colostomy
B. Total Colectomy
C. Hartmann's procedure
D. Abdominoperineal resection
Correct answer: C

96 . Patient having CA colon of left side, comes to emergency department with


obstruction. What will be the best treatment to be given -
A. Hartman's procedure
B. Defunctioning colostomy
C. Ileotransverse anastomosis
D. Transverse colostomy
Correct answer: A

97 . Colonic polyps which have no risk for malignancy is -


A. Juvenile polyposis syndrome
B. Familial adenomatous polyposis syndrome
C. Juvenile polyps
D. Hamartomatous polyps associated with Peutz-Jegher's syndrome
Correct answer: C

98 . A 25 years male, on tour, presents with bright red painless bleed since 7 days
and abdominal pain. External and per rectal examinations are normal. As
attending general practioner, your next step is-
A. Refer to surgeon for sigmoidoscopy
B. Refer to gastroenterologist for colonoscopy
C. Proctoscopy
D. Do a barium enema
Correct answer: C

99 . Based on Epidemiological studies, which of the bellow has been seen to be


most protective against CA colon?
A. Low selenium diet
B. Low protein diet
C. High fiber diet
D. Low fat diet
Correct answer: C

- 249 -
100 . True statements about congenital megacolon include all of the following
except:
A. Loud borborygmi
B. Dilatation and hyper trophy of pelvic colon
C. Large stool
D. Symptoms appear within 3 days following birth
Correct answer: D

101 . In which of the following conditions acquired(secondary) megacolon is seen


A. Absence of sympathetic ganglion cells
B. Rectal malignancy
C. Fissure in-ano
D. Complete absence of parasympathetic ganglion cells
Correct answer: B

102 . Gardener's syndrome is a rare hereditary disorder involving the colon,


which is characterized by:
A. Polyposis in jejunum, pituitary adenoma and skin tomours.
B. Polyposis colon, cancer thyroid, skin tumours.
C. Polyposis of gastrointestinal tract, cholangiocarcinoma and skin tumours.
D. Polyposis colon, osteomas, epidermal inclusion cysts and fibrous tumours in the
skin.
Correct answer: D

103 . Villous adenoma of colon causes all except:


A. Hypokalemia
B. Watery diarrhea
C. Hypocalcemia and hypophosphatemia
D. Prolapse with bleeding
Correct answer: C

104 . Which of the following is associated with Lynch syndrome-?


A. Breast CA, Endometrial CA and Ovarian CA
B. Breast CA, Stomach CA and Colon CA
C. Endometrial CA, Colon CA and Ovarian CA
D. Breast CA, Colon CA and Ovarian CA
Correct answer: C

105 . Following genes may be involved in development of CA colon, except-


A. Beta-Catenin
B. APC
C. Mismatch Repair Genes
D. K-ras
Correct answer: A

106 . Which of the following carries the least risk of Colonic malignancy ?
A. Familial adenomatous polyposis
B. Gardner's syndrome
C. Villous adenoma

- 250 -
D. Peutz-Jegher's syndrome
Correct answer: D

107 . Chance of development of colonic malignancy in all of the following


condition except-
A. FAP
B. Villus adenoma
C. Pseudopolyp in ulcerative colitis
D. Lynch syndrome
Correct answer: C

108 . Which of the following colonic polyps is not premaliganant -


A. Villous adenoma
B. Tubular adenomas
C. Juvenile polyps
D. Hamartomatous potyps associated with Peutz-Jegher's syndrome
Correct answer: C

109 . A patient suffered bullet injury to left side of the colon and presented in the
causality department after 12 hour. What will be the m/n(management):
A. Proximal defunctioning colostomy
B. Primary closure
C. Proximal colostomy and bringing out the distal end as mucus fistula
D. Resection and primary anastomosis
Correct answer: C

110 . Commonest site for volvulus is ?


A. Caecum
B. Stomach
C. Proximal jejunum
D. Sigmoid colon
Correct answer: D

111 . The diagnosis of congenital megacolon is confirmed by -


A. Clinical features
B. Barium enema
C. Rectal biopsy
D. Recto - sigmoidoscopy
Correct answer: C

112 . Colon cancer is seen in people taking ?


A. High fiber diet
B. High fat diet
C. High protein diet
D. Low fiber diet
Correct answer: B

- 251 -
113 . In which case immunoguided surgery is done -
A. CA colon
B. CA pancreas
C. CA jejunum
D. CA anal canal
Correct answer: A

114. Adjuvant chemotherapy is of definite value in -


A. CA gall bladder
B. CA oesophagus
C. CA colon
D. CA pancreas
Correct answer: C

115 . A middle aged man sustains a bullet injury to abdomen leading to


perforation of left colon THE TREATMENT of choice is-
A. Primary closure of fistula
B. Hemicolectomy
C. Abdominal colostomy and mucus fistula
D. Primary closure with drainage
Correct answer: C

116 . The overall survival is increased by screening procedure in which disease?


A. Lung cancer.
B. Prostate cancer.
C. Ovarian cancer.
D. Colon cancer.
Correct answer: D

117 . In a child who disease is misdiagnosed as appendicitis.


A. Gastroenteritis
B. Lymphadenitis
C. Intussusception
D. All of the above
Correct answer: D

118 .Most commonly associated with high risk for carcinoma colon is ?
A. Peutz-Jegher syndrome
B. Juvenile Familial Polyposis
C. Familial Adenomatous Polyposis
D. Lynch syndrome
Correct answer: C

119 . Obstructive symptoms are most common in lesions of -


A. Transverse colon
B. Ascending colon
C. Descending colon
D. Jejunum
Correct answer: C

- 252 -
120 . A three year old male child presents with history of constipation and
abdominal distension for the last two years. The plain radiograph of abdomen
reveals fecal matter containing distended bowel loops. A barium enema study done
subsequently shows a transition zone at the recta-sigmoid junction with reversal
of recta-sigmoid ratio. The most probable diagnosis is:
A. Anal atresia
B. Malrotation of the gut
C. Hirschsprung's disease
D. Congenital megacolon
Correct answer: C

121 . An elderly male presents with alteration of bowel habits, features of intestinal
obstruction along with hematochezia. Probable diagnosis is:
A. Tuberculosis
B. Diverticulosis
C. Ulcerative colitis
D. Colon carcinoma
Correct answer: D

122 . Risk factor for carcinoma colon:


A. Low fiber diet
B. Low fiber diet and high fat content
C. High fatty meal
D. Smoked fish
Correct answer: B

123 . Colonic polyps have no risk for malignancy is-


A. Juvenile polyposis syndrome
B. Familial adenomatous polyposis syndrome
C. Juvenile polyps
D. Hamartomata’s polyps associated with Peutz - Jegher's syndrome
Correct answer: C

124 . Which screening test for colon cancer is proven effective in RCT ?
A. Colonoscopy
B. Flexible Sigmoidoscopy
C. Occult blood in stool
D. Upper GI endoscopy
Correct answer: A

125 . Massive colonic bleeding in a patient with Diverticulosis is from:


A. Superior mesentric artery
B. Inferior mesentric artery
C. Gastro-duodenal artery
D. Coeliac artery
Correct answer: A

- 253 -
126 . A 50 years female has not passed stools for the last 14 days. X-ray shows no
air fluid levels. Possible diagnosis is:
A. Aganglionosis of the colon
B. Paralytic ileus
C. Duodenal obstruction
D. Intestinal pseuda-obstruction
Correct answer: D

127 . Which of the following has least malignant potential


A. Juvenile polyps in Juvenile Polyposis Syndrome
B. Hamartomatous Polyps in Peutz Jaghers Syndrome
C. Adenomatous polyps in HNPCC
D. Adenomatous polyps in Familial colonic Polyposis
Correct answer: B

128 . Most operative ileus is maximum in -


A. Colon
B. Ileum
C. Duodenum
D. Stomach
Correct answer: A

129 . Colonic diverticulosis is best diagnosed by -


A. Nuclear scan
B. Colonscopy
C. CT scan
D. Barium enema
Correct answer: D

130 . True regarding Ca colon is:


A. Lesion on Lt. side of the colon presents with features of anemia
B. Mucinous Ca. has a good prognosis
C. Duke's A stage should receive adjuvant chemotherapy
D. Solitary liver metastasis is not a C/I for surgery
Correct answer: D

131 . Complete bowel preparation is done in a case of-


A. Colonic carcinoma
B. Hirchsprung's disease
C. Irritable bowel disease
D. Ulcerative colitis
Correct answer: A

132 . Terminal group of lymph node for colon is-


A. Epicolic
B. Paracolic
C. Ileocolic
D. Preaortic
Correct answer: D

- 254 -
133 . Following genetic counselling in a family for Familial polyposis coli(FPC)
next screening test is -
A. Occult blood in stools
B. APC gene
C. Flexible sigmoidoscopy
D. Colonoscopy
Correct answer: B

134 . Commonest site for ischemic colitis is -


A. Hepatic flexure
B. Splenic flexure
C. Descending colon
D. Ascending colon
Correct answer: B

135 . Colo- vesical fistula most commonly arises from:


A. Carcinoma colon
B. Abdomina-perineal resection
C. Crohn's disease
D. Ulcerative colitis
Correct answer: C

136 . The tendency of colonic carcinoma to metastasize is best assessed by:


A. Size of tumor
B. Proportion of bowel circumference involved
C. Carcinoembryonic antigen (CEA) levels
D. Depth of penetration of bowel wall
Correct answer: D

137 . The defective migration of neural crest cells results in -


A. Adrenogenital hypoplasia
B. Dentinogenetic imperfecta
C. Congenital megacolon
D. Albinism
Correct answer: C

138 . Risk factors for colonic carcinoma in an adenomatous polyp except:


A. Size > 2cm
B. Atypia
C. Pedunculated polyp.
D. Villous histology
Correct answer: C

139 . In a child which disease is misdiagnosed as appendicitis ?


A. Gastroenteritis
B. Lymphadenitis
C. Intussusception
D. All of the above
Correct answer: D

- 255 -
140. Prolonged intake of refined foods leads to which of the following carcinoma
A.stomach
B.gall bladder
C.colon
D.pancreas
Correct answer C

141 . Most important prognostic factor for colorectal carcinoma is


A. Stage of lesion
B. Site of lesion
C. Lymph node status
D. Age of patient
Correct answer: A

142 . CEA level used in colorectal carcinoma-


A. To detect recurrence following surgery
B. For screening test of colorectal carcinoma
C. Complete disappearance means complete removal of tumor
D. Indicate metastasis
Correct answer: A

13- Vermiforme Appendix

1 . True regarding carcinoid of appendix is -


A. Most commonly occurs at the base of the appendix
B. Most common neoplasm of the appendix
C. Most cases require right hemicolectomy
D. Metastases are quite common
Correct answer: B

2 . Most common neoplasm of appendix is -


A. Lymphoma
B. Adenocarcinoma
C. Leiomyosarcoma
D. Argentaffin Oma
Correct answer: D

3 . TRUE regarding carcinoid of appendix is -


A. Most commonly occurs at the base of the appendix
B. Most common neoplasm of the appendix
C. Most cases require right hemicolectomy
D. Metastases are quite common
Correct answer: B

- 256 -
4 . All are true about appendix except:
A. Most malignancies are adenocarcinoma
B. Carcinoid is found in 0.5% cases of excised patients
C. Pseudomyxoma peritonei occurs following rupture of mucocele of appendix
D. Hot flush, diarrhea, and bronchospasm may occur
Correct answer: A

5 . What is the treatment of patient with carcinoid tumor of appendix of size


more than 2 cm:
A. Right hemicolectomy
B. Appendicectomy
C. Appendicectomy + abdominal CT scan
D. Appendicectomy + 24 hrs urinary HIAA
Correct answer: A

6 . Which of the following organisms produces signs and symptoms that mimic
acute appendicitis ?
A. Enteropathic E. coli
B. Enterobius vermicularis
C. Trichomonas hominis
D. Yersinia enetrocolitica
Correct answer: B

D.D of acut abdomen

14- Rectum and anal canal

Imperforate anus

1. Which of the following statements regarding the congenital anomalies of the


anal canal is false?
a. The dorsal part of the cloacal membrane is called the anal membrane.
b. Imperforate anus is divided into two types – high and low – depending on the level
of rectal termination in relation to the urinary bladder.
c. Low defects are easier to correct but more prone to constipation.
d. Postanal dermoid usually remains asymptomatic until adult life.
e. A postanal dimple is of no clinical significance.
Correct answer: B

- 257 -
2. Most common cause of death in imperforate anus is:
a. Toxemia.
b. Peritonitis.
c. Electrolyte imbalance.
d. Associated anomalies.
Correct answer: D

Anatomy, Anal fissure, Hemorrhoids, Rectal prolapse

1 . Anal fissure best diagnosed by:


A. History and superficial clinical examination
B. Anoscope
C. US
D. PR examination
Correct answer: A

2- Which of the following is NOT CORRECT regarding anal fissure ?


A. An elongated ulcer in the mid-line of anal canal
B. Painful on defecation
C. Cause mucus discharge
D. Associated with sentinel pile in acute stage
Correct answer: D

3. Multiple anal fissures are suggestive of:


a. Anal carcinoma.
b. Crohn's disease.
c. T.B.
d. Ulcerative colitis.
e. Any of the above.
Correct answer: E

4. Lateral sphincterotomy:
a. ls better than posterior sphincterotomy.
b. ls usually curative.
c. Must be combined with fissurectomy in chronic fissures.
d. All of the above.
Correct answer: D

5. The complications of chronic anal fissure include the following except:


a. Formation of "sentinel pile".
b. Dorsal abscess.
c. Anal fistula.
d. Anal contracture.
e. Malignant transformation.
Correct answer: E

- 258 -
6. Which of the following statements regarding anal fissure is false
a. It is most common in the posterior wall.
b. The common finding is a hypotonic anal sphincter.
c. Operation can be avoided in about 50 per cent of patients by the use of local agents
like glyceryl trinitrate (GTN) or Diltiazem.
d. The most important complication after lateral internal sphincterotomy is
incontinence, which may affect up to 30 per cent of patients.
Correct answer: C

7 . Which of the following is not a Causes of acute anal pain:


A. Acute anal fissure
B. Thrombosed hemorrhoids
C. Perianal abscess
D. Fistula in ano
Correct answer: D

8 . Most common causes of lower GI bleed in India is -


A. Benign tumour
B. Non specific ulcer
C. Cancer rectosigmoid
D. Hemorrhoids
Correct answer: D

9 . Which of the following is ideal for the treatment with injection of sclerosing
agents?
A. Internal hemorrhoids
B. External hemorrhoids
C. Strangulated hemorrhoids
D. Prolapsed hemorrhoids
Correct answer: A

10 . External hemorrhoids below the dentate line are -


A. Painful
B. Ligation is done as management
C. Skin tag is not seen in these cases
D. May turn malignant
Correct answer: A

11 . Pain of External hemorrhoids is carried by ?


A. Pudendal nerve
B. Perineal nerve
C. Superior rectal nerve
D. Dorsal nerve of penis or clitoris
Correct answer: A

- 259 -
12- Which of the following is accurate about the treatment of internal
hemorrhoids by grade?
A- Surgery is often recommended as a first-line treatment for grade I internal
hemorrhoids.
B- Stapled hemorrhoid surgery is typically the initial treatment for grade II or II
internal hemorrhoids.
C- Significantly symptomatic grade III and grade IV hemorrhoids are best treated
with surgical hemorrhoidectomy.
D- Surgical consultation is typically unnecessary in grade IV internal hemorrhoids,
because in-office procedures are still the recommended choice.
Correct answer: C

13. The following disease(s) can result in secondary hemorrhoids:


a. Cancer rectum.
b. Acquired megacolon.
c. Benign prostatic hyperplasia.
d. All of the above.
Correct answer: D

14. Prolapsing piles that reduces spontaneously at the end of the act is of
which degree:
a. First degree.
b. 2nd degree.
c. 3rd degree.
d. 4th degree.
Correct answer: B

15 . External hemorrhoids below the dentate line are -


A. Skin tag is not seen in these cases
B. May turn malignant
C. Painful
D. Ligation is done as management
Correct answer: C

16 . Injection sclerotherapy is ideal for which of the bellow:


A. Internal haemorrhoids
B. External haemorrhoids
C. Strangulated haemorrhoids
D. Prolapsed haemorrhoids
Correct answer: A

17 . De Lorme procedure is done for:


A. Rectal prolapse
B. Hemorrhoids prolapse
C. Rectal carcinoma
D. Hirschprung's disease
Correct answer: A

- 260 -
18 . A thrombosed external hemorrhoid(or perianal hematoma) is -
A. Painless, with graduate onset
B. The 5 day, painful , self curing lesion
C. Part of internal hemorrhoids
D. A sentinel pile
Correct answer: B

19- The following statements about partial rectal prolapse are true except that it:
a. Consists of a double layer of mucous membrane.
b. ls commonest in elderly people.
c. ls often associated with poor sphincter tone.
d. ls rarely associated with hemorrhoids.
e. ls best treated by ligature-excision of prolapsing mucosa.
Correct answer: D

20-Which of the following statement(s) about complete rectal prolapse is/are true:
a- Rectal prolapse results from intussusception of the rectum and
rectosigmoid.
b- The disorder is more common in men thon in women.
c- Continence nearly always is recovered after correction of the prolapse.
d- All of the above ore true.
Correct answer: A

21- Sigmoid volvulus has been associated with each of the following except:
a. Chronic constipation and laxative abuse.
b. Chronic rectal prolapse.
c. Chronic traumatic paralysis.
d. Medical management of Parkinson's disease.
Correct answer: B

22 . Rectal prolapse in child seen in:


A. 12 months -6 years
B. 4-6 months
C. 8 -12 months
D. 9 months -1 year
Correct answer: A

23 . Treatment of rectal prolapse in childhood is -


A. Lahaut's operation
B. Incision of prolapsed mucosa
C. Thiersch wiring
D. Ripstein operation
Correct answer: C

- 261 -
24 . A 32 years patient presents with complete rectal prolapse, which of the
following procedure is associated with lowest risk of recurrence:
A. Thiersh Procedure
B. Delorme's Procedure
C. Altemier's Procedure
D. Abdominal Rectopexy
Correct answer: D

25 . Solitary rectal ulcer is -


A. Due to anterior rectal wall prolapsed
B. Malignant
C. Always treated by surgery
D. Easy to treat
Correct answer: A

26 . A 25 year male patient presents with complete rectal prolapse, surgery of


choice is:
A. Anterior resection
B. Goodsall's procedure
C. Abdominal Rectopexy
D. Delerome's procedure
Correct answer: C

27. Which of the following is most accurate about the etiology and epidemiology
of hemorrhoids?
A-External hemorrhoids occur more commonly among older adults (>65 years) than
among young and middle-aged adults
B- Internal hemorrhoids often cause cutaneous pain because they are innervated by
cutaneous nerves.
C- Hemorrhoids have a male-to-female predilection of nearly 4:1.
D- Recognized risk factors associated with the development of hemorrhoids include
lack of erect posture, spinal cord injury, and episiotomy
Correct answer: D

28- Which of the following is an indication for complete colon evaluation in the
setting of symptomatic hemorrhoids and rectal bleeding, according to guidelines
from the American Society of Colon and Rectal Surgeons?
A- Age 45 years if no complete examination done within the past 5 years
B- Age 10 years older than the age of a single, first-degree relative with
adenocarcinoma diagnosed at younger than 50 years.
C- Age 10 years younger than the age of two first-degree relatives with colorectal
cancer.
D- Age older than 50 years, regardless of other factors, including negative fecal
immunochemical testing (FIT).
Correct answer: C

- 262 -
29- Which of the following is most accurate about the workup of hemorrhoids?
A- Anoscopy is indicated for viewing internal hemorrhoids and should include use of
a side-viewing anoscope.
B- Proctoscopy is typically preferred to anoscopy as an initial investigation in patients
with suspected internal hemorrhoids.
C- Routine histologic examination of hemorrhoidal tissue is mandatory for diagnosis.
D- Anemia due to hemorrhoidal bleeding occurs in nearly half of all patients and
helps confirm diagnosis.
Correct answer: A

30- Which of the following is most accurate about the treatment of


hemorrhoids, according to American College of Gastroenterology guidelines?
A- Excision is contraindicated in patients who present with a thrombosed external
hemorrhoid within 3 days of onset.
B- Increased fiber intake and adequate fluids are first-line treatment in patients with
symptomatic hemorrhoids.
C- Ligation is the least effective option for most patients with hemorrhoids that
remain symptomatic after other treatment.
D- Surgery is inadvisable in patients with large, symptomatic external tags along with
hemorrhoids.
Correct answer: B

Cancer rectum

1 . Commonest type of anal canal Ca is -


A. Sq. cell Ca
B. Adeno Ca
C. Adenocarcinoma
D. Papillary type
Correct answer: A

2 . Which of the following statements is true for Nigro's regimen?


A. It is a regimen for anal canal neoplasm
B. It incorporates chemotherapy with radiation as an alternative to surgery
C. Has the advantage of preserving continence
D. All of the above
Correct answer: D

3 . The preferred initial treatment for carcinoma of the anal canal is -


A. Abdominoperineal resection
B. Abdominoperineal resection followed by radiotherapy
C. Radiotherapy alone
D. Chemoradiation
Correct answer: D

- 263 -
4 . In lateral anal sphincterotomy, the following sphincter is divided -
A. Subcutaneous part of external sphincter
B. Deep part of external sphincter
C. Internal sphincter
D. Puborectalis sphincter
Correct answer: C

5 .The best acceptable operation for a rectal carcinoma at 5cm from the anal
verge is:
A. Abdominal perineal resection
B. Anterior resection
C. Local resection
D. Posterior resection
Correct answer: A

Anatomy, Anaorectal suppuration, Anal fistula

1. What is the management of a low, straight, 2cm long peri anal fistula.
A. Fistulectomy
B. Seton repair
C. Proximal colostomy
D. Anal dilatation
Correct answer A

2. Which of the following statements about anal fistula is true?


a. It may be found in association with ulcerative colitis.
b. Trans-sphincteric fistulae usually have an external opening close to the anal verge.
c. Posterior fistulae are more likely to have a curved track and a horseshoe
communication.
d. Fistulotomy is the treatment of choice in trans-sphincteric fistula involving more
than 30 per cent of the external sphincter.
Correct answer: C

Bleeding per rectum

1 . Most common cause of fresh bleeding per rectum in a 5-year-old child is -


A. Volvulus
B. Trauma
C. Worm infestation
D. Rectal polyp
Correct answer: D

- 264 -
2 . A Toddler has few drops of blood coming out of rectum.Probable diagnosis is
A. Juvenile Rectal Polyp
B. Adenoid Polyposis Coli
C. Rectal Ulcer
D. Piles
Correct answer: A

3 . All the following statements regarding malignant potential of colorectal


polyps are true except:
A. Polyps of the familial polyposis coli could invariably undergo malignant change
B. Pseudo polyps of ulcerative colitis has high risk of malignancy
C. Villous adenoma is associated with high risk of malignancy
D. Juvenile polyps have little or no risk
Correct answer: B

4 . Bleeding per rectum is present in all, except -


A. Meckel's diverticulum
B. Sigmoid volvulus
C. Carcinoma rectum
D. Ulcerative colitis
Correct answer: B

5 . Massive bleeding per rectum in a 70 year old patient is due to:


A. Colitis
B. Polyps
C. Diverticulosis
D. Carcinoma colon
Correct answer: C

6. Bleeding per rectum is recognized feature in all except:


a- Cancer colon
b- Hirschsprung disease
c- Ulcerative colitis
d- Diverticular disease
e- internal piles
Correct answer: B

15- Perineum, omentum and mesentery.

1 . A patient gives chronic history of Diarrhea and blood in stool presents with
multiple fistulae in the perineum and multiple strictures in small intestine. The
diagnosis is-
A. Crohn's disease
B. Radiation enteritis
C. Ulcerative Colitis

- 265 -
D. Ischemic bowel disease
Correct answer: A

2 . All of the following statements about Necrotizing fascitis are true, except
A. Most commonly caused by Group A beta hemolytic streptococci
B. Infection of fascia and subcutaneous tissue
C. Surgical debridement is Mandatory
D. Most common site is perineum followed by trunk and extremities
Correct answer: D

3 . Locate the renal stone with pain radiating to medial side of thigh and
perineum due to slipping of stone in males
A. At pelvic brim
B. Intramural opening of ureter
C. Junction of ureter and renal pelvis
D. At crossing of gonadal vessels and ureter
Correct answer: A

4 . Perineal hematoma after trauma is due to ?


A. Rupture of membranous urethra
B. Rupture of bulbar urethra
C. Pelvic organ blunt trauma
D. Rupture of bladder
Correct answer: B

5 . Content of epiplocele is ?
A. Omentum
B. Intestine
C. Colon
D. Urinary bladder
Correct answer: A

6. A 79-year-old man has had abdominal pain for 4 days. An operation is performed,
and a gangrenous appendix is removed. The stump is inverted. Why does acute
appendicitis in elderly patients and in children have a worse prognosis?
(A) The appendix is retrocecal.
(B) The appendix is in the preileal position.
(C) The appendix is in the pelvic position.
(D) The omentum and peritoneal cavity appear to be less efficient in localizing the
disease in these age groups.
(E) The appendix is longer in these age groups
Correct answer: D

7 . Most common position of appendix is ?


A. Pelvic
B. Paracolic
C. Retrocaecal
D. Retroperitoneal
Correct answer: C

- 266 -
8. All of the following conditions are derived from the primitive embryonic
foregut except:
A. Bronchogenic cyst.
B. Cystic adenomatoid malformation.
C. Gastric duplication.
D. Mesenteric cyst.
E. Pulmonary sequestration.
Correct answer: D

9 . Mesenteric tumors are -


A. Usually solid
B. Usually cystic
C. Highly malignant
D. Highly vascular
Correct answer: B

10 . Treatment of choice in a case of low, straight perianal fistula of 2 cm in size:


A. Fistulotomy
B. Excision of rectum
C. Dilatation
D. Sphincterotomy
Correct answer: A

11 . Acute mesenteric lymphadenitis is caused by


A. E.coli
B. Alpha-hemolytic streptococci
C. Hemophilus
D. Yersinia
Correct answer: D

16- Hernia

Anatomy and surgical importance

1. The most common hernia in females is:


A. Femoral hernia.
B. Direct inguinal hernia.
C. Indirect inguinal hernia.
D. Obturator hernia.
E. Umbilical hernia.
Correct answer: C

- 267 -
2. Which of the following statements about the causes of inguinal hernia is correct?
A. Excessive hydroxyproline has been demonstrated in the aponeuroses of hernia
patients.
B. Obliteration of the processes vaginalis is a contributing factor for the development
of an indirect inguinal hernia.
C. Physical activity and athletics have been shown to have a protective effect toward
the development of inguinal hernias.
D. Elevated levels of circulating serum elastalytic activity have been demonstrated in
patients with direct herniation who smoke.
E. The majority of inguinal hernias are acquired.
Correct answer: D

3. The following statements about the repair of inguinal hernias are true except:
A. The conjoined tendon is sutured to Cooper's ligament in the Bassini hernia repair.
B. The McVay repair is a suitable option for the repair of femoral hernias.
C. The Shouldice repair involves a multilayer, imbricated repair of the floor of the
inguinal canal.
D. The Lichtenstein repair is accomplished by prosthetic mesh repair of the inguinal
canal floor in a tension-free manner.
E. The laparoscopic transabdominal preperitoneal (TAPP) and totally extraperitoneal
approach (TEPA) repairs are based on the preperitoneal repairs of Cheattle, Henry,
Nyhus, and Stoppa.
Correct answer: A

4. Which of the following statements concerning the abdominal wall layers are
correct?
A. Scarpa's fascia affords little strength in wound closure.
B. The internal abdominal oblique muscles have fibers that continue into the scrotum
as cremasteric muscles.
C. The transversalis fascia is the most important layer of the abdominal wall in
preventing hernias.
D. The lymphatics of the abdominal wall drain into the ipsilateral axillary lymph
nodes above the umbilicus and into the ipsilateral superficial inguinal lymph nodes
below the umbilicus.
Correct answer: ABCD

5. Which of the following congenital abnormalities are correctly defined?


A. Omphalocele represents a defect in the abdominal wall lateral to the umbilical
cord.
B. The herniated viscera associated with omphaloceles are usually covered with a
membranous sac.
C. An umbilical polyp is a small excrescence of omphalomesenteric duct mucosa that
is retained in the umbilicus.
D. Meckel's diverticulum results when the intestinal end of the omphalomesenteric
duct persists and represents a true diverticulum.
Correct answer: BCD

- 268 -
6. The following statement(s) is/are true concerning the indications for treatment of an
inguinal hernia.
a. Most adult hernias will remain stable in size, therefore, delay seldom affects the
technical aspects of a surgical repair
b. There is a direct correlation between the length of time that a hernia is present and
the risk of major complications
c. The morbidity and mortality associated with emergent operation due to hernia
complications is significantly greater than for elective repair of the identical hernia
d. A truss maintains a hernia in the reduced state, therefore, minimizing the risk of
incarceration and strangulation
Correct answer: b, c

7. Which of the following statement(s) is/are true concerning the diagnosis and
management of epigastric hernias?
a. A large peritoneal sac containing abdominal viscera is common
b. At the time of surgical repair, a careful search for other defects should be
performed
c. Recurrent epigastric hernias after simple closure is uncommon
d. Patients with symptoms of a painful midline abdominal mass frequently will
contain incarcerated small bowel
Correct answer: b

8. The following statement(s) is/are true concerning neurovascular structures in the


inguinal region.
a. The inferior epigastric artery and vein run upward in the preperitoneal fat posterior
to the transversalis fascia close to the lateral margin of the internal inguinal ring
b. The Ili hypogastric and ilioinguinal are motor and sensory nerves in the inguinal
region which lie beneath the external oblique aponeurosis
c. The ilioinguinal nerve runs anterior to the spermatic cord in the inguinal canal and
at the superficial inguinal ring, branches into the sensory supply to the pubic region
and the upper scrotum or labium majors
d. The genital branch of the genitofemoral nerve is a sensory nerve only to the upper
thigh and genital area
Correct answer: b, c

9. Potential complications of operative treatment of an inguinal hernia, which of the


following statement(s) is/are true?
a. Severe symptoms due to sensory nerve entrapment or injury can occur
b. The most common vascular structure injured during the course of a groin hernia
repair is the femoral artery
c. Recurrent hernia after primary groin repair should occur in less than 10% of cases
d. Wound infection increases the risk of recurrent hernia
Correct answer: a, c, d

10. Which of the statement(s) is/are true concerning laparoscopic hernia repair?
a. General anesthesia is required
b. Either an abdominal or preperitoneal approach is possible
c. The use of prosthetic mesh is required in all variations
d. Long-term results suggest that the laparoscopic approach is equal or better than

- 269 -
traditional repairs
Correct answer: a, b, c

11. Which of the following statement(s) is/are true concerning repair of inguinal
hernias?
a. The Bassini repair approximates the transversus abdominis aponeurosis and
transversalis fascia and the shelving edge of the inguinal ligament.
b. The Bassini repair is an adequate repair for a femoral hernia
c. A relaxing incision is important for repairs of direct and large indirect inguinal
hernias to prevent excessive tension in the closure
d. An advantage to the use of prosthetic material is the mesh incites formation of scar
tissue to further increase tensile strength provided by the mesh alone
Correct answer: a, c, d

12. The following statement(s) is/are true concerning the epidemiology of inguinal
hernias.
a. Inguinal hernias occur with a male-ta-female ratio of about 7:1
b. Femoral and umbilical hernias are more common in women, with a female-ta-male
ratio of 4:1
c. The frequency of inguinal hernias increases with age
d. Almost all umbilical hernias occur in the pediatric age group
Correct answer: a, c

13. The following statement(s) is/are true concerning umbilical hernias in adults.
a. Most umbilical hernias in adults are the result of a congenital defect carried into
adulthood
b. A paraumbilical hernia typically occurs in multiparous females
c. The presence of ascites is a contraindication to elective umbilical hernia repair.
d. Incarceration is uncommon with umbilical hernias
Correct answer: b

14. The following statement(s) is/are true concerning the anterior abdominal wall
musculature.
a. The lateral musculature of the abdominal wall consists of three muscle layers.
These are, from external to internal, the external oblique, the transversus abdominis,
and the internal oblique muscles
b. The transversalis fascia lies on the deep side of the transversus muscle and extends
to form an essentially complete fascial envelope of the abdominal cavity
c. Above the semicircular line, the internal oblique aponeurosis splits into posterior
and anterior laminae
d. The rectus abdominis muscles originate on the ribs superiorly and on the pubis
inferiorly and are clearly distinct throughout their entire length
Correct answer: b, c

15. The following statement(s) is/are true concerning incarceration of an inguinal


hernia.
a. All incarcerated hernias are surgical emergencies and require prompt surgical
intervention
b. Attempt at reduction of an incarcerated symptomatic hernia is generally considered
safe

- 270 -
c. Vigorous attempts at reduction of an incarcerated hernia may result in reduction en
masse with continued entrapment and possible progression to obstruction or
strangulation
d. Incarcerated hernias frequently cause both small and large bowel obstruction
Correct answer: b, c

16. A careful history is necessary in all patients being considered for inguinal hernia
repair. Symptoms which deserve investigation and appropriate treatment prior to
proceeding with inguinal hernia repair include:
a. Chronic cough
b. Urinary hesitancy and straining
c. Change in bowel habit
d. A specific episode of muscular straining with associated discomfort
Correct answer: a, b, c

17. The following statement(s) is/are true concerning abdominal incisional hernias.
a. Large incisional hernias are associated with a high recurrence rate when closed
primarily
b. A large potential space remains anterior to the abdominal wall closure in most
patients indicating a need for postoperative wound drainage
c. The use of prosthetic mesh can often be avoided by employing relaxing incisions in
the anterior fascia parallel to the midline
d. Incisional hernias are frequently associated with a tissue deficit either due to
chronic retraction and scarring or the result of tissue necrosis from either infection or
tension at the initial closure
Correct answer: a, b, c, d

18. Which of the following structures are derived from the external oblique muscle
and its aponeurosis?
a. The inguinal or Poupart’s ligament
b. The lacunar ligament
c. The superficial inguinal ring
d. The conjoined tendon
Correct answer: a, b, c

19. A number of special circumstances exist in the repair of inguinal hernias. The
following statement(s) is/are correct.
a. Simultaneous repair of bilateral direct inguinal hernias can be performed with no
significant increased risk of recurrence
b. The preperitoneal approach may be appropriate for repair of a multiple recurrent
hernia
c. A femoral hernia repair can best be accomplished using a Bassini or Shouldice
repair
d. Management of an incarcerated inguinal hernia with obstruction is best approached
via laparotomy incision
Correct answer: b

20. Which of the following statements about acute salpingitis are true?
A. The disease rarely occurs after menopause.
B. Gonococcal infection is most common.

- 271 -
C. There is minimal cervical tenderness to palpation.
D. Vaginal discharge occurs rarely.
Correct answer: AB

21. Meckel's diverticulitis most often occurs in the:


A. Proximal jejunum.
B. Distal jejunum.
C. Proximal ileum.
D. Distal ileum.
Correct answer:D

22 . Spigelian hernia is seen in:


A. Subumblical region
B. Lumbar triangle
C. Supraumblical region
D. Paraumblical region
Correct answer: A

23 . Hernia that is least likely to strangulate is -


A. Femoral hernia
B. Indirect inguinal hernia
C. Indirect inguinal hernia
D. Umbilical hernia
Correct answer: B

24. Which of the following is not a predisposing factor for a hernia?


a. Chronic obstructive pulmonary disease
b. Obesity
c. Urinary stones
d. Pregnancy
e. Peritoneal dialysis.
Correct answer: C

25. Which of the following are more common in multiparous women?


a. Indirect inguinal hernia.
b. Lumbar hernia.
c. Umbilical.
d. Direct inguinal hernia.
Correct answer: C

26. Which of the following is not a complication of an inguinal hernia?


a. Irreducibility.
b. Inflammation.
c. Strangulation.
d. Obstruction.
e. Bleeding.
Correct answer: E

- 272 -
27. Which of the following is not true in relation to strangulated hernias?
a. They present with local and then generalized abdominal pain and vomiting.
b. A normal hernia can strangulate at any time.
c. This is more common in femoral hernia.
d. They can be reliably excluded in irreducible hernias on clinical examination.
e. They require urgent surgery.
Correct answer: D

28. Regarding the anatomy of the inguinal canal, which of the following are true?
a. In infants the internal and external rings are almost superimposed.
b. The inferior epigastric vessels lie posterior and lateral to the internal ring.
c. The inguinal canal is about 10 cm long and is directed downwards, medially and
forwards.
d. The internal ring is a U-shaped opening in the external oblique aponeurosis 1.25 cm
above the mid-inguinal point.
Correct answer: A

29. Which of the following are true about a sliding inguinal hernia?
a. It is common in men.
b. It should be suspected clinically in small hernias confined to the inguinal canal.
c. It is more common in the young patient.
d. It is impossible to control with a truss, and hence an operation is indicated.
Correct answer: D

30. Which of the following statement regarding a femoral hernia are true?
a. The femoral canal occupies the most lateral component of the femoral sheath.
b. It can be easily controlled by truss.
c. Strangulation is the initial presentation in 40 per cent of patients.
d. An operation is only occasionally required.
Correct answer: D

31. Which of the following statements are true regarding umbilical hernia?
a. Men are affected more frequently than women.
b. Irreducibility is common due to omental adhesions.
c. Mayo’s operation involves mesh repair of the hernia.
d. Infantile umbilical hernias need immediate surgery.
Correct answer: C

32. Which of the following are true regarding ‘burst abdomens’?


a. The incidence is around 10–15 per cent.
b. Midline and vertical incisions are more likely to burst than transverse incisions.
c. Catgut is associated with a lower risk of burst abdomen.
d. Most cases are managed conservatively.
Correct answer: B

33. In incompletely descended testis, which of the following statements is false?


a. The incidence is 4 per cent.
b. A testis absent from the scrotum after 3 months is unlikely to descend.
c. An incompletely descended testis tends to atrophy as puberty approaches.
d. Early orchidopexy can preserve function.

- 273 -
e. Orchidopexy reduces the chances of developing a testicular tumour.
Correct answer: E

34 . Acute mesenteric lymphadenitis is caused by


A. E.coli
B. Alpha-hemolytic streptococci
C. Hemophilus
D. Yersinia
Correct answer: D

35. In testicular tumours, which of the following statements are false?


a. A scrotal lump that is inseparable from the testis is likely to be a tumour.
b. Lymphatic spread is usually to the inguinal lymph nodes.
c. Teratomas occur in the third decade and seminomas in the fourth decade.
d. Seminomas usually spread via the lymphatics.
Correct answer: B

36- The following are True about the inguinal canal EXCEPT:
a. It is a 4 cm tunnel in the lower abdominal muscles.
b. It contains spermatic cord in males and Round ligament in female.
c. The floor is formed by the inguinal ligament and the roof is formed by conjoint
tendon.
d. Boundaries include external oblique muscle posterior and fascia transversalis
anteriorly.
Correct answer: D

37- Following organs can be found within the hernia sac EXCEPT:
a) Colon.
b) Ovary.
c) Urinary Bladder.
d) Prostate.
Correct answer: D

38- The following is the most common type of hernias:


a) Femoral.
b) Umbilical.
c) Inguinal.
d) Incisional
Correct answer: D

39- The following are causes of acquired hernia EXCEPT:


a) Cough.
b) Unobliterated processes vaginalis.
c) Pregnancy.
d) Ascites.
Correct answer: B

40- Richter's hernia is a type of hernia with:


a) Two loop of the bowel in the sac.
b) Portion of the circumference of the intestine in the sac.

- 274 -
c) Portion of the Urinary Bladder in the sac.
d) Omentum in the sac
Correct answer: B

41- All the following are True when the sac content in omentum EXCEPT:
a) Doughy in consistency.
b) Gives No gurgling sensation.
c) Dull on percussion.
d) Reduced by difficulty in Beginning.
Correct answer: D

42- Regarding the external ring, all are true EXCEPT:


a) 1/2 inch above Mid inguinal point.
b) Named also as (superficial ring).
c) Slit in the external oblique aponeurosis
d) It is not involved in the direct hernia
Correct answer: A.

43- Injury of the ilia-inguinal nerve during hernia repair can cause:
a) Direct hernia due to paralysis of conjoint tendon.
b) Loss of erectile function.
c) Numbness of the scrotum and medial aspect of the upper thigh.
d) Testicular dragging pain.
Correct answer: C

44- Boundaries of the Hasselbach's triangle are:


a. Inferior epigastric a. medially and conjoint tendon laterally.
b. Lateral border of rectus muscle medially and inferior epigastric a. laterally.
c. Ilia-inguinal Nerve laterally and inguinal ligament medially.
d. ASIS laterally and pubic tubercle medially.
Correct answer: B

45- Indirect (=oblique) inguinal hernia:


a) Directed forward.
b) Occur in old age.
c) Can occur in any age and bilateral in 30 % of cases.
d) Commonly irreducible
Correct answer: C.

46- All of the following is True about the clinical picture of the oblique hernia
EXCEPT:
a) The swelling increase in size with cough.
b) Giving gurgling sensation during reduction
c) Can reach the scrotum.
d) Direction of the reduction is "backward"
Correct answer: D

47- About the internal ring test, all are true EXCEPT:
a) Can differentiate between indirect, direct and femoral hernia.

- 275 -
b) Not usually done.
c) Oblique hernia does not come out except after thumb release.
d) The test starts by asking the patient to reduce hernia.
Correct answer: A

48- Hernial sac can be identified by all of the following EXCEPT:


a. Opaque.
b. Pearly white in color.
c. Containing the vas deferens and spermatic a.
d. Has defined edges.
Correct answer: C

49- Herniorrhaphy has the following characters EXCEPT:


a) Repair of the hernia by absorbable sutures.
b) Should not be performed under tension.
c) Indication in large hernias and good muscular in elderly.
d) Bassini repair is one of its techniques
Correct answer: A

50- Regarding the direct hernia, all the following are true EXCEPT:
a) Neck of the sac is Medical to the inferior epigastric a.
b) The sac is within the cord including its contents.
c) Usually accuse in elderly and > 50 % is bilateral.
d) Un-commonly complicated.
Correct answer: B

51- Any of the following can be a complication of the hernia EXCEPT:


a) Bleeding.
b) Strangulation.
c) Obstruction.
d) Rupture due to massive trauma.
Correct answer: A

52- Irreducible hernia can be presented by any of the following EXCEPT:


a. Not tense.
b. Not tender.
c. No sign of intestinal obstruction.
d. Lost impulse on cough.
Correct answer: D

53- Strangulation hernia can be presented by:


a) Lost expansible an cough, tenderness tension and irreducibly.
b) Tenderness with intact impulse on cough.
c) No tension
d) Hydrocele of hernia sac.
Correct answer: A

54- The most common type of hernia liable for strangulation is:
a) Incisional.
b) Para-umbilical.

- 276 -
c) Femoral.
d) Direct.
Correct answer: C

55- The commonest hernia type that can be seen strangulated is:
a) Incisional
b) Inguinal.
c) Para-umbilical.
d) Femoral.
Correct answer: B

56- The pathological sequence of the strangulated hernia includes all of the following
except:
a) Impeded venous return.
b) Intestinal Obstruction.
c) Impaired arterial blood flow.
d) Gangrene.
Correct answer: B

57- During the hernial repair, all the following are "wrong" except:
a. Viable intestine is returned back to the abdomen.
b. The omentum never to be excised.
c. The mesh should be applied to cover the cord.
d. Herniorrhaphy should be performed with good tension.
Correct answer: A

58- The ONLY cause of lost expansile impulses on cough during hernia examination
is:
a) Irreducible hernia.
b) Obstructed hernia.
c) Strangulated hernia.
d) Retrosternal goiter.
Correct answer: C
59- The synthetic material used in herniopalsty is:
a) Prolene mesh
b) Vicryl mesh
c) Double faced mesh
d) Silk mesh
Correct answer: A

60- Which of the following is not a predisposing factor for a hernia?


a. Chronic obstructive pulmonary disease
b. Obesity
c. Urinary stones
d. Pregnancy
e. Peritoneal dialysis.
Correct answer: C

- 277 -
61. Which of the following are more common in multiparous women?
a. Indirect inguinal hernia
b. Lumbar hernia
c. Umbilical
d. Direct inguinal hernia
Correct answer: C

62. Which of the following is not a complication of an inguinal hernia?


a. Irreducibility
b. Inflammation
c. Strangulation
d. Obstruction
e. Bleeding.
Correct answer: E

63. Which of the following is not true in relation to strangulated hernias?


a. They present with local and then generalized abdominal pain and vomiting.
b. A normal hernia can strangulate at any time.
c. This is more common in femoral hernia.
d. They can be reliably excluded in irreducible hernias on clinical examination.
e. They require urgent surgery.
Correct answer: D

64. Regarding the anatomy of the inguinal canal, which of the following are true?
a. In infants the internal and external rings are almost superimposed.
b. The inferior epigastric vessels lie posterior and lateral to the internal ring.
c. The inguinal canal is about 10 cm long and is directed downwards, medially and
forwards.
d. The internal ring is a U-shaped opening in the external oblique aponeurosis 1.25 cm
above the mid-inguinal point.
Correct answer: A

65. Which of the following is false about a sliding inguinal hernia?


a. It is far more common in men.
b. It should be suspected clinically in small hernias confined to the inguinal canal.
c. It is impossible to control with a truss, and hence an operation is indicated.
d. It is unnecessary to excise the sac and attempts to dissect the bowel wall can be
dangerous.
Correct answer: B

66. Which of the following statements regarding a femoral hernia is true?


a. It is more common in women.
b. The femoral canal occupies the most lateral component of the femoral sheath.
c. It can be easily controlled by truss.
d. An operation is only occasionally required.
Correct answer: A

67. Which of the following statements is true regarding umbilical hernia?


a. The umbilical hernia in an infant is through the umbilical cicatrix but is usually
paraumbilical in adults.

- 278 -
b. Men are affected more frequently than women.
c. Mayo’s operation involves mesh repair of the hernia.
d. Infantile umbilical hernias need immediate surgery.
Correct answer: A

68. Which of the following are causes of umbilical discharge?


a. Persistent vitellointestinal duct
b. Persistent urachus
c. Pilonidal sinus
d. Omphalitis
e. all of the above
Correct answer: E

69. Which of the following is true regarding ‘burst abdomens’?


a. The incidence is around 10–15 per cent.
b. Midline and vertical incisions are more likely to burst than transverse incisions.
c. Catgut is associated with a lower risk of burst abdomen.
d. Most cases are managed conservatively.
Correct answer: B

70. A64-year-old woman presents with a strangulated femoral hernia. At operation,


what is the criterion used to determine the viability of a loop of bowel?
(A) Increased peristalsis
(B) Absent arterial pulsation
(C) Venous engorgement
(D) Intraoperative CT scan
(E) Serum amylase
Correct answer: B

Inguinal hernia.

1 . In case of female commonest hernia is -


A. Direct inguinal hernia
B. Indirect inguinal hernia
C. Femoral Hernia
D. Incisional hernia
Correct answer: B

2 . Which one of the following is not performed in Lichtenstein tension free


hernioplasty ?
A. High ligation on indirect hernia sac
B. Mesh sutured to the conjoint tendon and inguinal ligament
C. Conjoint tendon sutured to inguinal ligament
D. Spermatic cord is placed in two tails of the mesh at the internal rings
Correct answer: C

- 279 -
3 . False regarding Bochdalek hernia is -
A. Spleen and kidney can herniate
B. Occurs posterolateral
C. Always occurs on right side
D. Hernia may or may not have sac
Correct answer: C

4. Inter parietal hernia at the level of arcuate line and lateral border of rectus is called
A. Spigelian hernia
B. Lumbar hernia
C. Richter's hernia
D. Epigastric hernia
Correct answer: A

5. Parastomal hernia is most frequently seen with:


A. Loop Colostomy
B. End Colostomy
C. Loop Iliostomy
D. End Iliostomy
Correct answer: D

6 . The ratio of males to female in the incidence of strangulated inguinal hernia in


infancy is -
A. 5:1
B. 1:2
C. 2:1
D. 1:5
Correct answer: A

7. False about hernia is :


A. Absorbable mesh not used
B. Long standing hernia increases changes of incarceration
C. Conservative management
D. In Child hernia treatment with herniotomy
Correct answer: B

8 . A patient operated for direct inguinal hernia developed anethesia at the root of the
penis and adjacent part of the scrotum the nerve likely to be injured is:
A. Genital br. of genitofemoral nerve (supply dartos muscle)
B. Femoral br. of genitofemoral nerve
C. Iliohypogastric nerve
D. Ilioinguinal nerve
Correct answer: D

9 . What is giant hiatal hernia ?


A. Bochdalek hernia
B. Sliding hernia
C. Paraesophageal
D. Sliding
Correct answer: C

- 280 -
10 . All should be done in a case of strangulated inguinal hernia except:
A. Abdominal x-ray
B. IV fluids for resuscitation
C. Needle aspiration for diagnosis
D. Immediate surgery
Correct answer: C

11 . Hernia that is least likely to strangulate is


A. Femoral hernia
B. Direct inguinal hernia
C. Indirect inguinal hernia
D. Umbilical hernia
Correct answer: B

12. During repair of indirect inguinal hernia, while releasing the constriction at the deep
inguinal ring,the surgeon takes care not to damage one of the following structures
A. Flax inguinalis (conjoint tendon)
B. Interfoveolar ligament
C. Inferior epigastric artery
D. Spermatic cord
Correct answer: C

13 . The person whose work on the radical cure of hernia immortalized his name was -
A. William Halsted
B. Eduardo Bassini
C. Kocher
D. Koontz
Correct answer: B

14 . Classic diagnostic point for an uncomplicated indirect inguinal hernia is -


A. Impulse on coughing
B. Positive deep ring occlusion test
C. Groin swelling
D. Reducibility
Correct answer: B

Femoral

ventral hernias, Incisional hernia, recurrent hernia, burst abdomen.

1 . Burst abdomen most commonly occurs on the


A. 2nd day
B. 3rd day

- 281 -
C. 7th day
D. 9th day
Correct answer: C

17- Scrotum

Varicocele and hydrocele.

1 . Not true about varicocele is:


A. Common on the right side
B. Can present as a late sign of renal cell carcinoma
C. Has bag of worm like feeling
D. Can lead to infertility
Correct answer: A

2 . Ligation of varicocele (testicular vein) should be done:


A. Above the inguinal ligament
B. Below the inguinal ligament
C. In the scrotum
D. Individual veins
Correct answer: A

3 . Varicocele is common on left testis because:


A. Left testicular vein drains into left renal vein which has high pressure
B. Left testicular vein drains into IVC which has high pressure
C. Compression of testicular vein by rectum
D. Left lestis is lower situated
Correct answer: A

4-Complications of varicocele include the following except:


a. Sub fertility.
b. Secondary hydrocele.
c. Malignancy.
d. Testicular atrophy.
Correct answer: C

5-lndications of surgical correction of varicocele include the following except:


a. Severe persistent pain.
b. Thrombophlebitis.
c. Secondary varicocele.
d. All cases should be treated surgically for fear of complications
Correct answer: D.

- 282 -
6-The main disadvantage of palomo operation is:
a. Testicular artery injury.
b. High recurrence.
c. Hernia formation.
d. lnjury to vas.
Correct answer: B

7-The vaginal hydrocele is characterized by all of the following except:


a. Fluctuation is positive
b. Eversion and excision of the tunica is the main line of treatment
c. The swelling is usually found above the neck of the scrotum
d. Aspiration is followed by recurrence
Correct answer: C

8. Size fluctuation is a character of:


a. lnfantile hydrocele.
b. Congenitol hydrocele.
c. Both A ond B
d. None of the above.
Correct answer: B

9- The following statements are true about a hydrocele except


a. A hydrocele is a collection of fluid within the tunica vaginalis.
b. A congenital hydrocele causes an intermittent swelling.
c. An acute hydrocele in a young man may be a sinister finding.
d. Testicular pathology may cause a hydrocele.
e. Drainage is an effective treatment.
Correct answer: E

10. The following statements are true about a hydrocele except


a. A hydrocele is a collection of fluid within the tunica vaginalis.
b. A congenital hydrocele causes an intermittent swelling.
c. An acute hydrocele in a young man may be a sinister finding.
d. Testicular pathology may cause a hydrocele.
e. Drainage is an effective treatment.
Correct answer: E

11 . Congenital hydrocele is best the treatment by:


A. Eversion of sac
B. Excision of sac
C. Lords procedure
D. Herniotomy
Correct answer: D

12 . For which of the following,Lord's Plication is done?


A. Testicular cancer
B. Inguinal Hernia
C. Testicular varices
D. Hydrocele

- 283 -
Correct answer: D

Diseases of testis and epididymis.

1 . High inguinal orchiectomy for teratoma testes with involved epididymis is


what stage ?
A. Stage I
B. Stage II
C. Stage IIIA
D. Stage IV
Correct answer: A

2 . Barley water like fluid is present in -


A. Chylocele
B. Hydrocele
C. Epididymic cyst
D. Spermatocele
Correct answer: D

D.D of inguina-scrotal swelling

18- Pediatric surgery

1. Polyhydramnios is frequently observed in all of the following conditions


except:
A. Esophageal atresia.
B. Duodenal atresia.
C. Pyloric atresia.
D. Hirschsprung's disease.
E. Congenital diaphragmatic hernia.
Correct answer: D

2. Which of the following statements about Hirschsprung's disease is/are true?


A. There are no ganglion cells seen in Auerbach's plexus.
B. There is an increased incidence of Down syndrome.
C. It is more common in girls.
D. It may be associated with enterocolitis.
E. It may involve the small intestine.
Correct answer: ABDE

- 284 -
3. Which of the following statements is/are true of infants with gastroschisis?
A. It is associated with malrotation.
B. There is a high incidence of associated anomalies.
C. There is prolonged adynamic ileus following repair.
D. It is complicated by intestinal atresia in 10% to 12% of cases.
E. It is associated with chromosomal syndromes.
Correct answer: ACD

4. In neonates with congenital diaphragmatic hernia, which of the following


statements is true?
A. The defect is more common on the right side.
B. Survival is significantly improved by administration of pulmonary vasodilators.
C. An oxygen index of 20 is an indication for extracorporeal membrane oxygenation
(ECMO).
D. Oligohydramnios is a frequent occurrence.
E. Mortality is the result of pulmonary hypoplasia.
Correct answer: E

5. Which of the following statements are true regarding the premature neonate?
A. A 15% to 20% right-ta-left shunt occurs across the foramen ovale and patent
ductus arteriosus.
B. Surfactant levels are normal after 30 weeks' gestation.
C. Fluid requirements are higher than in a full-term baby.
D. Rectal temperature is the best indicator of core body temperature.
E. They are more at risk for infection than the full-term infant.
Correct answer: ACE

6. In neonates with necrotizing enterocolitis, which of the following findings is an


indication of significant bowel ischemia?
A. Increased gastric residuals.
B. Septic shock.
C. Cardiac failure due to a patent ductus arteriosus.
D. Elevated platelet count.
E. Erythema of the abdominal wall.
Correct answer: E

7. The treatment of choice for neonates with uncomplicated meconium ileus is:
A. Observation.
B. Emergency laparotomy, bowel resection, and Bishop-Koop enterostomy.
C. Intravenous hydration and a gastrograffin enema.
D. Emergency laparotomy, bowel resection, and anastomosis.
E. Sweat chloride test and pancreatic enzyme therapy.
Correct answer: C

8. The pentalogy of Cantrell includes all of the following except:


A. Epigastric omphalocele.
B. Sternal cleft.
C. Intracardiac defect.
D. Pericardial cyst.

- 285 -
E. Ectopia cordis.
Correct answer: D

9. In infants with duodenal atresia all the following statements are true except:
A. There is an increased incidence of Down syndrome.
B. Duodenal atresia can be detected by prenatal ultrasound examination.
C. It may occur in infants with situs inversus, malrotation, annular pancreas, and
anterior portal vein.
D. It is best treated by gastroenterostomy.
E. There is a high incidence of associated cardiac defects.
Correct answer: D

10. The initial treatment of choice for a 2.5-kg. infant with a 20.0-cm. long
proximal jejunal atresia and 8.0 cm. of distal ileum is:
A. Laparotomy, nasogastric suction, proximal dilatation to lengthen the atretic
jejunum, total parenteral nutrition, and delayed anastomosis.
B. Laparotomy and proximal end-jejunostomy.
C. Laparotomy and immediate small bowel transplantation.
D. Laparotomy and double-barrel enterostomy (jejunum and ileum), with refeeding of
jejunal contents into distal ileum and delayed anastomosis.
E. Laparotomy, tapering jejunoplasty, and end-ta-oblique jejunoileal anastomosis.
Correct answer: E

11. A 2.8-kg. neonate with excessive salivation develops respiratory distress.


Attempts to pass an orogastric catheter fail because the catheter coils in the back
of the throat. A chest film is obtained and shows right upper lobe atelectasis and
a gasless abdomen. The most likely diagnosis is:
A. Proximal esophageal atresia without a fistula.
B. Proximal esophageal atresia with a distal tracheoesophageal (TE) fistula.
C. “H-type” TE fistula.
D. Esophageal atresia with both proximal and distal TE fistula.
E. Congenital esophageal stricture.
Correct answer: A

12. Neonates with NEC may demonstrate all of the following findings on
abdominal films except:
A. Pneumatosis intestinalis.
B. Portal vein air.
C. Pneumoperitoneum.
D. Colovesical fistula.
E. Fixed and thickened bowel loops.
Correct answer: D

13. The most common type of congenital diaphragmatic hernia is caused by:
A. A defect in the central tendon.
B. Eventration of the diaphragm in the fetus.
C. A defect through the space of Larrey.
D. An abnormally wide esophageal hiatus.
E. A defect through the pleuroperitoneal fold.

- 286 -
Correct answer: E

14. The calorie-nitrogen ratio for an infant should be maintained at:


A. 75:1.
B. 100:1.
C. 50:1.
D. 150:1.
E. 25:1.
Correct answer: D

16. For a 22-kg infant, the maintenance daily fluid requirement is approximately
which of the following?
a. 1100 ml
b. 1250 ml
c. 1550 ml
d. 1700 ml
e. 1850 ml
Correct answer: c

18. A term infant 48 hours of age suddenly develops hypoxemia, irritability, and
glucose and temperature instability. Which of the following statements are true?
a. Empiric antibiotic coverage for b-hemolytic Streptococci and Escherichia coli
should be initiated
b. An intravenous infusion of prostaglandin E1 should be initiated immediately
c. Exogenous surfactant should be given immediately
d. The mortality rate for this child is approximately 50%
Correct answer: a, d

19. Which of the following statements about pulmonary surfactant are true?
a. Endogenous surfactant deficiency is the key physiologic problem in preterm infants
with the infant respiratory distress syndrome
b. Surfactant function can be restored to normal using aerosolized
phosphatidylcholine administration
c. Exogenous surfactant replacement has been shown to reduce mortality in preterm
infants with the infant respiratory distress syndrome
d. Surfactant is produced by Type I alveolar epithelial cells
Correct answer: a, c

20. Which of the following statements regarding premature infants are true?
a. Complications of prematurity account for approximately 85% of fetal deaths
b. Prematurity is defined by the World Health Organization as birth prior to 35 weeks
gestation
c. Infants with intrauterine growth retardation have physiologic problems which are
more dependent on the birth weight than the gestational age
d. Preterm infants are at increased risk for hypocalcemia and hypoglycemia when
compared to term infants
Correct answer: a, d

- 287 -
21. Other than the history and physical exam, which of the following tests is
considered an essential feature of the preoperative evaluation of a patient with a
suspected thyroglossal duct cyst?
a. Cervical ultrasound
b. Thyroid scan
c. Serum T3 and T4 levels
d. Needle aspiration
e. None of the above
Correct answer: e

22. Suppurative cervical lymphadenitis in a 3-year-old child is commonly related


to which of the following organisms?
a. Staphylococcus aureus
b. Atypical mycobacterial organisms
c. Streptococcal organisms
d. Lymphoma with secondary pyogenic organisms
e. Cat scratch
Correct answer: a, c

23. Branchial cleft remnants most often present with which of the following
clinical problems?
a. Infection
b. Airway obstruction
c. Hemorrhage
d. Malignant degeneration
e. Pain
Correct answer: a

24. Proximity to which of the following structures places it at risk during surgical
excision of a second branchial cleft remnant?
a. Internal carotid artery
b. External carotid artery
c. Hypoglossal nerve
d. All of the above
e. None of the above
Correct answer: d

25. Standard therapy for acute epiglottitis in a child is:


a. Tracheostomy
b. Intravenous antibiotic treatment in an ICU setting
c. Endotracheal intubation in the operating room and intravenous antibiotic therapy
d. Indirect laryngoscopy and intravenous antibiotics
e. Intravenous steroids and antibiotics
Correct answer: c

26. Which of the following statements regarding congenital diaphragmatic


hernia are true?
a. The incidence of right and left-sided lesions is equal
b. Malrotation is to be expected
c. Left-ta-right shunting via a patent ductus arteriosus is a serious but expected

- 288 -
physiologic consequence of pulmonary hypoplasia
d. Survival rates of 75% are reported in several contemporary series
e. Congenital heart disease is present in approximately 20% of these infants
Correct answer: b, d, e

27. Of the following cystic malformations of the tracheobronchial tree, which is


most likely to be asymptomatic when discovered?
a. Intralobar pulmonary sequestration
b. Extralobar pulmonary sequestration
c. Congenital cystic adenomatoid malformation
d. Congenital lobar emphysema
Correct answer: b

28. Infants with a double aortic arch most commonly present with which of the
following problems?
a. Dysphagia
b. High output cardiac failure related to a patent ductus arteriosus
c. Positional hyperemia and edema of the right upper extremity
d. Symptomatic tracheal compression
Correct answer: d

29. Which of the following is most common after primary esophagostomy for
esophageal atresia with a distal tracheoesophageal fistula?
a. Anastomotic leak
b. Esophageal stricture
c. Recurrent tracheoesophageal fistula
d. Gastroesophageal reflux
e. Tracheomalacia requiring aortopexy
Correct answer: d

30. Which of the following is the most common primary lung tumor in infants
and children?
a. Pulmonary blastoma
b. Squamous cell carcinoma
c. Endobronchial carcinoid
d. Leiomyoma
e. Metastatic osteogenic sarcoma
Correct answer: c

31. Which of the following statements regarding congenital chest wall


deformities are true?
a. Children with pectus excavatum deformities typically have physiologically
insignificant limitation of exercise tolerance
b. The rate of recurrence after operative repair of a pectus excavatum deformity is
between 5% and 10%
c. Pectus carinatum is the most common congenital chest wall defect
d. The most common indication for operative repair of congenital chest wall
deformities is cosmesis
Correct answer: a, d

- 289 -
32. The definitive evaluation of a child with a suspected congenital cystic
abnormality of the tracheobronchial tree is best done using which of the
following?
a. Rigid bronchoscopy
b. Computerized tomography or magnetic resonance imaging
c. Chest x-ray
d. Angiography
e. Barium esophagogram
Correct answer: b

33. A newborn infant develops coughing, choking and cyanosis with his first
feeding. He is noted to have excessive drooling. What are the important
associated anomalies that must be screened for prior to surgical intervention?
a. Right-sided aortic arch
b. Hydrocephalus
c. Genitourinary obstruction
d. Congenital heart disease
Correct answer: c

34. Which of the following ventilation strategies is the best initial approach for a
neonate with a left congenital diaphragmatic hernia and the following post
ductal arterial blood gases: PaO2 50 mm Hg, PaCO2 60 mm Hg, pH 7.35?
a. High-frequency jet ventilation
b. Permissive hypercapnia with convential pressure controlled ventilation
c. Extracorporeal membrane oxygenation (ECMO)
d. Induced respiratory alkalosis
e. Inhaled nitric oxide with conventional volume controlled ventilation
Correct answer: b

35. There is an emerging consensus that the surgical repair for congenital
diaphragmatic hernia is best done:
a. Emergently at the bedside, eliminating the risks of transporting an unstable neonate
b. While on extracorporeal membrane oxygenation
c. When the infant is potentially extubatable
d. Within the first 48 to 72 hours of life
Correct answer: c

36. Meckel’s diverticulum may present with which of the following signs or
symptoms?
a. Hemorrhage
b. Intussusception
c. Volvulus
d. Patent omphalomesenteric duct
e. Right lower quadrant peritoneal findings
Correct answer: a, b, c, d, e

37. A 3-week old infant has a barium upper gastrointestinal series to evaluate
vomiting. The duodenojejunal flexure is found to be to the right of the midline as
well as more caudal and anterior than a normal ligament of Treitz. The child is
seen to reflux barium spontaneously to the level of the mid-thoracic esophagus.

- 290 -
You would recommend which of the following?
a. Barium enema
b. Emergency laparotomy
c. A trial of H2, blockade and cisapride therapy
d. Upper gastrointestinal endoscopy
e. Overnight pH probe analysis
Correct answer: b

38. Which of the following statements regarding duodenal atresia are true?
a. 20% to 40% of these infants have Trisomy 21
b. When associated with an annular pancreas, division of the pancreas at the site of
obstruction is curative
c. Bilious vomiting is typical because the obstruction is usually distal to the ampulla
of Vater
d. Reconstruction is best achieved with Roux-en-Y duodenojejunostomy
Correct answer: a, c

39. A 1500-gram, 30-week gestation neonate is fed at 2 weeks of age. He develops


abdominal distention, bilious vomiting, and guaiac-positive stool. A plain film of
the abdomen demonstrates pneumatosis intestinalis. Which of the following
related statements are true?
a. An emergency barium upper GI series should be done to rule out malrotation
b. The child should have a nasogastric tube placed, broad spectrum intravenous
antibiotics begun, and sequential abdominal films obtained.
c. The likelihood of intestinal perforation is in excess of 50%
d. The expected survival rate is in the range of 70%
Correct answer: b, d

41. Which of the following’s statements regarding an infant with meconium ileus
are true?
a. The probability is 100% that he will have cystic fibrosis
b. Nonoperative therapy resolves this problem in approximately twa-thirds of patients
c. The average life expectancy is approximately 26 to 28 years for this infant at
present
d. The finding illustrated below on plain film is an absolute operative indication
Correct answer: a, b, c, d

42. You are asked to recommend therapy for an asymptomatic 2 year old who
swallowed a small alkaline watch battery 4 hours Aga. A plain film shows the
intact battery in the intestine beyond the stomach. The best course of therapy is?
a. Immediate laparotomy, enterotomy and removal of the battery
b. Enteroscopy with extraction
c. Laparoscopy with ultrasound localization and extraction
d. Cathartics and a follow-up plain film in 48 hours if the child remains asymptomatic
Correct answer: d

- 291 -
44. Of the following, which is the most likely cause of hemodynamically
significant lower gastrointestinal bleeding in a 6-month-old male child?
a. Meckel diverticulum
b. Henoch-Schonlein purpura
c. Intussusception
d. Crohn’s colitis
e. Hemolytic uremic syndrome
Correct answer: a, c

45. Which of the followings statement regarding Hirschprung’s disease are true?
a. Suction rectal biopsy is virtually always diagnostic if the specimen includes
submucosa
b. Hirschprung’s disease is the result of a sex linked dominant gene
c. The endorectal pullthrough is demonstrably superior to other forms of surgical
construction
d. Ninety percent or more of patients have an excellent or good functional result
following reconstructive surgery
e. The important cause of mortality in contemporary practice is enterocolitis
Correct answer: a, d, e

46. The operative procedure of choice for managing the most common type of
choledochal cyst is which of the following?
a. Cyst gastrostomy
b. Cyst jejunostomy
c. Excision with Roux-en-Y hepaticojejunostomy
d. Transduodenal marsupialization
e. Endoscopic sphincterotomy
Correct answer: c

49. The most common cause of acute pancreatitis in childhood is which of the
following?
a. Pancreas divisum
b. Cholelithiasis
c. Trauma
d. Valproic acid
e. Annular pancreas
Correct answer: c

50 . Which one is not poor prognostic factor for acute Pancreatitis:


A. Hyperglycemia
B. Hypocalcemia
C. Raised LDH level in blood
D. Hyperamylasemia
Correct answer: D

51 . Which of the following is not a prognostic factor for acute pancreatitis -


A. AST elevation
B. Increased serum amylase
C. Hypocalcemia
D. Hyperglycemia

- 292 -
Correct answer: B

52 . A 21 years old patient attended a party the previous night and gives the
following symptoms. pain in abdomen radiating to back, pulse 100/min, BP
100/76 . temp 39 degree C and vomiting before coming Most probable dx is:
A. Acute appendicitis
B. Acute cholecystitis
C. Acute diverticulitis
D. Acute pancreatitis
Correct answer: D

53 . Difference between acute and chronic pancreatitis is ?


A. Acute pancreatitis gas reversible changes
B. Alcohol causes only acute pancreatitis
C. Chronic pancreatitis shows no signs of inflammation
D. Acute pancreatitis affects mainly younger population
Correct answer: A

54 . All are used in treatment of acute pancreatitis except -


A. Analgesics
B. IV fluids
C. Antibiotics
D. Nasojejunal feeds
Correct answer: C

55 . Most common complication after ERCP is -


A. Acute cholecystitis
B. Duodenal perforation
C. Acute pancreatitis
D. Acute cholangitis
Correct answer: C

56 . Which of the following is NOT include in Ranson's score for acute


pancreatitis ?
A. Age
B. WBC count
C. Blood glucose
D. Serum amylase
Correct answer: D

57 . All of the following can be used to predict severe acute pancreatitis except -
A. Glassow score33
B. APACHE II39
C. CT severity36
D. C-reactive protein < 100
Correct answer: D

58 . In acute pancreatitis there is -


A. Generalized or localized ileus
B. Increases in serum amylase 2 times above normal

- 293 -
C. Pseuda-cyst occurs in the first week after the attack
D. Abdominal pain is uncommon
Correct answer: A

59 . Greyish discoloration of flank seen in Acute pancreatitis is referred to as ?


A. Cullen's sign
B. Grey Turner sign
C. Ballance's sign
D. Alvaradao's sign
Correct answer: B

60 . Severity of acute pancreatitis correlate with levels of all of the following


levels -
A. Amylase
B. Glucose
C. Calcium
D. Transaminase
Correct answer: A

61 . Which of the following does not correct with severity of acute pancreatitis -
A. Serum Glucose
B. Serum amylase
C. Serum calcium
D. AST
Correct answer: B

62 .Factors that produce acute pancreatitis include all of the following except -
A. Alcohol
B. Hypercalcemia of parathyroid origin
C. Gall stones
D. Hemochromatosis
Correct answer: D

63 . All are prognostic criteria for acute pancreatitis except -


A. Age >50 year
B. Serum amylase >100 S.U.
C. LDH > 200 I.U.
D. Blood sugar >200 mg
Correct answer: B

50. Which of the following statements regarding gastroschisis are true?


a. Primary fascial closure can be achieved in only about 25% of these infants
b. These infants have an incidence of approximately 40% to 50% of associated
anomalies
c. Overall survival is approximately 80% to 90%
d. When the diagnosis is known prenatally, planned cesarean section is the safest
method of delivery
Correct answer: c

- 294 -
51. Which of the following are typical causes of neonatal intestinal obstruction?
a. Intussusception
b. Meconium ileus
c. Hirschsprung’s disease
d. Meckel’s diverticulum
e. Incarcerated hernia
Correct answer: b, c, e

52. A 6-week-old child presents with generalized seizures, a serum glucose of 30


mg/dL and concurrent hyperinsulinemia. This child’s first priority is which of
the following?
a. Permanent central venous access and glucose infusion
b. Administration of cortisone and adrenocorticotropic hormone
c. Computerized tomographic scan of the abdomen to look for an islet cell adenoma
d. Urgent pancreatic resection
Correct answer: a

53. At what age is surgical orchiopexy recommended for a child with a unilateral
undescended testis?
a. Promptly upon discovery, regardless of age
b. 1 year
c. 5 to 6 years
d. Any time prior to puberty
Correct answer: b

55. The medical indications for circumcision include which of the following?
a. Infants with a history of urinary tract infection
b. Hypospadias
c. Phimosis
d. Enuresis
e. Vesicoureteral reflux
Correct answer: a, c, e

56. Which of the following statements regarding neuroblastoma are true?


a. Neuroblastoma is the most common abdominal malignancy of childhood
b. Approximately 80% of neuroblastoma patients are diagnosed prior to age 4 years
c. N-myc oncogene copy number in neuroblastoma tissue is inversely related to
survival probability
d. Trk prota-oncogene expression in neuroblastoma tissue is inversely related to
survival probability
e. All of the above
Correct answer: a, b, c

57. Which of the following are considered low risk features for neuroblastoma
patients?
a. Age less than one year
b. Stage 2A and 2B disease (International Staging Criteria)
c. Stage 4S disease (International Staging Criteria)
d. Neuron specific enolase plasma level less than 100 ng/ml
e. None of the above

- 295 -
Correct answer: a, b, c, d

58. A one month old female infant is brought to you for evaluation of afriable
polypoid mass prolapsing through the vaginal introitus. Your presumptive
diagnosis is which of the following?
a. Ectopic ureterocele
b. Rectal prolapse
c. Congenital adrenal hyperplasia with ambiguous genitalia
d. Embryonal rhabdomyosarcoma
Correct answer: d

59. Which of the following approaches is considered standard care for most
Wilms’ tumor patients today?
a. Adriamycin and vincristine therapy followed by surgical resection
b. Needle biopsy followed by either chemotherapy or resection depending upon the
histology
c. Primary surgical resection followed by chemotherapy
d. Radiation therapy if judged unresectable on CT or MRI imaging
Correct answer: c

60. Which of the following statements regarding rhabdomyosarcoma are true?


a. Surgical resection of the primary tumor results in cure of approximately 80 to 90%
of all patients
b. Currently recommended therapy includes complete resection of primary tumors
prior to chemotherapy for small noninvasive lesions, or after documented response
with more formidable primary tumors
c. Alveolar histology is a favorable prognostic finding
d. Overall survival of all patients is now approximately 50%
Correct answer: b, d

61. Patients with Wilms’ tumors most frequently present with which of the
following?
a. Bilateral metachronous lesions
b. Bilateral synchronous lesions
c. An extrarenal primary
d. A multicentric primary lesion
e. A unifocal, unilateral lesion
Correct answer: e

63. Common sites of neuroblastoma metastasis are which of the following?


a. Lung
b. Regional lymph nodes
c. Bone marrow
d. Cortical bone
e. Liver
Correct answer: b, c, d

- 296 -
64. Which of the following statements regarding renal tumors of childhood and
adolescence are true?
a. Clear cell sarcoma is presently considered a variant of Wilms’ tumor with a poor
prognosis
b. Clear cell sarcoma of the kidney has a high rate of metastasis to bone
c. Rhabdoid tumors may arise in the kidney, mediastinum or brain
d. Childhood rhabdoid tumors of the kidney carry an excellent prognosis
Correct answer: b, c

65. Which of the following syndromes are associated with the development of
Wilms’ tumor?
a. Beckwith-Wiedemann Syndrome (hemi-hypertrophy, macroglossia, aniridia)
b. Neurofibromatosis
c. Denys-Drash syndrome (pseudo hermaphroditism, glomerulopathy)
d. Gonadal dysgenesis
e. Hemolytic uremic syndrome
Correct answer: a, b, c, d

66 . Which of the following statement is false for a Wilms-tumor ?


A. Pyrexia
B. Hematuria denotes tumor extension in renal pelvis
C. Bone secondaries are common
D. Lymphatic spread uncommon
Correct answer: C

67 . All are associated with Wilms tumour except one -


A. Anirida
B. Male pseudohermaphrodite
C. Arthogryposis multiplex congenita
D. Hemihypertrophy
Correct answer: C

66. In meconium aspiration syndrome all of the following are true except .
A.There is always respiratory distress
B. Steroids are not useful
C. Extracorporeal membrane oxygenation (ECMO) may be useful
Correct answer A

67. What is the percentage of steroid responders in a case of Minimal Change


disease .
A.65%
B.75%
C.85%
D.95%
Correct answer D

68. At what age a child is able to state correctly his name and sex .
A.24months
B.36months
C.30months

- 297 -
D.42months
Correct answer A?

71. In Phenylketonuria all are true except


A.EEG abnormality in 50% of cases
B.Convulsion in 25% of cases
C.Blood is drawn for testing immediately after birth
D.Profuse vomiting
Correct answer C

72. A 15 month old child is able to do all the following except


A.crawl upstairs
B.make a tower of 5 cubes
C.name familiar objects
D.point to familiar things
Correct answer B

73. Most common cause of ambiguous genitalia is


A.Congenital adrenal hyperplasia
B.Klinefelter's syndrome
C.Testicular feminization
Correct answer A

74. Commonest type of hypospadias is


A.glandular
B.penile
C.coronal
D.scrotal
Correct answer A

76. What is the incubation period of vaccine strain of measles virus.


A. 7days
B. 10days
C. 14days
D. 18days
Correct answer A

77. Physical Quality of Life Index (PQLI) does not include


A.Infant Mortality rate
B.Gross national Product
C.Litreacy rate
D.Life expectancy at 1 year of age.
Correct answer B

78 . Epispadias is associates with?


A. Chordee
B. Bifid pubic symphysis
C. Intestinal obstruction
D. Anal atresia
Correct answer: A

- 298 -
79 . 4 years old female child with bone mets and abdominal mass - seen in -
A. Neuroblastoma
B. Wilms tumour
C. Dysgerminoma ovary
D. Burkitt's
Correct answer: A

80 . A new born girl not passed meconium for 48 hours, has abdominal
distension and vomiting. Initial investigation of choice would be -
A. Genotyping for cystic fibrosis
B. Manometry
C. Serum trypsin immunoblot
D. Lower GI contrast study
Correct answer: D

81. Regarding inguinal hernias in children- what is/are true.


(a) Boys are affected more often than girls
(b) Are less common in preterm infants
(c) In infants (less than one years old) approximately 50% present with incarceration
(d) 50% of hernias are bilateral
(e) Approximately 20% of patients will develop a contralateral hernia
Correct answer: a , c , e are true .

82. Regarding undescended testes- what is/are true.


(a) 75% of undescended testes descend in the first year of life
(b) Is associated with a reduced risk of testicular malignancy
(c) Is associated with an increased risk of infertility
(d) Surgery should be considered in the neonatal period
(e) Laparoscopy is indicated for impalpable testes
Correct answer: a , c , e are true .

83. Regarding testicular torsion- what is wrong?


(a) The highest incidence is in the neonatal and peripubertal periods
(b) May present with abdominal pain and vomiting and few testicular symptoms
(c) Radiological investigation with doppler ultrasound or isotope scanning is
invariably indicated
(d) Testicular viability is reduced if surgery is delayed more than 6 hours
(e) Contralateral orchidopexy should also be performed if a torsion is confirmed
Correct answer: only c is wrong.

84. Regarding exomphalos and gastroschisis- what is/are right.


(a) A gastroschisis has a sac
(b) Gastroschisis is associated with major congenital abnormalities
(c) The postoperative mortality of surgery for gastroschisis approaches 50%
(d) Both conditions may be diagnosed prenatally with ultrasound
(e) Both conditions require delivery by caesarian section
Correct answer: only d is right .

85. Congenital diaphragmatic hernia- what is/are true.


(a) Usually occur through the foramen of Bochdalek

- 299 -
(b) Are more common on the right than the left
(c) Are rarely associated with other congenital anomalies
(d) Present with respiratory distress in the neonatal period
(e) Neonates usually require sedation, ventilation and intestinal decompression prior
to surgery
Correct answer: a , d , e are true .

86. Regarding oesophageal atresia and trachea-oesophageal fistula (TOF) - what


is/are true.
(a) Most cases of oesophageal atresia are associated with a proximal TOF to the lower
oesophagus
(b) Often present prenatally with oligohydramnios
(c) Postnatally present with difficulty swallowing and aspiration
(d) The diagnosis may be confirmed by the inability to pass a nasogastric catheter
(e) Radiological evidence of gas in the stomach confirms the presence of distal TOF
Correct answer: c , d , e are true .

87. Regarding meconium ileus - what is/are true.


(a) Is a rare cause of neonatal intestinal obstruction
(b) Less than 5% of cases are associated with cystic fibrosis
(c) Obstruction usually occurs in the distal ileum
(d) Presents with neonatal bile-stained vomiting and abdominal distension
(e) A plain x-ray may show an intra-luminal 'ground glass' appearance
Correct answer: c , d , e are true .

88. Regarding infantile hypertrophic pyloric stenosis - what is/are right.


(a) Usually presents between 6 and 12 months of age
(b) The female : male ratio is 4:1
(c) Has a strong familial predisposition
(d) Pathologically shows hypertrophy of the longitudinal muscle layer of the pylorus
(e) Presents with bile-stained projectile vomiting
Correct answer: only c is right .

89. Which one is not true regarding pyloric stenosis-


A. Hypokalemia
B. Alkaline urine
C. Metabolic alkalosis
D. Hypochloremia
Correct answer: B

90. The metabolic disturbance in a case of hypertrophic pyloric stenosis is


A. Metabolic acidosis
B. Respiratory alkalosis
C. Metabolic alkalosis with alkaline urine
D. Metabolic alkalosis with paradoxical aciduria
Correct answer: D

91 . Make the diagnosis of a 26 day old Infant presenting with recurrent


nonbilious vomiting with constipation and loss of weight:
A. Oesophageal atresia

- 300 -
B. Choledochal cyst
C. ileal atresia
D. Pyloric stenosis
Correct answer: D

92 . Metabolic abnormality seen in congenital hypertrophic pyloric stenosis is -


A. Hypochloremic hypokalemic metabolic acidosis
B. Hyperchloremic hypokalemic metabolic acidosis
C. Hypochloremic hypokalemic metabolic alkalosis
D. Hyperchloremic hypokalemic metabolic alkalosis
Correct answer: C

93 . Pyloric stenosis is associated with all of the following EXCEPT


A. Hypochloremia
B. Hyperkalemia
C. Alkalosis
D. Aciduria
Correct answer: B

94 . Congenital hypertrophic pyloric stenosis associated with which of the


bellow?
A. Hypokalemic alkalosis
B. Hypokalemic acidosis
C. Hyperkalemic alkalosis
D. Hyperkalemic acidosis
Correct answer: A

95 . The electrolyte abnormality seen in patients with longstanding pyloric


stenosis is -
A. Hyponatremic hyperkalemic Acidosis
B. Hyponatremic hypokalemic acidosis
C. Hyponatremic hypokalemic alkalosis
D. Hypernatremic hypokalemic alkalosis
Correct answer: C

96 . The surgery for hypertrophic pyloric stenosis of infancy is -


A. Ramsted's operation
B. Truncal vagotomy
C. Heller's operation
D. Delorome's operation
Correct answer: A

97 . Which among the following is not a ultrasound feature of Congenital


Hypertrophic Pyloric Stenosis ?
A. 95% sensitivity by ultrasound
B. Thickness of pylorus>4mm
C. Canal length >16mm
D. High gastrial residue
Correct answer: D

- 301 -
98 . In pyloric stenosis, which one occurs:
A. Hypokalemic metabolic acidosis
B. Hyperchloremic metabolic acidosis
C. Hyperkalemic metabolic alkalosis
D. None
Correct answer: D

99 . Most common biochemical abnormality in congenital pyloric stenosis -


A. Hyperkalemic metabolic alkalosis
B. Hypokalemic metabolic alkalosis
C. Hyperkalemic metabolic acidosis
D. Hypokalemic metabolic acidosis
Correct answer: B

100 . Hypokalemic alkalosis is commonly seen in surgical wards in -


A. Excessive absorption of alkali
B. Excessive vomiting in pyloric stenosis
C. Excessive steroid use
D. Diarrhea with dehydration
Correct answer: B

101 . Which is Not true regarding Hypertrophic pyloric stenosis:


A. Ultrasound is not useful for diagnosis
B. Usually presents between 2-8 weeks
C. Common in males
D. Dilated stomach with minimal gas
Correct answer: A

102 . The metabolic disturbance, in a case of hypertrophic pyloric stenosis, is


A. Metabolic acidosis
B. Respiratory alkalosis
C. Metabolic alkalosis with alkaline urine
D. Metabolic alkalosis with paradoxical aciduria
Correct answer: D

103. Regarding intussusception - what is/are true.


(a) It is the commonest cause of intestinal obstruction between 6 and 18 months
(b) Usually occurs in the jejunum
(c) The lead point may be a Peyer's patch or Meckel's diverticulum
(d) Usually presents with few clinical signs
(e) Has a characteristic 'doughnut' appearance on ultrasound
Correct answer: a , c , e are true .

104. Regarding Hirschsprung's disease - what is/are true.


(a) Often presents with neonatal large bowel obstruction
(b) Results from absence of ganglion cells in both the Meissner's and Auberbach's
plexus
(c) A contrast-study will show dilatation of the aganglionic segment
(d) The diagnosis can be confirmed by histological evidence of reduced

- 302 -
acetylcholinesterase in the aganglionic segment
(e) Early treatment may involve rectal irrigation or an emergency colostomy
Correct answer: a , b , e are true .

105. Which of the following statements about Hirschsprung's disease is/are true?
A. There are no ganglion cells seen in Auerbach's plexus.
B. There is an increased incidence of Down syndrome.
C. It is more common in girls.
D. It may be associated with enterocolitis.
E. It may involve the small intestine.
Correct answer: ABDE

106. Which of the following statements is/are true of infants with gastroschisis?
A. It is associated with malrotation.
B. There is a high incidence of associated anomalies.
C. There is prolonged adynamic ileus following repair.
D. It is complicated by intestinal atresia in 10% to 12% of cases.
E. It is associated with chromosomal syndromes.
Correct answer: ACD

107. In neonates with congenital diaphragmatic hernia, which of the following


statements is true?
A. The defect is more common on the right side.
B. Survival is significantly improved by administration of pulmonary vasodilators.
C. An oxygen index of 20 is an indication for extracorporeal membrane oxygenation
(ECMO).
D. Oligohydramnios is a frequent occurrence.
E. Mortality is the result of pulmonary hypoplasia.
Correct answer: E

108. Which of the following statements are true regarding the premature
neonate?
A 15% to 20% right-ta-left shunt occurs across the foramen oval and patent ductus
arteriosus.
B. Surfactant levels are normal after 30 weeks' gestation.
C. Fluid requirements are higher than in a full-term baby.
D. Rectal temperature is the best indicator of core body temperature.
E. They are more at risk for infection than the full-term infant.
Correct answer: ACE

109. In neonates with necrotizing enterocolitis, which of the following findings is


an indication of significant bowel ischemia?
A. Increased gastric residuals.
B. Septic shock.
C. Cardiac failure due to a patent ductus arteriosus.
D. Elevated platelet count.
E. Erythema of the abdominal wall.
Correct answer: E

- 303 -
110. A male infant weighing 3 kg is born via spontaneous vaginal delivery at 37
weeks gestation. His Apgar score is 6/9 at 1 and 5 minutes. The patient is in no
apparent distress. Physical examination reveals no anus. What is the most
appropriate initial step in this patient’s management?
(A) Colostomy
(B) Continued observation for 24 hours
(C) Intubation and mechanical ventilation
(D) Magnetic resonance imaging (MRI) of the abdomen and pelvis
(E) Posterior sagittal anorectoplasty
Correct answers : (B)

111- A previously healthy 5-week-old boy born at 39 weeks’ gestation following


an uncomplicated pregnancy is brought to the emergency department (ED) with
a 3-day history of forceful vomiting after feeding. The vomitus looks like
undigested food. On physical examination, a distinct 1-cm mass is palpable in the
epigastrium. Surgical management is indicated.
i. Which of the following interventions is the most effective for reducing serious
complications associated with the indicated operation?
(A) Concurrent fundoplication
(B) Perioperative antibiotics
(C) Postoperative food/fluid restriction for 36 hours
(D) Postoperative supplemental oxygen
(E) Preoperative fluid resuscitation
Correct answers : E

ii. The patient undergoes pyloromyotomy. During the procedure, a deep mucosal
injury is noted at the distal aspect of the incision. How should this patient’s mucosal
injury be managed?
(A) Closure of the mucosal defect with the myotomy intact
(B) Closure of the mucosal and muscular defects with repeat myotomy at a later date
(C) Closure of the mucosal and muscular defects with repeat myotomy on another
side of the pylorus
(D) Wide local drainage and antibiotics
(E) Wide local drainage, antibiotics, and 10 to 14 days of postpyloric tube feeding
correct answers: C

112. A previously healthy 5-year-old girl presents to the ED with her parents
with a temperature of 100.8°F (38.2°C) and a 2-day history of decreased appetite
and persistent vague abdominal pain with tenderness in the mid-abdomen and
right lower quadrant. Her parents report that she has had no appetite and felt
nauseous but has not vomited. Laboratory results are unremarkable except for a
white blood cell count of 16,000 cells/mL (normal, 4500– 11,000 cells/mL).
Ultrasound of the abdomen and pelvis is inconclusive, and the patient is admitted
to the hospital for observation. Eighteen hours into her hospital stay, she passes
copious amounts of bloody stool. She remains hemodynamically stable with
normal vital signs and no change in her abdominal pain. What is this patient’s
most likely diagnosis?
A) Appendicitis
(B) Colonic arteriovenous malformation
(C) Colonic diverticulitis

- 304 -
(D) Gastric stress ulcer
(E) Meckel’s diverticulitis
correct answers: E

113. A previously healthy 2-month-old girl is brought to the ED with a 1-day


history of abdominal pain and emesis that developed over the last several hours.
The parents describe the vomitus as yellow-green and nonblood. Physical
examination reveals a fair appearing child with moderate diffuse tenderness of
the abdomen. Vital signs are normal. Laboratory results are unremarkable
except for a white blood cell count of 14,000 cells/mL. What is the next step in
this patient’s management?
(A) Empiric antibiotics and observation
(B) Immediate appendectomy
(C) Immediate exploratory laparotomy
(D) Ultrasound of the abdomen
(E) Upper gastrointestinal (GI) contrast study .
correct answers : E

114. A 1-day-old male infant has been intolerant of oral feeds and is vomiting.
The vomitus sometimes appears bile-colored. On examination, the patient’s
abdomen is mildly distended and somewhat tympanitic but is nontender to
palpation. A radiograph is performed in the following figure. What is this
patient’s most likely diagnosis?
(A) Annular pancreas
(B) Duodenal atresia
(C) Intestinal malrotation
(D) Jejunoileal atresia
(E) Pyloric stenosis
correct answers : B

19- Surgical instruments and tubes.

1 . Shoulder pain post laparoscopy is due to -


A. CO2 retention
B. Subphrenic abscess
C. Compressing of the lung
D. Positioning of the patient
Correct answer: A

2 . Vicryl, the commonly used suture material is a -


A. Homopolymer of polydiaxone
B. Ca-polymer of glycoside and lactide
C. Homopolymer of glycoside
D. Homopolymer of lactide
Correct answer: B

- 305 -
20- X-rays

1. Which among the following is least likely to be affected by radiation


A.skin
B.muscle
C.bone marrow
D.intestine
Correct answer B

2. Bull's eye lesion in ultrasonography is seen in


A. Candidiasis
B. Aspergillosis
C. Sporotrichosis
D. Cryptococcosis
Correct answer A

3. Unit of absorbed dose of radiation is


A.Becqueral
B.Gray
C.Stewart
D.Roentgen
Correct answer B

4. Which of the following is a characteristic radiological finding in neonatal


necrotizing enterocolitis
A. Gas in the portal system
B. Gas in the intestinal wall
C. Pneumoperitoneum
D. Air fluid levels
Correct answer B

5. The most common neurologic abnormality that occurs with head injury is -
A. Hemiplegia
B. Ocular verve palsy
C. Altered consciousness
D. Convulsions
Correct answer: C

6. 'Flower vase' appearance in IVP is seen in


A.Poly cystic kidney disease
B.Horse Shoe Kidney
C.Hydronephrosis
D.Ectopic kidney
Correct answer B

- 306 -
21- Jars and surgical pathology

22- Basic surgical skills (models)

Rectal examination

Wound care and stitch

1 . Non Absorbable suture among the following is ?


A. Polydiaxanone
B. Polyester
C. Polyglactic
D. Monocryl
Correct answer: B

2 . Silk is associated with infection because


A. Cannot be properly sterilized
B. It has crevices lodging bacteria
C. Infection goes deeper during bite
D. It is non-absorbable
Correct answer: B

3 . Which of the following is an absorb-able suture material ?


A. Polyamide
B. Polydioxanone
C. Polypropylene
D. Silk
Correct answer: B

4 . Which of the following is an absorbable suture material ?


A. Polyamide
B. Polydiaxone
C. Polypropeline
D. Silk
Correct answer: B

- 307 -
Uretheral catheterization

1 . Urethral catheterization is done in all except:


A. Postoperative acute retention
B. Traumatic urethral rupture
C. Urethral stenosis
D. Prostatic CA leading to retention
Correct answer: B

2 . Not true about urethral injuries is:


A. Catheterize the patient immediately
B. Can be associated with fracture pelvis
C. Bladder injury is associated with post urethral injuries
D. Blood at the external urethral meatus is an imp feature
Correct answer: A

3 . Chronic urethral obstruction due to benign prostatic hypertrophy can lead to the
following change in the kidney parenchyma -
A. Hyperplasia
B. Hypertrophy
C. Atrophy
D. Dysplasia
Correct answer: C

4 . For posterior urethral valve- investigation of choice is -


A. Cystoscopy
B. MCU
C. Cystourethroscopy
D. Retrograde urethroscopy
Correct answer: B

5 . Investigation of choice for a child having posterior urethral valve:


A. USG of kidney, ureter, bladder
B. Micturating cystourethrogram
C. Endoscopic examination
D. X-ray of pelvis
Correct answer: B

6 . Which of the following is not an appropriate investigation for anterior urethral


stricture?
A. Magnetic Resonance Imaging
B. Retrograde urethrogram
C. Micturating cystourethrogram
D. High frequency ultrasound
Correct answer: A

- 308 -
7 . All are true about bulbar urethral rupture, except -
A. Perineal hematoma
B. Floating prostate on per rectal examination
C. Collecting of urine in perineum
D. Bleeding per urethra
Correct answer: B

8 . Which of the following is the most common site of urethral carcinoma in men?
A. Prostatic urethra
B. Fossa Navicularis
C. Bulb membranous urethra
D. Penile urethra
Correct answer: C

Nasogastric tube

Examination of trauma patient

Vascular acess

23- Cardiothoracic surgery

Bronchogenic carcinoma

1. About bronchogenic carcinoma all are correct except:


a. Common in old smoker males
b. Can present by any chest symptom
c. Usually inoperable at diagnosis
d. 5 years survival rate is 60%
Correct answer: D
- 309 -
2. A 65 year old minor has lost 7 kgs weight within two months, has presented with
cough, and blood streaked sputum. He was treated for pulmonary tuberculosis 10
years ago. He also has drooping of his left eyelid for one month. On physical
examination, there is ptosis of the left eye and papillary Molisches X-ray revealed
round opacification in the left upper apical lobe. What is the most probable
diagnosis ?
A. Squamous cell carcinoma
B. Adenocarcinoma
C. Secondary tuberculosis
D. Asbestosis
Correct answer: A

3. Most common site of cholangiocarcinoma?


A. Hilum
B. Distal biliary duct
C. Multifocal
D. Intrahepatic duct
Correct answer: A

4 . Marker of Small cell cancer of lung is ?


A. Synaptobrevin
B. Chromogranin
C. Cytokeratin
D. Vimentin
Correct answer: B

5. Which of the following statements regarding the epidemiology of lung cancer


is correct?
(A) Chronic obstructive pulmonary disease (COPD) is not associated with an increased risk of
lung cancer
(B) Cigar smoking is associated with an equal risk of lung cancer as compared with cigarette
smoking
(C) Occupational exposure to radon is associated with an increased risk of lung cancer
(D) The risk of developing lung cancer for cigarette smokers exposed to asbestos is equal to
that of those not exposed to asbestos
Correct answer: C

6. A 53-year-old male smoker presents to the emergency department with


exertional dyspnea that has progressed to dyspnea at rest and a cough for 1 month.
Two weeks ago, he started having headaches and noticed swelling of his face.
Physical examination reveals prominent neck veins and edema of the upper limbs.
A chest radiograph demonstrates a suspicious nodule in the right hilar region.
Which of the following types of lung cancer is most commonly associated with the
signs and symptoms this patient is experiencing?
(A) Adenocarcinoma of the lung
(B) Carcinoid tumor of the lung
(C) Large cell lung cancer
(D) Small cell lung cancer (SCLC)
(E) Squamous cell lung cancer

- 310 -
Correct answer: D

7. Paraneoplastic syndromes are clinical disorders associated with malignant


diseases but not directly related to the physical effects of primary or metastatic
tumors’ Several paraneoplastic syndromes are more closely associated with a
particular subtype of bronchogenic carcinoma Which of the following
paraneoplastic syndrome is appropriately matched with its most commonly
associated histologic type of lung cancer?
(A) Ectopic Cushing’s syndrome/bronchial carcinoid tumor
(B) Hypercalcemia/adenocarcinoma
(C) Hypertrophic pulmonary osteoarthropathy and digital clubbing/SCLC
(D) Lambert-Eaton myasthenic syndrome (LEMS)/ squamous cell carcinoma
(E) Syndrome of inappropriate antidiuretic hormone (SIADH)/SCLC
Correct answer: E

8. A 60-year-old man with known squamous cell cancer of the lung presents to the
clinic for a follow-up appointment with complaints of dyspnea on exertion. The
patient has completed 3 cycles of cisplatin and gemcitabine. Surveillance
computed tomography (CT) scan of the thorax performed 2 days prior indicates a
stable 2-cm right lower lung lesion, a stable 1-cm right mediastinal lymph node,
and a new moderate-sized right pleural effusion. The patient subsequently
undergoes a right thoracentesis, which yields an exudative effusion with cytology
positive for squamous cells. Which of the following is the patient’s current stage
of disease?
A) Stage IIA
(B) Stage IIB
(C) Stage IIIB
(D) Stage IV
Correct answer: C

9. A 64-year-old nonsmoking woman presents to her primary care physician with


a 2-month history of cough, progressive shortness of breath, and recent
development of hemoptysis. She has been depressed recently with the death of her
69-year-old husband, who had a 60 pack-year smoking history and passed away
due to end-stage COPD. A chest radiograph reveals a suspicious lesion in the left
hilar region. CT of the thorax confirms a 3-cm left central hilar lesion and a 1-cm
ipsilateral peribranchial lymph node. Bronchoscopy yields bronchial washings
indicative of lymphocyte-like small round cells growing in sheets of scant
cytoplasm. What is the best treatment for this patient’s condition?
(A) Combination chemotherapy alone with etoposide and cisplatin
(B) Combination chemotherapy with a platinum-based regimen and concurrent thoracic
radiotherapy
(C) Single-agent chemotherapy and sequential thoracic radiotherapy with prophylactic
cranial irradiation
(D) Surgical resection of the 3-cm left hilar lesion, combination chemotherapy with a
platinum based regimen, and sequential thoracic radiotherapy.
Correct answer: B

- 311 -
10. A 13-year-old was referred to you with a chest mass. Following biopsy of
the mass the pathologist tells you that this is a small round blue cell tumor and
that immunostains are positive for neuron-specific enolase, synaptophysin, and
vimentin. Which of the following findings is most likely present in the tumor cells?
A. t(2;13)(q35;q14)
B. t(11;22)(q24;q12)
C. Ring chromosome
D. N-MYC amplification
E. t(X;18)(p11;q11)
Correct answer is B.

11. A 13-year-old was referred to you with a chest mass. Following biopsy of
the mass, the pathologist tells you that this is a small round blue cell tumor and
that immunostains are positive for neuron-specific enolase, synaptophysin, and
vimentin. Which of the following findings is most likely present in the tumor cells?
A. t(2;13)(q35;q14)
B. t(11;22)(q24;q12)
C. Ring chromosome
D. N-MYC amplification
E. t(X;18)(p11;q11)
Correct answer is B

Mediastinum and Chest Tube

1 . Most common tumor in the posterior mediastinum is:


A. Lymphoma
B. Bronchogenic cyst
C. Neurofibroma
D. Teratoma
Correct answer: C

2 . MOST COMMON anterior mediastinum tumor is -


A. Thymoma
B. Neurogenic Fibroma
C. Lymphoma
D. Meningocele
Correct answer: A

3 . Which tumour among the following is not found in Ant. Mediastinum -


A. Retrosternal goitre
B. Thymoma
C. Teratomatous mass
D. Neurogenic tumour
Correct answer: D

- 312 -
4 . Chest drain insertion is safest when performed -
A. In the apex of the chest for pneumothorax
B. In the base of the chest for hemothorax
C. In the 'triangle of safety' on the anterolateral chest wall
D. With the smallest bore chest tube available
Correct answer: C

5 . Most common site for putting chest drain in case of pleural effusion
A. 2nd intercostal space midclavicular line
B. 7th intercostal space midaxillary line
C. 5th I.C. space mid clavicular line
D. 5th I.C. space just lateral to vertebral column
Correct answer: B

Chest trauma

1 . The best treatment for uncomplicated fractures of the ribs is -


A. Leave it alone
B. Immediate surgery and wiring of the fragments
C. Strapping of the chest
D. Injection of local anesthetic around the involved intercostal nerve
Correct answer: A

2 . Which of the following is MOST COMMON cause of hypotension in fracture


ribs (T10 - T12):
A. Abdominal solid visceral organ injury
B. Injury to aorta
C. Inter costal artery damage
D. Pulmonary contusion
Correct answer: A

3. Which of the following statements about traumatic hemothorax is untrue?


a. May absorb spontaneously.
b. May be associated with signs of internal hemorrhage.
c. Cannot be diagnosed radiologically with less than 500 ml of blood.
d. Should be treated by repeated needle aspirations.
e. May require decortications of both lung and pleura.
Correct answer: A

4. The treatment of choice in flail chest is:


a. Tracheotomy.
b. Intercostal nerve block.
c. Adhesive strapping.
d. Elevation of broken ribs with towel clips.
e. Positive pressure ventilation.
Correct answer: E

- 313 -
5. Open chest wounds are characterized by the following except that they:
a. May cause "pendulum respiration".
b. May produce "mediastinal flutter.
c. May be associated with abdominal injuries.
d. Need immediate sealing of the wound.
e. Always require thoracotomy for definitive treatment.
Correct answer: E

6 . A man presented with fractures of 4th to 10th ribs and respiratory distress after
a Road Traffic Accident (RTA). He is diagnosed to have flail chest and a Pa O2 of
<60%, management is:
A. Tracheostomy
B. IPPV with oral intubation
C. Fixation of ribs
D. Strapping of chest
Correct answer: B

7 . A 50 year old smoker male presents with pain along the left arm and ptosis.
His chest radiograph shows a soft tissue opacity at the left lung apex with
destruction of adjacent ribs. The picture is suggestive of:
A. Adenocarcinoma lung
B. Bronchial carcinoid
C. Pancoast tumour
D. Bronchoalveolar carcinoma
Correct answer: C

8. The most life-threatening chest wall lesion is:


a- Emphysema
b- Tension pneumothorax
c- Flail chest
d- Hemothorax
Correct answer: B

9. All ore causes of hemothorax except:


a- Trauma
b- Bronchogenic carcinoma
c- Rupture of emphysematous bullae
d- Anticoagulant therapy
e- Hemophilia
Correct answer: C

10. About treatment of hemothorax all are correct except:


a- there is no need to decorticate of all
b- Resuscitation and stabilization of general condition has the priority
c- Encysted hemothorax is an indication of thoracotomy
d- Fibrinolysis may prevent clotting giving chance to conservation
Correct answer: A

- 314 -
11. As regards pathology of hemothorax all are correct except:
a- Blood always coagulates completely
b- Blood is defibrinated by continuous diaphragmatic motion
c- Hemothorax can result in large hydrothorax by transudation
d- lt can be complicated by fibrothorax or emphysema
Correct answer: A

12. In flail chest limitation of movement of flail part is done to:


a- Decrease risk of injury to intercostals structure
b- Prevent paradoxical movement and pendulum respiration
c- Prevent mediastinal flutter
d- Decreases pain
e- All of the above
Correct answer: E

13. About etiology of pneumothorax:


a- Spontaneous pneumothorax is always secondary to lung disease
b- Accidental pneumothorax needs a penetrating injury
c- Mechanical ventilation can result in pneumothorax
d- Central venous line insertion is not a reported cause
Correct answer: C

14. All of the following are radiological features of pneumothorax except:


a- Lung collapse on some side
b- Jet black opacity on some side
c- Shift of cardiac shadow to some side
d- Flat diaphragm on some side
Correct answer: C

15. The main line of treatment in closed pneumothorax without mediastinal shift
is:
a- Chest tube
b- Mechanical ventilation
c- Needle aspiration
d- Conservative treatment
Correct answer: D

16. The following aid(s) in diagnosis of open pneumothorax


a- Cyanosis
b- Congested non-pulsating neck veins
c- Harsh whistling sound following trauma
d- All of the above
Correct answer: D

17. For treatment of pneumothorax intercostals tube is inserted in


a- 5th space of parasternal line
b- 2nd space of parasternal line
c- 2nd space of midclavicular line
d- Any of the above
Correct answer: C

- 315 -
18. Circulatory failure in open pneumothorax is due to the following except:
a- Mediastinal flutter
b- Exaggerated negativity of intrathoracic pressure
c- Associated great vessel injury
d- Diminished cardiac function due to respiration failure
Correct answer: B

19. All of the following are differential diagnoses for sudden chest pain without
shock except:
a- Tension pneumothorax
b- Massive pulmonary embolism
c- Massive myocardial infarction
d- Flail chest
Correct answer: D

20. During the insertion of a subclavian catheter for hyperalimentation in a patient


with Crohn's disease the patient become dyspneic with a respiratory role of
32/min, pulse rate of l20/min. and drop of the B.P. to 80/60. The appropriate
immediate action is:
a. Chest X-ray,
b. Lung scan.
c. Intubation and mechanical ventilation.
d. Chest tube.
e. Vasopressors.
Correct answer: D

21. The following statements about spontaneous pneumothorax are true except
that it :
A. May occur in the absence of any pulmonary lesion.
B. is readily diagnosed clinically.
C. Usually resolves spontaneously.
D. May require intercostal catheter drainage.
E .Usually requires exploratory thoracotomy.
Correct answer: E

22. When should thoracotomy be considered in traumatic hemothorax?


a) if initially drain 800ml
b) if initially drain 1000ml
c) if sustained loss of 50ml/hr for 2 hours
d) if sustained loss of 200ml/hr for 2 hours
e) always
Correct answer: D

23. What is the largest size hemothorax that an injury to an intercostals artery can
cause?
a) Negligible-50ml
b) Small= 200ml
c) Moderate=600ml

- 316 -
d) Large = 1200ml
e) Massive=>1600ml
Correct answer: E

24.What percentage of rib fractures may not be apparent on CXR?


a) 10%
b) 20%
c) 30%
d) 40%
e) 50%
Correct answer: E

25.Which is incorrect with regards to the use of investigations to investigate


myocardial trauma in sternal fractures?
a) troponin should be requested in most patients
b) ECG should be done in most patients
c) If ECG normal and normal vital signs, no need to investigate further
d) If abnormal ECG they should be admitted and monitored
e) Transthoracic Echo can be used to assess myocardial wall motion
Correct answer: A

26.Which part of the heart is most often damaged in blunt cardiac trauma?
a) RV
b) LV
c) RA
d) RV
e) Interventricular septum
Correct answer: A
27.What percentage of pts with blunt trauma to the aorta who survive to hospital
will be dead within 24 hours if left untreated?
a) 10%
b) 20%
c) 30%
d) 40%
e) 50%
Correct answer: E

28.What is the next step if you see a stable patient with a good mechanism of injury
for traumatic aortic dissection and a widdenend mediastinum on CXR?
a) MRI
b) OT
c) Spiral CT
d) Aortography
e) TOE
Correct answer: B

29.Which sign on CXR is most specific and sensitive for traumatic aortic
dissection?
a) widened mediastinum

- 317 -
b) oesophageal deviation of >2cm to right at level of T4
c) obscuration of the aortic knob
d) small left hemothorax
e) depression of the R main bronchus >40 degrees below horizontal
Correct answer: A

30. Radiopaque shadow seen immediately after trauma to the chest could be due
to
A.pulmonary embolism
B.pulmonary contusion
C.hemothorax
D.pneumonia
Correct answer C

Suppurative lung diseases

1. The following organisms cause a sever type of empyema complicated by dense


adhesions:
a- Staph
b- Streptococci
c- Pneumococci
d- E. Coli
Correct answer: C

2. All of the following ore indications of open surgical drainage of pus in acute
empyema except:
a- thick pus
b- Rapid recollection of pus
c- Streptococcal empyema
d- Extensive conditions
Correct answer: C

3. As regards Empyema necessitans all are correct except:


a- ls an encysted empyema
b- Gives expansible impulse on cough
c- Can perforate skin leading to skin sinus
d- is a subcutaneous abscess communicated with empyemic pleura
Correct answer: A

4. The following are signs of chronic empyema except:


a- Paroxysmal fever and chills
b- Flattening of diaphragm
c- Sinus in chest wall discharging pus
d- Scoliosis
Correct answer: B

- 318 -
5. All of the following ore complications of chronic empyema except:
a- Empyema necessitans
b- Amyloidosis
c- Mesothelioma
d- Toxic arthritis
Correct answer: C

6. The following factor(s) predispose to chronic empyema:


a- Pulmonary tuberculosis
b- DM
c- Inadequate drainage
d- Bronchogenic carcinoma
e- All of the above
Correct answer: E

7. ln a case of pleural effusion. The following statements are correct except that:
a. The diagnosis can be made with as little as 100 ml of fluid.
b. The trachea may be displaced to the opposite side.
c. Aspiration of blood-stained fluid is highly suspicious of bronchogenic carcinoma.
d. Open biopsy of the pleura is necessary.
e. Rapid aspiration of the fluid is not advisable.
Correct answer: A

8. Pleural fluid with the appearance of "anchovy sauce” pus is characteristic of:
a. E. coli.
b. Staph. aureus.
c. Echinococcus.
d. Entamoeba histolytica.
e. Hemothorax.
Correct answer: D

9. The primary treatment of Tuberculous empyema is:


a. Systemic chemotherapy.
b. Aspiration.
c. Closed tube drainage.
d. Open drainage.
Correct answer: A

10 . A 23years man with pulmonary TB presents with massive recurrent


hemoptysis.For angiographic treatment,Which vascular structure should be
evaluated first:
A. Bronchial artery
B. Pulmonary artery
C. Superior vena cava
D. Pulmonary vein
Correct answer: A

- 319 -
Ischemic Heart Disease

1. Dichrotic pulse is seen in


A. Cardiac tamponade
B. Aortic regurgitation
C. Dilated cardiomyopathy
D. Restrictive cardiomyopathy
Correct answer C

2. Which of the following ECG finding is diagnostic of cardiac ischaemia in


treadmill test
A. flat ST depression
B. upsloping ST depression
C. ventricular tachycardia
D. T wave inversion
Correct answer A

3. In acute myocardial infarction the best drug to cause thrombolysis and to


start reperfusion is
A. streptokinase
B. urokinase
C. tissue plasminogen activator
D. anisoylated plasminogen streptokinase activator complex
Correct answer C

Valvular heart diseases

Cong-Heart Diseases

Diaphragm

1 . About Diaphragmatic injury, true statement is -


A. The treatment is conservative
B. Resolves spontaneously

- 320 -
C. Left side more common
D. Associated with pneumothorax
Correct answer: C

2 . Which of the following is the most important prognostic factor in congenital


diaphragmatic hernia?
A. Timing of surgery
B. Gestational age
C. Pulmonary hypertension
D. Size of hernia
Correct answer: C

24- Neurosurgery

Introduction and CNS examination

1.Which cranial nerve does not travel through the orbital foramen?
a) 2
b) 3
c) 4
d) 5
e) 6
Correct answer: C

2. Which nerve root is incorrect?


a. Elbow extension – C7
b. Finger abduction- T1
c. Hip Flexion- T12
d. Ankle dorsiflexion-L4
e. Ankle plantar flexion – S1
Correct answer: C

3. Senile plaques in brain is a feature of


A. Multiple sclerosis
B. Parkinsonism
C. Alzheimer's disease
Correct answer C

- 321 -
4. In the surveillance for Acute Flaccid Paralysis, the age group that is included
is below
A. 5years
B. 12years
C. 8years
D. 16years
Correct answer B?

5 . Most common tumor which metastasis to brain is ?


A. Lung
B. Breast
C. Bone
D. Prostate
Correct answer: A

6 . Which of the following brain tumours is highly vascular in nature?


A. C P angle epidermoid
B. Pituitary adenomas
C. Glioblastoma
D. Meningiomas
Correct answer: C

7 . Most common site of brain metastasis?


A. Brainstem
B. Cerebellum
C. Cerebral cortex
D. Thalamus
Correct answer: C

8 . A person has been brought in casualty with history of road accident. He had
lost consciousness transiently and then gained consciousness but again become
unconscious. Most likely, he is having brain hemorrhage of -
A. Intracerebral
B. Sub arachnoid
C. Sub Dural
D. Extra Dural
Correct answer: D

9 . Clinical features of anterior cranial fossa bleeding with fracture at base of


skull includes:
A. Anosmia
B. Periorbital hematoma
C. CSF rhinorrhea
D. CSF otorrhea
Correct answer: A

10 . In skull fracture, the condition in which an operation is not done


immediately is
A. Depressed fracture
B. Compound fracture

- 322 -
C. CSF leak
D. Increased size of head
Correct answer: C

11 . A patient presents in emergency with a cervical spine fracture. First thing to


do is:
A. Locate the fracture by shifting the patient side to side
B. X-ray of spine
C. Clear the airway and intubate him
D. Immobilize the cervical spine
Correct answer: D

12 . Regarding Glasgow coma scale, which is not true -


A. Range from 6-12
B. Low score indicates deteriorating brain function
C. Based on eye opening, verbal response and motor response
D. Score below 5 show poor prognosis
Correct answer: A

13 . Maximum minimum score in Glasgow coma scale is ?


A. 15 and 0
B. 14 and 3
C. 15 and 3
D. 15 and 1
Correct answer: C

14 . All are true about Glasgow coma scale, except -


A. Score between 3-15
B. Obeying motor command is given maximum score
C. Consists of eye opening, motor and verbal response
D. Increased score indicates poor prognosis
Correct answer: D

15 . According to the Glasgow Coma Scale (GCS),a verbal score of 1 indicates


which of the following?
A. Incomprehensible sounds.
B. Disoriented response.
C. No response.
D. Inappropriate words.
Correct answer: C

16 . Prognosis in head injury is best given by -


A. Mode of injury
B. CT head
C. Glasgow coma scale
D. Age of patient
Correct answer: C

- 323 -
17 . A 45 years RTA patient with 'Inappropriate speech' evaluated by the
'Glasgow Coma Scale' will have a verbal score of:
A. 3
B. 4
C. 1
D. 2
Correct answer: A

18 . For a conscious person, total score in Glasgow Coma Scale is:


A. 3
B. 8
C. 10
D. 15
Correct answer: D

19 . The diagnosis of concussion is established by which one of the following.


A. Pupillary constriction
B. Nausea and vomiting
C. Brief loss of consciousness and amnesia for events
D. A positive Babinski's sign
Correct answer: C

20 . In Erb-Duchene paralysis all occurs except:


A. Increased angle between neck and shoulder during delivery
B. Adduction of shoulder, mid-pronation and wrist flexion
C. Paralysis of pronator
D. Weakness of flexors and medial rotators
Correct answer: C

21. A 43-year-old man experiences lower back pain after lifting a heavy object off the
ground. The following morning, he notices that the pain has begun to radiate down the
posterolateral aspect of the right leg and across the top of the foot to the big toe. The
pain is severe, electric in quality, associated with paresthesia over the same distribution,
and made worse by coughing. On examination, it is found that he has an area of
diminished sensation to pinprick over the dorsum of the right foot and mild weakness
in his right extensor hallucis longus muscle. The deep tendon reflexes are all intact.
What is the most likely diagnosis?
(A) Lumbar spinal fracture with compression of the caudaequina
(B) Herniated lumbar disk on the right at the level of L4–L5
(C) Herniated lumbar disk on the left at the level of L4–L5
(D) Herniated lumbar disk on the right at the level of S1–S2
(E) Intermittent claudication
Correct answer: B

22. A 48-year-old woman has a lower back pain and hypoesthesia in the left S1
dermatomal distribution (left calf and lateral left foot). What is the most likely cause?
(A) A lesion at the right L4–L5 interspace
(B) Pathology where the nerve exits the spinal canal immediately above the pedicle of
S3 vertebra
(C) A herniated nucleus pulposus

- 324 -
(D) Compression by the L5 lamina
(E) A lesion outside the vertebral column
Correct answer: C

23. A 35-year-old secretary complains of severe pain in the neck that radiates down the
right arm. The pain is electric in quality and affects specifically the radial aspect of the
right forearm and the thumb. She also describes numbness and paresthesia over the
same distribution. On physical examination, she is found to have an area of diminished
sensation to pinprick over the right wrist and thumb. The right biceps tendon reflex is
diminished, but there is no loss of muscle strength. She has right C5–C6 disk
compression and radiculopathy affecting which of the following?
(A) The right C4 root
(B) The right C4 mixed spinal nerve
(C) The right C4 anterior primary rami
(D) The right C6 root
(E) The right C6 spinal ganglion
Correct answer: D

24. A 57-year-old woman presents to the emergency department with new-onset


seizures. She was witnessed by her husband to have a generalized seizure lasting
approximately 1 minute. She has smoked 1 pack of cigarettes a day for over 40 years.
In the past 3 months, she has lost 25 lb in weight. On examination, she appears thin and
nervous but findings on her neurologic examination are otherwise essentially within
normal limits. Plain radiographs of the chest obtained in the emergency department
show a 4-cm nodule in the upper lobe of her right lung. To exclude cerebral metastasis
as a cause of her seizure, what should the next test requested be?
(A) An electroencephalogram (EEG)
(B) A CT scan of the brain with intravenous contrast
(C) Aspinal tap to measure opening pressure and obtain CSF for cytology
(D) An MRI of the brain with intravenous contrast
(E) Doppler ultrasound.
Correct answer: D

25. A 64-year-old man presents with headache and left-sided upper extremity weakness.
The MRI findings suggest that this is a glioblastoma multiform. This is because the
tumor exhibits which of the following?
(A) It is regular in shape.
(B) It is well demarcated from surrounding brain tissue.
(C) It shows a ring pattern of enhancement with intravenous contrast and has a
nonenhancing necrotic center.
(D) It shows an absence of surrounding white-matter edema.
(E) It arises from the carotid body
Correct answer: C

26. A 63-year-old woman presents for workup to determine the reason for a gradual
hearing loss over approximately 5 years and intermittent tinnitus over the last several
months. Findings on physical and neurologic examination are entirely within normal
limits, except for the presence of sensorineural hearing loss in the left ear. She has no
cranial nerve deficits. An MRI of the brain with gadolinium reveals the presence of an

- 325 -
extra-axial tumor in the region of the left cerebella-pontine angle. What is the most
likely diagnosis?
(A) Epidermoid tumor (cholesteatoma)
(B) Glioblastoma multiforme
(C) Meningioma
(D) Acoustic neuroma
(E) Glomus tumor
Correct answer: D

27. In the management of a 64-year-old woman struck by a car, mannitol is given to do


which of the following?
(A) Increase CSF formation
(B) Increase the respiratory rate
(C) Increase the pulse rate
(D) Replace extensive fluid loss
(E) Lower raised ICP
Correct answer: E

28. Following a sudden impact in an accident, the 34-year-old race car driver becomes
unconscious and is admitted to the hospital. A CT scan is performed, and a right space-
occupying lesion is noted (Fig. 11–1). What is the most likely diagnosis?
(A) Corpus callosum injury
(B) Pituitary apoplexies
(C) Acute subdural hematoma
(D) Acute epidural hematoma
(E) Chronic subdural hematoma
Correct answer: C

29. A 64-year-old woman complains of gait imbalance, headache and deterioration of


mental status over the past several months. Her vision is normal. A CT scan reveals
hydrocephalus, but the lumbar puncture pressure is unexpectedly low. What does she
have?
(A) Meningitis
(B) Normal-pressure hydrocephalus
(C) Sigmoid sinus thrombosis
(D) Echinococcus
(E) Glioblastoma multiforme
Correct answer: B

30. A 23-year-old woman complains of progressive loss of vision and papilledema.


Investigations show normal findings on CT scan. A lumbar puncture shows marked
elevation of pressure. What is the most likely diagnosis?
(A) Pseudotumorcerebri
(B) Corpus cavernous thrombosis
(C) Cavernous sinus thrombosis
(D) Retinoblastoma
(E) Chordoma
Correct answer: A

- 326 -
31. During her eighth month of pregnancy, a 29-year-old woman is noted to have
hydramnios. Further testing shows anencephalous. In this case hydramnios is caused by
which of the following?
(A) Impairment of the fetus’s swallowing mechanism
(B) Tumor of the fetus’s brain
(C) A secretory peptide from the placenta
(D) Excess antidiuretic hormone (ADH) from the fetus
(E) Renal agenesis
Correct answer: A

32. A 28-year-old man presents with a history of chronic headache. The headache is
intermittent, severe, poorly localized, and most often present when he arises in the
morning. He suffered a severe blow to the head and sustained a skull fracture at the age
of 15. Findings on his physical and neurologic examinations are within normal limits.
An MRI of the brain with gadolinium reveals the presence of a large, nonenhancing
extra-axial cyst in the region of the right temporal tip. This most likely represents which
of the following?
(A) An arachnoid cyst
(B) A cystic astrocytoma
(C) Rathke’s cleft cyst
(D) A Dandy-Walker cyst (failure of proper formation of the foramina of Lushka and
Magendie)
(E) Polycystic disease
Correct answer: A

33. A 15-year-old boy complains of right-sided weakness and gait impairment. ACT
scan shows a large, nonenhancing cyst in the posterior cranial fossa, with an enhancing
tumor nodule in the left cerebellum. What is the most likely diagnosis?
(A) An arachnoid cyst
(B) A cystic astrocytoma
(C) Rathke’s cleft cyst
(D) Glioblastomamultiforme
(E) A large sebaceous cyst
Correct answer: B

34. A 43-year-old man is treated with pyridostigmine for facial, ocular, and pharyngeal
weakness due to myasthenia gravis. Which statement is true of pyridostigmine?
(A) It is unrelated to neostigmine.
(B) It has far more side effects than neostigmine.
(C) Pyridostigmine and neostigmine reverse depolarizing neuromuscular blockade.
(D) It causes greater muscarinic effect than neostigmine.
(E) It is an anticholinesterase agent
Correct answer: E

35. A 17-year-old male presents with 3-month history of headache, weight gain,
decreased concentration, polyuria, and polydipsia. His headaches are mostly in morning
and involves the frontal region. On examination he was found to have bitemporal visual
field defect and no facial hair. MRI scan revealed a suprasellar partially calcified cystic
lesion with displacement of optic chiasm. The most likely pathology is:
(A) Giant aneurysm of carotid artery

- 327 -
(B) Pituitary macroadenoma
(C) Glioblastoma multiforme
(D) Craniopharyngioma
(E) Testicular metastasis
Correct answer: D

36. He underwent a craniotomy for resection of his lesion. Twelve hours


postoperatively, he developed diuresis of over 500 mL/h. The diagnosis of (DI) was
entertained. What laboratory findings are most consistent with the clinical impression?
(A) Urine specific gravity of over 1010
(B) Serum sodium of less than 135
(C) Decreased both serum and urine osmolality
(D) Increased serum osmolality and decreased urine osmolality
(E) Increased both serum and urine osmolarities
Correct answer: D

37. A 45-year-old woman was brought to emergency department for sudden onset of
severe headache associated with photophobia, nausea, and transient loss of
consciousness. On examination, she is awake and alert with normal cranial nerve
function. She also exhibits normal muscle strength and sensation. Her past medical
history is significant for sickle cell disease (SCD) and hypertension. CT scan confirms
the diagnosis of SAH without any intraparenchymal abnormality. What is the least
likely cause of SAH?
(A) Aneurysmal bleed
(B) Sickle cell angiopathy
(C) Arteriovenous malformation (AVM)
(D) Hemorrhagic meningioma
(E) Blood dyscrasia
Correct answer: D

38. What is the most definitive diagnostic test in this condition?


(A) CT angiography
(B) Magnetic resonance angiography (MRA)
(C) Cerebral angiogram
(D) MR spectroscopy
(E) Positron emission tomography (PET) Scan
Correct answer: C

39. What is the most likely complication of angiography in this patient?


(A) Cerebral stroke
(B) Aneurysmal rupture
(C) Increased intracranial pressure
(D) Vascular wall damage
(E) Sickle cell crisis
Correct answer: E

- 328 -
Congenital CNS anomalies

1 . A newborn with meningomyelocele has been posted for surgery. By which of the
following, the defect should be immediately covered-
A. Tincture Benzoin Gauze
B. Methylene Blue Gauze
C. Normal Saline Gauze
D. Povidone Iodine Gauze
Correct answer: C

Head injuries

1. Where is Zone 1 in the Roon and Christensen classification of neck trauma?


a) Clavicles to cricoid cartilage
b) Cricoid cartilage to angle of mandible
c) Angle of mandible to base of skull
d) Midline to anterior border of sternomastoid
e) Post border of sternomastoid to ant border of trapezius
Correct answer: A

2.Which facial view x ray is the best for examining the orbits and midface?
a) OPG
b) Waters or occipitomental
c) Caldwell or PA view
d) Submental vertex
e) Towne
Correct answer: B

3. A tripod fracture involves what?


a) it is another word for le fort fracture
b) zygomatico - temperal and zygomatico- frontal suture diastasis and inferiororbital
rim fracture
c) fracture through maxilla, zygomatic arch and nasal bones
d) fracture through neck angle and body of the mandible
e) superior and inferior orbital rim fracture and diastasis of zygomaticofrontal suture
Correct answer: B

4.What is the name of the fracture that is clinically detected by tugging on the
maxilla/hard palate causing the nose to move.
a) le fort 5
b) le fort 4
c) le fort 3
d) le fort 2
e) le fort 1
- 329 -
Correct answer: D

5 . Clinical feature of fracture of Zygomatic bone include all of the following except:
A. Bleeding
B. C.S.F. rhinorrhea
C. Diplopia
D. Trismus
Correct answer: B

6. Bloody discharge from the ear occurs in:


a. Fracture base of the skull.
b. Glomus jugular tumor.
c. Hemorrhagic otitis media.
d. Rupture drum.
e. All of the above.
f. Non of the above.
Correct answer: E

7. Pain in tempro-mandibular joint is referred to the ear through:


a. The 5th nerve.
b. The 9th nerve.
c. The 10th nerve.
d. 2nd and 3rd cervical nerve.
Correct answer: A

8. Pain in cancer larynx is referred to the ear through:


a. The 5th nerve.
b. The 9th nerve.
c. The 10th nerve.
d. 2nd and 3rd cervical nerve.
Correct answer: C

9 . Which of the following bacterial meningitis is associated with subdural effusion ?


A. H Influenza
B. Neisseria meningitis
C. Streptococcus pneumonia
D. Enterococcus
Correct answer: A

10. Watery discharge from the ear occurs in all of the following EXCEPT:
a. Cerebra-spinal otorrhea.
b. Parotid fistula.
c. Acute otitis media.
d. Endolymphatic sac surgery.
Correct answer: C

11. In lesion of facial nerve at vertical part below the nerve of stapedius"
there is:
a. Loss of taste.
b. Impairment of salivation.
c. Impairment of lacrimation.

- 330 -
d. Hyperacusis.
Correct answer: A

12. In lesion of the facial nerve at horizontal part, there is:


a. Loss of taste.
b. Impairment of salivation.
c. Impairment of lacrimation.
d. Hyperacusis.
e. All of the above.
Correct answer: E

13. In lesion of the facial nerve at the geniculate ganglion, there is:
a. Loss of taste.
b. Impairment of salivation.
c. Impairment of lacrimation.
d. Hyperacusis.
e. All of the above.
Correct answer: E

14. In UMNL of the facial nerve, there is:


a. Paralysis of the muscles of the lower 1/2 of the face on the opposite side.
b. Paralysis involves the voluntary but spares the emotional and associative movement.
c. Hypotonia.
d. Hyporeflexia.
Correct answer: A and B

15. In LMNL of the facial nerve, there is:


a. Paralysis of the muscles of the lower 1/2 of the face on the opposite side.
b. Paralysis involves the voluntary but spares the emotional and associative movement.
c. Hypotonia.
d. Hyporeflexia.
e. Reaction of degeneration.
Correct answer: C; D and E .

16. In LMNL of the facial nerve, there is:


a. Paralysis of the muscles of the lower and upper 1/2 of the face on the opposite side.
b. Paralysis involves the voluntary, emotional and associative movement.
c. Paralysis of the muscles of the lower and upper 1/2 of the face on the same side.
Correct answer: B and C.

17. Bell's palsy is LMNL at the level of:


a. Geniculate ganglion.
b. Internal facial auditory meatus.
c. Stylomastoid foramen.
d. Cerebellopontine angle.
Correct answer: C

18. The cause of Bell's palsy may be one of the following EXCEPT:
a. Vascular ischemia.
b. Virus infection.
c. Bacterial infection.

- 331 -
d. Auto immune.
Correct answer: A; B and D.

19. The early symptom of Bell's palsy is:


a. Dropping of angle of the affected side.
b. Obliteration of the angle of the mouth.
c. Pain of acute onset behind the ear.
d. Inability to close the eye.
Correct answer: C.

20. The earliest manifestation of cavernous sinus thrombosis is:


a. Fever.
b. Ptosis.
c. Proptosis.
d. Ophthalmoplegia.
Correct answer: d.

21. The pathology in case of Bell's palsy is:


a. Facial nerve tumor.
b. Cut in the tympanic segment in the facial nerve.
c. Edema of the facial nerve inside its bony canal.
d. Hemorrhage in the facial nerve nucleus.
Correct answer: C.

22. The most accurate diagnostic test to detect degeneration of the facial nerve:
a. Nerve excitability test.
b. Electromyography.
c. Electroneurography.
d. Stapedial reflex.
Correct answer: C.

23. Facial palsy is most commonly:


a. Neoplastic.
b. Traumatic.
c. Herpetic.
d. Bell's palsy.
Correct answer: D.

24 . After 4 weeks of Head trauma, patient presents with features of irritability and
altered sensorium. Commonest cause will be:
A. Chronic subdural hematoma
B. Extradural hematoma
C. Intraparenchymal bleed
D. Electrolyte imbalance
Correct answer: A

- 332 -
Subarachniod hemorrhage and neurovascular diseases

Brain tumours

1 . FACIAL Nerve is most commonly involved in


A. Warthin's tumour
B. Pleomorphic adenoma
C. Metastatic
D. Adenitis
Correct answer: C

Spinal tumours

1. Most common cause of myelophthisic anemia (disorder of the spinal cord or


of bone marrow.) is
A. Multiple myeloma
B. NHL
C. Lukaemia
D. Multiple secondaries
Correct answer D?

Spinal fracture

Peripheral nerve injuries

1. All are true about amyotropic lateral sclerosis except


A. Bowel and bladder involvement is present
B. LMN in upper limb
C. UMN in lower limb
D. No sensory involvement
Correct answer A

- 333 -
2. Sphenoid wing dysplasia is seen in
A.Von Hippel Lindau's disease
B.Neurofibromatosis
C.Sturge Weber syndrome
D.Bournville's disease
Correct answer B

3 . Causalgia in a limb can be sequel to


A. Arterial spasm
B. Nerve injury
C. Venous insufficiency
D. All of the above
Correct answer: B

25- Urology:

Urological Case taking

1- Sample collection for renal tuberculosis is -


A. Single early morning urine sample
B. 24 hours urine collection
C. 3 random urine collection
D. 3 morning samples of urine
Correct answer: D

2- In a BPH patient, most important indication of surgery is :


A. Can not use medication due to hypertension
B. Bilateral hydronephrosis
C. Prostate size >70 gm
D. Single episode of UTI requiring 3 day antibiotics
Correct answer: B

3- Indication of surgery in benign prostatic hypertrophy are all except -


A. Prostatism
B. Chronic retention
C. Hemorrhage
D. Enlargement prostate
Correct answer: D

4. Renal papillary necrosis is seen in all the following except


A. analgesic nephropathy
B. hydronephrosis

- 334 -
C. acute pyelonephritis
D. sickle cell disease
Correct answer B

5. Investigation of choice for Posterior urethral valve is


A. Retrograde cystourethrogram
B. MCU
C. IVP
D. USG
Correct answer B

6 . Prostatic carcinoma commonly metastasis to the vertebrae:


A. Because valveless communication exist with batsons periprostatic plexus
B. Via drainage to sacral lymph node
C. Of direct Spread
D. None of the above
Correct answer: A

7 . A 62 year old male was found to have a solid renal tumor on CT scan which
had invaded the renal vein and inferior vena cava. All are true except:
A. Pre-op biopsy is not indicated
B. IVC invasion means inoperable condition
C. Pre-op radiotherapy or chemotherapy is not done
D. Chest x-ray is done to rule out pulmonary metastasis
Correct answer: B

8 . A patient presented with renal cell carcinoma invading IVC and the renal
vein. False statement is:
A. Preop biopsy is not necessary
B. IVC involvement indicates inoperability
C. Pre op radiotherapy is not essential
D. Chest x-ray should be done to rule out pulmonary metastasis
Correct answer: B

9 . Tributaries of Lt renal vein are all except -


A. Left lumbar vein
B. Left gonadal vein
C. Left suprarenal vein
D. Left diaphragmatic vein
Correct answer: A

10 . Renal collar which surrounds the aorta has its two limbs split by:
A. Left renal vein
B. Left renal artery
C. Isthmus of horseshoe kidney
D. All of the above
Correct answer: A

11 . 7 cm suprarenal mass diagnosed with hypokalemia, hypertension. Diagnosis:


A. Cushing's syndrome

- 335 -
B. Pheochromocytoma
C. Adrenocortical carcinoma
D. Hypernephroma
Correct answer: C

12 . Not correct regarding renal cell carcinoma is -


A. May be associated with varicocele
B. May invade renal vein
C. More common in females
D. Arises from proximal convoluted tubule
Correct answer: C

13 . All are true regarding renal call carcinoma except one -


A. Females > Males
B. Polycythemia common
C. Is radioresistant
D. Invades renal vein
Correct answer: A

14 . Not true about renal cast -


A. Albumin secreted from glomeruli form cast
B. Hyaline cast may be found in normal condition
C. Broad cast is a feature of CRF
D. Red cast is a feature of glomerulonephritis
Correct answer: A

15 . Not a CT feature of Adrenal adenoma.


A. Homogeneous density and well-defined borders
B. Low attenuation
C. Calcification is rare
D. Enhances rapidly, contrast stays in it for relatively longer time and washes out late
Correct answer: D

16 . Sterile pyuria is seen in -


A. Wilms' tumor
B. Renal tuberculosis
C. Neuroblastoma
D. Chronic pyelonephritis
Correct answer: B

17 . A 33 years patients with a suspected renal tumor / mass is diagnosed by


exfoliative urinary cytology. Which histological types is most likely to be
diagnosed on urinary exfoliative cytology:
A. Well differentiated carcinoma (low grade)
B. All three types can be easily detected on exfoliative cytology
C. Transitional cell carcinoma
D. Adenocarcinoma
Correct answer: C

- 336 -
18 . Renal Calculi associated with proteus infection
A. Triple Phosphate
B. Uric Acid
C. Xanthine
D. Calcium Oxalate
Correct answer: A

19 . Which of the following is not a contraindication for extra corporeal


Shockwave Lithotripsy (ESWL) for renal calculi?
A. Uncorrected bleeding diathesis
B. Pregnancy
C. Ureteric stricture
D. Stone in a calyceal diverticulum
Correct answer: D

20 . Which of the following is the most important prognostic indicator for Renal
cell carcinoma?
A. Size
B. Pathological staging
C. Nuclear grade
D. Histological type
Correct answer: B

21 . Most common cause of painless hematuria -


A. Hypernephroma (Renal cell carcinoma)
B. Bulent trauma to abdomen
C. T.B. of renal parenchyma
D. Hydronephrosis
Correct answer: C

22 . The most common renal vascular anomaly is-


A. Double renal arteries
B. Double renal veins
C. Supernumerary renal arteries
D. Supernumerary renal veins
Correct answer: C

23 . Which of the following renal calculi is associated with proteus infection?


A. Triple Phosphate
B. Uric Acid
C. Xanthine
D. Calcium Oxalate
Correct answer: A

24 . Statements about acute adrenal insufficiency are true Except-


A. Acute adrenal insufficiency presents with weakness, vomiting, fever, and
hypotension.
B. Hyponatremia occurs because of impaired renal tubule sodium resorption
C. Hyperglycemia is usually present

- 337 -
D. Acute adrenal insufficiency usually is secondary to exogenous glucocorticoid
administration.
Correct answer: C

25 . Which of the following renal calculi is associated with proteus infection?


A. Triple phosphate
B. Uric acid
C. Xanthine
D. Calcium oxalate
Correct answer: A

26 . In 'Three glass Test' shreds are present in 1st glass only.The most probable
diagnosis should be ?
A. Urethritis
B. Cystitis
C. Prostatitis
D. Renal pathology
Correct answer: A

27 . There is a high risk of renal dysplasia in


A. Posterior urethral valve
B. Bladder exstrophy
C. Anorectal malformation
D. Neonatal sepsis
Correct answer: A

28 . Triad of Renal colic, swelling in loin which disappears after passing urine is
called
A. Kocher's triad
B. Saint's triad
C. Diestel’s crisis
D. Charcot's triad
Correct answer: C

29 . Nephroureterectomy is indicated in ?
A. Renal carcinoma
B. Chronic pyelonephritis
C. Polycystic kidney disease
D. Transitional carcinoma of the pelvis extending till ureter
Correct answer: D

30 . Not correct regarding Renal Cell carcinoma -


A. May be associated with varicocele
B. May invade renal vein
C. More common in female
D. Arises from proximal convoluted tubule
Correct answer: C

- 338 -
31 . All of the following are associated with renal cell carcinoma, except:
A. Amyloidosis
B. Polycythemia
C. Hypertension
D. Cushing's syndrome
Correct answer: D

32 . Which drug is not a part of the Triple Therapy immunosuppression for post-
renal transplant patients?
A. Azathioprine
B. Cyclosporine
C. Prednisolone
D. FK 506
Correct answer: D

33 . Whitaker test is done for ?


A. Renal carcinoma
B. Renal tuberculosis
C. Wilm's tumor
D. Hydronephrosis
Correct answer: D

34 . Ca urinary bladder commonly presents as -


A. Hematuria
B. Frequency
C. Dysuria
D. Abdominal lump
Correct answer: A

35 . One of the following disease will show urinary bladder calcification


radiologically which resemble fetal head in pelvis
A. Tuberculosis
B. Schistosomiasis
C. Chronic cystitis
D. Malignancy
Correct answer: B

36 . All of the following are true of urinary bladder calculus EXCEPT


A. Pain is referred to the tip of the penis
B. Initial hematuria is present
C. Urinary stream gets interrupted
D. Frequency occurs early
Correct answer: B

37 . Regarding urinary bladder stone one is not true -


A. Common in Pediatric patients in tropics than that of non tropical areas
B. Uric acid stone are dropped from above
C. Jack stone is due to protein splitting bacteria
D. Commonly distal passage obstruction cauuse stone
Correct answer: B

- 339 -
38 . Commonest symptom of CA urinary bladder is -
A. Increase frequency
B. Hematuria
C. Lower abdominal pain
D. Perineal pain
Correct answer: B

39 . The most common site for an acquired bladder diverticulum is


A. In the dome of the bladder
B. In the anterior bladder wall
C. Adjacent to the ureteric orifice
D. In the midline posteriorly
Correct answer: C

40 . Not true about rt. kidney is:


A. Rt. kidney is preferred over the left for transplantation
B. It is lower than the left kidney
C. Rt. renal vein is shorter than the left
D. Rt. kidney is related to the duodenum
Correct answer: A

41 . Gleason score: all are true except -


A. Scores range from 1-10
B. Used for grading prostate cancer
C. Helps in planning management
D. Higher the scores, poorer the prognosis
Correct answer: A

42 . Penile intraepithelial neoplasia is predisposed by:


A. Phimosis
B. Balanitis obliterans xerotica
C. HSV infection
D. Condyloma acuminata
Correct answer: B

43. What is not true of drug treatment of BPH:


a. Alfa adrenergic inhibition reduces bladder outlet obstruction
b. 5 Alpha reductase inhibitors decrease level of dihydrotestosterone
c. These drugs are expensive rather than effective
d. All are true
Correct answer : D

44. Atypical presentation of hypernephroma includes:


a. Fever of unknown origin
b. Polycythemia
c. Hemoptysis/pathological fracture
d. All of the above
Correct answer : D

- 340 -
45. The common malignant tumor of renal pelvis and ureter is:
a. Squamous carcinoma
b. Adenocarcinoma
c. Transitional cell carcinoma
d. Papillary carcinoma
Correct answer : C

46 . Investigation of choice in case of 50 years old patient with low back pain and
urinary retention:
A. Acid phosphatase
B. Alkaline phosphatase
C. Ultrasound abdomen
D. Electrophoresis
Correct answer: A

47 . Not true about paraphimosis is -


A. Iatrogenic
B. Seen in diabetes mellitus
C. Gangrene of glans
D. Circumcision is the the treatment
Correct answer: B

48 . Evaluation of renal functions before IVU is best done by measuring:


a. Urine specific gravity.
b. Urea level.
c. Creatinine level.
d. None of the above.
Correct answer: B

49. The following is incorrect regarding urinary investigations:


a. With IVP the kidney may not be visualized if congenitally absent, surgically
removed.
b. The best advantage in performing CT in urinary tumors is to assess exact extent and
invasion.
c. U/S is indicated for any renal cyst.
d. IVP is not indicated after any abdominal trauma.
e. isotope imaging shows fractional renal function.
Correct answer: D

50. Benign cyst by U/S all are correct except:


a. Is smooth containing clear fluid.
b. Leaves residual moss after aspiration.
c. No rapid recollection.
d. All ore true.
Correct answer: B

51. Urinary system investigations:


a. IVP is safe in renal impairment.
b. Cystogram is the most diagnostic method of urinary bladder carcinoma.
c. Vesicoureteral reflux is diagnosed by MCU.

- 341 -
d. Cystoscope of bladder carcinoma shows pseudo tubercle, sandy patches and
ulceration.
Correct answer: C

52. As regard polycystic kidney all are correct except:


a. Hereditary condition.
b. Infantile type is a rare condition inherited as autosomal recessive.
c. May cause renal hypertension.
d. Cannot be complicated by infections (sterile condition).
Correct answer: D

53. As regard inheritance of polycystic kidney all are correct except:


a. Adult type (autosomal dominant.)
b. infantile type (autosomal recessive.)
c. Percentage of inheritance is l0%.
d. May be associated with other cystic changes.
Correct answer: C

54. Solitary cyst of the kidney may contain the following except:
a. Clear fluid.
b. Cholesterol crystals.
c. May contain altered blood.
d. Pyogenic material if infected.
Correct answer: B

55. As regard ectopia vesica all are correct except:


a. It is more common in females.
b. it may lead to concern bladder.
c. Usually present with ambiguous genitalia (undefined sex).
d. Always complicated by recurrent UTI.
Correct answer: A

56. Double ureter may present with:


a. Recurrent UTI in non diabetic patient.
b. Recurrent stone formation especially triple phosphate.
c. May be asymptomatic.
d. All of the above.
Correct answer: 10

57. Histologically benign prostatic hyperplasia is:


a. Fibroadenoma
b. Fibromyo adenoma
c. None of the above
Correct answer : C

58. The formation of middle lobe in BPH arises from which zone:
a. Transitional zone
b. Central zone
c. Peripheral zone
d. None of the above

- 342 -
Correct answer : B

59. Localized carcinoma of the prostate is BEST treated by which of the


following modalities:
a. Bilateral orchiectomy
b. Estrogens
c. Orchiectomy followed by estrogens
d. Radical prostatectomy
e. Transurethral resection of the prostate
Correct answer : D

60. About urinary bladder diverticula all are true except:


a. May be congenital or acquired.
b. May be complicated by infection, hydronephrosis or even neoplasm.
c. Most commonly traction diverticula.
d. None of the above.
Correct answer : C

61. About posterior urethral valve all ore true except:


a. Is best diagnosed by urethroscopy.
b. Treatment is most commonly by transurethral resection.
c. Causes acute retention of urine.
d. All of the above.
Correct answer : B

62. in penile hypospadias:


a. The urethra opens midway between the scrotum and the onus.
b. The prepuce is deficient superiorly.
c. The penis is curved downwards.
d. No treatment is required till the age of l0 years.
e. Circumcision should be done during the neonatal period.
Correct answer : C

63. Complications of rupture kidney include:


a. Shock.
b. Renal failure if solitary kidney.
c. A-V fistula.
d. Pseudo hydronephrosis.
e. All of the above
Correct answer : E

64. As regard incidence of rupture bladder all are correct except:


a. It is more common in males.
b. Intraperitoneal rupture is more common.
c. Extraperitoneal is always associated with fracture pelvis.
d. All of the above.
Correct answer : B

- 343 -
65. Treatment of rupture bladder includes the following except:
a. Resuscitation.
b. Watertight repair without drain.
c. Urethral catheter for 2 weeks.
d. 2ry survey and treatment of associated injuries.
Correct answer : B

66. Conservative treatment of rupture kidney is indicated in:


a. Complete tear.
b. Deep tear.
c. Intraperitoneal rupture with minimal shock.
d. Retroperitoneal rupture with minimal shock.
Correct answer : D

67. Tear drop sign is seen in:


a. Under screen in plain x-ray in rupture kidney.
b. in IVP in rupture kidney.
c. in U/S in rupture kidney.
d. None of the above.
Correct answer : B

68. Rupture urinary bladder:


a. Most commonly extraperitoneal.
b. More common in males.
c. May cause hematuria.
d. All of the above.
Correct answer : D

69. About rupture urethra:


a. Commonly is prostatic urethra.
b. Causes bleeding per rectum.
c. Fracture of pubic and ischial rami causes rupture membranous urethra.
d. The 1st step of treatment is usually urethral catheterization.
Correct answer : C

70. An absolute indication of surgical treatment of renal injury is:


a. 20% devitalized renal parenchyma.
b. Major urinary extravasation.
c. Vascular injury.
d. Expanding perineal hematoma.
Correct answer : D

71. The triad of signs of rupture urethra includes the following except:
a. Perineal hematoma
b. Bleeding per urethra
c. Retention of urine
d. Bleeding per rectum
Correct answer : D

- 344 -
72. Male 25 years old arrived at the hospital in severe shock after a road traffic
accident. Abdominal examination revealed tenderness and rigidity in the loin
and fullness in the suprapubic region. He failed to pass any urine. He should be
suspected lo have:
a. Rupture of the spleen
b. Rupture of the left kidney
c. Retroperitoneal hematoma
d. Extraperitoneal rupture of the bladder
e. lntrapelvic rupture of the urethra
Correct answer : B

73. Renal tuberculosis all are correct except:


a. The kidney is affected usually by hematogenous spread.
b. Earliest symptom is frequency.
c. Sterile pyuria increase suspicion.
d. Renal calcification excludes T.B.
e. May result in auto nephrectomy.
Correct answer : D

74. The following may be a presentation of perinephric abscess:


a. Fever.
b. Psoas spasm.
c. Hiccough.
d. Throbbing pain in loin increased with breathing.
e. All of the above
Correct answer : E

75. As regards types of pyonephrosis all are correct except:


a. In primary type the kidney may be hugely enlarged.
b. 2ry type is due to superimposed infection and chronic obstruction.
c. Closed pyonephrosis is an emergency condition.
d. Urine analysis may be -ve in closed type.
Correct answer : A

76. Complications of urinary bilharziasis include the following except:


a. lnfertility.
b. Corpulmonale.
c. Hydronephrosis.
d. Squamous cell carcinoma.
e. All of the above
Correct answer : E

77. All of the following findings give sure diagnosis of renal TB except:
a. Acid fast, alcohol fast bacilli on ZN stain.
b. Sterile pyuria.
c. +ve culture on Lowenstein medium.
d. +ve culture on BancTec medium.
Correct answer : B

- 345 -
78. About urinary bilharziasis:
a. Most common organism is Schistosoma mansoni.
b. May lead to transitional cell carcinoma.
c. Characteristic sandy patches on macroscopic exam.
d. Nonspecific histopathological pattern may be seen.
Correct answer : C

79. All of the following lesions in bilharziasis are precancerous except:


a. Cystitis cystica.
b. Leukoplakia.
c. Cystitis glandularis.
d. Sandy patches.
Correct answer : D

80. Disease of urinary bladder causes:


a. Initial hematuria.
b. Terminal hematuria.
c. Total hematuria.
d. Con not cause hematuria.
Correct answer : B

81. About urinary tract infection all are true except:


a. It is the most common cause of phosphate stone.
b. The most common organism is proteus merabiles.
c. Causes of stasis are very common predisposing factors.
d. More in females.
Correct answer : B

82. Regarding urinary bladder bilharziasis all ore true except:


a. Diagnosed by presence of sandy patches, pseudo tubercles or ulcerations in
cystoscope.
b. Leads to urinary bladder carcinoma.
c. Causes total hematuria.
d. All of the above.
Correct answer : C

83. Radiological signs of perinephric abscess include the following except:


a. Enlargement of the renal shadow on the plain film
b. Homolateral scoliosis
c. Obliteration of the psoas shadow
d. Elevation and fixation of the diaphragm
e. Positive mother's sign
Correct answer : B

84. The following statements about pyonephrosis are correct except that it
a. Is a chronic retention of infected urine in the kidney
b. Usually results from ca-incident infection and obstruction as in renal calculi and
pyelonephritis
c. ls characterized by dilatation of the pelvicalyceal system with extensive excavation
of the parenchyma

- 346 -
d. Presents with pain, swelling, fever and pyuria
e. ls best treated by primary nephrectomy
Correct answer : E

85. Renal tuberculosis should be suspected in presence of:


a. Sterile pyuria
b. Renal calcification
c. indistinct outline of papillae in IVP
d. Urinary frequency and hematuria
e. Each of the above
Correct answer : E

86. Presentations of urinary stones may be the following except:


a. Renal pelvic stone pain restricted to loin.
b. Upper ureter stone has no radiating pain.
c. Bladder neck stone pain radiating to neck of penis.
d. Urinary stone may be asymptomatic especially triphosphate stone.
Correct answer : B

87. The following are common sites of impaction of ureteric stone:


a. Pelviureteric junction.
b. Crossing of iliac arteries.
c. Crossing of brood ligaments in females or vas in males.
d. Ureteric orifice.
e. All of the above.
Correct answer : E

88. Conservative treatment of urinary stone include:


a. Ample fluid intake.
b. Antibiotics.
c. Analgesics.
d. Acidification of urine.
e. All of the above.
Correct answer : E

89. The following type of stone is radiolucent in (KUB) films:


a. Calcium stones.
b. Urate stones.
c. Cysteine stones.
d. Oxalate stones.
Correct answer : B

90. All of the following are predisposing factors to Ca oxalate stone except:
a. Hyperparathyroidism.
b. increase consumption of mongo, tomato and milk.
c. Bone secondaries.
d. Giant tumor lysis.
e. Prolonged recumbency.
Correct answer : D

- 347 -
91. The most symptomatic urinary stone is:
a. Oxalate.
b. Urate.
c. Phosphate.
d. Cysteine.
Correct answer : A

92. The least symptomatic stone is:


a. Oxalate.
b. Urate.
c. Phosphate.
d. Cysteine.
Correct answer : C

93. DD of ureteric stone (opacity) in x-ray film include:


a. Gall bladder stone on rt side.
b. Phlebolith.
c. Calcified LN.
d. All of the above.
Correct answer : D

94. The stone causing strangury must irritate:


a. Ureter.
b. Pelviureteric junction.
c. Trigone.
d. Ureteric orifice.
Correct answer : C

95. Regarding urinary stones all are true except:


a. Urate stones are translucent.
b. Oxalate stone is spiky.
c. Triple phosphate stones consist of magnesium, calcium and sodium.
d. None of the above.
Correct answer : C

96. Predisposing factors of urinary stones include:


a. Urinary bladder diverticula.
b. Recurrent urinary tract infection.
c. Metabolic causes.
d. All of the above.
Correct answer : D

97. Renal calculus formation has been shown to be related to all of the following
except:
a. Vitamin D metabolism.
b. Urea splitting bacteria.
c. Immobilization.
d. Stenosis of ureteropelvic junction.
e. Malabsorption.
Correct answer : E

- 348 -
98. A ureteric stone needs surgical removal when:
a. Enlarging
b. Obstructing
c. Too large to pass
d. Causing urinary infection
e. All of the above
Correct answer : E

99. Which of the following statements regarding ureteric colic is false:


a. The pain is restricted to the loin when the stone is coming out of the kidney
b. Pain radiates to the groin when the stone is in upper ureter
c. In lower ureteric stones, pain radiates to the perineum
d. All of the above
Correct answer : D

100. The main aim in emergency treatment of anuria is to do the following


except:
a. Relieve obstruction.
b. Prevent infection.
c. Relieve pain.
d. Detect type of stone and remove it.
Correct answer : D

101. As regards treatment of renal malignancies all are correct except:


a. Urinary bladder is removed during radical nephrectomy.
b. When unresectable radiotherapy and chemotherapy may be tried in Wilm's tumor.
c. Chemotherapy and radiotherapy are not effective against RCC as it is adeno
carcinoma.
d. RTH is the 2nd line after surgery in RCC.
e. Survival rate after surgery in Wilm's tumor is 5O%.
f. Radical nephrectomy for renal tumors is best done transabdominal.
Correct answer : A

102. All of the following are presentations of BPH:


a. Asymptomatic.
b. Chronic retention.
c. Hematuria.
d. Chronic renal failure.
e. All of the above.
Correct answer : E

103. As regards Prostatectomy all are correct except:


a. Best way for benign condition is TURP.
b. Is indicated in all cases with BPH.
c. Radical prostatectomy is indicated in locally active adenocarcinoma.
d. Local latent adenocarcinoma follow up.
e. Metastasizing adenocarcinoma hormonal therapy.
Correct answer : B

- 349 -
104. Risk factors of transitional cell carcinoma include all of the following
except:
a. Smoking.
b. Cyclophosphamide.
c. Pelvic irradiation.
d. Exposure to benzidine.
e. Exposure to schistosomiasis.
Correct answer : E

105. All of the following primary sites metastasize classically with osteolytic
lesion except:
a. Kidney.
b. Prostate.
c. Bladder.
d. Breast.
e. Lung.
Correct answer : B

106. As regard pathology of BPH:


a. It arises from submucous glands.
b. It leads to shortening and compression of urethra.
c. It is fibromyo adenoma.
d. All of the above.
Correct answer : D

107. As regard DRE, BPH differs from cancer prostate in that:


a. Cancer prostate feel hard and irregular.
b. BPH feels smooth, firm and elastic.
c. In BPH rectal mucosa is freely mobile.
d. All of the above.
Correct answer : D

108. As regard complications of prostate cancer:


a. Most common site for distant metastasis is vertebral column.
b. Urine retention is a late complication.
c. Most common sexual complication after prostatectomy is retrograde ejaculation.
d. All of the above.
Correct answer : D

109. Diagnosis of urinary bladder carcinoma:


a. The most effective diagnostic tool is cystoscopy.
b. Best assessment of penetration is by contrast enhanced CT.
c. irregular filling defect in cystogram.
d. All of the above.
Correct answer : D

110. The commonest presentation of nephroblastoma:


a. Abdominal mass.
b. Hematuria.
c. Renal pain.

- 350 -
d. Pulmonary metastasis.
e. Polycythemia.
Correct answer : A

111. Carcinoma of bladder all are true except:


a. Is usually very painful even in early stages.
b. Is usually associated with hematuria.
c. Most often occur in the vault of the bladder.
d. Is best diagnosed by cystoscopy.
Correct answer : C

112. Radiological findings of benign hypertrophy of the prostate include all of the
following except:
a. Filling defect at base of the bladder.
b. Diverticula of the bladder.
c. Uretheral dilatation.
d. Ureteral dilatation.
Correct answer : D

113. The earliest symptom of benign prostatic hyperplasia is:


a. Dysuria.
b. Nocturnal frequency.
c. Hesitancy.
d. Post micturition drippling.
e. Urine retention.
Correct answer : B

114. Bladder irrigation during TUR prostatectomy may produce:


a. Hemolysis.
b. Hyponatremia.
c. Hypercalcemia.
d. Hyperkalemia.
e. Alkalosis.
Correct answer : B

115. The following statements concerning hypernephroma are true except that it:
a. Is a carcinoma arising from renal tubular cells
b. Commonly arises in one pole of the kidney
c. Usually manifests itself by recurrent renal hematuria
d. Never causes pain
e. May form a palpable moss
Correct answer : D

- 351 -
Congenital anomalies

1. Congenital bladder diverticulum differs from acquired one in the following


except:
a. Congenital is a true diverticulum.
b. Acquired is always solitary.
c. Congenital may be o port of anomalous urachus.
d. Acquired is always symptomatic.
Correct answer: B

2 . Not true about congenital PUJ obstruction is:


A. Can be associated with renal agenesis
B. Can be diagnosed antenatally
C. b/1 in 10-15% of cases
D. Aberrant vessels is the most common cause
Correct answer: D

3. Ectopio vesico is common to be associated with all of the following anomalies


except:
a. Spina bifida.
b. Rudimentary penis.
c. Congenital heart disease.
d. Cleft palate.
Correct answer: C

4. In ectopia vesica:
a. absent anterior abdominal wall.
b. Absent anterior bladder wall.
c. Associated late rupture of cloacal membrane.
d. All of the above.
Correct answer: D

Hematuria

1 . A patient presents with hematuria for many days. On investigations he is


found to have renal calculi, calcifications in the wall of urinary bladder and
small contracted bladder; most probable cause is:
A. Schistosomiasis
B. Amyloidosis
C. Tuberculosis
D. Ca urinary bladder
Correct answer: A

- 352 -
2 . Terminal hematuria is most commonly caused by
A. Renal calculus
B. Ureteric calculus
C. Bladder calculus
D. Urethral calculus
Correct answer: C

3 . Painless gross hematuria occurs in


A. Renal cell carcinoma
B. Polycystic kidney
C. Stricture of urethra
D. Wilm's tumor
Correct answer: A

4 . A 65 years chronic smoker presents gross hematuria of 1 day duration.


Investigation of choice to know the cause is -
A. Urine routine
B. Urine microscopy for malignant cytology cells
C. USG
D. X-ray KUB
Correct answer: B

5 . A 60 year old smoker came with the history of painless gross hematuria for
one day. The investigation of choice would be -
A. USB KUB
B. Urine for malignant cytology
C. Urine routine and microscopy
D. Plain X-ray KUB
Correct answer: B

6 . A 50 years smoker presents with history of five episodes of macroscopic


hematuria each lasting for about 4-5 days in the past five years. The following
investigations should be done to evaluate the suspected diagnosis is-
A. Ultrasound KUB
B. DTPA scan
C. Urine microscopy and cytology
D. X-Ray KUB
Correct answer: C

7 . A 50 years female presented with hematuria was found to have Stage II


Transitional Cell Carcinoma of bladder. True: statements about management of
her condition is-
A. 70% chance of requiring cystectomy in 5 years after TUR
B. Cystoscopy fulguration is the standard treatment
C. There is no role of chemotherapy
D. History of smoking is not a risk factor
Correct answer: A

- 353 -
8 . A 67-year-old man presented with initially after a single episode of gross
painless hematuria. An excretory urogram demonstrated a 1.5 mm round filling
defect in the right lower renal infundibulum. The best study to obtain next is -
A. Urinary cytology
B. Cystoscopy
C. Ultrasonography
D. Retrograde pyelography
Correct answer: C

9 . A patient presents with hematuria of several days and dysmorphic RBC casts
in urine. The site of origin is:
A. Kidney
B. Ureter
C. Bladder
D. Urethra
Correct answer: A

10 . A 30 year old male presents with pain on the right flank and hematuria. A
CECT abdomen reveals a large 8X8 cm sized solid mass in the right kidney and
3x3 cm solid mass occupying the upper pole of left kidney. The most appropriate
surgical treatment for this patient is:
A. Bilateral radical nephrectomy
B. Right radical nephrectomy and biopsy of the mass from opposite kidney
C. Right radical nephrectomy and left partial nephrectomy
D. Right radical nephrectomy only
Correct answer: C

11 . A 50 years male presented with painless terminal hematuria. Cystoscopy


examination revealed a solitary papillary tumor. Histopathological examination of
completely resected tumor is suggestive of grade III transitional cell carcinoma
with no muscle invasion. Further best management done by:
A. Intravesical chemotherapy
B. Just follow up
C. Cystectomy
D. Intravesical BCG
Correct answer: D

12 . Painless hematuria occurs in:


A. Nephritic syndrome
B. Bladder stone
C. TB Bladder
D. Ureteric stones
Correct answer: A

13 . After a single episode of painless gross hematuria in a boy. Doctor performed


an excretory urogram showing a filling defect towards the lower renal
infundibulum 1.5 cm. in size. What will be the next investigation to be done:
A. Cystoscopy
B. Urine cytology
C. USG

- 354 -
D. Retrograde pyelography
Correct answer: C

14 . A 23 year old male who is otherwise normal complains of mild pain in his right
iliac fossa in a waveform pattern which increases during the night and he becomes
exhausted and is admitted in the hospital. On examination there is mild hematuria.
Urine examination levels plenty of RBC's 50 WBCs/hpf. Urine pH is 5.5. Most
likely diagnosis is -
A. Ca-Urinary bladder
B. Glomerulonephritis
C. Cystitis
D. Ureteral calculus
Correct answer: D

15. A 60-year-old male presenting with painless hematuria gives a history of two
similar attacks during the last six months. He had no pain, dysuria or frequency
and examination revealed no abnormality. He should be suspected to be
suffering from:
a. Polycystic kidney
b. Renal calculus
c. Carcinoma of the bladder
d. Hypernephroma
e. Senile enlargement of the prostate
Correct answer : D

16 . Commonest finding in a patient with raised IgA levels is:


A. Proteinuria
B. GI bleed
C. Hematuria
D. Hypertension
Correct answer: C

Hydronephrosis

1 . Fetal hydronephrosis is diagnosed in a mother at 34 weeks gestation. The


amniotic fluid is normal. Which of the following is the most appropriate
management?
A. Fetal intervention to decompress hydronephrotic kidney
B. Premature termination of pregnancy followed by pyeloplasty
C. Delivery at term followed by radiological evaluation
D. Delivery at term followed by early pyeloplasty within
Correct answer: C

2 . A 45 years lady is admitted with abdominal pain and anuria. Radiological


studies revealed bilateral impacted ureteric stones with hydronephrosis. Urine
analysis showed RBCs with pus cells in urine. Serum creatinine level was 14

- 355 -
mg/dl and urea level were 180 mmol/1. Which should be the immediate
treatment
A. 'J' stent drainage
B. Hemodialysis
C. Ureteroscopic removal of stones
D. Lithotripsy
Correct answer: A

3 . A 63 years male with poor stream of urine, post void residual urine is 400 ml,
bilateral hydronephrosis and prostate weighing 75 g. His Ur- 118 and Cr- 3.4.
The next steps of management is-
A. CT to rule out ca
B. MRI pelvis
C. Catheterize with Foley catheter
D. b/l PC Nephrostomies
Correct answer: C

4. An infant is noted to have a left flank mass shortly after birth and an
ultrasound examination demonstrates left hydronephrosis. The most common
cause of this finding is which of the following?
a. Neonatal Wilm’s tumor
b. Congenital ureteropelvic junction obstruction
c. Multicyclic dysplastic kidney
d. Vesicoureteral reflux
Correct answer: b

5 . A patient presents with pain and tenderness in left iliac fossa. USG shows a 3
cm stone in the renal pelvis without any hydronephrosis. Most appropriate
management is -
A. PCNL
B. ESWL
C. Diuretics
D. Medical dissolution therapy with potassium citrate
Correct answer: A

Stones

1 . For the ureteric stones, conservative management is indicated when:


A. a stone less than 6 mm in size
B. Hydronephrosis and infection is present
C. Associated congenital anomalies of kidney
D. Stone does not descend for 2 weeks of follow up
Correct answer: A

2 . Not true about 'Struvite Stones' is:


A. better k /a staghorn calculus
B. these are triple phosphate stones

- 356 -
C. common in infected urine
D. usually seen in acidic urine
Correct answer: D

3 . Idiopathic (endemic ) bladder stones - false statement -


A. Always recurs after surgery
B. Made up of ammonium acid urate sometimes mixed with ca oxalate
C. predominantly associated with children having cereal-based diet
D. Peak incidence in children 3 years
Correct answer: A

4 . Commonest presentation of bilateral ureteric stones -


A. CRF
B. UTI
C. Pain
D. Hematuria
Correct answer: A

5 . Secondary vesical calculus refers to stones formed due to -


A. Hypercalciuria
B. Injury
C. Infection
D. Migrating from kidney pelvis
Correct answer: C

6 . Following stones is hard to break by ESWL


A. Uric acid
B. Struvite
C. Calcium Oxalate Monohydrate
D. Calcium Oxalate Dihydrate
Correct answer: C

7 .Not true about bladder stones is:


A. Rare in Indian children
B. Primary stones are rare
C. Small stones can be removed per urethra
D. Max stones are radiopaque
Correct answer: A

8. Stag horn calculi associated with Proteus infection are ?


A. Uric acid stones
B. Triple phosphate stones
C. Calcium oxalate stones
D. Cysteine stones
Correct answer: B

9 . Lithotripsy is the equipment used to crush the stones -


A. High frequency (CW) US used
B. High frequency pulsed wave
C. Low frequency continuous wave

- 357 -
D. Medium range (2-5 MHz) pulsed wave are used
Correct answer: A

urinary tract trauma

Urinary tract infections

1 . Fruit Juice which helps in preventing UTI -


A. Orange
B. Grape
C. Cranberry
D. Raspberry
Correct answer: C

2 . What will be the next investigation to be done in a case of a 2 year old female
child with 1st episode of UTI -
A. Abdominal Ultrasound
B. DMSA scan
C. 6 monthly urine culture
D. Nothing actively needed
Correct answer: A

3 . A symptom of medullary sponge kidney disease is


A. Nocturia
B. Anemia
C. Azotemia
D. UTI
Correct answer: D

4 . The treatment of choice for grade IV vesicoureteric reflux with recurrent


UTI
A. Cotrimoxazole
B. Bilateral reimplantation of ureter
C. Injection of Collagen in the ureter
D. Endoscopic resection of ureter
Correct answer: A

5 . A 9 year boy presented with abdominal pain and recurrent UTI, IVP reveals
duplication of left ureter. The most likely site of ectopic opening would be:
A. Seminal vesicle

- 358 -
B. Vas-deference
C. Prostatic urethra
D. Ejaculatory duct
Correct answer: C

Renal tumors

Bladder cancer

BPH, prostate cancer

1- FDG-PET scanning appears to be of greatest value in evaluating patients with prostate


cancer in assessing what situation?
1. Localized disease
2. Regional node metastases
3. Bone metastases
4. Response to therapy
Correct answer: 2

2 . An 85-year-old man underwent Transurethral resection of prostate. A histological


examination of his specimen showed foci of adenocarcinoma management will be:
A. Endocrine therapy
B. Radical surgery
C. Hormone therapy
D. No further The treatment.
Correct answer: D

3 . Commonest cause of periumbilical pain after 30 min.of TURP done under spinal
anesthesia with Bupivacaine
A. Meteorism
B. Perforation of bladder
C. Recovery from bupivacaine anesthesia
D. Mesentery artery ischemia
Correct answer: B

4 . A 55years male with positive family history of prostate cancer has come to you for a
screening test.The most sensitive screening test is -
A. PSA
B. DRE (digital rectal examination)
C. Endorectal Coil MRI with T1W and T2 W images

- 359 -
D. DRE +PSA
Correct answer: D

5 . All of the following can be seen after transurethral resection of prostate except:
A. Congestive cardiac failure
B. Transient blindness
C. Convulsion
D. Hypernatremia
Correct answer: D

6 . 60 years patient with fever, dysuria, PSA 8.5 ng/ml on DRE prostate is n tender, diffuse
back pain - next line of management is -
A. Emergency TURP
B. Repeat PSA
C. Admit and antibiotics
D. TRUS guided biopsy
Correct answer: C

7 . Most common site of carcinoma prostate is ?


A. Peripheral
B. Centre
C. Verumontum
D. None
Correct answer: A

8 . Prostate cancer is best diagnosed by ?


A. Prostate specific antigen
B. Trans urethral ultrasound
C. FNAC of prostate gland
D. DRE
Correct answer: B

9 .Trans rectal ultrasonogram in evaluation of carcinoma prostate most useful for:


A. Nodal sampling
B. Measuring in extent of invasion
C. Taking guided biopsy
D. Identifying seminal vesicle invasion
Correct answer: C

10 . For Carcinoma of the prostate the commonest site is:


A. Anterior zone
B. Peripheral zone
C. Central zone
D. Transitional zone
Correct answer: B

- 360 -
Anuria / Retention

Testicular tumors

1- Which of the following is not done in of testicular tumour:


A. Tran scrotal biopsy
B. Tumour marker
C. Inguinal exploration
D. CT abdomen
Correct answer: A

2 . After sexual intercourse, a person develops pain in the left testes that does not
get relieved on elevation of scrotum. Possible diagnosis is:
A. Torsion testis
B. Epididymo-orchitis
C. Tumor testes
D. Fourniers gangrene
Correct answer: A

3 . A 25 year old married male presents with infertility. He had undergone


retroperitoneal lymph node dissection at age of 15 years for embryonal carcinoma
of right testis. Semen analysis shows-quantity-0.5 ml, no. sperm, no fructose.
Biopsy of testis shows normal spermatogenesis. Best treatment here would be:
A. Artificial insemination of donor
B. Penile-prosthesis
C. Microtesticular aspiration and intracyto plasmic injection
D. None of the above
Correct answer: C

4 . Testis is involved but epididymis is spared in ?


A. Syphilis
B. Gonorrhea
C. Chancroid
D. Chlamydia
Correct answer: A

5 . Stage I seminoma testis, the treatment of choice is:


A. high inguinal orchidectomy
B. high inguinal orchidectomy and radiotherapy
C. radiotherapy and chemotherapy
D. Trans-scrotal orchidectomy

- 361 -
Correct answer: B

6 . All of the following clinicopathologic features are seen more often in seminomas
as compared to non-seminomatous germ cell tumors of the testis except -
A. Tumors remain localized to testis for long time
B. They are radiosensitive
C. They metastasize predominantly by lymphatics
D. They are often associated with raised levels of serum AFP and HCG
Correct answer: D

7 . Bony metastasis is common with all of the following except:


A. CA lung
B. CA breast
C. CA prostate
D. CA testis
Correct answer: D

8 . A 28-year-old man presents with a single descended testis and a 6 cm solid mass
seen by CT. The most likely diagnosis is -
A. Metastatic prostate cancer
B. Teratoma
C. Hyperplastic cryptorchid testis
D. Seminoma
Correct answer: B

9 . A 40-year-old men is suffering from fever and pain in scrotum which is not
relieved by elevation of test is 3 days before had sexual contact with a young
female:
A. Testicular tumour
B. Acute epididymitis
C. Torsion of testis
D. Acute orchitis
Correct answer: C

10 . Which of the following age is the best time for surgery of undescended testis?
A. 6 months of age
B. Just after birth
C. 24 months of age
D. 12 months of age
Correct answer: D

11 . All of the following are predisposing factors for torsion of testis except:
A. Long pedicle
B. Long mesorchium
C. Inversion of testis
D. Varicocele
Correct answer: D

12 . True about cryptorchidism are A/E-


A. Contralateral testis is also at risk

- 362 -
B. Orchidopexy reduces the risk of malignancy
C. Cryptorchidism is a risk factor for testicular tumor
D. Seminoma is the most common tumor
Correct answer: B

13 . At which of the following age, surgery for undescended testis is recommended?


A. 24 months
B. 36 months
C. 6 months
D. 12 months
Correct answer: C

14 . Following statements are true about germ cell tumors of testis except
A. They constitute 90-95% of all primary testicular tumors
B. Seminoma is the most common tumor developing in the pts with cryptorchid testis
C. Alpha-feto protein is markedly raised in all germ cell tumors
D. High inguinal orchidectomy is the initial surgical procedure
Correct answer: C

15 . At which age, Unilateral undescended testis is ideally operated ?


A. 6 months of age
B. 2 months of age
C. 24 months of age
D. 12 months of age
Correct answer: D

16 . A patient presented with a hard swelling in his right testis.All are true
statements except:
A. Trans-scrotal biopsy is needed
B. Inguinal exploration is done
C. High inguinal exploration should be done
D. Scrotal U/S is done
Correct answer: A

17 . A 12 year old boy experiences sudden severe pain in the right testis. On
palpation there is exquisite tenderness. The most likely diagnosis -
A. Spontaneous hemorrhage
B. Torsion
C. Strangulated hernia
D. Epidydimitis
Correct answer: B

18 . What is not seen in Undescended Testis -


A. Hydrocele
B. Hernia
C. Teratoma
D. Seminoma
Correct answer: A

- 363 -
19 . A 16 year old boy presents with acute onset pain in the left testis.The following
statements about his management is true except -
A. The patient should be prescribed antibiotics and asked to come after a week
B. Color flow Doppler will be very useful in diagnosis
C. Scrotal exploration should be done without delay if doppler is not available
D. If left testis is not viable on exploration, patient should undergo left Orchidectomy
and right orchidopexy
Correct answer: A

20 . Orchiectomy is not indicated in:


A. Seminoma testis
B. Prostatic carcinoma
C. Tubercular epididymitis
D. Male breast cancer
Correct answer: C

21. As regard the embryology of testis all are correct except:


a. Develops from the genital ridge.
b. Is embryologically on intraperitoneal structure.
c. Derives its blood supply directly from aorta.
d. Torsion only occurs on top of anomalous testis.
Correct answer: B

22-The tunica vaginalis corresponds to:


a. Internal oblique.
b. Transversalis fascia.
c. Peritoneum.
d. None of the above.
Correct answer: C

23. Empty scrotum may be due lo:


a. Testicular agenesis.
b. Fetal testicular torsion.
c. Arrested testis.
d. Retractile testis.
e. All of the above
Correct answer: E

24-lnversion of testis may be:


a. Anterior.
b. Anterolateral.
c. Lateral.
d. A and C
Correct answer: D

25-All of the following are causes of unilateral testicular arrest except:


a. Testicular dysgenesis.
b. Low maternal HCG.
c. Malformed inguinal canal.
d. Short testicular artery.

- 364 -
Correct answer: B

26- Testicular arrest is more:


a. On left side.
b. On right side.
c. Equal on both sides.
Correct answer: B

27-Risk factors of testicular arrest include:


a. +ve family history.
b. Down syndrome"
c. Teratogenic drugs.
d. Chronic maternal illness.
e. All of the above.
Correct answer: E

28. Testicular arrest is associated with urinary anomalies in:


a. 5% of cases.
b. 15%of cases.
c. 40% of cases.
d. 50% of cases.
Correct answer: B

29- Most common site of arrest of testis is:


a. Abdominal cavity.
b. inguinal canal.
c. Pelvis.
d. Superficial inguinal pouch.
Correct answer: B

30- The following ore complications of arrested testis:


a. Torsion.
b. Trauma.
c. Tumor.
d. All of the above.
Correct answer: D

31- The most diagnostic investigation in arrested testis is:


a. US.
b. CT.
c. Laparoscopy.
d. Doppler.
Correct answer: C

32- Testis not found by laparoscopy may be due to:


a. Testicular agenesis.
b. Fetal testicular torsion.
c. Mal descended testis.
d. All of the above.
Correct answer: D

- 365 -
33- Best time for orchiopexy is:
a. At l st day of life.
b. 6-l5 months.
c. At 5 years.
d. At puberty.
Correct answer: B

34- Best treatment of retractile testis is:


a. Reassurance.
b. Medical treatment.
c. Orchiopexy.
d. Orchidectomy.
Correct answer: A

35- Concerning undescended testis, the following statements ore true except that
it:
a . Affects about 1% of all males
b. is commoner on the right than on the left side
c. is bilateral in about 20% of cases
d. May be intra-abdominal or extra-abdominal
e. is rarely associated with inguinal hernia
Correct answer: E

36. The following statements about ectopic testis ore true except that it:
a. is probably due to rupture of the scrotal tail of the gubernaculum
b. May lie in the groin or pubic region.
c. Does not develop normally.
d. Has normal long spermatic cord.
e. Can be easily replaced in the scrotum.
Correct answer: C

37. Best treatment for testicular torsion is:


a. Resuscitation.
b. Early operation to untwist the testis.
c. Orchiopexy for other testis.
d. All of the above in sequence.
Correct answer: D

38. Regarding choriocarcinoma all are correct except:


a. It is a subtype of teratoma.
b. Rarely gives lung metastasis.
c. Secretes HCG in large amounts.
d. All are true
Correct answer: B

39. Regarding incidence of testicular neoplasm:


a. Most testicular neoplasms are malignant.
b. Mostly occurs above 60 years of age.
c. Most common neoplasm is teratoma.
d. All of the above.

- 366 -
Correct answer: A

40. The first LN station draining testis is:


a-Inguinal.
b-Internal iliac.
c- Paraaortic.
d. Supraclavicular.
Correct answer: C

41- Most common presentation of testicular neoplasm is


a. Severe dull aching pain.
b. Accidentally discovered painless mass.
c. Secondary hydrocele.
d. Bone pain and/or hemoptysis.
Correct answer: B

42-Radiosensitive testicular neoplasm is:


a. Seminoma
b. Teratoma.
c. None of the above.
d. All of the above
Correct answer: A

43-Precocious puberty is character of:


a. Sertoli cell tumor.
b. Leydig cell tumor.
c. Seminoma.
d. Teratoma.
Correct answer: B

44. The most malignant testicular tumor is:


a. Seminoma
b. Embryonal carcinoma
c. Choriocarcinoma
d. Teroto carcinoma
e. Teratoma
Correct answer: C

45-The management of testicular tumors includes the following except:


a. Testicular biopsy
b. Radical orchidectomy
c. Simple orchidectomy
d. Radiotherapy
e. Chemotherapy
Correct answer: A

46-What is not true of interstitial cell tumors of testis:


a. Leydig cell tumor masculinizes.
b. Sertoli cell tumor feminizes.
c. Prepubertal tumors are from Sertoli cell.

- 367 -
d. Sertoli cell tumors are benign and orchidectomy is curative.
Correct answer: C

47-Which of the following has earlier pulmonary metastases:


a. Seminoma
b. Teratoma.
c. Chorio -carcinoma.
d. Embryonol cell carcinoma.
Correct answer: C

48-Regarding testicular tumors, the untrue is:


a. seminomas and teratomas are more common thon non-germ cell tumors.
b. Seminomas send pulmonary metastasis.
c. They have on increased incidence in undescended testis.
d. a-fetoprotein and B-HCG are tumor markers.
e. The treatment of choice is radical orchidectomy.
Correct answer: B

49. In incompletely descended testis, which of the following statements is false?


a. The incidence is 4 per cent.
b. A testis absent from the scrotum after 3 months is unlikely to descend.
c. An incompletely descended testis tends to atrophy as puberty approaches.
d. Early orchidopexy can preserve function.
e. Orchidopexy reduces the chances of developing a testicular tumour.
Correct answer: E

50. The hazards of incomplete descent of testis include all the following except:
a. Sterility in bilateral cases
b. Trauma
c. Torsion
d. Increased liability to malignant change in later life
e. Epididymo-orchitis.
Correct answer: E

51. Which of the following statements are true with regard to testicular torsion?
a. The typical symptom is sudden agonizing pain in the scrotum.
b. It is most common above 25 years of age.
c. Inversion of the testis and a transverse lie are not a cause.
d. Doppler ultrasound (US) scan should be done to confirm the diagnosis.
e. The emergency operation of fixation of the testis to the tunica albuginea with non-
absorbable sutures should be done on both sides.
Correct answer: E

52. In epididymo-orchitis, which of the following is false?


a. The causes are chlamydia and gonococci as sexually transmitted infections.
b. The initial symptoms are those of urinary tract infection (UTI).
c. In mumps 18 per cent of men may develop the condition.
d. It is a common postoperative complication after prostatectomy.
e. Treatment is doxycycline for chlamydia or a broad-spectrum antibiotic.
Correct answer: D

- 368 -
53. In testicular tumours, which of the following statements is false?
a. A scrotal lump that is inseparable from the testis is likely to be a tumour.
b. Lymphatic spread is usually to the paraaortic lymph nodes.
c. Teratomas occur in the third decade and seminomas in the fourth decade.
d. Seminomas usually spread via the lymphatics.
Correct answer: D

54. Which of the following statements is false with regard to the management of
testicular tumours?
a. Tumour markers are measured, and a chest X-ray carried out.
b. Initial surgical treatment is orchidectomy through the groin.
c. Staging is done by CT scan and MRI.
d. Seminomas are radiosensitive.
e. Secondaries from teratoma are always treated by surgery.
Correct answer: D

55. A 9-year-old with a paratesticular rhabdomyosarcoma. The primary tumor


was completely resected at an outside institution. CT scan shows enlarged
ipsilateral retroperitoneal nodes that are obviously involved with tumor. No
metastatic disease was detected on bone scan, bone marrow, or chest CT. What is
the Stage/Group of this patient?
A. Stage 1, Group I
B. Stage 1, Group II
C. Stage 1, Group III
D. Stage 3, Group II
E. Stage 4, Group IV
Correct answer is C.

56.A 16-year old adolescent presents with a left sided scrotal mass. Ultrasound
shows a paratesticular mass. Which of the following is the most appropriate
surgical approach?
A. Tran scrotal orchidectomy and resection of the entire spermatic cord
B. Inguinal orchidectomy and resection of the entire spermatic cord
C. Open biopsy of the mass through an inguinal incision followed by inguinal
orchidectomy and resection of the entire spermatic cord after biopsy
D. Open biopsy of the mass through a Tran scrotal approach followed by Tran scrotal
orchiectomy and resection of the entire spermatic cord
Correct answer is B.

57 . All are sex cord tumor except:


A. Luteoma
B. Gynandro blastoma
C. Theca fibroma
D. Hilus cell tumor
Correct answer: A

- 369 -
Erectile dysfunction and Infertility

1 . Following are indications for penile angiography, except-


A. Erectile dysfunction
B. Arterio venous malformation
C. Painful priapism
D. Peyronies disease
Correct answer: C

Renal transplantation

1 . The most common indication of liver transplantation in adults is -


A. Portal vein thrombosis
B. Cirrhosis
C. Biliary atresia
D. None of the above
Correct answer: B

2 . A 45 years male with a single Kidney presents with a solitary exophytic mass of
4cm localized at its lower pole. The best recommended management option is-
A. Radical Nephrectomy with dialysis
B. Radical Nephrectomy with immediate Renal Transplantation
C. Partial Nephrectomy
D. Radical Nephrectomy
Correct answer: C

3 . A patient had undergone a renal transplantation 2 months back and now


presented with difficulty breathing. X-ray showed bilateral diffuse intestinal
pneumonia. The probable etiologic agent would be:
A. CMV
B. Histoplasma
C. Candida
D. Pneumocystic carinii
Correct answer: A

4 . Who did the First autologous renal transplantation?


A. Higgins
B. Studor
C. Hardy
D. Kavosis
Correct answer: C

- 370 -
26- Orthopedic surgery:

General principles of fractures and dislocations

1. Which of the following fractures are more common to be impacted


a. Neck femur fracture.
b. Humeral supracondylar fracture.
c. Colle's fracture.
d. Clavicle fracture.
Correct answer: C

2. In stage of primary callus in bone healing


a. Acidic.
b. Alkaline.
c. Neutral.
d. Any of the above.
Correct answer: B

3. Which of the following are factors affecting bone healing:


a. Type of bone.
b. lntra-articular fracture.
c. Drugs.
d. Surgical intervention.
e. All of the above
Correct answer: E

4. As regards nonunion all are correct except:


a. Diagnosed if there is no healing after 6 months.
b. Characterized by thinning of bone ends in avascular nonunion.
c. Should be treated conservatively in avascular nonunion.
d. May results from excess union triols.
e. Is rare in fractures of highly vascularized bones, e.g. clavicle.
Correct answer: C

5. Malunion may result in the following except:


a. Angulations.
b. Rotation.
c. Elongation.
d. Shortening.
Correct answer: C

6. As regards open fracture all are correct except:


a. Gustilo et al classification is useful in determining method of fixation.
b. Wound debridement means removal of foreign bodies and devitalized tissues.

- 371 -
c. Closure of skin wounds is always indicated.
d. All bluish non contracting non bleeding muscles should be excised.
Correct answer: C

7. O.R.I.F is indicated in the following except:


a. Unstable fractures.
b. Open fracture.
c. With neurovascular injuries (when exploration is needed).
d. Pathological fractures.
Correct answer: B

8. The following fractures ore more common in postmenopausal females except:


a. Fracture surgical neck of humeurs.
b. Fracture neck femur.
c. Supracondylar fracture humeurs.
d. Colle's fracture.
Correct answer : C

9. Delayed union:
a. May be caused by infection.
b. Systemic steroids may cause delayed union.
c. ls diagnosed when there is non-union after 1.5 times of the expected time for union.
d. ls treated by treatment of the cause + rigid fixation.
e. All of the above.
Correct answer : E

10. The followings are complications of fracture except:


a. Neurogenic shock.
b. Stroke.
c. Fat embolism.
d. Renal failure.
e. Sepsis.
Correct answer : B

11. Immobilization of fractures of long bones should include


a. Fractured bone only
b. Joint involved in the fracture
c. Proximal joint
d. Both proximal and distal joints
e. Distal joint
Correct answer : D

12. Causes of gangrene after fracture in a limb do not include:


a. Direct crushing of the tissues
b. injury to the main vessels
c. Tight plasters
d. Septic infection
e. Clostridial infection
Correct answer : D

- 372 -
13. The most common site of fracture clavicle is:
a. Middle third.
b. Medial end.
c. Between middle and lateral thirds.
d. Between middle and medial thirds.
Correct answer : C

14. Most common complication of fracture clavicle is:


a. Brachial plexus injury.
b. Shoulder stiffness.
c. Mal union (of no functional significance).
d. None of the above.
Correct answer : C

15. The most common type of shoulder dislocation is:


a. Posterior.
b. Anterior.
c. lnferior.
d. Superior.
Correct answer : B

16. The commonest complication of anterior shoulder dislocation is:


a. Axillary artery injury.
b. Rotator cuff tear.
c. Recurrent dislocation.
d. None of the above.
Correct answer : C

17. The treatment of recurrent shoulder dislocation is:


a. Kocher's method.
b. Hippocrotic method.
c. Bonkort's operation.
d. Arthrodesis.
Correct answer : C

18. Which fracture description does not match its name?


A Lisfranc = dorsal dislocation of tarsometatarsal joint +/- # 1st cuneiform or 2nd MT
B Monteggia = # proximal third ulna and dislocation of radial head
C Clay-shoveller’s = avulsion of C3, 4,5 spinous processes
D Bennett’s = intra-articular # base of thumb metacarpal and proximal, radial and
dorsal displacement of distal fragment
Correct answer : C

19. Which of the following is FALSE regarding C-spine injuries?


A Hangman’s # is a hyperextension injury
B Anterior wedge # is usually mechanically stable
C Anterior tear-drop # is nearly always mechanically unstable
D C2 and C3 are the most commonly injured vertebra.
Correct answer : D

- 373 -
20. Most common hand infection is:
a. Distal pulp space infection (felon).
b. Paronychia.
c. Ulnar bursitis.
d. Thenar space infection.
Correct answer: B

21. The most desirable position to immobilize the hand:


a. Wrist is flexed, MCP joints ore extended and lP joints ore flexed.
b. Wrist is flexed. MCP joints ore flexed and lP joints ore extended.
c. Wrist is extended, MCP joints ore extended and lP joints ore flexed.
d. Wrist is extended. MCP joints ore flexed and lP joints ore flexed.
e. Wrist is extended, MCP joints ore flexed and lP joints ore extended.
Correct answer:E

Congenital orthopedic disorders and birth injuries

1. Earliest bone to ossify is


A. Clavicle
B. Tibia
C. Sternum
D. Ribs
Correct answer A

2. Features seen on a child’s cervical spine X Ray that are different from an
adult’s include all of the following EXCEPT
A Anterior pseudo subluxation C2 on C3 can be normal until 8 years
B Increased depth of anterior soft tissue space
C Predental space (C1) up to 5mm
D Prominent lordosis
Correct answer : D

3. First bone to ossify


a. Femur.
b. Tibia.
c. Clavicle.
d. Sternum.
Correct answer: C

- 374 -
Shoulder and upper limb fractures

1.Which of the following is not a complication of Colle's fracture


A.Sudek's osteodystrophy
B.Non Union
C.Shoulder stiffness
D.Malunion
Correct answer B

2. Median nerve involvement is most common with


A. elbow dislocation
B. supracondylar fracture
C. lateral condyle fracture
D. olecranon fracture
Correct answer B

3. Which is a CORRECT indication for replantation post amputation?


A Single digit amputation proximal to the FDS insertion
B Sharp amputations with minimal to moderate avulsion proximal to the elbow
C Multiple level amputations
D Self- Inflicted amputations
Answer: B

4. Which of the following regarding elbow x rays is FALSE?


A The anterior humeral line passing through the middle third of the capitellum
indicates a supracondylar fracture
B A visible posterior fat indicates an intracapsular fracture
C A small anterior fat pad is normal
D The radia-capitellar line should pass through the middle 1/3 of the capitellum
Answer: A

5. Regarding forearm fractures


A Bartons fracture involves the distal radia-ulnar joint
B Monteggia fracture dislocation involves fracture of the radius proximally and distal
radia-ulnar joint disruption
C Monteggia fracture requires POP application in supination
D Galeazzi fractures require fixation of the related joint disruption in most cases as
well as ORIF of the bone fracture
Correct answer : C

6. With regard to scaphoid #’s which statement is TRUE?


A Risk of # if not seen on initial X-ray is 10-20%
B Most common location is proximal pole ≈ 50%
C Can get AVN of tubercle #s
D MRI sensitivity approaches 100%
Answe: D

- 375 -
7. Regarding forearm fractures which is FALSE?
A All Galeazzi fracture-dislocations require surgery
B All Monteggia fracture-dislocations require ORIF
C Radial head fractures with up to 200 tilts can be managed conservatively
D Smith’s fracture is managed with below elbow plaster with wrist in pronation and
full dorsiflexion
Answe: D

8. Radial head fractures are commonly associated with all of the following
EXCEPT
A Lateral collateral ligament injury
B Medial collateral ligament injury
C Coronoid fracture
D Olecranon fracture
Correct answer : A

9. With regards to isolated medial epicondyle fractures, which is FALSE?


A Patients present with pain on flexion of the digits
B They are extraarticular injuries
C The medial epicondyle may become entrapped in the joint space in Medial
Collateral ligament tears.
D They affect adults more commonly than children
Correct answer : D

10. Which association is the least likely?


A Brachial plexus injury and fracture of the coracoid process
B Pseudo subluxation in proximal humeral fractures
C Wrist drop in Type III supracondylar fractures
D Volkmanns Ischaemic Contracture in supracondylar fractures
Correct answer : C

11. Which is FALSE regarding supracondylar fractures?


A Posterior displacement of the distal fragment are more common than
anterior displacement
B Fat pad sign may be negative in severe supracondylar fractures involving capsular
rupture
C Injuries to the anterior interosseous nerve are easily missed and are detected by an
inability to use FDP on the radial side
D An absent radial pulse as an isolated finding generally warrants urgent surgical
exploration
Correct answer : D

12. Regarding arm injuries which of the following is TRUE?


A Radial neck fractures with >10° angulation should be reduced.
B Colles fractures in post-menopausal women require only orthopaedic follow-up.
C A Barton fracture with minimal displacement will require operative reduction.
D Galeazzi fracture should not be diagnosed when there is <4mm of distal radia-ulnar
separation.
Correct answer : C

- 376 -
13. In regard to elbow dislocation, which of the following is FALSE?
A It is associated with significant soft tissue injury and resultant instability.
B It is one of the three most common large joint dislocations – the others being
glenohumeral and patellofemoral
C Patients typically present with their arm held in near full extension
D The commonest associated neurovascular injury is to the Ulnar nerve.
Correct answer : C

14. The following statements are true EXCEPT


A Volkmann’s ischaemic contracture of the fore-arm and hand may develop despite
wrist pulses being present and the forearm skin being well perfused
B ‘Fat pad’ sign is helpful for diagnosing occult #’s outside the elbow joint capsule
C On the lateral XR of the elbow, the radia-capitellar line should intersect the middle
1/3 of the capitellum
D 25 % of supra-condylar #’s are undisplaced and is the most common # involving
the elbow in children
Correct answer : B

15. In which area do clavicular fractures most commonly occur?


A Inner 1/3.
B Junction of inner 1/3 and middle 1/3.
C Proximal end
D Junction of middle 1/3 and outer 1/3.
Correct answer : D

16. Which of the following regarding scapular fractures is FALSE?


A Most commonly involve the blade.
B Are commonly overlooked.
C Are associated with thoracic injury in 80% of cases.
D Commonly managed non-operatively.
Correct answer : C

17. Regarding forearm fractures, which of the following is TRUE?


A Radial neck fractures with up to 30 degrees tilt can be managed conservatively
B Posterior interosseous nerve injury is common with Monteggia fracture dislocation
C X-rays in Galeazzi injury shows a radial fracture with dorsolateral tilt
D Radial nerve injury is common in Colles fracture
Correct answer : B

- 377 -
Pelvis and lower limb fractures

1 . A 25 year old male presents to emergency department following a road traffic


accident. On examination there is pelvic fracture and blood at urethral meatus.
Following are true about patient except
A. Anterior urethra is the most likely site of injury
B. Retrograde urethrography should be done after the pt is stabilized
C. Foley catheter may be carefully passed if the RGU is normal
D. Rectal examination may reveal a large pelvic hematoma with the prostate
displaced superiorly
Correct answer: A

2. Which of the following is TRUE?


A An ipsilateral Pelvic fracture is commonly associated with an L5 transverse process
fracture
B The posterior column of the thoracolumbar spine involves the posterior longitudinal
ligament
C Intracapsular hip fractures are more common than extracapsular fractures
D The Hawkins classification can be used for hip fractures
Answer: A

3. What is the position of lower limb in anterior dislocation of hip


A.flexion adduction and internal rotation
B.flexion adduction and external rotation
C.flexion abduction and internal rotation
D.flexion abduction and external rotation
Correct answer D

4. A segmental compound fracture of tibia with 1 cm skin wound is classified as


A.Type I
B.Type II
C.Type IIIA
D.Type IIIB
Correct answer A?

5. What is the diagnostic radiological finding in skeletal flurosis


A. sclerosis of sacroiliac joint
B. interroseous membrane ossification
C. osteosclerosis of vertebral body
D. osssification of ligaments of knee joint
Correct answer D?

6. All of the following is true with regards to Perthes disease, EXCEPT:


A All children with irritable hips should be referred to an orthopaedic surgeon
B It is mostly unilateral
C It occurs more often in malnourished children

- 378 -
D When followed up 20 years later; most patients are pain free and active
Answer: A

7. In regard to slipped upper femoral epiphysis (SUFE):


A On an AP X-ray, a line drawn up along the superior femoral neck will not intersect
the femoral head
B Obesity does not play a role in its pathogenesis
C Ultrasound has low sensitivity to detect slip of the epiphysis
D Most patients will develop avascular necrosis of the hip
Answer: A

8. Which of the following statements regarding ankle fractures is TRUE?


A A Maisonneuve fracture is produced by forced external rotation
B The Pott’s system is based on the level of the fibula fracture
C The Weber classification is based on the number of malleoli involved
D The Ottawa ankle rules have not been prospectively validated in children
Answer: A

9. Regarding Nerve Injuries that characteristically accompany certain


Orthopedic Injuries which pairing is CORRECT?
A Lunate dislocation - Ulnar nerve
B Femoral Shaft Fracture- Femoral Nerve
C Shoulder Dislocation- Radial Nerve
D Knee Dislocation- Peroneal Nerve
Answer: D

10. Regarding Compartment Syndrome which bone fracture is MOST commonly


implicated?
A Femur
B Radius
C Tibia
D Humerus
Answer: C

11. In regards to Slipped Upper Femoral Epiphysis (SUFE), which is


CORRECT?
A More common in females
B In chronic slip passive flexion is associated with external rotation
C In unstable SUFE patient can still weight bear
D More common in children under the age of 10
Answer: B

12. What is the key event required for a Triplane Ankle Fracture to Occur?
A Inversion Injury
B Ossification of Epiphysis
C Axial Loading Injury
D Partial Fusion of Growth Plate
Answer: D

- 379 -
13. Which of the following is TRUE?
A Most tibial plateau fractures involve the medial plateau
B A Segond fracture suggests PCL disruption and anterolateral rotator instability
C The younger the age of onset of Perthes disease, the better the prognosis
D The more proximal the scaphoid fracture, the lower the incidence of
avascular necrosis
Answer: C

14. Which of the following pairs is CORRECT?


A Femoral head fractures – Pipkin classification
B Femoral head fractures -Gardiner classification
C Patellar fractures – Hohl classification
D Ankle factures – Boehler classification
Answer: A

15. Which of the following is CORRECT when treating neck of femur fractures?
A Alignment maintained by pre operative skin traction helps post operative mobility
B Indwelling urinary catheters are not associated with post operative infection
C Antibiotics at time of anaesthetic induction reduce post operative infection rates
D Patients do not need nursing on pressure mattresses in the pre operative
period where surgery is conducted within 24 hrs
Answer: C

16. Regarding Segond fractures which is TRUE?


A Small avulsion-fracture proximal medial tibia
B Signifies tear of the menisci-tibial attachment of the middle 1/3 of the medial
capsular ligament
C All have anterior cruciate tears
D Few have meniscal tears
Answer: C

17. Regarding hip injuries, which is TRUE?


A Garden I and II fractures are non-displaced fractures and can be managed
conservatively
B Intracapsular fractures are more common than extracapsular fractures
C The hip is least stable when it is flexed and abducted
D Superior femoral head fractures are often associated with anterior hip dislocations
Answer: D

18. Regarding knee injuries, which is FALSE?


A. Lateral tibial plateau fractures are associated with MCL and ACL injury
B. The knee joint is the most commonly injured joint in the body
C. Patella often dislocated laterally due to the weakening of the vastus lateralis tendon
insertion
D. Dislocation of the knee is associated with popliteal artery and tibial nerve injury
Answer: C

- 380 -
19. In knee injuries which of the following is TRUE?
A A common triad of injuries is ACL/MCL and medial meniscus.
B Proximal tib-fib dislocation occurs with a twisting force to the extended knee.
C Tears of the patellar tendon need repair due to the high stress of quadriceps
contraction.
D 60% of adolescents with a knee hemarthrosis on x-ray will have an
osteochondral fracture.
Correct answer : D

20. Which of the following statement regarding femoral neck fractures is TRUE?
A They are more common in men
B Intracapsular fractures are more common than extracapsular fractures
C Displaced fractures should be treated with skin traction in the ED
D Intermittent catheterization is preferable to indwelling catheters
Correct answer : D

21. In the child with Slipped Upper Femoral Epiphysis (SUFE), which statement
is FALSE?
A SUFE is a Salter Harris type fracture.
B Bilateral SUFE in a child under 10 years of age is more likely to be associated with
endocrine / metabolic disorders.
C The ‘Klein line’ is helpful in diagnosing SUFE
D A ‘Frog leg’ plain XR view of the hips is a helpful investigation to diagnose SUFE
Correct answer : D

22. In the child with Leg-Calve-Perthes Disease (LCPD), which statement is


TRUE?
A It is a form of capital femoral epiphysis avascular necrosis secondary to trauma
B CT imaging is preferred above plain ‘frog leg’ XR’s of the hips
C Patients with suspected LCPD disease require admission for further in patient
management.
D Is clinically indistinguishable from acute synovitis
Correct answer : D

23. Which of the following is TRUE with regards to knee injuries?


A Usually a varus force with axial loading will produce a lateral condylar fracture of
the tibia
B Elderly patients with a lateral tibial plateau fracture usually require a bone scan as
part of their investigation
C A Segond fracture is a marker of collateral ligament injury and ACL disruption
D Popliteal artery is relatively protected from injury due to its anatomical position
Correct answer : C

24. Regarding knee injuries, which of the following is TRUE?


A Segond fractures are important markers of PCL disruption
B Fracture types I, II and III involve the medial tibial plateau with increasing articular
depression
C Medial tibial plateau fractures are twice as common as lateral fractures
D Lateral plateau fractures are associated with ACL and MCL disruption
Correct answer : D

- 381 -
Osteomyelitis and arthritis

1.Which of the following childhood disorders is a common differential diagnosis


of osteomyelitis
A.chondrosarcoma
B.Ewing's sarcoma
C.osteosarcoma
Correct answer B

2. Rigid flat foot is not seen in


A. Osteo arthritis
B. rheumatoid arthritis
C. peroneal spasm
D. congenital tarsal fusion
Correct answer A

3. Regarding Paediatric Arthritis,


A The arthritis of psoriatic arthritis always follows the skin lesions
B Enthesitis is associated with HLAB27
C Iritis is associated particularly with the polyarticular form
D Constitutional features are common with the pauci-articular form
Answer: B

4. Which of the following is FALSE Regarding the systemic form of Juvenile


Chronic Arthritis, (Stills disease)?
A A temperature of 39.5 for two weeks is required for the diagnosis of the systemic
form
B Pericarditis is an association
C Lymphadenopathy is common
D The child generally appears well
Answer: D

5. Which statement is unique regarding Sudek's atrophy:


a. Occurs most common after wrist and ankle injuries.
b. ls characterized by severe pain and stiffness.
c. Followed by osteoarthritis of the near joints.
d. May require sympathectomy.
e. ls a type of osteodystrophy.
Correct answer : C

Bone tumors

1.A patient with an embryonal rhabdomyosarcoma of the paratesticular region


demonstrates multiple enlarged unilateral retroperitoneal lymph nodes on CT
scan. The primary tumor is completely resected and a single lymph node is

- 382 -
removed which demonstrates involvement with tumor. No other disease is found
on staging evaluation. What is stage and group of this patient’s tumor?
a. Stage 1, Group I
b. Stage 1, Group II
c. Stage 1, Group III
d. Stage 2, Group I
e. Stage 2, Group II
Correct answer: C

2. Regarding Multiple Myeloma :


a. It is the commonest 1ry neoplasm of bone
b. Generalized osteopenia is a recognized appearance
c. Scintigraphy over-estimates disease extent
d. Lesions becomes sclerotic following chemotherapy
e. Vertebral pedicle destruction is an early event
Correct answer: A; B; D.

3- Erosion of the lateral 3rd of the clavicle is seen in the following:


a. Rheumatoid arthritis
b. Ankylosing spondylitis
c. Langerhans cell histiocytosis
d. Hypoparathyroidism
e. Multiple myeloma
Correct answer: A;E

4. Concerning Malignant bone lesion:


a. Chordoma is most common in the thoracolumbar spine
b. Fibrosarcoma is the commonest tumour 2ry to Paget’s disease
c. Adamantinoma occurs in the tibia in over 90% of cases
d. Angiosarcoma has a soap bubble appearance at radiography
e. Chondroblastoma are typically found in the diaphysis
Correct answer: B; C

5. Concerning Malignant bone lesion:


a. Chordoma is most common in the thoracolumbar spine
b. Fibrosarcoma is the commonest tumour 2ry to Paget’s disease
c. Adamantinoma occurs in the tibia in over 90% of cases
d. Angiosarcoma has a soap bubble appearance at radiography
e. Chondroblastoma are typically found in the diaphysis
Correct answer: B; C; D

6. What is the most common primary malignant tumour of bone?


1. Ewing’s sarcoma
2. Osteosarcoma
3. Chondrosarcoma INDIAN
4. Giant cell tumour
Correct answer: 1

- 383 -
7. The peak incidence of Ewing’s sarcoma occurs during which decade?
1. First
2. Second
3. Third
4. Sixth
Correct answer: 2

8. During which decade is the peak incidence for osteosarcoma?


1. First
2. Second
3. Third
4. Sixth
Correct answer: 3

9. A 11 – years – old Caucasian girl with tumor arises in the diaphysis of the
femur and X – ray shows an onion-skin pattern. Biopsy of a bone tumor reveals
small round blue cells. Immunostains show that the tumor is CD99 positive. You
and the pathologist decide to do which of the following?
A. Initiate treatment for small round cell osteosarcoma
B. Repeat the biopsy
C. Do FISH using FLI1 break-apart probes
D. Do RT PCR for the t(1;13)(p36;q14)
E. Do FISH using EWS break-apart probes
Correct answer is E.

10. A 7-year old girl is receiving chemotherapy for a stage 3, group III
rhabdomyosarcoma of the extremity. After her second course of chemotherapy
she is admitted to the hospital with right upper quadrant pain, jaundice,
abdominal distension, and weight gain. Which of the following is the most
appropriate next step in the evaluation/treatment of this patient?
A. Echocardiogram
B. Nutrition consult
C. Doppler study of the liver
D. Serum hepatitis screen
E. Urinalysis
Correct answer is C.

11. A patient with a 4 cm rhabdomyosarcoma of the lower extremity. The


tumor has been completely resected (margins negative for tumor on histology)
and regional lymph node sampling has been done. No other sites of metastatic
disease are present. Which of the following statements is correct concerning
stage and Group assignment?
A. If a regional lymph node is not enlarged on imaging or physical examination but is
removed and shows rhabdomyosarcoma on histology, then the patient has stage 2,
Group I rhabdomyosarcoma
B. If a regional lymph node is not enlarged on imaging or physical examination but is
removed and shows rhabdomyosarcoma on histology, then the patient has stage 3,
Group II rhabdomyosarcoma

- 384 -
C. If a regional lymph node is enlarged on imaging and physical examination and is
removed but does not show rhabdomyosarcoma, then the patient has stage 3, Group
I rhabdomyosarcoma
D. If a regional lymph node is enlarged on imaging and physical examination and is
removed but does not show rhabdomyosarcoma, then the patient has stage 2, Group
I rhabdomyosarcoma
E. If a regional lymph node is enlarged on imaging and physical examination and is
removed and shows rhabdomyosarcoma, then the patient has stage 3, Group III
rhabdomyosarcoma
Correct answer is C.

12. A 3-month old presents with paraspinal infantile fibrosarcoma. Molecular


studies show the presence of the characteristic chromosomal translocation.
Based on cytogenetic analysis, this tumor seems to share a genetic origin with
which of the following tumors?
A. Mesoblastic nephroma
B. Aggressive fibromatosis (Desmoid tumor)
C. Infantile hemangiopericytoma
D. Inflammatory myofibroblastic tumor
E. Malignant rhabdoid tumor
Correct answer is A.

13. A 16-year old is referred to you with an osteoblastic osteosarcoma in the


scapula. When taking a history, you learn that she had been treated for vaginal
rhabdomyosarcoma when she was 2 years of age. Which genetic mutation best
explains this cancer pattern?
A. Germline p53 mutation
B. Germline Rb mutation
C. Germline RECQL4 mutation
D. Germline PAX3 mutation
E. Germline LOH at 11p15.5
Correct answer is A.

14. A patient completed therapy for localized osteosarcoma using high-dose


methotrexate, doxorubicin and cisplatin chemotherapy and is now concerned
about ringing in the ears. You tell her that,
A. This is most likely not a result of any therapy that she received
B. This is most likely a result of antibiotics that she received during her chemotherapy
C. This is most likely a result of cisplatin
D. This is most likely a result of methotrexate
E. This is most likely a result of doxorubicin
Correct answer is C.

15. You are caring for a new patient with a large localized Ewing sarcoma of
the soft tissues of the arm. The surgeon believes that the tumor can be resected
without amputation but asks whether you can give some chemotherapy to
shrink the tumor before surgery. Which of the following would you tell the
surgeon?

- 385 -
A. If the tumor can be resected without amputation, then the best time to do the resection
is before any chemotherapy in order to improve the prognosis
B. You agree with waiting to do the resection until week 12 of therapy and will begin
chemotherapy; you recognize that radiotherapy will not be necessary if the tumor is
completely resected at week 12 of therapy
C. You agree with waiting to do the resection until week 12 of therapy and will begin
chemotherapy; you recognize that radiotherapy will be necessary even if the tumor
is completely resected at week 12 of therapy
D. If the tumor can be resected without amputation, then the best time to do the resection
is before any chemotherapy; you recognize that this is the only way to avoid
radiotherapy
Correct answer is B.

16. You are reviewing the MRI of a patient with a tumor of the distal femur.
The radiologist tells you that the tumor is on the surface of the bone and does
not invade the medullary cavity. The radiologist suspects osteosarcoma. The
MRI suggests which of the following?
A. This is not an osteosarcoma based on this pattern.
B. This is a low grade osteosarcoma that can be treated with resection alone.
C. This is a high grade osteosarcoma that can be treated with resection alone.
D. This is an osteosarcoma and all osteosarcomas require chemotherapy.
Correct answer is B

17. A 16-year-old is referred to you with an osteoblastic osteosarcoma in the


scapula. When taking a history, you learn that she had been treated for vaginal
rhabdomyosarcoma when she was 2 years of age. Which genetic mutation best
explains this cancer pattern?
A. Germline LOH at 11p15.5
B. Germline Rb mutation
C. Germline RECQL4 mutation
D. Germline PAX3 mutation
E. Germline p53 mutation
Correct answer is E

18. You treated a patient for a localized Ewing sarcoma of the chest wall on the
right side at the age of 7 with cycles of vincristine, doxorubicin, and
cyclophosphamide (VDC) alternating with cycles of etoposide and ifosfamide
(IE) and radiotherapy. At age 16 he presents with an osteosarcoma of the right
scapula. Which of the following best explains why the osteosarcoma developed?
A. This is most likely secondary to previous treatment with an alkylating agent.
B. This is most likely secondary to previous treatment with a topoisomerase II inhibitor.
C. This is most likely secondary to radiotherapy.
D. This is most likely secondary to a germline mutation in a tumor suppressor gene.
E. This is most likely secondary to previous treatment with anthracycline.
Correct answer is C

19. You are caring for a new patient with a large localized Ewing sarcoma of
the soft tissues of the arm. The surgeon believes that the tumor can be resected
without amputation but asks whether you can give some chemotherapy to

- 386 -
shrink the tumor before surgery. Which of the following would you tell the
surgeon?
A. If the tumor can be resected without amputation, then the best time to do the resection
is before any chemotherapy to improve the prognosis.
B. You agree with waiting to do the resection until week 12 of therapy and will begin
chemotherapy; you recognize that radiotherapy will not be necessary if the tumor is
completely resected at week 12 of therapy.
C. You agree with waiting to do the resection until week 12 of therapy and will begin
chemotherapy; you recognize that radiotherapy will be necessary even if the tumor
is completely resected at week 12 of therapy.
D. If the tumor can be resected without amputation, then the best time to do the resection
is before any chemotherapy; you recognize that this is the only way to avoid
radiotherapy.
Correct answer is B

20. A surgeon refers a patient to you with osteosarcoma of the distal femur. He
has already completed what appears to be an appropriate and uncomplicated
limb-sparing surgical resection. You complete a staging evaluation that shows
no evidence for metastases. The patient’s family has learned that most patients
receive some chemotherapy before surgery, and they wonder whether their
child should have received chemotherapy before surgery. You indicate that the
surgeon should have referred the patient to you before surgery and that
A. You will treat the patient with high-dose methotrexate, doxorubicin, and cisplatin
(MAP) chemotherapy and the outcome will not be adversely affected by the early
surgery.
B. The patient’s prognosis is now definitely worse because you will not be able to
optimize the chemotherapy regimen based on the response to chemotherapy.
C. You could have administered preoperative chemotherapy, which would have
changed the surgical approach dramatically.
D. You will now have to treat the patient with an intensified chemotherapy regimen that
was designed for patients with unresectable/metastatic osteosarcoma.
E. You will treat the patient with MAP chemotherapy and optimize local control by
using radiotherapy so that the outcome should not be affected.
Correct answer is A

21. A 10-year-old presents with a localized Ewing sarcoma of the ilium. Which
of the following best describes prognostic features for this patient?
A. A pelvic primary site is the single most unfavorable prognostic feature for Ewing
sarcoma.
B. Prognosis for this patient depends primarily on whether surgery or radiotherapy is
used as local therapy.
C. Prognosis for this patient has improved with the addition of ifosfamide and etoposide
to vincristine, doxorubicin, and cyclophosphamide.
D. This patient’s age represents a more important negative prognostic factor than the
primary site.
Correct answer is C

- 387 -
Others

1. Which of the following is FALSE in regard to the Mangled Extremity Severity


Score (MESS)?
A The presence and degree of other major injuries is a parameter that contributes to the
final score
B Limb ischemia is a parameter that contributes to the final score
C A score of 7 or more is associated with 100% chance of amputation
D The degree of soft tissue injury is a parameter that contributes to the final score
Answer: A

2. Which of the following is TRUE?


A Clinical examination is more sensitive than x-ray in picking up a fracture
B A 'fat pad' sign of a fracture is less reliable in obese individuals
C x-ray is less specific than clinical examination in regards to fracture diagnosis
D X-rays with 2 views is 95% sensitive and specific for major joint dislocation
Answer: A

27- Anaesthesia and intensive care:

Preoperative assessment and premedication

1. What is the BMI (body mass index) of a male with weight 45kg and height
1.5m
A.19
B.20
C.21
D.18
Correct answer: B

2 . All are important causes of hyponatremia, except-


A. Excessive vomiting
B. Gastric fistula
C. Prolonged Ryle's tube aspiration
D. Excessive Sweating
Correct answer: D

3 . All electrolyte abnormalities are seen in immediate postoperative period,


except -
A. Negative Nitrogen balance
B. Hypokalemia
C. Glucose intolerance

- 388 -
D. Hyponatremia
Correct answer: D

4 . A patient underwent prostatic surgery and during surgery glycine was used
for bladder irrigation. Which abnormality will give vomiting and clouding of
consciousness on 3rd postop day:
A. Hypomagnesemia
B. Hypokalemia
C. Hyponatremia
D. Hypercalcemia
Correct answer: C

5 . 60-year male undergoes TURF. After 3 days patient develops altered


sensitiveness and drowsiness. Most probable diagnosis is -
A. Hyponatremia
B. Hypernatremia
C. Meningitis due to spinal anaesthesia
D. Stroke
Correct answer: A

General anaesthesia (inhalational), (intravenous)

1. Which of the following induction agent is used to produce a 'street-fit' person


following surgery
A. midazolam
B. propofol
C. alfentanyl
D. thiopentone
Correct answer B

2. Which of the following reacts with Soda lime in anaesthetic circuits


A. trichloroethylene
B. isoflurane
C. halothane
D. methoxyflurane
Correct answer A

3. Which of the following anesthetics sensitizes the heart to catecholamines


A. Halothane
B. Isoflurane
C. Enflurane
Correct answer A

- 389 -
4. Thiopentone sodium has a short duration of action because
A. it is rapidly metabolized
B. it is rapidly redistributed
Correct answer B

5. In emergency caesarian section rapid induction of anesthesia is done to


A. prevent gastric aspiration
B. prevent fetal depression
C. to decrease awareness
Correct answer A

Complications of General Anasthesia

1 . Post operative Pulmonary Thromboembolism is seen in all, except:


A. Pregnant female
B. Oestrogen therapy
C. Tall and thin man
D. Obese male
Correct answer: C

2 . Which of these is not a risk factor for thromboembolism:


A. Myocardial infarction
B. Hypertension
C. Estrogen therapy
D. Superficial thrombophlebitis
Correct answer: B

Regional anaesthesia

1 . Upper does limit of plain lignocaine in a 70 kg adult is -


A. 100 mg
B. 150 mg
C. 200 mg
D. 250 mg
Correct answer: C

- 390 -
Basic life support

28- TRAUMA

1.Which investigation of the abdomen in trauma is described by: rapid, sensitive, not
specific, not examine retroperitoneum, repeatable, non invasive?
a) physical examination
b) CT
c) FAST (Focused assessment with sonography in trauma)
d) DPL (Diagnostic peritoneal lavage)
e) Plain x- ray
Correct answer: C

2.What RBC count constitutes a positive Diagnostic peritoneal lavage (DPL)?


a) >30 000/cm3
b) >60 000/cm3
c) >100 000/cm3
d) >200 000/cm3
e) >500 000/cm3
Correct answer: C

3.Which statement is incorrect with regards to penetrating neck injury?


a) if the platysma muscle is clearly intact local wound repair is all that is required
b) if the platysma has been violated it must be assumed that significant injury has
occurred
c) in stable patient with a zone 1 injury angiography and oesophagoscopy are
mandatory +/- bronchoscopy
d) in a stable patient with a zone 3 injury angiography and oesophagoscopy are
mandatory +/- bronchoscopy
e) Zone 2 injuries some people advocate mandatory exploration
Correct answer: D

4.Which statement is true regarding c spine fracture in hanging?


a) this is the usual mechanism of death with associated spinal cord transection
b) it is often seen but is not usually associated with spinal cord injury
c) it is only seen if the person falls the distance of their height and their feet do not
touch the ground
d) the lower C spine is usually involved when it occurs
e) spinal cord injury tends to happen without C spine fracture
Correct answer: C

5- A 23 years male is brought to the emergency as a case of road traffic accident. He


is hypotensive. Most possible ruptured organ is:
A. Kidney
B. Rectum
C. Spleen
D. Mesentry
- 391 -
Correct answer: C

6 . Which of the following is done to expose the coeliac axis, left renal artery, superior
mesenteric artery and abdominal aorta in a case of trauma?
A. Cranial visceral rotation
B. Caudal visceral rotation
C. Left medial visceral rotation
D. Right medial visceral rotation
Correct answer: C

7. Which is incorrect of trauma in pregnancy?


a) Fluid in vagina with pH of > 7.0 is suggestive of amniotic fluid.
b) The Kleihaur-Betke test will indicate which Rh –ve patients require anti-D.
c) Ultrasound is less sensitive than CTG in identifying patients with placental
abruption.
d) Peri mortem C-section should be done at 4 mins into maternal arrest.
e) The trauma views of C-spine, CXR and pelvis have combined radiation of less than
1.0 rad and so are relatively safe in pregnancy.
Correct answer: B

8. Which is incorrect of paediatric trauma?


a) In haemorrhagic shock BP will not fall until approx. 45% blood loss.
b) Duodenal haematoma is more common in children than adults.
c) Subluxation of C2 on C3 is seen in up to 40% of children < 7 years normally.
d) An infant with eyes open to shouting, crying inconsolably, and localizing to pain
has a GCS of 11.
e) Haemorrhagic shock can not be due to subgaleal haematomas in children.
Correct answer: E

9 . Most common cause of Rupture of Tendon is:


A. Congenital defect
B. Fall from height
C. Overuse
D. Trauma
Correct answer: C

10 . A hemodynamically stable patient with blunt abdominal trauma, the best


investigation is ?
A. CECT abdomen
B. MRI abdomen
C. DPL
D. FAST
Correct answer: A

11 . A child presents in causality in stable condition after a blunt abdominal trauma


associated with splenic trauma the treatment of choice is:
A. Observation
B. Splenectomy
C. Arterial embolization
D. Splenorrhaphy

- 392 -
Correct answer: A

12 . After trauma, a patient presents with a drop of blood at the tip of urinary meatus;
He complains of inability to pass urine. Next step of management should be:
A. Catheterize, drain bladder, and remove the catheter thereafter
B. Catheterize, drain bladder, and retain the catheter thereafter
C. IVP should be done
D. MCU should be done
Correct answer: B

13 . In order to expose the coeliac axis, left renal artery, superior mesenteric artery
and blood abdominal aorta in a case of trauma, which of the following is performed -
A. Cranial visceral rotation
B. Caudal visceral rotation
C. Left medial visceral rotation
D. Right medial visceral rotation
Correct answer: C

14 . A patient died after a blunt trauma to chest most common cause of death in blunt
trauma to chest is -
A. Oesophageal rupture
B. Tracheo bronchial rupture
C. Pulmonary laceration
D. Pneumothorax
Correct answer: B

15 . All except one are correct regarding renal trauma -


A. Observation is best
B. IVP is indicated
C. Exploration indicated in all cases
D. Haematuria is a cardinal sign
Correct answer: C

16 . After Acute trauma Fluid replacement is best assessed by which:


A. CVP
B. Urine output
C. Pulse
D. BP
Correct answer: A

17 . FAST stands for


A. Focused Assessment with sonography for trauma
B. Focused Assessment with sonography and Tomography
C. Fast Assessment with sonography for Trauma
D. Fast Assessment with sonography and Tomography
Correct answer: A

18 . What should be the approach for doing laparotomy in blunt trauma abdomen?
A. Always midline incision
B. Depends on organ injured

- 393 -
C. Depends upon type of injury
D. Always transverse
Correct answer: A

19 . A patient sustained Traumatic injury to major abdominal vessels. It has been


planned to explore the Suprarenal Aorta, the Coeliac Axis, the Superior Mesenteric
Artery, and the Left Renal Artery. What manoeuvre for exposure is recommended:
A. Cranial visceral Rotation
B. Caudal visceral Rotation
C. Left Medial Visceral Rotation
D. Right Medial Visceral Rotation
Correct answer: C

20 . What is most useful for fluid replacement in post traumatic period -


A. Blood pressure
B. Pulse rate
C. Urine output
D. Central venous pressure
Correct answer: C

21 . What is the treatment for blunt trauma of kidney ?


A. Conservative
B. Nephrectomy
C. Nephrotomy
D. Nephroplexy
Correct answer: A

22 . A 35 years man with blunt abdominal trauma with h/o pelvic fracture has presented
to ER. He has passed only few drops of blood per meatus and no urine in the past 8
hours. His bladder is palpable per abdomen. The correct diagnosis is-
A. Urethral injury
B. Anuria due to hemorrhagic shock
C. Bladder rupture
D. Ureteral rupture leading to extravasation of urine in retroperitoneum
Correct answer: A

23 . Which of the following is the most common urethral injury ?


A. Rupture of bulbar urethra
B. Rupture of penile urethra
C. Rupture of prostatic urethra
D. Rupture of membranous urethra
Correct answer: A

24 . A patient is brought to the hospital with history of RTA 8 hours back. A few drops
of blood were noted at the external urethral meatus. He has not passed urine and his
bladder is palpable per abdomen. What is the probable diagnosis?
A. Ureteral injury with extravasations of urine in the retro peritoneum
B. Urethral injury
C. Rupture bladder
D. Anuria due to hypovolemia

- 394 -
Correct answer: B

25 . A patient was brought to the hospital with a history of RTA eight hours back.A few
drops of blood is noted at the external urethral meatus.He has not passed urine and his
bladder is palpable per abdomen. The probable diagnosis is -
A. Urethral injury with extravasation of urine in the retroperitoneum
B. Anuria due to hypovolemia
C. Urethral injury
D. Rupture bladder
Correct answer: C

26 . Which among the following is best method to assess intake of fluid in Polytrauma
patient
A. Urine output
B. CVP
C. Pulse
D. BP
Correct answer: A

27 . The term post traumatic epilepsy refers to seizures occurring


A. Within moments of head injury
B. Within 7 days of head injury
C. Within serval weeks to months after head injury
D. Many years after head injury
Correct answer: C

28 . Trauma and Injury Severity Score (TRISS) includes which of the bellow:
A. RTS+ISS+Age
B. GCS+BP+RR
C. RTS+GCS+BP
D. RTS+ISS+GCS
Correct answer: A

29 . Criteria for Positive Peritoneal Lavage in a case of blunt abdominal trauma


include:
A. WBC>1000
B. RBC>500
C. Presence of enteric contents
D. Absence of blood
Correct answer: C

30 . A 25 years male complains of upper abdominal and lower chest pain exacerbated
by deep breathing, ten days after a splenectomy for blunt abdominal trauma. He is
anorectic but ambulatory and otherwise making satisfactory progress. On physical
examination, his temperature is 38.2 degree C(108 degree C) reptilian he has decreased
breath sounds at the left lung base. His abdominal wound appears to be healing well,
bowel sound are active and there are no peritoneal sings. Rectal examination is
negative. The W.B.C count is 12,500mm3 with a shift to left. Chest X-rays show
platelike atelectasis of the left lung field. Abdominal X-rays show a nonspecific gas

- 395 -
pattern in the bowel and an air-fluid level in the left upper quadrant. Serum amylase is
150 Somogyi units dl(normal 60 to 80).most Possible diagnosis is
A. Pulmonary embolism
B. Subfascial wound infection
C. Subphrenic abscess
D. Pancreatitis
Correct answer: C

31 . Decision regarding surgery in a case of hemothorax due to blunt trauma chest


should be based on:
A. Hemodynamic status
B. Chest symptoms
C. X-ray finding
D. Nature of chest tube output
Correct answer: A

32 . Which of the following is true about renal trauma


A. Urgent IVP is indicated
B. Exploration of the kidney to be done in all cases
C. Lumbar approach to kidney is preferred
D. Renal artery aneurysm is common
Correct answer: A

33 . All of the following are true about cranial trauma, except-


A. Arteriovenous fistula occur in base skull fracture
B. Post traumatic epilepsy seen in 15%
C. Raccoon eyes seen in subgalea hemorrhage
D. Depressed skull fracture is associated with brain injury at the immediate area of
impact
Correct answer: C

34 . After trauma to neck causing fracture of cervical spine what will be the first step:
A. Immobilization
B. Intubation
C. Maintenance of airway
D. X-ray cervical spine
Correct answer: A

35 . In a case of polytrauma, the first management is:


A. Airway control
B. Breathing
C. Circulation
D. Emergency laparotomy
Correct answer: A

36 . All of the following can be done in a case of pelvic fracture with pelvic
hematoma and had not passed urine since trauma except:
A. Pass indwelling urethral catheter
B. IV fluid infusion
C. IV pylography

- 396 -
D. Digital per rectal examination
Correct answer: A

37 . Which of the following is the best parameter to assess fluid intake in a poly-
trauma patient
A. Urine output
B. BP
C. Pulse
D. Pulse oximetry
Correct answer: A

38 . Example of indirect traumatic gangrene are all except -


A. Use of local anesthetic with adrenaline and tourniquet for digits
B. Pressure of fractured bone on main artery to a limb
C. Thrombosis of large artery following injury
D. That resulting from crush or pressure injury
Correct answer: D

39 . A male patient with blunt Trauma Abdomen is hemodynamically stable. What is


the next line of management?
A. Further imaging of abdomen
B. Observation
C. Laparoscopy
D. Exploratory Laparotomy
Correct answer: A

40 . In hand injury first structure to be repaired should be:


A. Muscle
B. Bone
C. Skin
D. Nerve
Correct answer: C

41 . A 23 years male falls on the ground when he is struck in the right temple by a
baseball. While being driven to the hospital, he lapses into coma. He is unresponsive
with the dilated right pupil when he reaches the emergency department. Which of the
following is the most important step in initial management ?
A. X-ray of the skull and cervical spine
B. Doppler ultrasound examination of the neck
C. Craniotomy
D. CT scan of the head
Correct answer: C

42 . Which of the following is used to define Penetrating Neck Injury?


A. Injury to vital structures
B. 2 cm depth of wound
C. Through and through wound
D. Breach of platysma
Correct answer: D

- 397 -
43 . In hand injuries the first structure to be repaired is
A. Bone
B. Skin
C. Tendon
D. Verve
Correct answer: B

44 . Most common tumor in hand is ?


A. Enchondroma
B. Chondroblaastoma
C. Giant cell tumour
D. Osteoid osteoma
Correct answer: A

29- Head and neck

1- Which one among the following statements about cystic hygroma is untrue?
a. Affects infants and young children.
b. Occurs chiefly in the neck, axilla and groin.
c. Presents as a large soft fluctuating translucent swelling.
d. is localized to the subcutaneous tissues.
e. May rupture subcutaneously with spontaneous cure.
Correct answer : D

2. Among the following statements about branchial fistula, the incorrect one is that it
a. Commonly results from rupture of a branchial cyst.
b. Usually at the lower third of the anterior border of the sternomastoid muscle.
c. is often bilateral.
d. Discharges clear mucoid fluid.
e. Requires removal of the whole track by the "step-ladder' operation.
Correct answer : C

3. Concerning fibrous epulis the incorrect statement is that it:


a. is a soft fibroma of the mucoperiosteum around a carious tooth.
b. Forms a small pedunculated swelling arising between two incisor teeth.
c. is covered by intact epithelium.
d. May be highly vascular or fibrous.
e. is treated by local excision.
Correct answer : E

4. A patient is diagnosed to have malignant metastasis in right upper deep cervical


lymph nodes. There are no symptoms to indicate the site of the primary. The possible
primary lesion may be any of the following, except
a. Nasopharyngeal carcinoma
b. Carcinoma of maxillary antrum
c. Papillary carcinoma of thyroid
d. Carcinoma of pyriform fossa of hypopharynx
e. Laryngeal carcinoma
Correct answer : E

- 398 -
5. The commonest swellings in the carotid triangle are:
a.Thyroid nodules
b. Lymphadenopathy
c. Carotid body tumour
d.Branchial cyst
e.Lipoma
Correct answer : B

6. About cold abscess of the neck, all the following statements are true, except:
a.It is usually caused by caseation of tuberculous lymphadenitis
b. The abscess may have two components, one superficial, and one deep to the deep fascia
c.The condition is mildly painful with low-grade fever
d. Anti-tuberculous treatment is an essential part of treatment
e. Drainage is achieved by incision
Correct answer : E

7. About branchial cyst and fistula, all are true, except:


a.A branchial cyst is observed in the neonate
b.A branchial fistula is present at birth
c.A branchial cyst is partially covered by the sternomastoid muscle
d.The main differential diagnosis of branchial cyst is cold abscess
e.A branchial cyst contains cholesterol crystals
Correct answer : A

8. About tracheostomy, all the following statements are true, except:


a. Tracheostomy is indicated if mechanical ventilation is needed for a long time
b. Emergency tracheostomy is done at the scene of an accident if there is upper airway
obstruction
c. Percutaneous tracheostomy is suitable for critically-ill ICU patients
d. In adults, an ellipse is removed from the anterior wall of 2nd, 3rd and 4th tracheal rings
e.One of its possible complications is subglottic stenosis
Correct answer : B

9. The commonest site for lip cancer is :


a) Angle of the mouth.
b) Lower lip at junction of middle, outer thirds.
c) Upper lip central part.
d) Upper lip at junction of middle, outer thirds.
Correct answer : B

10. All of the following is true about ameloblastoma (previously called adamantinoma)
except………….:
a) It is located near symphysis menti usually.
b) It originates from Mallasez cells.
c) It grows both in horizontal and vertical rami.
d) It has equal lobulations.
Correct answer : A

- 399 -
11-A43-year-old teacher underwent left parotidectomy. Upon awakening from
surgery, paralysis of the left lower lip was observed. This complication was most likely
due to injury to which of the following:
(A) Parotid duct
(B) Facial nerve - temporal branch
(C) Facial nerve – marginal mandibular branch
(D) Facial nerve - main trunk
(E) Platysma muscle
Correct answer : C

12. A9-year-old boy complains of a swelling on the left side of his neck in the
supraclavicular region. The swelling is translucent; a diagnosis of cystic hygroma is
established. What is true of cystic hygroma?
(A) It arises from sweat glands in the neck.
(B) It is usually an anterior midline structure.
(C) It may occur in the mediastinum.
(D) Its lesions are usually easy to enucleate.
(E) It is premalignant.
Correct answer : C

13-A 62-year-old man undergoes excision of a cylindroma of the submandibular


gland. He is most likely to have an injury to which of the following?
(A) Maxillary branch of the trigeminal nerve
(B) Lingual nerve
(C) Vagus nerve
(D) Floor of the maxilla
(E) Frontozygomatic branch of the facial nerve
Correct answer : B

14-A 58-year-old woman undergoes excision biopsy of a tumor in the left posterior
triangle of her neck. Histology suggests that this is a metastatic cancer. What is the
most likely site of the primary tumor?
(A) Ovary
(B) Adrenal gland
(C) Kidney
(D) Piriform fossa
(E) Stomach
Correct answer : D

15-The classic complete neck dissection for palpable adenopathy in the posterior
triangle of the neck includes removal of which of the following?
(A) The transverse process, C2–C4
(B) The spinal accessory nerve
(C) Both thyroid lobes
(D) The trapezius
(E) The vagus
Correct answer : B

16. A 40-year-old woman is suspected of having a carotid body tumor. Which one of
the following is most characteristic of such a tumor?

- 400 -
(A) They secrete catecholamines.
(B) They are more common at sea level.
(C) They arise from structures that respond to changes in blood volume.
(D) They arise from the structures that respond to changes in PO2.
(E) They are usually highly malignant.
Correct answer : D

17. Following surgical resection of a large thyroid mass, a patient complains of


persistent hoarseness and a weak voice. What is the most likely cause of these
symptoms?
(A) Traumatic intubation
(B) Prolonged intubation
(C) Injury to the recurrent laryngeal nerve
(D) Injury to the superior laryngeal nerve
(E) Scar tissue extending to the vocal cords
Correct answer : C

18. What is the most common site for foreign bodies in the head and neck?
(A) Eye
(B) Ear
(C) Nose
(D) Throat
(E) Esophagus
Correct answer : B

19-A 33-year-old female noted a discharge from a sinus in the overlying skin below the
right angle of the mandible. She recalls previous episodes of fullness and mild pain in
this region over the past several years. What is the most likely cause?
(A) Thyroglossal duct cyst
(B) Branchial cyst
(C) Teratoma
(D) Myeloma
(E) Trauma to the neck
Correct answer : B

20-Acute bacterial sialadenitis most commonly affects


a) Parotid
b) Submandibular
c) Submaxillary
d) All equal
Correct answer : A

21-Pharyngeal pouch should be suspected in presence of


a) Dysphagia
b) Regurgitation of undigested food
c) Aspiration pneumonitis
d) All of the above
Correct answer : D

- 401 -
22-regarding the pharyngeal pouch which statement is incorrect
a) It protrudes through killian 's dehiscence
b) It usually turns to the left side of the neck
c) It may be visible in the neck
d) It is twice as common in males as in females
Correct answer : D

23- branchial cyst commonly arises from remnant of


a) First branchial arch
b) Second branchial arch
c) Third branchial arch
d) All of the above
Correct answer : B

24-regarding branchial cyst which statement is incorrect


a) Arise from the second branchial cleft
b) Usually appears between the ages of 20-25 years
c) Protrudes beneath the anterior border of the sternomastoid
d) Is usually lined by squamous epithelium
e) All are correct
Correct answer : E

25-branchial cyst is best differentiated from cold abscess by


a) Fluctuation
b) Transillumination
c) Contains cholesterol crystals
d) None of the above
Correct answer : C

26 . The commonest site of Branchial cysts is


A. Upper 1/3rd of the SCM
B. Lower 1/3rd of the SCM
C. Upper 2/3rd of the SCM
D. Lower 2/3rd of the SCM
Correct answer: A

27 . True about branchial anomaly -


A. Cysts present with dysphagia and hoarseness of voice
B. Most commonly due to 2nd branchial remnant
C. Cysts are more common than sinuses
D. For sinuses surgery is not always indicated
Correct answer: B

28 . Internal opening of branchial fistula is situated-


A. Anterior aspect of posterior faucial pilar near the tonsil
B. Posterior aspect of anterior faucial pilar near the tonsil
C. Lower third of the neck near the anterior border of the sternocleidomastoid
D. Lower third of the neck near the posterior border of the sternocleidomastoid
Correct answer: A

- 402 -
29 . True about Branchial cyst is -
A. Mostly arises from 2nd branchial system
B. Cysts are more common than sinuses
C. Sinus should always be operated
D. Causes dysphagia and hoarseness
Correct answer: A

30 .Which of the following is true about branchial anomaly?


A. Cysts present with dysphagia and hoarseness of voice
B. Most commonly due to 2nd branchial remnant
C. Cysts are more common than sinuses
D. For sinuses surgery is not always indicated
Correct answer: B

31 . True regarding branchial fistula is:


A. Internal opening is situated near the tonsil
B. Externally located in the middle of sternocleidomastoid
C. Single transverse incision is the treatment of choice
D. It's a third arch anomaly
Correct answer: A

32 . A branchial cyst develops from the vestigial remnants of the -


A. 2nd branchial cleft
B. 3rd branchial cleft
C. 1st branchial cleft
D. 4th branchial cleft
Correct answer: A

33 . Branchial cyst is found in -


A. Middle of neck
B. Upper and middle third of the sternomastoid muscle at its anterior border
C. Upper one -third along anterior tender of trapezius
D. Nape of neck
Correct answer: B

34-cystic hygroma includes all except


a) Develops from jugular lymph sacs
b) Brilliantly translucent
c) Occupy the middle third of the neck
d) Enlarge with crying
Correct answer : C

35-Inappropriate to cystic hygroma


a) Type of cavernous haemangioma
b) It can be earliest swelling of neck to appear in life
c) It can obstruct labour
d) It is brilliantly translucent
Correct answer : A

- 403 -
36- Treatment of Ludwig angina includes
a) Amoxicillin plus metronidazole
b) Decompression of both submandibular triangles
c) Tracheostomy
d) All of the above
Correct answer : D

37-The potato tumour of the neck is


a) Sternomastoid tumor
b) Carotid body tumour
c) Thyroid tumour
d) Parotid tumour
Correct answer : B

38-most common presentation of hodgkin lymphoma is


a) Leukocytosis
b) Fever
c) Painless LN enlargement
d) Pruritis
Correct answer : C

39-symptoms of of hodgkin lymphoma include all except


a) Pruritis
b) Headache
c) Weight loss
d) Fever
Correct answer : B

40-All true regarding adamantinoma exept:


a) X-ray honeycomb appearance
b) Pathologically mimic b.c.c
c) Common in females
d) Can cross symphysis menti
Correct answer : D

41- All true regarding osteoclastoma except:


a) Fluid inside chocolate like
b) X ray appearance soap bubble appearance
c) Common in children
d) Pathologically giant cell tumor
Correct answer : C

42- Nerve involvement in Frey's syndrome is ?


A.Trigeminal
B. Mandibular
C. Auriculo temporal
D. Lingual
Correct answer: C

- 404 -
43- All of the following are true regarding cystic hygroma except -
A. May be present at birth
B. Brilliantly trans illuminant
C. Increases in size on coughing
D. Sclerotherapy may be beneficial
Correct answer: D

44- Which of the following is not a landmark for facial nerve during parotid surgery?
A. Inferior belly of omohyoid
B. Digastric muscle
C. Retrograde dissection of distal branch.
D. Tragal pointer
Correct answer: A

45- Which of the following is true about branchial anomaly?


A. Cysts present with dysphagia and hoarseness of voice
B. Most commonly due to 2nd branchial remnant
C. Cysts are more common than sinuses
D. For sinuses surgery is not always indicated
Correct answer: B

46 . One of the following is an example of painless midline swelling in the neck-


A. Branchial cyst
B. Thyroglossal cyst
C. Cystic hygroma
D. Carotid body tumour
Correct answer: B

47. Operoble carcinoma of parotid is best treated by:


a. Radiotherapy alone.
b. Chemotherapy alone.
c. Superficial conservative parotidectomy.
d. Radical parotidectomy with block dissection of LNs.
Correct answer: D

48.Concerning fibrous epulis the incorrect statement is that it :


a. is a soft fibroma of the mucoperiosteum around a carious tooth.
b. Forms a small pedunculated swelling arising between two incisor teeth.
c. is covered by intact epithelium.
d. May be highly vascular or fibrous.
e. is treated by local excision.
Correct answer: E

49. A patinet is diagnosed to have malignant metstasis in right upper deep cervical
lymph nodes. There are no symptoms to indicate the site of the primary .The possible
primary lesion may be any of the following, except
a. Nasopharyngeal carcinoma
b. Carcinoma of maxillary antrum
c. Papillary carcinoma of thyroid
d. Carcinoma of pyriform fossa of hypopharynx

- 405 -
e. Laryngeal carcinoma
Correct answer: E

50. The commonest site for lip cancer is …………..:


a) Angle of the mouth.
b) Lower lip at junction of middle, outer thirds.
c) Upper lip central part.
d) Upper lip at junction of middle, outer thirds.
Correct answer: B

51.All of the following is true about ameloblastoma except………….:


a)It is located near symphsismenti usually.
b)It originates from Mallasez cells.
c)It grows both in horizontal and vertical rami.
d)It has equal lobulations.
Correct answer: A

52. A 58-year-old woman undergoes excision biopsy of a tumor in the left posterior
triangleof her neck. Histology suggests that this is a metastatic cancer. What is the
most likely site of the primary tumor?
(A) Ovary
(B) Adrenal gland
(C) Kidney
(D) Piriform fossa
(E) Stomach
Correct answer: D

53.The classic complete neck dissection for palpable adenopathy in the posterior
triangle ofthe neck includes removal of which of the following?
(A) The transverse process, C2–C4
(B) The spinal accessory nerve
(C) Both thyroid lobes
(D) The trapezius
(E) The vagus
Correct answer: B

54. A 40-year-old woman is suspected of having a carotid body tumor. Which one of
the following is most characteristic of such a tumor?
(A) They secrete catecholamines.
(B) They are more common at sea level.
(C) They arise from structures that respond to changes in blood volume.
(D) They arise from the structures that respond to changes in PO2.
(E) They are usually highly malignant.
Correct answer: D

55. Following surgical resection of a large thyroid mass, a patient complains of


persistent hoarsenessand a weak voice. What is the most likely cause of these
symptoms?
(A) Traumatic intubation
(B) Prolonged intubation

- 406 -
(C) Injury to the recurrent laryngeal nerve
(D) Injury to the superior laryngeal nerve
(E) Scar tissue extending to the vocal cords
Correct answer: C

56.All true except adamantinoma:


a) x-ray honey comb appearance
b) pathologically mimic b.c.c
c) commom in females
d) can cross symphysismenti
Correct answer: D

57. all true except osteoclastoma:


a) fluid inside chicklet like
b) x ray appearance soap bubble appearance
c) common in children
d) pathologically giant cell tumor
Correct answer: C

58. Medullary carcinoma of thyroid arises from ?


a- parafollicular cells
b- cells lining the acini
c- capsule of thyroid
d- stroma of the gland
Correct answer: A

59. Which of the following is used in the management of thyroid malignancy?


a- Iodine 131
b- Iodine 125
c- Technetium 99
d- Phosphorus 32
e- strontium
Correct answer: A

60.Hurthle cells are seen in?


a- Hashimoto’s thyroiditis
b- follicular cell carcinoma
c- hurtle cell thyroid adenoma
d- all the above
Correct answer:D

61. Treatment of medullary carcinoma of the thyroid with lymphnode metastasis?


a- subtotal thyroidectomy + radioiodine
b- subtotal thyroidectomy + radiotherapy
c- near total thyroidectomy + radioiodine
d- near total thyroidectomy + radiotherapy
e- total thyroidectomy + radiotherapy
Correct answer: E

- 407 -
62. Thyroglossal cyst may occasionally give rise to which carcinoma ?
a- papillary
b- anaplastic
c- medullary
d- follicular
Correct answer: A

63. HURTHLE cell carcinomas are derived from which cells ?


a- follicular cells
b- para follicular cells
c- oxyphilic cells
d- lymphocytes
Correct answer: C

64.The malignancy which is common on long standing goiter ?


a- PTC
b- MTC
c- FTC
d- Anaplastic
Correct answer: C

65. Cancer common in iodine deficient areas ?


a- papillary
b- medullary
c- anaplastic
d- follicular
e- C+D
Correct answer: E

66. Least malignant thyroid cancer ?


a- papillary
b- medullary
c- anaplastic
d- follicular
Correct answer: A

67. Which of the following gene defects is associated with development of the
medullary carcinoma of the thyroid ?
a- RET proto oncogene
b- FAP gene
c- RB gene
d- BRCA 1 gene
Correct answer: A

68. RET prota-oncogene encodes for ?


a- tyrosine kinase receptor on the cytoplasmic membrane
b- protein kinase
c- IP3
d- All the above
Correct answer: A

- 408 -
69. RET gene is located on which chromosome ?
a- 5th chromosome.
b-13chromosome.
c-10thchromosome.
d.17 chromosome.
Correct answer: C

70. Rearrangement of RET gene leads to ?


a- PTC
b- MTC
c- FCT
d- Lymphoma
Correct answer: A

71. Psammoma bodies are seen in all of the following conditions except ?
a- serous cystadenoma of ovary
b- papillary carcinoma of thyroid
c- meningioma
d- mucinouscystadenoma of ovary
Correct answer: D

72.Amyloid stroma is seen in which carcinoma of thyroid?


a-PTC.
b-lymphoma.
c-MTC.
d-FTC.
Correct answer: C

73. Lateral aberrant thyroid refers to ?


a- congenital thyroid abnormality
b- metastatic foci from primary in the thyroid
c- strumaovarii
d- lingual thyroid.
Correct answer: B

74. a patient presented with headache and flushing . he has a family history of his
relative having died of a thyroid tumor. The investigation that would be required for
this patient will be ?
a- chest x-ray
b- measurement of 5-HIAA
c- measurement of catecholamine
d- intravenous pyelography
Correct answer: C

75. Cancer that develops after irradiation ?


a- PCT
b- MCT
c- FCT
d- Anaplastic
Correct answer: A

- 409 -
76. Mutated p53 gene is formed in most of ?
a- anaplastic carcinomas
b- PCT
c- MCT
d- FCT
Correct answer: A

77. The most precise diagnostic screening procedure for differentiating benign thyroid
nodules from malignant ones is:
A. Thyroid ultrasonography.
B. Thyroid scant scan.
C. Fine-needle-aspiration biopsy (FNAB).
D. Thyroid hormone suppression.
Correct answer: C

78. The preferred operation for initial management of a thyroid nodule that is
considered suspicious for malignancy by FNAB is:
A. Excision.
B. Partial lobectomy.
C. Total lobectomy and isthmectomy.
D. Total thyroidectomy.
Correct answer: C

79. A familial form of medullary thyroid carcinoma (MTC) should be suspected


whenever:
A. The tumor is multifocal.
B. The tumor is bilateral (foci of tumor are present in both thyroid lobes).
C. Pathologic examination of the resected thyroid gland reveals the presence of
C-cell hyperplasia in areas of the gland adjacent to foci of MTC.
D. All of the above.
Correct answer: D

80. All of the following are components of the MEN type 2B syndrome except:
A. Multiple neuromas on the lips, tongue, and oral mucosa.
B. Hyperparathyroidism.
C. MTC.
D. Pheochromocytoma.
Correct answer: B

81. MEN 2A and MEN 2B syndromes are associated with germline mutations in:
A. The p53 tumor suppressor gene.
B. The H-ras gene.
C. The N-myc gene.
D. The RET prota-oncogene.
Correct answer: D

82. Radioactive iodine is effective treatment for metastatic lung disease for which of
the following thyroid neoplasms?
a. Hürthle cell carcinoma
b. Papillary carcinoma

- 410 -
c. Follicular carcinoma
d. B+C
e. Medullary carcinoma
f. Anaplastic carcinoma
Correct answer: D

83. A 45-year-old woman has a solitary, nonfunctioning thyroid nodule and fine
needle cytology is non-diagnostic. Which of the following is the initial surgical
procedure of choice?
a. Total extracapsular thyroidectomy
b. Subtotal thyroid lobectomy and resection of the isthmus
c. Total extracapsular thyroid lobectomy, resection of the isthmus, and modified
unilateral neck dissection
d. Total extracapsular thyroid lobectomy and resection of the isthmus
Correct answer: D

84. Which of the following statements regarding medullary carcinoma of the thyroid
are true?
a. Approximately 75% of all cases are hereditary
b. The overall 10-year survival rate is less than 10%
c. Medullary carcinoma of the thyroid is associated with both multiple endocrine
neoplasia IIa (MEN IIa) and multiple endocrine neoplasia IIb (MEN IIb) syndromes
d. Prophylactic total thyroidectomy is recommended for MEN IIa and MEN IIb
patients after the age of 10 years
Correct answer: C

85. A 50-year-old male has undergone an ipsilateral thyroid lobectomy and isthmus
resection for what appeared on frozen section to be a benign nodular lesion 2.0 cm in
diameter. 72 hours later the final pathology returns and the diagnosis is a high-grade
angioinvasive follicular carcinoma. What do you recommend?
a. 99mTc bone scan to rule out occult bone metastases
b. Ipsilateral radical neck dissection
c. Total thyroidectomy
d.Observation with sequential 131I scans every 3 months
Correct answer: C

86. Multiple Endocrine Neoplasia (MEN) 2b is characterized by which of the following


findings?
a. Medullary carcinoma of the thyroid, pheochromocytoma, mucosal neuromas, and a
distinctive marfanoid habitus
b. Parathyroid hyperplasia, pancreatic islet cell tumors, and pituitary adenomas
c. Medullary carcinoma of the thyroid, pheochromocytoma, and parathyroid
hyperplasia
d. Parathyroid carcinoma, pheochromocytoma and chronic pancreatitis
Correct answer: A

87. Which of the following signs/symptoms are pathognomonic of


hyperparathyroidism?
a. Pathologic fractures of the metacarpals
b. Calcium oxalate nephrolithiasis

- 411 -
c. Hypercalcemia causing mental status changes
d. Atrophy of Type II muscle fibers
e. Osteitisfibrosacystica
Correct answer: E

88. Hyperthyroidism can be caused by all of the following except:


A. Graves' disease.
B. Plummer's disease.
C. Struma ovarii.
D. Hashimoto's disease.
E. Medullary carcinoma of the thyroid.
Correct answer: E

89 . First step taken in case of Multiple injuries of face and neck -


A. Blood transmission
B. IV fluids
C. Reconstruction
D. Maintainence of airways
Correct answer: D

90 . Secondary metastasis to neck-most common primary site is-


A. Nasopharyngeal carcinoma
B. Carcinoma of lung
C. Breast carcinoma
D. Soft palate carcinoma
Correct answer: A

91 . A 5-year-old patient has a midline neck mass that moves with swallowing and is
located just the hyoid. The mass is most likely to be a -
A. Branchial cleft cyst
B. Cystic hygroma
C. Teratoma
D. Thyroglossal duct cyst
Correct answer: D

92 . Structure preserved in functional neck dissection is -


A. Carotid artery, Vagus nerve
B. Strenomastoid muscle, Internal Jugular vein
C. Spinal accessory nerve, Submandibular salivary gland
D. Neck nodes
Correct answer: B

93 . Which is used to define Penetrating Neck Injury?


A. Injury to vital structures
B. 2 cm depth of wound
C. Through and through wound
D. Breach of platysma
Correct answer: D

- 412 -
94 .Cystic hygroma in the neck -
A. Develops from sequestration of jugular lymph channels
B. Is lined by stratified epithelium
C. Is nontransilluminant
D. Is lined by columnar epithelium
Correct answer: A

95 . Treatment of T4 N0 M0 stage of head and neck carcinoma is ?


A. Surgery alone
B. Radiotherapy alone
C. Chemoradiation
D. Surgery and Radiotherapy
Correct answer: D

96 . Structures preserved in Radial Neck Dissection is:


A. Sternocleidomastoid
B. Internal Jugular Vein
C. Vagus nerve
D. Submandibular gland
Correct answer: C

97 . In post radical neck dissection shoulder syndrome, are all seen except -
A. Shoulder drooping
B. Normal electromyographic finding
C. Restricted range of movement
D. Pain
Correct answer: B

98 . Prineural invasion in head and neck cancer is most commonly seen in-
A. Adenoid cystic carcinoma.
B. Adenocarcinoma.
C. Squamous cell carcinoma.
D. Basal Cell Adenoma.
Correct answer: A

99-Laryngocele is common to
a) Trumpet blowers
b) Glass blowers
c) Patient of chronic cough
d) All of the above
Correct answer : D

100-tuberculous cervical lymphadenitis commonly affects


a) Posterior triangle nodes
b) Upper deep cervical nodes
c) lower deep cervical nodes
d) submaxillary nodes
Correct answer : B

- 413 -
101-of the 800 lymph node of the body lymph nodes in the neck is aroud
a) 100
b) 200
c) 300
d) 400
Correct answer : C

102-in radical neck dissection all the following are removed except
a) Accessory nerve
b) Internal jugular vein
c) Carotid artery
d) Submandibular gland
e) Sternomastoid muscle
Correct answer : C

103-cystic hygroma is a
a) Haemangioma
b) Sebaceous gland tumour
c) Meningioma
d) Lymphangioma
Correct answer : D

104 . True about Lymphangioma is ?


A. Common in puberty
B. Respond in low doses to radiotherapy
C. Lymphangioma progress slowly and may invade local tissue
D. Predispose to cancers
Correct answer: C

105- Which of the following is brilliantly translucent


a) Sebaceous cyst
b) Dermoid cyst
c) Cystic hygroma
d) branchialcyst
Correct answer : C

106-What is wrong about Hodgkin lymphoma


a) Better prognosis than NHL
b) Few malignant cells outnumbered by inflammatory cells
c) Characterized by reed Sternberg cells
d) Diagnostic laparotomy is essential
Correct answer : D

107-most commom site of lymph nodes enlargement in Hodgkin lymphoma is


a) Abdominal
b) cervical
c) Axillary
d) Mediastinal
Correct answer : B

- 414 -
30- ORAL CAVITY

1. A T2 malignant tumor of the oral cavity near the mandible is best treated
with .
A.Radiotherapy
B.Chemotherapy
C.Surgery
D.Laser ablation
Correct answer C

2 . A 22years male presented with chronic constipation,headache and


palpitations.On examination he had marfanoid habitus,neuromas of tongue
,medullated corneal nerve fibers and nodule of 2x2 cm size in the left lobe of
thyroid gland.The possible diagnosis is:
A. MEN IIA
B. MEN IIB
C. Sporadic medullary carcinoma of thyroid
D. Familial medullary carcinoma of thyroid
Correct answer: B

3 . Most frequent site of mandibular fracture is -


A. Condylar neck
B. Canine fossa
C. Mandibular body
D. Symphysis mentii
Correct answer: A

4 . The commonest pre-malignant condition of oral cancer is


A. Leukoplakia
B. Aphthous ulcer
C. Lichen planus
D. Erythra-leukoplakia
Correct answer: A

31- ENT
1.Bezold's abscess is seen at
A.petrous tip
B.Mastoid tip
C.digastric triangle
D.subdurally
Correct answer B

2. Which of the following is not a feature of Rhinosporidiosis


A.Bleeding polyp
B.Russel bodies are seen
C.Oral Dapsone is useful in treatment

- 415 -
D.bleeding is present
Correct answer B

3. In a patient with hypertrophied adenoids the voice abnormality that is seen is


A. rhinolalia clausa
B. rhinolalia aperta
C. hotpotato voice
Correct answer A

4. Abductor of the vocal cord is


A.lateral cricoarytenoid
B.cricothyroid
C.posterior cricoarytenoid
Correct answer C

5. Cottel's test is used to test the patency of the nares in


A.Atrophic rhinitis
B.Rhinosporidiosis
C.Deviated nasal septum
D.Hypertrophied inferior turbinate
Correct answer C

6. Which of the following is not seen in Menier's disease


A.vertigo
B.tinnitus
C.conductive deafness
Correct answer C

7. Which of the following ECG finding is associated with congenital deafness


A.delta wave
B.long QT
C.short PR interval
Correct answer B

8. Which among the following is not true about otosclerosis


A.hearing better in louder conditions
B.Eustachian tube is always abnormal
C.tympanic membrane is normal
D.bilateral progressive conductive deafness
Correct answer B

9. For Grommet tube insertion in serous otitis media, the incision on tympanic
membrane is made on
A.anteroinferior quadrant
B.anterosuperior quadrant
C.posterosuperior quadrant
D.posteroinferior quadrant
Correct answer A

- 416 -
10. What is the symptom of a person with unilateral abductor paralysis
A.dyspnea on exertion
B.transient hoarseness
C.husky voice
D.inspiratory stridor
Correct answer B

11. Regarding acute vestibular neuronitis all are true except


A.tinnitus and vertigo developing over 1-2 weeks
B.high frequency sensory hearing loss
C.spontaneous nystagmus is present
D.usually follows an upper respiratory tract infection
Correct answer B

12. Most common complication of tracheostomy is


A.pneumothorax
B.surgical emphysema
C.injury to the large vessels
D.injury to esophagus
Correct answer B

13. Which substances is found in CSF but not simple rhinorhea?


a) glucose
b) lactate
c) acid
d) base
e) galactose
Correct answer: A

14. Rhinopharyngitis motilins is not seen in


A.Yaws
B.Pinta
C.Leprosy
D.Kala azar
Correct answer B

32- EYE

1. Cherry red spot in retina is a feature of


A.Infantile Gaucher's disease
B.Niemann Pick disease
C.Fabry's disease
Correct answer B

2. Preganglionic and post ganglionic Horner's syndrome are differentiated by


application of
A.Atropine
B.Pilocarpine
C.Timolol

- 417 -
D.Hydroxy amphetamine
Correct answer D

3. Axial proptosis is caused by


A. blow out fracture of the orbit
B. maxillary tumor invading the orbit
C. frontal mucocele
D. optic glioma
Correct answer D

4. Which of the following is used for treatment of myopia


A. Nd YAG laser
B. Excimer laser
C. Argon laser
D. Holmium laser
Correct answer B

5. Sun flower cataract is seen in


A.Blunt trauma
B.Diabetes
C.Chalcosis
D.Argyriosis
Correct answer C

6. Steroid may be used in the treatment of which of the following type of herpes
keratitis
A. Superficial punctate keratitis
B. Dendritic ulcer
C. Geographic ulcer
D. Disciform keratitis
Correct answer D

7. Which of the following is the cause of blindness in partial retinal branch vein
occlusion
A. Cystoid macular edema
B. Macular hemorrhage
C. Vitreous hemorrhage
Correct answer A

8. Miotics are useful in which type of squint


A. Accomodative squint
B. Paralytic squint
C. Divergant squint
D. Congenital squint
Correct answer A

9. Extra capsular extraction of lens is not possible in


A. hypermature cataract
B. immature cataract
C. lens subluxation

- 418 -
D. developmental cataract
Correct answer C

10. KF ring in Wilson's disease is seen at the level of


A. Deschmet's membrane
B. epithelium
C. endothelium
D. stroma
Correct answer A

11. Treatment of congenital dacryocystitis is


A.dacryocystectomy
B.hydrostaticmassage
C.probing and syringing with antibiotics
D.dacryocystorhinostomy
Correct answer B

12.Cattle truck appearance on fundus examination is a feature of


A.CRVO
B.Incomplete CRAO
C.Incomplete CRVO
D.Hypertensive retinopathy
Correct answer B

13.Whorled keratopathy is seen with the use of


A. Digoxin
B. Amiodarone
C. Ethambutol
Correct answer B?

33- soft tissue

1. A43-year-old window cleaner fell off a scaffold. He sustained an open wound on the
right leg. Debridement was carried out in the emergency department, and the edges of
the wound were left open. The wound measures 4 cm x 6 cm. What is TRUE of wound
contraction?
(A) It occurs within 12 hours of injury.
(B) It is more prominent over the tibia than gluteal region.
(C) It is accelerated if wound is excised 3 days after injury.
(D) It accounts for excessive fibrous tissue formation and fixation of tissue around a
joint.
(E) It is experimentally less affected by excision of tissue from center of wound rather
than at the periphery.
Correct answer: E

- 419 -
2. Which factor is least likely to inhibit wound contraction?
(A) Radiation
(B) Cytolytic drug
(C) Transformation growth factor b
(D) Full-thickness skin graft
(E) External splints
Correct answer: E

3. A 43-year-old male undergoes a total proctocolectomy for ulcerative colitis. The


terminal ileum is brought out on the anterior abdominal wall as an end (Brooks)
ileostomy. What is necessary to obtain optimal healing?
(A) The ileostomy should be circular rather than square.
(B) The seromuscular layer is sutured to the epithelium of the skin to avoid
inflammatory changes.
(C) The ileostomy must be constructed to avoid fixing the mesentery.
(D) The mesentery of the ileal loop should be widely cut to increase its mobility.
(E) The ileostomy must be constructed on the right side.
Correct answerA

4. A12-year-old boy has multiple skin lesions that are diagnosed as von
Recklinghausen’s syndrome (NF 1). What is TRUE of this condition?
(A) It does not show other malignant lesions.
(B) It is autosomal recessive.
(C) It is associated with optic nerve gliomas.
(D) It is characterized by AV malformation.
(E) It is associated with dermoid
Correct answer: C

5- When the swelling is deep to the muscle:


a) Will disappear during contraction.
b) Will keep the same size during contraction.
c) Will become more prominent during contraction.
d) Will show transmitted pulsation.
Correct answer: A

6- Where is the "Gaiter" area:


a) Just above the lateral malleolus.
b) Front of the shaft of the tibia.
c) 2,4,6 inches above medial malleolus.
d) Back of the leg on the calf muscle.
Correct answer: C

7. Congenital dermoid cysts are characterized the following features


(except) that they:
a. Occur at lines of embryological fusion.
b. Are commonest on the face.
c. Are often pitched to the overlying skin.
d. May Couse hollowing of subjacent bone.
e. Are lined by stratified squamous epithelium.
Correct answer: c

- 420 -
8. Sebaceous cyst is characterized by the following except that it:
a. ls due to obstruction of a sebaceous gland.
b. ls lined by stratified squamous epithelium.
c. Contains yellow poultices greasy material known as sebum.
d. Moy occur on the palms and soles
e. ls always anchored to the overlying skin at punctum of the obstructed gland.
Correct answer: d

9. The untrue statement about desmoid tumors of the abdominal wall is that they
:
a. Arise from the rectus muscle or its sheath.
b. Occur most often in multiparous women.
c. Are locally invasive.
d. May give rise to metastasis.
e. Should be treated by wide local excision
Correct answer: D

10. Condyloma acuminatum is produced by which type of papilloma virus


A.HPV 6-16
B.HPV 6-11
C.HPV 16-18
D.HPV 11-18
Correct answer: B

11. Herald patch is seen in


A.pityriasis rosea
B.lichen planus
C.DLE
Correct answer: A

12. Which of the following is not a pyoderma


A. Furuncle
B. Ecthyma
C. Pyoderma gangrenosum
D. Carbuncle
Correct answer: C

13.Treatment of choice for disseminated gonococcal infection is


A. penicillin
B. ceftriaxone
Correct answer: B

14. Epidemic tenia capitis is caused by


A. Trichopyton mentagrophytes
B. Trichophyton rubrum
C. Microsporum audoni
Correct answer: C

- 421 -
15. Scabies in children differs from that in adults in that it affects
A. webspace
B. face
C. genitalia
D. axilla
Correct answer: B

16. A smear from a genital lesion shows Donovan bodies. The diagnosis is
A. Lymphogranuloma venereal
B. Chancroid
C. Granuloma inguinal
Correct answer C

17. Photosensitivity is seen in all except


A. SLE
B. Xeroderma pigmentosa
C. Porphyria cutanea tarda
D. Pseudoxanthoma elasticum
Correct answer D

18 . A patient presents with malena, hyperpigmentation over lips, oral mucosa


and skin; and his sister is also having similar complaints. The diagnosis is:
A. Gardner's Syndrome
B. Villous Adenoma
C. Peutz-Jegher's Syndrome
D. Familial Adenomatous Polyposis
Correct answer: C

19 . Commonest malignancy in renal transplant recipient is -


A. Skin cancer
B. Renal cell carcinoma
C. Non Hodgkin’s lymphoma
D. Hodgkin’s lymphoma
Correct answer: A

20 . Bowen's disease of the skin is


A. A premalignant intradermal condition
B. A tumor of sweat glands
C. A complication of a sebaceous cyst
D. A type of dermatitis
Correct answer: A

21 . In which of the following, multiple Cutaneous Sebaceous adenomas are seen?


A. Turcot's syndrome
B. Gardner's syndrome
C. Cowden syndrome
D. Muir Torre syndrome
Correct answer: D

- 422 -
22 . A keratin horns
A. Arises from sebaceous cyst
B. Is a papilloma with excess keratin formation
C. Is due to matting of hair
D. Is congenital
Correct answer: A

23 . A 35 year old premenopausal patient has as recently developed a 1.5 cm


sized pigmented lesion on her back. Which of the following forms of tissue
diagnosis will be recommend for her ?
A. Trucut biopsy
B. Needle biopsy
C. Incisional biopsy
D. Excision biopsy
Correct answer: D

24 . All are true about skin grafting, except:


A. Full thickness graft includes epidermis, dermis, without subcutaneous tissue
B. Partial thickness graft involves epidermis and part of dermis
C. Full thickness graft has cosmetic value
D. For large areas full thickness graft is used
Correct answer: D

25 . Best procedure to be done after an injury to leg associated with exposure of


underlying bone and skin loss -
A. Pedicle flap
B. Split skin grafting
C. Full thickness grafting
D. Skin flap
Correct answer: A

26 . A patient comes with rectal carcinoma situated 6 cm above dentate line with
no nodal metastasis The treatment of choice will be -
A. Ant resection
B. APR
C. Radiotherapy
D. Hartman's procedure
Correct answer: A

27. A12-year-old boy has multiple skin lesions that are diagnosed as von
Recklinghausen’s syndrome (NF-1). What is TRUE of this condition?
(A) It does not show other malignant lesions.
(B) It is autosomal recessive.
(C) It is associated with optic nerve gliomas.
(D) It is characterized by atrioventricular (AV) malformation.
(E) It is associated with dermoid
Correct answer: C

- 423 -
28 . Most common tumor to produce metastasis to cervical lymph nodes
A. Glottic Ca
B. Nasopharyngeal carcinoma
C. Ca Base of tongue
D. Ca lip
Correct answer: B

29 . Which of the following is the most common cause of isolated splenic


metastasis?
A. Carcinoma stomach
B. Carcinoma pancreas
C. Carcinoma cervix
D. Carcinoma ovary
Correct answer: D

30 . Predominantly osteoblastic metastasis is seen in ?


A. Renal cell carcinoma
B. Prostate carcinoma
C. Breast carcinoma
D. Thyroid carcinoma
Correct answer: B

31 . Metastasis of ca buccal mucosa goes to -


A. Regional lymph node
B. Liver
C. Heart
D. Brain
Correct answer: A

32 . Which of the following causes second degree upper cervical lymph node
metastasis:
A. Nasopharyngeal carcinoma
B. Breast carcinoma
C. Lung carcinoma
D. Thyroid carcinoma
Correct answer: A

33 . MOST COMMON site for secondary metastasis is a case of Hypernephroma


A. Adrenal
B. Lungs
C. Brain
D. Bones
Correct answer: B

34- GYNAE and OBSTATRIC

1. Which of the following is the commonest cause of urethritis with leucorrhea


A. Chlamydia trachomatis
B. H.vaginalis

- 424 -
C. Trichomonas
D. Candida
Correct answer A

2. A pregnant women presents with a placenta praevia of a major degree. The foetus is
malformed. Which of the following will be the best line on management
A. cesarian section
B. oxytocin drip
C. rupture of membranes
D. instillation of PG E2
Correct answer A

3. Transverse lie is most likely to occur in


A. subseptate uterus
B. uterus didelphys
C. hypoplastic uterus
D. unicornuate uterus
Correct answer A

4. Osiander's sign means


A. Pulsation in the lateral vaginal fornix
B. Bluish colour change in the vagina
C.Softening of the cervix
D.On bimanual palpation the fingers can be approximated as if nothing is in between
Correct answer A

5. Which of the following does not cross placenta


A.heparin
B.propranolol
C.warfarin
Correct answer A

6. Magnesium sulphate as an uterine relaxant is contraindicated in following except


A.premature rupture of membrane
B.intrauterine infection
C.myasthenia gravis
D.renal failure
Correct answer B

7. Anti epileptic agent given in pregnancy is


A. phenytoin
B. sodium valproate
C. carbamazepine
D. lamotrigine
Correct answer D

8. A pregnant woman with Mitral stenosis is most likely to undergo failure in which
period
A.16wks
B.24wks

- 425 -
C.20wks
D.28wks
Correct answer D

9. CA-125 is the tumor marker for


A. colon
B. ovary
Correct answer B

10. Incidence of scar rupture in a pregnant lady with previous LSCS is


A. 0.2
B. 0.5
C. 0.7
D. 0.9
Correct answer

11. Karyotype of Turner's syndrome is


A. XO
B. XX
C. XXY
D. XY
Correct answer A

12. What is true about shoulder presentation


A. Cord prolapse is common
B. 3rd degree perineal tear is common
C. Deep transverse arrest can occur
D. More common in prime
Correct answer

13. Variable deceleration indicates


A.head compression
B.cord compression
C.fetal hypoxaemia
D.maternal sedation
Correct answer B

14. Regarding monozygotic twins which of the following is not true


A. USG is more useful in the first half of pregnancy
B. sex discordance can occur rarely
C. always monochorionic
D. more common following ovulation induction
Correct answer C

15. The placenta is formed from


A.decidua basalis
B.decidua vera
C.chorion levae
D.decidua capsularis

- 426 -
Correct answer A

16. The level of hCG is maximum during which day of pregnancy


A.50-60
B.60-70
C.70-80
D.80-90
Correct answer B

17. Failure rate of tubal sterilization is one in


A. 100
B. 200
C. 500
D. 1000
Correct answer B

18. Which cervical fibroid is most likely to cause urinary retention


A.Anterior
B.Posterior
C.Submucous
D.Subserous
Correct answer A

19. Fetal lung maturity is assessed by all the following except


A. Lecithin sphingomyelin ratio
B. Nile blue sulfate test
C. Nitrazine paper test
D. Creatinine estimation of amniotic fluid
Correct answer C

20. If a patient comes with complaints of post dated pregnancy what is the first thing
that you will do
A. USG
B. NST
C. Review the menstrual history once more
D. X-ray abdomen
Correct answer C

21. First trimester USG is not indicated in


A.Hydramnios
B.Hyper emesis gravidarum
Correct answer A

22. A 25-year-old woman is noted to have a solid and cystic right ovarian mass
measuring 10 cm on ultrasound. Which of the following is the most likely histologic
subtype?
A. Serous
B. Mucinous
C. Brenner

- 427 -
D. Teratoma
E. Fibroma
Correct answer: D

23. Paracusis willisi is seen in


A.stapedial fixation
B.ASOM
Correct answer A

24. Mesonephric duct remnant seen in anterolateral vaginal wall is


A. Gartner's cyst
B. endometriotic cyst
C. Inclusion cyst
Correct answer A

25. Which of the following is NOT a risk factor for cervical cancer?
a. Multiple sexual partners
b. Condom usage
c. HPV infection
d. HIV infection
e. Smoking
Correct answer: B

26. Usually, the first treatment for ovarian cancer is...


A. Surgery
B. Chemotherapy
C. Radiation
D. Any of the above
Correct answer: A

27. A 58-year-old woman is noted to have bilateral adnexal masses on physical


examination. Which of the following is most suggestive of these adnexal masses being
malignant?
A. CT imaging revealing that they are primarily cystic
B. Elevation of the serum alkyline phosphatase level
C. Family history of lung cancer
D. The presence of ascites
E. The presence of low-grade fever
Correct answer: D

28. A 44-year-old woman undergoes an exploratory laparotomy for suspected ovarian


cancer. Upon removal of the right ovary, a frozen section reveals “signet ring” cells.
Which of the following is the most likely etiology?
A. Dysgerminoma
B. Metastatic
C. Mucinous
D. Serous
E. Teratoma
Correct answer: B

- 428 -
29. A 4-year-old girl is noted to have breast enlargement and vaginal bleeding. On
physical examination, she is noted to have a 9-cm pelvic mass. Which of the
following is the most likely etiology?
A. Cystic teratoma
B. Dysgerminoma
C. Endodermal sinus tumor
D. Granulosa cell tumor
E. Mucinous tumor
Correct answer: D

30. Which is not a normal physiological change in pregnancy?


a. Heart rate increase 10-20 bpm.
b. Tidal volume increase 40%
c. Respiratory alkalosis
d. Residual volume increased by 25%
e. Hct decreased to 30-35%.
Correct answer: D

35- Lymphoma and LN

(1) Common lymphomas in pediatric age group include all except


1. Anaplastic large cell lymphoma
2. Burkitt’s lymphoma
3. Follicular lymphoma
4. Diffuse large B-cell lymphoma
Correct answer: 3

(2) Follicular lymphomas in childhood differ from adult-onset disease in all except:
1. Bcl-6 expression
2. Prognosis
3. Treatment
4. Pathogenesis
Correct answer: 1

(3) All are usually expressed in pediatric follicular lymphomas except:


1. CD20
2. Bcl-2
3. Bcl-6
4. CD10
Correct answer: 2

(4) Follicular lymphomas in children should be treated with


1. Chemotherapy
2. Surgical resection of disease
3. Surgical resection of disease and chemotherapy
4. Surgical resection of disease, chemotherapy and radiotherapy
Correct answer: 2

- 429 -
(5) Most common site of myoepithelial carcinoma is-
1. Submandibular salivary gland
2. Parotid gland
3. Sartorius muscle
4. Breast
Correct answer: 2

(6) The characteristic pathology of myoepithelial carcinoma is-


1. Non-encapsulated with trabeculae of spindle-shaped cells without vascularisation
2. Encapsulated with trabeculae of synovial cells with mitosis
3. Non-encapsulated with trabeculae of spindle-shaped cells with mitosis
4. Encapsulated with trabeculae of spindle-shaped cells with vascularisation
Correct answer: 4

(7) Myoepithelial carcinoma is immunoreactive for


1. S-100, CD1a and p53
2. EMA, desmin and fibrin
3. Cytokeratin, S-100 and calponin
4. CD20, CD3 and vimentin
Correct answer: 3

(8) Which one of the following is the characteristic radiological feature of myoepithelial
carcinoma
1. Low T1 and low T2 signal intensity and indistinct margin on MRI scan
2. Low T1 but high T2 signal intensity with distinct margin on MRI scan
3. Low T1 and low T2 signal intensity and distinct margin on MRI scan
4. Low T1 but high T2 signal intensity with indistinct margin on MRI scan
Correct answer: 1

(9) What criterion is customarily used to define mediastinal lymph nodes visualized by
computed tomography (CT) as being pathologic?
1. >1 cm on short axis
2. >1 cm on long axis
3. 2.5 cm on short axis
4. >2.5 cm on long axis
Correct answer: 1

(10) In patients with NSCLC, what proportion of patients with enlarged nodes on CT have no
evidence of neoplastic disease at surgical exploration of the mediastinum?
1. 5%
2. 15%
3. 30%
4. 45%
Correct answer: 3

(11) Which of the following most accurately characterizes the value of magnetic resonance
imaging (MRI) in comparison with CT in evaluation of mediastinal lymph nodes?
1. More sensitive
2. More specific
3. Less effective
4. Similar outcomes

- 430 -
Correct answer: 4

(12) A 50 year old man presented with 3 month history of right sided cervical lymphnode
enlargements.The best investigation to establish the diagnosis in this case would be:
A. Tuberaulin test
B. Lymph-node biopsy
C. FNAC
D. X-ray
Correct answer: B

36- GIT

1- As regard Meckel's diverticulum all are correct except:


a- lt occurs due to persistence of proximal urachus
b- lt arises from antimesenteric border
c- lt lies 60 cm (2 feet) from coecum
d- lt's 2 inches long
Correct answer: D

2-indications of surgical removal in accidentally discovered Meckel's


during laparotomy include the following except:
a- Narrow mouthed
b- Adhesions
c- Diabetics
d- Young age of patient
Correct answer: B

3- Most common complication of Meckel's is:


a- Diverticulitis
b- lntestinal obstruction
c- Peptic ulceration
d- Littre's hernia
Correct answer: C

4- Meckel's diverticulum most commonly presents by:


a. Gastrointestinal bleeding.
b. Obstruction.
c. Diverticulitis.
d. lntermittent abdominal pain.
Correct answer: C

- 431 -
5-The following are radiologic finding ln TB enteritis:
a- Sterlin's sign is characteristic of ulcerative type
b- Hypertrophic type always show distortion and elevation of coecum
c- Hypertrophic type always show narrow ileum
d- None of the above
Correct answer: D

6-The common sites of carcinoid tumor in Gl tract are all except:


a. Appendix.
b. Jejunum.
c. lleum.
d. Rectum
Correct answer: B

7-True statements about chronic duodenal ulcers do not lnclude that they:
a- Are never malignant.
b- Occur equally in both sexes.
c- Produce epigastric pain several hours after eating.
d- Hove a periodic clinical course.
e- May heal under medical treatment.
Correct answer: B

8. The cardinal symptoms of uncomplicated duodenal ulcer do not include: -


a. Anorexia.
b. Localized mid epigastric pain.
c. Hunger pain.
d. Nocturnal pain.
e. Periodic remissions and exacerbations.
Correct answer: A

9- Perforated duodenal ulcer is best treated by:


a. Gastroduodenal suction and antibiotics.
b. Simple closure over on omental patch.
c. Truncal vagotomy.
d. Super selective vagotomy.
e. Partial gastrectomy.
Correct answer: B

10- Among the following statements about Crohn's disease, lt is untrue that it :
a. ls commonest in the third decode.
b. Always involves the terminal ileum,
c. ls a segmental granulomatous lesion affecting all coots of the bowel
and the related mesentery.
d. May be associated with skip areas and anorectal lesions.

- 432 -
e. Should be managed medically whenever possible.
Correct answer: B

11- The most frequent complication of regional ileitis ls


a. Abscess formation.
b. internal fistulae.
c. Hemorrhage.
d. Perforation.
e. intestinal obstruction.
Correct answer: B

12- The least common site for cancer in the alimentary tract is:
a. Pharynx.
b. Esophagus.
c. Stomach.
d. Small bowel.
e. Colon and rectum.
Correct answer: D

13- The following ore examples of strangulation except:


a. Mesenteric vascular occlusion.
b. Volvulus.
c. lntussusceptions.
d. Meconium ileus.
Correct answer: D

14- Common causes of secondary intussusception include the following except:


a. Small intestinal tumor.
b. Cancer sigmoid.
c. Meckel's diverticulum.
d. Henoch shonlein purpra.
Correct answer: B

15- Conservative treatment is indicated in early uncomplicated in the following cases


except:
a. Adhesive l.O.
b. Primary intussusception.
c. Secondary intussusception.
d. Volvulus.
Correct answer: C

16- As regard hydrostatic barium reduction of intussusception all are correct except:
a. Should be tried in all cases.
b. Successful in about half of cases.
c. Success is proved by filling of terminal ileum.
d. May be complicated by perforation.
- 433 -
Correct answer: A

17 . Most common benign tumor of small intestine is:


A. Adenoma
B. Leiomyoma
C. Hamartoma
D. Fibroma
Correct answer: A

18- As regard Volvulus neonatorum all are correct except:


a. Never to occur without anomalies.
b. Usual presentation is screaming and non-bile-stained vomiting.
c. Bleeding per rectum is common.
d. Primary resection is usually incompatible with life.
Correct answer: B

19- The most common cause of death in low intestinal obstruction is:
a. Toxemia.
b. Dehydration.
c. Electrolyte imbalance.
d. Generalized peritonitis.
Correct answer: A

20- Colicky pain is absent in the following types of intestinal obstruction:


a. Strangulated hernia.
b. Volvulus.
c. Paralytic ileus.
d. lntussusception.
Correct answer: C

21- The following types of intestinal obstruction may present without absolute
constipation except:
a. Early cases of high intestinal obstruction
b. Gall stone ileus.
c. Mesenteric vascular occlusion.
d. Paralytic ileus.
e. Richter's hernia.
Correct answer: D

22- Signs of strangulated intestinal obstruction include:


a. Nasogastric suction does not relief pain.
b. Rebound tenderness.
c. Leucocytosis.
d. All of the obove.
Correct answer: D

- 434 -
23- The following are recognized causes of paralytic ileus. The most common is:
a. Diabetic ketoacidosis.
b. Drugs.
c. Peritoneal irritation.
d. Postoperative.
e. Spinal injury.
Correct answer: D

24- As regard meconium ileus:


a. Occur during first few days of neonatal life.
b. Due to inspissated meconium.
c. May be associated with cystic fibrosis.
d. All of the above.
Correct answer: D

25- The most common injured artery in mesenteric vascular occlusion is:
a. Celiac artery.
b. Gastroduodenal artery.
c. Superior mesenteric artery.
d. inferior mesenteric artery.
Correct answer: C

26- In children and adolescents, the commonest cause of intestinal obstruction is:
a. Adhesions
b. lntussusception
c. Strangulated hernias
d. Neoplasm
e. Paralytic ileus
Correct answer: C

27 . Commonest tumour of small intestine -


A. Leiomyoma
B. Lymphoma
C. Adenoma
D. Hemangioma
Correct answer: A

28- Uncomplicated meconeum ileus is best treated by:


a. Laparotomy and evacuation
b. Transverse colostomy
c. N-acetyl cysteine barium enema
d. Wait and watch
Correct answer: C

- 435 -
29- Example of third space sequestration is:
a. Burn
b. Fracture hematoma
c. Small bowel obstruction
d. All of the above
Correct answer: D

30- A 20-year-old man swallowed two open safety pins. X-rays show pins in the small
intestine, the most appropriate management of this point is:
a. lV antibiotics
b. 250 ml magnesium citrate orally
c. lmmediate surgery
d. Serial abdominal exam and x-rays if required
Correct answer: D

31-In generalized peritonitis, the symptomatology does not include:


a. Fever and tachycardia.
b. Severe colicky abdominal pain.
c. Diffuse tenderness and rigidity.
d. Silent abdomen on auscultation.
e. Shifting dullness.
Correct answer: B

32 . True about intestinal obstruction radio-graphically except


A. Small intestine dilation > 3 cm
B. Distal large intestine > 9 cm and proximal intestine > 3 cm
C. On lying supine fluid air gap absence does not signify obstruction
D. Volvulus shows a characteristic appearance on radiograph
Correct answer: B

33 . Commonest malignancy of the small intestine is


A. Leiomyoma
B. Lymphoma
C. Adenocarcinoma
D. Hemangioma
Correct answer: A

34 . In case of choledochal cyst, bile diversion into the small intestine is not done
because of the risk of:
A. Malignancy
B. Pancreatitis
C. Recurrent Cholangitis
D. Gall stones
Correct answer: C

- 436 -
37- PSYCHIATRY

1. Which of the following is not a classical symptom of depression


A.altered sleep pattern
B.weight loss
C.guilt
D.decreased apettite
Correct answer C?

2. Psychomotor symptoms are associated with which type of schizophrenia


A.hebephrenic
B.catatonic
C.paranoid
D.simple
Correct answer B

3. Which of the following is a first rank symptom in Schizophrenia


A.perceptional delusions
B.thought insertion
C.perceptional hallucinations
Correct answer B

4. Which of the following is not seen in Korsakoff's syndrome


A.clear consciousness
B.inabiliity to learn new things
C.hallucinations
D.confabulation
Correct answer C

5. Wernicke Korsakoff's syndrome is due to the deficiency of which of the following


A.Pyridoxine
B.Thiamine
C.Vitamin B12
D.Riboflavin
Correct answer B

6. The type of treatment which can be used for mental retardation is


A.Psychoanalysis
B.Family therapy
C.Transaction analysis
D.Rapid emotive treatment and response
Correct answer

7. deJavu phenomenon means


A.feeling that familiar things look unfamiliar
B.individual feels that he knows the surroundings even though he is new to it
Correct answer B

- 437 -
280 Others
38- Others

1. Validity of a screening test includes all except


A.sensitivity
B.specificity
C.repeatability
D.accuracy
Correct answer C

2.Which of the following is not sexually transmitted


A. Entamoeba histolytica
B. Giardia lamblia
C. Trichomonas vaginalis
D. Leishmania Brasiliense
Correct answer D

3. Which of the following is true about Botulism


A.high fever
B.brisk reflexes
C.salivation
D.sore throat
Correct answer D

4. Most common age group of benign cystic teratoma is


A. <10 years
B. 20-40 years
C. 40-60 years
D. >60 years
Correct answer

5. Minimum level of free chlorine in potable water is


A. 1mg
B. 0.5mg
C. 0.2 mg
D. 0.7mg
Correct answer B

6. Which of the following is true about Cohort study


A.incidence can be calculated
B.it is from effect to cause
C.it is inexpensive
D.shorter time than case control
Correct answer A

7. The size of sand particle in slow sand filter is


A. 0.1 -0.2 mm
B. 0.2-0.3 mm
- 438 -
C. 0.3-0.4 mm
D. 0.4-0.5 mm
Correct answer: B

8. What is the potassium concentration in WHO ORS solution


A. 10meq
B. 20meq
C. 30meq
D. 40meq
Correct answer: B

9 . In the acronym 'SWELLING' used for the history and examination of a lump
or swelling, the letter 'N' stands for -
A. Nodes
B. Noise (Thrill/bruit)
C. Numbness
D. Neurological effects
Correct answer: B

10. Most common hand infection is:


a. Distal pulp space infection (felon).
b. Paronychia.
c. Ulnar bursitis.
d. Thenar space infection.
Correct answer: B

11. The most desirable position to immobilize the hand:


a. Wrist is flexed, MCP joints are extended and lP joints are flexed.
b. Wrist is flexed. MCP joints are flexed and lP joints are extended.
c. Wrist is extended, MCP joints are extended and lP joints are flexed.
d. Wrist is extended. MCP joints are flexed and lP joints are flexed.
e. Wrist is extended, MCP joints are flexed and lP joints are extended.
Correct answer: E

12 . All are correct regarding pneumatosis cystoides except one -


A. Spontaneous regression is seen
B. Surgical resection indicated
C. May cause tension pneumoperitoneum
D. May cause severe bleeding
Correct answer: B

13 . Pseudo claudication is caused by


A. Popliteal Artery stenosis
B. Femoral Artery stenosis
C. Radial Artery stenosis
D. Lumber canal stenosis
Correct answer: D

- 439 -
14 . Which of the following is the most common site of cholangiocarcinoma?
A. Hilum
B. Distal biliary duct
C. Multifocal
D. Intrahepatic duct
Correct answer: A

15 . In the Forrest classification for bleeding peptic ulcer with a visible vessel it is
classified as -
A. FI
B. FI Ia
C. FII b
D. FII
Correct answer: B

16 . Accidental ligation of ureter may result in:


A. Kidney atrophy
B. Hydronephrosis
C. Acute renal failure
D. None of the above
Correct answer: B

17 . In a patient with diffusive liver disease, platelet count of less than 50000, the
procedure of choice for liver biopsy is -
A. Plugged liver biopsy
B. Laparoscopic liver biopsy
C. Trans jugular liver biopsy
D. Laparotomy and wedge biopsy
Correct answer: C

18 . A comatose patient with GCS 6, suffered a motor cycle accident with


intraventricular bleed. Next immediate step in management is
A. intubation and airway
B. immediate ventriculostomy
C. intraventricular catheter to monitor raised ICT and hyperventilation to reduce it
D. burr hole
Correct answer: A

19 . A patient presents with altered bowel habits thickening of terminal ileum,


caecum, dilatation of distal and proximal ileum, antimesenteric sacculation’s,
increased mesenteric fat, increased blood supply to adjacent mesentery. All are
true except -
A. Crohn's
B. Ulcerative colitis
C. Ischemic colitis
D. Ileocecal TB
Correct answer: C

- 440 -
20 . True about Warthin's tumor is ?
A. Pleomorphic adenoma
B. Cystadenoma lymphomatous
C. Most common parotid tumor
D. Malignant tumor
Correct answer: B

21 . A 35 years male patient presented with left sided abdominal pain to the
emergency room 6 hours after an RTA. He was hemodynamically stable and
FAST positive. Contrast Enhanced CT(CECT)scan showed grade III splenic
laceration. The most appropriate management is-
A. Splenorrhaphy
B. Splenectomy
C. Conservative Management
D. Splenic Artery Embolization
Correct answer: D

22 . In carcinoma cheek what is the best drug for single drug chemotherapy -
A. Vincristine
B. Cyclophosphamide
C. Cisplatin
D. Daunorubicin
Correct answer: C

23 . A 16 years male patient with ITP has a platelet count of 50,000 and is being
planned for splenectomy. The best time for platelet infusion in this patient is-
A. At the time of skin incision
B. 2 hours before surgery
C. Immediately after removal of spleen
D. After ligating the splenic artery
Correct answer: D

24 . In which of the following tumors alpha fetoprotein is elevated -


A. Choriocarcinoma
B. Neuroblastoma
C. Hepatocellular carcinoma
D. Seminoma
Correct answer: C

25 . The Gray hack shunt is established between


A. Corpora cavernosa and saphenous vein
B. Corpora cavernosa and corpora spongiosa
C. Corpora cavernosa and glans
D. Corpora cavernosa and dorsal vein
Correct answer: A

26 . Treatment of choice for squamous cell carcinoma of anus below dentate line
is ?
A. Chemotherapy
B. Wide local excision

- 441 -
C. Sphincter sparing surgery
D. Monoclonal antibodies
Correct answer: B

27 . The most acceptable method of treatment of esophageal perforation during


endoscopy, which is associated with a leak into pleural space, is -
A. Observation alone
B. Observation + antibiotics
C. Insertion of left chest tube
D. Drainage and surgical repair of the injury
Correct answer: D

28 . Clostridial organisms are -


A. Gram positive aerobes
B. Gram positive obligate anaerobe
C. Gram negative aerobes
D. Gram negative anaerobes
Correct answer: B

29 . For annular pancreas, treatment of choice is-


A. Duodeno duodenostomy
B. Division of pancreas
C. Roux-en-Y loop
D. Duodenojejunostomy
Correct answer: D

30 . Postoperative fever, causes are


A. Atelectasis
B. Subphrenic abscess
C. Catheter related phlebitis
D. All
Correct answer: D

31 . A lady comes with polyps in intestine, melanotic pigmentation of lip and


positive family history. Most probable diagnosis is ?
A. Peutz-Jeghers Syndrome
B. Gartner's syndrome
C. Turcot's syndrome
D. Lynch syndrome
Correct answer: A

32 . Which is not an indication of thoracotomy -


A. Massive pneumothorax
B. Pulmonary contusion
C. Bleeding more than 200ml/hr. in thoracotomy tube
D. Oesophageal rupture
Correct answer: B

33 . In hypertrophic pyloric stenosis-metabolic changes seen-


A. Metabolic alkalosis with paradoxical aciduria

- 442 -
B. Metabolic acidosis with paradoxical alkaluria
C. Metabolic alkalosis with paradoxical bicarbonaturia
D. Metabolic acidosis with paradoxical bicarbonaturia
Correct answer: A

34 . Critical limb ischemia occurs when the ankle-brachial index falls to below -
A. 0.3
B. 0.5
C. 0.9
D. 1.0
Correct answer: A

35 . In case of vesicoureteral reflux which will be inv. of choice -


A. Micturating cystourethrogram
B. IVP
C. Cystography
D. Radionucleotide study
Correct answer: A

36 . Ganglion is ?
A. Collection of neurons in CNS
B. Collection of neurons outside CNS
C. Collection of dendrites
D. Any collection of neurons
Correct answer: B

37 . Ca. Anal canal The treatment of choice is -


A. Radiotherapy
B. Radiotherapy + APR
C. Chemoradiation
D. Radiotherapy + chemotherapy APR
Correct answer: C

38 . Cause of chronic tropical pancreatitis is ?


A. Parasitic infection
B. Cassava ingestion
C. Idiopathic
D. Genetic
Correct answer: B

39 . True about transitional cell carcinoma are all except:


A. Nephrectomy is the best treatment
B. Painless hematuria is most characteristic symptom
C. Per urethral resection of tumor possible
D. Follow up cystoscopies are not essential
Correct answer: D

40 . Commonest causes of pyogenic liver abscess -


A. Aspiration
B. Hematogenous spread from a distant site

- 443 -
C. Direct contact
D. Lymphatic spread
Correct answer: B

41 . After 30% loss of blood volume in road traffic accident. What next m/n is to
be given:
A. IV fluid only
B. IV fluid with cardiac stimulant
C. Dopamine
D. Vasopressor drug
Correct answer: A

42 . 50-year-old male with positive family history of prostate cancer has come to
you for a screening test. The most sensitive screening test to pick up prostate
cancer is:
A. DRE
B. PSA
C. DRE+PSA
D. Endorectal Coil MRI with T1W and T2W images
Correct answer: C

43 . 15 years old girl, regular swimmer presents with sudden onset of pain in
abdomen, abdominal distension and fever of 39 degree C and obliteration of the
liver dullness. Most probable diagnosis is:
A. Ruptured typhoid ulcer
B. Primary bacterial peritonitis
C. Ruptured ectopic pregnancy
D. UTI with PID
Correct answer: A

44 . A women of 35-year old comes to emergency department with symptoms of


pain in abdomen and bilious vomiting but no distension of bowel. Abdominal x-
ray showed no air fluid level. Diagnosis is:
A. CA rectum
B. Duodenal obstruction
C. Adynamic ileus
D. Pseudo obstruction
Correct answer: B

45 . Majority of cysts occur in -


A. Mediastinum
B. Near carina
C. Base of the lung
D. Peribranchial tissue
Correct answer: A

46 . Which type of pancreatitis has the best prognosis?


A. Gall stone pancreatitis
B. Alcoholic pancreatitis

- 444 -
C. Idiopathic pancreatitis
D. Post operative pancreatitis
Correct answer: A

47 . Menetrier's disease presents with all of the following EXCEPT


A. Weight loss
B. Edema
C. Constipation
D. Skin rash
Correct answer: C

48 . Beta2 microglobulin is a marker for prognosis in


A. Multiple myeloma
B. SLE
C. Chronic pancreatitis
D. AIDS
Correct answer: A

49 . Ectopic ureter does not open in:


A. Prostatic urethra
B. Seminal vesicle
C. Bulbar urethra
D. Ejaculatory duct
Correct answer: C

50 . A patient of Crohn's disease, underwent resection anastomosis. Now presents


on 7th post-op day with anastomotic site leak from a fistula. Everyday leakage
volume adds up to 150-200 ml. There is no intra-abdominal collection, and the
patient is stable without any complains. What will be the next line of management
?
A. Do conservative treatment and leave him and hop for the spontaneous resolution
B. Perform Laparotomy and check for leakage site and healthy margins
C. Perform Laparotomy and completely exteriorize the fistula
D. Perform Laparotomy and place lateral drains and leave
Correct answer: A

51 . The most common position adopted for surgical procedures is -


A. Trendelenburg position
B. Lithotomy position
C. Supine position
D. Prone position
Correct answer: C

52 . Axillary vein thrombosis in NOT a complication of -


A. Thoracic outlet syndrome
B. Simple mastectomy
C. Excessive upper limb exercise
D. Straphangers
Correct answer: B

- 445 -
53 . In which of the following patients, thyrotoxicosis should not be suspected -
A. Patients with unexplained weight loss
B. Patients with unexplained diarrhea
C. Patients with distal muscle weakness
D. Patients with paroxysmal atrial tachycardia
Correct answer: C

54 . The most common major component of a urinary calculus is -


A. Uric acid
B. Calcium oxalate
C. Calcium carbonate
D. Ammonium magnesium phosphate
Correct answer: B

55 . A chronic Alcoholic present with abdominal pain radiating to the back that
responds to analgesics. At evaluation the pancreatic duct was found to be dilated
and stones were noted in the tail of pancreas. The most appropriate management
is:
A. Percutaneous removal of stone
B. Medical management
C. Pancreatic Tail Resection
D. Pancreatico Jejunostomy
Correct answer: B

56 . Interstitial cystitis is also known as ?


A. Eosinophilic cystitis
B. Radiation cystitis
C. Hunner's cystitis
D. Tubercular cystitis
Correct answer: C

57 . Choledoco-duodenostomy is not done for:


A. Duodenal diverticula
B. Stricture at lower end of CBD
C. Large impacted stone at CBD
D. Multiple intrahepatic stones
Correct answer: A

58 . A patient presented with obstructed inguinal hernia. All of the following


features should be present except:
A. Tense and tender swelling
B. Cough impulse is less
C. Always associated with intestinal obstruction
D. Less commonly occurs with direct hernia
Correct answer: A

59 . An 8-month-old infants present with acute intestinal obstruction. Contrast


enema X-ray shows the intussusceptions, Possible cause is:
A. Mucosal polyp
B. Duplication cyst

- 446 -
C. Payers patch hypertrophy
D. Mekell’s diverticulum
Correct answer: C

60 . A 45 year old female complains of progressive lower limb weakness,


spasticity, urinary hesitancy. MRI shows intradural enhancing mass lesion. Most
likely diagnosis is -
A. Neuroepithelial cyst
B. Meningioma
C. Dermoid cyst
D. Intradural lipoma
Correct answer: B

61 . Type 2 cholangiocarcinoma involves?


A. Division of both ducts and not extending outside
B. Common hepatic duct only
C. Secondary hepatic ducts
D. Extends beyond hilum
Correct answer: A

62 . A patient presential with local gigantism of the leg and increased pulsations
of the lower limb veins. Most probable diagnosis is:
A. Tumor
B. AV fistula
C. Varicose veins
D. Incompetence of the saphenofemoral junction
Correct answer: B

63 . Obstructive of the hepatic vein by thrombosis or a web is -


A. Schatzki's ring
B. Plummer Vinson syndrome
C. Budd Chiari syndrome
D. Caroli's disease
Correct answer: C

64 . Which of the following is the most common complication in hiatus hernia?


A. Volvulus.
B. Esophageal stricture.
C. Esophagitis.
D. Aspiration pneumonitis.
Correct answer: C

65 . Splenic abscess is best treated by -


A. Open splenectomy
B. Laparoscopic septectomy
C. Percutaneous drainage under image guidance
D. Antibiotic only
Correct answer: C

- 447 -
66 . Which of the following is resistant to lithotripsy?
A. Triple phosphate stone
B. Calcium oxalate
C. Uric acid stone
D. Cystine stone
Correct answer: D

67 . Which of the following is a non-absorbable suture?


A. Catgut
B. Polydioxanone
C. Polypropylene
D. Vicryl
Correct answer: C

68 . All are true regarding Early Post-cibal syndrome, except:


A. Hypermotility of intestine is common
B. Surgery is usually indicated
C. Distension of abdomen
D. Managed conservatively
Correct answer: B

69 . Following drug lowers intracranial pressure except:


A. Hyperventilation
B. None
C. Mannitol infusion
D. Furosemide
Correct answer: B

70 . Treatment of acutely developing massive left sided hemothorax in a young


male after an accident is -
A. Strapping of chest
B. Tube thoracostomy
C. Endotracheal intubation + IPPV + pleural fluid aspiration
D. Conservative, wait and watch
Correct answer: B

71 . A patient comes with stage III non seminomatous testicular tumor the
treatment of choice is
A. Radiotherapy
B. Chemotherapy
C. Hormonal therapy
D. Surgery
Correct answer: B

72 . Orchidopexy is done in case of undescended tests at the age of -


A. 5 years
B. Puberty
C. Infancy
D. 1-2 years
Correct answer: C

- 448 -
73 . Treatment of duodenal atresia is ?
A. Roux- en-y procedure
B. Duodenoduodenostomy
C. Duodenojejunostomy
D. Duodenal canalization
Correct answer: B

74 . Ureteric colic due to stone is caused by:


A. Increased peristalsis of ureter to overcome the obstruction
B. Stretching of renal capsule due to back pressure
C. Extravasation of urine.
D. Irritation of intramural ureter
Correct answer: A

75 . Alpha-Fetoprotein increases in all the following except


A. Hepatocellular carcinoma
B. Seminoma of the testes
C. GI neoplasms
D. Embryonal cell carcinoma
Correct answer: B

76 . Paradoxical aciduria seen in ?


A. Intestinal obstruction
B. Pyloric obstruction
C. Enterocutaneous fistula
D. Vesico vaginal fistula
Correct answer: B

77 . In which case pneumococcal vaccine is most effective -


A. When given preoperatively
B. When given post operatively
C. Against all strains of bacteria
D. Against gram negative bacteria
Correct answer: A

78 . Meconium Ileus is associated with:


A. Pyloric Stenosis
B. Hirschsprung's disease
C. Fibrocystic disease of the Pancreas
D. Duodenal Atresia
Correct answer: C

79 . A patient developed respiratory distress and hypoxemia after central venous


catheterization through internal jugular vein, reason for this is:
A. Pneumothorax
B. Hypovolemia
C. Septicemia
D. Cardiac tamponade
Correct answer: A

- 449 -
80 . A decubitus ulcer is
A. A venous ulcer
B. An ulcer in the region of the elbow
C. A pressure sores
D. An ulcer of the tongue
Correct answer: C

81 . In case of pyogenic liver abscess, commonest mood of infection is -


A. Ascending infection
B. Hematogenous through portal vein
C. Direct
D. Lymphatics
Correct answer: A

82 . Progressive dysphagia for solids and liquids is found in carcinoma of ?


A. Oropharynx
B. Hypopharynx
C. Nasopharynx
D. All of the above
Correct answer: B

83 . Which one is not true regarding hyperplastic ileocecal tuberculosis -


A. Mass in rt iliac fossa
B. Common site is ileo cecal region
C. X-ray shows indrawing of caecum from ileum
D. Conservative m/n is the treatment of choice
Correct answer: D

84 . MOST COMMON retro-peritoneal tumor is -


A. Fibrosarcoma
B. Liposarcoma
C. Dermoid cyst
D. Rhabdosarcoma
Correct answer: B

85 . Prophylactic removal of lymph nodes in sarcoma involving extremity is done


in -
A. Liposarcoma
B. Fibrosarcoma
C. Embryonal rhabdomyosarcoma
D. Leiomyosarcoma
Correct answer: C

86 . Out of the following locations of intracranial hemorrhage, surgery is most


useful in -
A. Putamen hemorrhage
B. Thalamic hemorrhage
C. Cerebellar hemorrhage
D. Pontine hemorrhage
Correct answer: C

- 450 -
87 . Stain used to diagnose premalignant lesion of lip is ?
A. Crystal violet
B. H and E
C. Toluidine blue
D. Giemsa
Correct answer: C

88 . Which of the following statements regarding myasthenia gravis and


thymectomy is not true -
A. Thymectomy achieves best results in bulbar myasthenia
B. Thymectomy achieves good remission in patients with benign thymomas
C. Invasive thymomas have less likelihood at remission after thymectomy
D. Best surgical approach is median sternotomy
Correct answer: A

89 . Which one is not true regarding pseudocyst of pancreas -


A. Epigastric mass
B. Increase level of amylase
C. Cysto - gastrostomy is the ideal treatment
D. Percutaneous aspiration is the treatment
Correct answer: D

90 . Which of the following indicate prognosis for sarcoma -


A. Grade of tumour
B. Depth of tumour
C. Size of tumour
D. Nature of tumour
Correct answer: A

91 . Which grading methods is used to evaluate the prognosis/outcome after


subarachnoid hemorrhage
A. Hess and Hunt Scale
B. Glasgow Coma Scale
C. Intracerebral Hemorrhage score
D. Glasgow - Blatchford Bleeding score
Correct answer: A

92 . Sympathectomy is indicated in A/E-


A. Intermittent claudication
B. Ischemic ulcers
C. Acrocyanosis
D. Anhidrosis
Correct answer: D

93 . Thoracic extension of cervical goiter is usually approached through which


one?
A. Combined cervico-thoracic route
B. Thoracoscopic
C. Neck
D. Chest

- 451 -
Correct answer: C

94 . A 54 years old man complains of recurrent epigastric pain, radiating to the


back. The pain is relieved on bending forward and increase on sitting up. There
is an associated history of alcohol intake. On examination serum bilirubin is 5
mg% and blood glucose 180 mg%, probable diagnosis is:
A. Chronic pancreatitis
B. Hepatitis
C. Cholecystitis and cholelithiasis
D. Carcinoma head of pancreas
Correct answer: A

95 . Treatment for CTEV should start from -


A. Immediately after birth
B. After 3 months
C. After 6 months
D. After 24 hours
Correct answer: A

96 . Occupations at risk for bladder cancer include all of the following EXCEPT
A. X-ray technicians
B. Textile workers
C. Petrol workers
D. Painters
Correct answer: A

97 . Which of the following statements is true of primary grade IV-V


vesicoureteral reflux in young children ?
A. Postnatal scarring may occur even in the absence of urinary tract infections
B. Renal scarring usually begins in the midpolar regions
C. Oral amoxicillin is the choice antibiotic for prophylaxis
D. Long-term outcome is comparable in patients treated with either antibiotic
prophylaxis or surgery
Correct answer: A

98 . Carcinoma of buccal mucosa commonly drain to the following lymph node


sites:
A. Sub mandibular
B. Sub mental
C. Cervical
D. Supraclavicular
Correct answer: A

99 . Ainhum is seen in:


A. Base of fingers tips
B. Base of great toe
C. Ankle
D. Base of toe
Correct answer: D

- 452 -
100 . Commonest site of a rodent ulcer is
A. Limbs
B. Face
C. Abdomen
D. Trunk
Correct answer: B

101 . Meconium ileus can be caused by


A. Fibrocystic disease of the pancreas
B. Liver aplasia
C. Cirrhosis of the liver
D. Malnutrition
Correct answer: A

102 . In organ preservation, Allopurinol is used as


A. Preservative
B. Antioxidant
C. Precursor for energy metabolism
D. Free radical scavenger
Correct answer: D

103 . Which of the following is the gold standard test for insulinoma?
A. C-peptide levels
B. Low glucose levels < 30 mg/dL
C. 72 hr fasting test
D. Plasma insulin levels
Correct answer: C

104 . Dumping syndrome occurs least with -


A. Selective vagotomy
B. Truncal vagotomy
C. Highly selective vagotomy
D. Syndrome occurs with all of the above commonly
Correct answer: C

105 . All of the following are features of Zollinger Ellison syndrome except -
A. Intractable peptic ulcers
B. Severe diarrhea
C. Beta cell tumours of the pancreas
D. Very high acid output
Correct answer: C

106 . Full bowel preparation is avoided in all, except -


A. Carcinoma colon
B. Hirschsprung disease
C. Ulcerative colitis
D. Irritable bowel syndrome
Correct answer: A

- 453 -
107 . All of the following are used in pneumatosis cystoids intestinalis except -
A. High flow oxygen
B. Albendazole
C. Metronidazole
D. Sulphasalazine
Correct answer: B

108 . Not true regarding solitary rectal ulcer -


A. 20% are multiple
B. Recurrent rectal prolapse is a cause
C. Involves Posterior wall
D. Managed by digital reposition
Correct answer: C

109 . Which of the following nerve is commonly damaged during Mc Burney's


incision?
A. Iliohypogastric
B. Subcostal
C. 10th thoracic
D. 11th thoracic
Correct answer: A

110 . Coronary graft is most commonly taken from ?


A. Femoral vein
B. Saphenous vein
C. Axillary vein
D. Cubital vein
Correct answer: B

111 . True about carcinoid tumors are A/E-


A. 5 year survival for carcinoid tumors is > 60%
B. Appendiceal carcinoids are more common in females
C. It is the most common malignant tumor of the small intestine
D. Extensive involvement of small intestine is associated with higher probability of
Lung metastasis
Correct answer: C

112 . Hippocrates facies is seen in -


A. Peritonitis
B. Pancreatitis
C. Facial nerve injury
D. Marginal mandibular nerve injury
Correct answer: A

113 . In which case lymph nodes are resected prophylactically -


A. Embryonal Rhabdomyosarcoma
B. Liposarcoma
C. Fibrosarcoma
D. Neurofibroma
Correct answer: A

- 454 -
114 . 66 years previously healthy man is admitted to a hospital because of a closed
head injury and ruptured spleen following a roadside automobile accident. During
the first 4 days of hospitalization, following laparotomy and splenectomy, he
receives 5% dextrose 0.5% normal saline solution at a rate of 125mL/h. Recorded
daily fluid outputs include 450 to 600mL of nasogastric drainage and 700 to 1000
mL of urine. The patient is somnolent but easily aroused until the morning of the
5th hospital day, when he noted to be in deep coma. By the after-noon, he begins
having seizures. The following laboratory data are obtained: Serum
electrolytes(meq L):Na+128:K+1.8;Cl-94;HCO3-19.Serum osmolality : 260m
Osm/L; Urine electrolytes (meq/L):Na+61;K+18.Which of the following about
diagnosis or treatment of this patient's is true:
A. Secondary to metabolic acidosis their is hypokalemia
B. Emergency carotid arteriogram to be done
C. IV infusion of 20ml of 50% MgSO4 given for 4 hrs.
D. A small quantity of hypertonic saline to be given
Correct answer: D

115 . DTPA scan of hypertensive young lady is normal, USG shows small kidney
on left side. Next investigation will be:
A. CT Scan abdomen
B. Retrograde pyelogram
C. Digital subtraction angiography
D. DMSA
Correct answer: C

116 . Which is not elevated in a child presenting with jaundice, icterus, pruritus
and clay colored stools -
A. Gamma glutamyl transpeptidase
B. Alkaline phosphatase
C. 5'-nucleosidase
D. Glutamate dehydrogenase
Correct answer: D

117 . Bilateral spider leg appearance is found in:


A. Hydronephrosis
B. Polycystic kidney
C. Renal cell carcinoma
D. Traumatic rupture of kidney
Correct answer: B

118 . Which among the following is not a feature of peripheral arterial occlusion?
A. Shock
B. Pallor
C. Pain
D. Pulselessness
Correct answer: A

- 455 -
119 . Following tumour may regress spontaneously except
A. Malignant melanoma
B. Retinoblastoma
C. Choriocarcinoma
D. Osteosarcoma
Correct answer: D

120 . Sentinel lymph node biopsy is not done in ?


A. Carcinoma breast
B. Carcinoma colon
C. Carcinoma penis
D. Melanoma
Correct answer: B

121 . A wide mouth Meckel’s diverticulum is found accidentally on laparotomy.


What will be the treatment of choice -?
A. Resection of diverticulum
B. Leave as such
C. Ligate at base
D. Resection with part of ileum
Correct answer: B

122 . All of the following can lead to retroperitoneal fibrosis EXCEPT


A. Retroperitoneal hemorrhage
B. Beta-adrenergic blocking agents
C. Urinary extravasation
D. Immunosuppressive drugs
Correct answer: D

123 . For differentiating Inguinal hernia and femoral hernia the landmark will
be -
A. Pubic symphysis
B. Femoral artery
C. Inferior epigastric vessel
D. Pubic tubercle
Correct answer: D

124 . In case of blunt injury thorax MOST COMMON complication is -


A. Pneumothorax
B. Rib
C. Hemopneumothorax
D. Aortic rupture
Correct answer: B

125 . Which of the following substances is not used as an irrigate during


transurethral resection of the prostate?
A. Normal Saline
B. 1.5% Glycine
C. 5% Dextrose
D. Distilled Water

- 456 -
Correct answer: A

126 . Cardiovascular findings in an elderly Thyrotoxicosis patient are all, except:


A. Scratch in left 2nd inter costal space
B. Irregularly Irregular pulse
C. Early diastolic murmur
D. Systolic ejection murmur
Correct answer: C

127 . Correct about axillary vein thrombosis except:


A. Treated with IV anticoagulant
B. May be caused by a cervical rib
C. May occur following excessive exercise
D. Embolectomy is done in all cases
Correct answer: D

128 . Which is not associated with elevation Right hemi-diaphragm?


A. Pyogenic abscess
B. Amebic abscess
C. Subdiaphragmatic abscess
D. Cholecystitis
Correct answer: D

129 . A 60 year diabetic and hypertensive male with second grade prostatism
admitted for prostatectomy developed myocardial infarction. Treatment now
would be:
A. Terazocin
B. Finasteride
C. Diethyl stilbestrol
D. Finasteride and terazocin
Correct answer: A

130 . Splenectomy is not done in


A. Chronic ITP
B. Hereditary spherocytosis
C. Porphyria
D. Niemann-Pick disease
Correct answer: C

131 . The term 'ulcer' by definition is


A. An erosion of the skin
B. A discontinuity of any epithelial surface
C. A lesion caused by infection
D. A neoplastic process
Correct answer: B

- 457 -
132 . Most common site for abscess after laparotomy is:
A. Pelvic
B. Subphrenic
C. Subhepatic
D. Paracolic
Correct answer: A

133 . Which neo-adjuvant chemotherapy is used in Esophageal carcinoma


A. Cisplatin
B. Cyclophosphamide
C. Doxorubicin
D. Methotrexate
Correct answer: A

134 . The following are true of benign prostatic hypertrophy EXCEPT


A. It occurs in the transitional zone of the prostate
B. It can cause bladder diverticula
C. The prostatic urethra is narrowed
D. It may be associated with a prominent median lobe
Correct answer: C

135 . Denver shunt used in ?


A. Ascites
B. Dialysis
C. Raised ICP
D. Raised IOP
Correct answer: A

136 . Commonest cause of pyogenic liver abscess -


A. Biliary Sepsis
B. Biliary Colic
C. Appendicitis
D. Sigmoid Diverticulitis
Correct answer: A

137 . A 8 years child presents with an expansile swelling on medial side of the
nose. Possible diagnosis is:
A. Meningocele
B. Teratoma
C. Lipoma
D. Dermoid cyst
Correct answer: A

138 . Which of the following is not an oncological emergency?


A. Spinal cord compression
B. Superior vena cava syndrome
C. Tumor lysis syndrome
D. Carcinoma cervix stage-III B with Pyometra
Correct answer: D

- 458 -
139 . How do you get an ideal scar ?
A. Achieve quiet, secondary healing
B. Ensure tight closure of the skin
C. Give antibiotics
D. Minimize tension
Correct answer: D

140 . Orchidectomy is not done in


A. Prostate cancer
B. Seminoma testes
C. Filarial epididmo-orchitis
D. Male breast cancer
Correct answer: C

141 . Malignant cells in urine is suggestive of -


A. Prostatic carcinoma
B. Urothelial carcinoma
C. Wilms' tumor
D. Hypernephroma
Correct answer: B

142 . Intermittent Claudication is defined as


A. Pain in muscle on first step
B. Pain in muscle at rest only
C. Pain in muscle on last step
D. Pain in muscle on exercise only
Correct answer: D

143. A child presents with raised ICT, on CT, scan a lesion is seen around
foramen of Monroe and multiple periventricular calcific foci. What is the
probable diagnosis ?
A. Central Neurocytoma
B. Ependymoma
C. Subependymal Giant cell Astrocytoma
D. Ganglioglioma
Correct answer: C

144 . 7 - years girl who has non-pitting edema of recent onset affecting her right
leg but no other symptoms is refer for evaluation. Which of the following is true
about her?
A. A lymphangiogram will show hypoplasia of the lymphatics
B. Prophylactic antibodies are indicated
C. A variety of operations will ultimately lead to a normal appearance of the limb
D. Elastic stocking and diuretics will lead to a normal appearance of the limb
Correct answer: A

145 . The best site for an intramuscular injection is the


A. Abdomen
B. Forearm
C. Outer upper quadrant of buttock

- 459 -
D. Inner lower quadrant of buttock
Correct answer: C

146 . The length of the feeding tube to be inserted for transpyloric feeding is
measured from the tip of:
A. Nose to the umbilicus
B. Ear lobe to the umbilicus
C. Nose to the knee joint
D. Ear lobe to the knee joint
Correct answer: B

147 . False about Peutz-Jegher's syndrome is ?


A. Benign
B. Familial
C. Malignant
D. Melanosis of lip
Correct answer: C

148 . Which of the following is the treatment for achalasia associated with high
rate of recurrence?
A. Open surgical myotomy
B. Botullin toxin
C. Pneumatic dilatation
D. Laproscopic myotomy
Correct answer: B

149 . A 60-year-old smoker came with a history of painless gross hematuria for
one day. Most logical investigation would be
A. Urine routine
B. Plain X-ray KUB
C. USG KUB
D. Urine microscopy for malignant cytology
Correct answer: D

150 . A 50 years old woman presented with history of recurrent episodes of right
upper abdominal pain for the last one year. She presented to casualty with history
of jaundice and fever for 4 days. On examination, the patient appeared toxic and
had a blood pressure of 90/60 mmHg. She was started on intravenous antibiotics.
Ultrasound of the abdomen showed presence of stones in the common bile duct.
What would be the best treatment option for her?
A. ERCP and bile duct stone extraction
B. Laparoscopic cholecystectomy
C. Open surgery and bile duct stone extraction
D. Lithotripsy
Correct answer: A

151 . Smoking may be associated with all of the following cancer's Except:
A. Ca Nasopharynx
B. Ca Larynx
C. Ca Esophagus

- 460 -
D. Ca Bladder
Correct answer: A

152 . Which one is not associated with peptic ulcer -


A. Smoking cigarrete
B. Zollinger Ellison's syndrome
C. Pulmonary Vinson’s syndrome
D. Cirrhosis
Correct answer: C

153 . Recovery is complete in the following injury -


A. Neurotmesis
B. Axonotmesis
C. Neuropraxia
D. Nerve avulsion
Correct answer: C

154 . A 30 year old male patient presents with sudden onset swelling and pain over
the right hemiscrotum. On examination the scrotum is reddened and tender.
Which of the following statement about the affecting condition is not true?
A. If torsion confirmed, treat with antibiotics and analgesics, and perform corrective
surgery immediately
B. If torsion confirmed, treat with antibiotics and analgesics, and perform corrective
surgery after 14 days
C. Probable diagnosis is torsion
D. The right testis is likely to ride high in the scrotal compartment.
Correct answer: B

155 . Management for CSF rhinorrhea is -


A. Plain X-ray and packing of nose
B. Nasal packing only
C. Antibiotics and observation
D. Immediate surgery
Correct answer: C

156 . Which of the following statements is true regarding fat embolism -?


A. Most patients with major trauma involving long bones have urinary fat globules
B. All patients with urinary fat globules develop fat embolism
C. Peak incidence of respiratory insufficiency for pulmonary fat embolism is around
day 7 after injury
D. Heparin as an anticoagulant decreases mortality and morbidity in fat embolism
syndrome
Correct answer: A

157 . All the following statements are true regarding Malignant Parotid Gland
Tumors except -
A. Painful
B. Presents with skin Ulceration
C. Cervical Lymphadenopathy present
D. Facial nerve involvement

- 461 -
Correct answer: B

158 . What will be the diagnosis of 45 years old male with history of chronic
smoking and pain in lower limb due to blockage of femoral artery -
A. Thromboangitis obliterans
B. Atherosclerosis
C. Embolism
D. Arteritis
Correct answer: B

159 . Meckel's diverticulum is a derivative of


A. Allantoic diverticulum
B. Vitello intestinal duct
C. Ventral mesogastrium
D. Ductus arteriosus
Correct answer: B

160 . A 27 years patient with head injury opens eyes to painful stimulus, uses
inappropriate words, and localizes pain. His GCS score is-
A. 10
B. 8
C. 14
D. 12
Correct answer: A

161 . Which of the following differentiates Hodgkin's lymphoma from non


Hodgkin's lymphoma ?
A. Fever, night sweat, weight loss
B. Generalized lymphadenopathy
C. Elderly
D. Reed stern berg cells

162 . Which of the following is false about hepatic duct?


A. Caudate lobe drains only left hepatic duct
B. Left hepatic duct formed in umbilical fissure
C. Left hepatic duct cross’s IV segment
D. Right hepatic duct formed by V and VIII segments
Correct answer: A

163 . Most important pathological cause of GERD is -


A. Hiatus hernia
B. Transient LES relaxation
C. LES hypotension
D. Inadequate esophageal clearance
Correct answer: B

164 . Correct about deltopectoral flap:


A. It is based on internal mammary artery
B. Myocutaneous flap
C. Cannot extend into shoulder

- 462 -
D. Cannot be used for reconstruction after excision of ORA cancer
Correct answer: A

165 . Neuroblastoma differs from Wilm's tumor radiologically by all except:


A. Calcification
B. Aorta and IVC are not eroded but pushed a side
C. Same location
D. Intraspinal extension of tumor
Correct answer: C

166 . Green discharge is M. commonly seen with -


A. Duct papilloma
B. Duct ectasia
C. Retention cyst
D. Fibro adenosis
Correct answer: B

167 . Triangle of Doom is bounded by all of the following except -


A. Gonadal vessels
B. Peritoneal reflection
C. Cooper's ligament
D. Vas deferens
Correct answer: C

168 . Commonest presentation of primary biliary cirrhosis


A. Jaundice
B. Fever
C. Pruritis
D. Pain
Correct answer: C

169 . The most common cause of small intestinal obstruction is


A. Intussusception
B. Iatrogenic adhesions
C. Trauma
D. Carcinoma
Correct answer: B

170 . In which case spontaneous regression is not seen -


A. Malignant Melanoma
B. Osteosarcoma
C. Neuroblastoma
D. Choriocarcinoma
Correct answer: B

171 . Treatment of pseudocyst of pancreas is -


A. Excision of head
B. Excision of tail
C. Internal drainage
D. Antibiotics + conservative treatment

- 463 -
Correct answer: C

172 . Treatment of hyperprolactinemia is ?


A. Bromocriptine
B. Methyl Dopamine
C. Haloperidol
D. Chlorpromazine
Correct answer: A

173 . Which is true regarding hydatid cyst of lung


A. Never ruptures
B. Calcification is common
C. Always associated with cyst in the liver
D. More common in lower lobes
Correct answer: D

174 . True about Flail chest are A/E-


A. PaO2 < 40 with FiO2 > 60 treated with intubation and PEEP
B. Paradoxical movement may not be seen in conscious patients
C. Fracture of at lest 3 ribs
D. If overlapping of fractured ribs with severe displacement is seen than patients are
treated surgically with open reduction and fixation
Correct answer: B

175 . All hernias involving bowel that reach the stage of vascular compromise, do
cause symptom signs of intestinal obstruction, except -
A. Sciatic hernia
B. Littre's hernia
C. Cloquet's hernia
D. Serafini's hernia
Correct answer: B

176 . A young patient presents with a massive injury to proximal duodenum,


head of pancreas and distal common bile duct. The procedure of choice in this
patient should be:
A. Pancreaticoduodenectomy (Whipple's operation)
B. Roux-en-Y anastomosis
C. Retrograde jejunostomy
D. Lateral tube jejunostomy
Correct answer: A

177 . Secondaries of all the following cause osteolytic lesions except


A. Prostate
B. Kidney
C. Bronchus
D. Thyroid
Correct answer: A

- 464 -
178 . All of the following are true of congenital cystic kidneys EXCEPT
A. It is transmitted as an autosomal dominant trait
B. It manifests clinically at a young age
C. The lungs may also be affected
D. Unilateral swelling may be palpable
Correct answer: B

179 . For stage T3 N2a M0 bronchoalveolar cell Ca, the treatment is -


A. Radiotherapy
B. Surgery + Radiotherapy
C. Surgery
D. Chemotherapy
Correct answer: B

180 . Laugier's hernia is a type of -


A. Inguinal hernia
B. Femoral hernia
C. Incisional hernia
D. Obstructed hernia
Correct answer: B

181 . All are true except:


A. Site of colostomy stoma should be marked preoperatively
B. Colostomy causes increase in renal and GB stone
C. Ileostomy preferably done over right side
D. Feeding jejunostomy done in severe malnourished patients
Correct answer: B

182 . Hypotension, failure of liver, decreased urine output in a 45-year-old male


patient is due to
A. SIRS
B. Septic shock
C. MODS
D. DIC
Correct answer: C

183 . Which one is NOT a feature of hypospadias is -


A. Incontinence
B. Hooded prepuce
C. Chordee
D. Meatal stenosis
Correct answer: A

184 . Best diagnostic modality for parotid swelling is -


A. Enucleation
B. FNAC
C. Superficial parotidectomy
D. Excisional biopsy
Correct answer: B

- 465 -
185 . Emergency Thoracotomy is done in
A. Cardiac tamponade
B. Massive pneumothorax
C. Clotted hemothorax
D. Haemorrhage of more than 200 ml/hr for more than three hours
Correct answer: D

186 . Gynecomastia may be seen in all of the following conditions except:


A. Cirrhosis of liver
B. Klinefelter syndrome
C. Sexcord tumour of sertoli cells
D. Cryptorchidism
Correct answer: D

187 . When PCNL is done through 11th intercostals space, one may expect
A. Damage to colon
B. Remnants fragments
C. Hydrothorax
D. Hematuria
Correct answer: C

188 . In all of the following sympathectomy is effective except one -


A. Intermittent claudication
B. Hyperhidrosis
C. Raynaud's disease
D. Causalgia
Correct answer: A

189 . Carcinoid tumor is most common in-


A. Stomach
B. Esophagus
C. Appendix.
D. Small bowel.
Correct answer: D

190 . Which of the following is commonest source of Extradural hemorrhage -?


A. Middle meningeal artery
B. Subdural venous sinus
C. Charcot's artery
D. Middle cerebral artery
Correct answer: A

191 . In emergency thoracotomy, the incision is made > 1 cm lateral to sternal


margin to preserve:
A. Superior Epigastric Artery
B. Intercostal Artery
C. Intercostal vein
D. Internal Mammary Artery

- 466 -
Correct answer: D

192 . Which of the following is not a feature of Zollinger Ellison Syndrome?


A. Post bulbar ulcer
B. Recurrence of peptic ulcer
C. Severe diarrhea
D. Excess HCI production following infection of histamine
Correct answer: D

193 . Chronically lymphadenopathy limb is predisposed to A/E-


A. Recurrent soft tissue infections
B. Thickening of the skin
C. Sarcoma
D. Marjolin's ulcer
Correct answer: D

194 . True statement for axial flap is:


A. Transverse flap
B. Carries its own nerve in it
C. Carries its own vessels within it
D. Kept in limb
Correct answer: C

195 . Which of the following is the treatment of choice for squamous cell anal
cancer?
A. Laser fulguration
B. Abdominoperineal resection
C. Platinum-based chemotherapy
D. Chemoradiotherapy
Correct answer: D

196 . The following are important in maintenance of normal fecal continence


except
A. Anorectal angulation
B. Rectal innervation
C. Internal sphincter
D. Haustral valve
Correct answer: D

197 . Commonest cause of pulmonary embolism is


A. IVC thrombosis
B. Thrombosis of internal pudendal artery
C. Thrombosis of leg veins
D. Thrombosis of prostatic veins
Correct answer: C

198 . Dangerous layer of the scalp is:


A. Skin
B. Galea
C. Pericranium

- 467 -
D. Loose areolar tissue
Correct answer: D

199 . Congenital hydrocoele is best treated by -


A. Eversion of sac
B. Excision of sac
C. Lord's procedure
D. Herniotomy
Correct answer: D
200 . Bascom technique is employed in surgery of -
A. Rectal prolapse
B. Anal incontinence
C. Pilonidal sinus
D. Fistula-in-ano
Correct answer: C

201 . Greenish nipple discharge most commonly seen in-


A. Duct ectasia
B. Intraductal papilloma
C. Intraductal carcinoma
D. Fibro adenosis
Correct answer: D

202 . 25 years old patient presented with mass in rt iliac fossa. Which after
laparotomy was found to be carcinoid of 2.5 cm in diameter. What will be the
next step in management?
A. Segmental resection
B. Appendicectomy
C. Rt. hemicolectomy
D. Do yearly 5 HIAA assay
Correct answer: C

203 . In which of the following transmission, meiosis occurs -


A. Germ cell to spermatogonium
B. Spermatogonium to spermatocyte
C. Primary to secondary spermatocyte
D. Secondary spermatocyte to globular spermatid
Correct answer: C

204 . Hamman's sign is seen in


A. Achalasia cardia
B. Esophageal perforation
C. Diffuse esophageal spasm
D. CA esophagus
Correct answer: B

205 . In hypernephroma, all are true except -


A. It is an adenocarcinoma
B. Rapidly developing varicocele may be a presenting complaint
C. It is radiosensitive

- 468 -
D. Blood borne metastasis are often seen
Correct answer: C

206 . Commonest intra-abdominal tumor below 2 years of age -


A. Neuroblastoma
B. Wilm's tumor
C. Hepatoblastoma
D. Lymphoma
Correct answer: A
207 . In emergency, which artery can safely ligate -
A. Internal iliac
B. Popliteal
C. Renal artery
D. Internal carotid
Correct answer: A

208 . Haemobilia is characterized by:


A. Gastrointestinal bleeding, biliary colic, jaundice
B. Pain, fever, jaundice
C. Melena, hematuria, abdominal pain
D. Hematemesis, jaundice, fever
Correct answer: A

209 . Most common organ involved in retroperitoneal fibrosis is -


A. Aorta
B. Ureter
C. Inferior vena cava
D. Sympathetic nerve plexus
Correct answer: B

210 . A middle-aged woman was examined for abdominal distension, epigastric


pain and vomiting. She was clinically conforming as a case of intestinal
obstruction. Next investigation to be done:
A. X-ray abdomen
B. USG abdomen
C. CT scan
D. Upper GI endoscopy
Correct answer: A

211 . A 33 years patient suddenly experienced pain radiating along the medial
border of the dorsum of foot. Which nerve is most likely to be accidently ligated-
?
A. Saphenous nerve
B. Sural nerve
C. Genicular nerve
D. Deep peroneal nerve
Correct answer: A

- 469 -
212 . The following are true about Hepatocellular carcinoma except:
A. It has a high incidence in East Africa and South-east Asia
B. Its worldwide incidence parallels the prevalence of Hepatitis B
C. Over 80% of tumours are surgically resectable
D. Liver transplantation offers the only chance of cure in those with irresectable
disease
Correct answer: C

213 . Which one is not the bad prognostic sign for pancreatitis?
A. TLC >16000
B. Calcium less than 8 mmol/L
C. Glucose > 200 mg%
D. Prothrombin >2 times the control
Correct answer: D

214 . In oesophageal Ca which Neoadjuvant chemotherapy is used -


A. Cisplatin
B. Cyclophosphamide
C. Doxorubicin
D. Methotrexate
Correct answer: A

215 . ROME III criteria is for ?


A. IBS
B. Ulcerative colitis
C. Colonic cancer
D. Colonic hemangiomata
Correct answer: A

216 . Which of the following is the most common cysts of the spleen?
A. Pseudo cyst
B. Lymphangioma
C. Hydatid cyst
D. Dermatoid cyst
Correct answer: C

217 . Tumor with least malignant potential is:


A. Teratoma
B. Seminoma
C. Choriocarcinoma
D. Leydig cell tumor
Correct answer: D

218 . Whipple's triad include all except -


A. Hypoglycemia
B. Corrected by IV glucose
C. Increased insulin levels
D. Symptoms of hypoglycemia
Correct answer: C

- 470 -
219 . Dysphagia Iusoria is due to:
A. Ca larynx
B. Esophageal web
C. Esophageal diverticulum
D. Compression by aberrant blood vessel
Correct answer: D

220 . All are true regarding gynecomastia, except -


A. May be seen Addison's disease
B. Usually unilateral in young males
C. Acini are not involved
D. Bilaterality is due to endocrinopathy
Correct answer: A

221 . Platelets can be stored at -


A. 20-240 C for 5 days
B. 20-240 C for 8 days
C. 4-80 C for 5 days
D. 4-80 C for 8 days
Correct answer: A

222 . Lumber sympathectomy is done due to management of-


A. Distal ischemia affecting the skin of the toes.
B. Intermittent claudication.
C. Back pain.
D. Arteriovenous Fistula.
Correct answer: A

223 . 'Charcot's triad' is


A. Fever,pain,vomiting
B. Fever,stone,jaundice
C. Fever,pain,jaundice
D. Gall stone,vomiting,jaundice
Correct answer: C

224 . A 36 years male with abdominal injury presents to the emergency


department with signs of peritonitis and shock. Airway and Breathing were
secured and IV fluids were started with 2 large bore cannulas. The next line of
management should be:
A. Exploratory Laparotomy under general anesthesia
B. FAST
C. Laparoscopy
D. Insertion of abdominal drain followed by Laparotomy
Correct answer: A

225 . Most common site of cholangiocarcinoma?


A. Distal biliary duct
B. Hilum
C. Intrahepatic duct
D. Multifocal

- 471 -
Correct answer: B

226 . Treatment of choice for pleomorphic adenoma


A. Superficial parotidectomy
B. Radical parotidectomy
C. Enucleation
D. Radiotherapy
Correct answer: A

227 . The most appropriate site for pluri sentesis done by inserting a needle in an
adult is:
A. 7th intercostal space in mid axillary's bone
B. 5th intercostals space in midclavicular line
C. 10th intercostal space adjacent to the vertebral column
D. 2nd intercostals space adjacent to the sternum
Correct answer: A

228 . A 60 year old man presented with dysphagia. After 4 months he improved
without any treatment. Investigation of choice in this case-
A. Upper GI endoscopy
B. CT scan
C. Manometric examination
D. Barium swallow
Correct answer: D

229 . Common type of mesenteric cyst is -


A. Enterogenous
B. Chylolymphatic
C. Dermoid
D. Urogenital remnant
Correct answer: B

230 . Resection of 90% of the ileum and jejunum causes all the following except
A. Hypogastrinemia
B. Steatorrhea
C. Anemia
D. Extracellular volume depletion
Correct answer: A

231 . Which of the following statement is not true -


A. Ankle brachial index <0.5 indicates critical limb ischemia
B. Ankle brachial index changes during exercise and rest
C. Ankle brachial index >1 is normal
D. Smoking is more specific for peripheral vascular disease than coronary artery
disease
Correct answer: D

- 472 -
232 . The least painful surgical approach to the heart and lungs is -
A. Median sternotomy
B. Posterolateral thoracotomy
C. Anterolateral thoracotomy
D. Transverse sternal splitting thoracotomy
Correct answer: A

233 . All of the following statements regarding bronchial cysts are true except:
A. Are commonly infected
B. Multilocular
C. Seen in mediastinum
D. 50-70% occur in lung
Correct answer: B

234 . Foreign body aspiration in supine position causes which of the following
parts of the lung commonly to be affected:
A. Apical left lobe
B. Apical lobe of right lung
C. Apical part of the lower lobe
D. Posterobasal segment of left lung
Correct answer: C

235 . A lady had Meningioma with inflammatory edematous lesion. She has
planned for surgery. Junior resident's mistake in writing pre-op notes is -
A. Stop Steroids
B. Wash head with shampoo
C. Antibiotic Sensitivity
D. Continue antiepileptics
Correct answer: A

236 . Which of the following is the most common site of Morgagni hernia?
A. Left anterior
B. Left posterior
C. Right anterior
D. Right posterior
Correct answer: C

237 . Which of the following statements is incorrect regarding Meckel's


diverticulum ?
A. Consists of mucosa without a muscle coat
B. Is found on anti-mesenteric border of the small intestine
C. A fibrous band between the apex and umbilicus can cause intestinal obstruction
D. Heterotopic gastric mucosa can ulcerate and cause a brisk gastrointestinal bleed
Correct answer: A

238 . In Mallory Weiss syndrome, the mucosal tear is usually located at -


A. Gastroesophgeal junction on the lesser
B. Gastroesophgeal junction on the greater curvature side
C. Gastroduodenal junction
D. Gricopharyngeus sphincter

- 473 -
Correct answer: A

239 . Madura foot is characterized by all of the following EXCEPT


A. Chronic granulomas
B. Filamentous organism
C. Spread to lymph nodes
D. Dapsone is drug of choice
Correct answer: C

240 . According to the Bismuth/Strasberg classification 'Cystic blow out' is


classified as which of the bellow:
A. Type C
B. Type D
C. Type A
D. Type B
Correct answer: C

241 . After rupture of middle meningeal artery bleeding occurs in which region -
A. Subdural bleed
B. Extradural bleed
C. Intracerebral bleed
D. Subarachnoid bleed
Correct answer: B

242 . 60 years woman, has a left femoral venous thrombosis during a pregnancy
30 year ago. The left greater saphenous vein had been stripped at age 21 She now
presents with a large non healing ulceration over the medical left calf, which has
continuously progressed despite bedrest, elevation, and use of a support stocking.
Descending phlebography of the left leg demonstrates a patent deep venous
system, with free flow of dye from the groin to foot. The first profunda femoris
valve is competent. Which of the following is appropriate management?
A. Ligated iliofemoral venous thrombectomy with creation of the temporary
arteriovenous fistula
B. Subfascial ligation of perforating veins in the left calf
C. Division of the superficial femoral vein in the groin and transposition of its distal
end onto the profunda femoris vein below the level of the competent profunda valve
D. Saphenous venous crossover graft with anastomosis of the end of the right
saphenous vein onto the side of competent femoral vein
Correct answer: C

243 . Metabolic changes associated with excessive vomiting includes the


following:
A. Hyperchloremia
B. Metabolic acidosis
C. Decreases bicarbonates
D. Hypokalemia
Correct answer: D

- 474 -
244 . In BPH most common lobe involved is
A. Lateral
B. Posterior
C. Median
D. Anterior
Correct answer: C

245 . After head injury, biconvex, lenticular shape hematoma in CT scan is


characteristic of which of the following:
A. Extradural haemorrhage
B. Subdural haemorrhage
C. Intracerebral hematoma
D. Diffuse-axonal injury
Correct answer: A

246 . Commonest tumour of lumbar region in children is -


A. Dermoid cyst
B. Neuroblastoma
C. Wilm's tumour
D. Mesenteric cyst
Correct answer: B

247 . A 23 years female presented in the emergency department with a stab


injury to the left side of the abdomen. She was hemodynamically stable, and a
contrast enhanced CT scan revealed a laceration in spleen. Laparoscopy was
planned however the patient's pO2 suddenly dropped as soon as the
pneumoperitoneum was created. The most possible cause is-
A. Inferior vena cava compression
B. Injury to colon
C. Gaseous Embolism through splenic vessels
D. Injury to the left lobe of the diaphragm
Correct answer: C

248 . A male baby with vigorous feeding and immediate vomiting at 2 months of
age. Most possible diagnosis is -
A. Brain tumor
B. Congenital hypertrophic pyloric stenosis
C. Paralytic ileus
D. Hirschsprung's disease
Correct answer: B

249 . 55 years old chronic smoker presents with complaints of hoarseness of


voice, and single enlarged painless lymph node in left supraclavicular region.
Next step to be done:
A. CT Scan of chest
B. Sputum exam. For AFB
C. Laryngoscopy and chest x-ray
D. Excision biopsy of the node
Correct answer: D

- 475 -
250 . Most common tumor of parotid gland is -
A. Squamous cell carcinoma
B. Pleomorphic adenoma
C. Adenolymphoma
D. None of the above
Correct answer: B

251 . Which is used in BHP:


A. Alpha agonist
B. Alpha antagonist
C. Beta agonist
D. Beta antagonist
Correct answer: B

252 . Which of the following is the preferred treatment modality for annual
pancreas
A. Whipple's operation
B. Duodena-jejunostomy
C. Gastro-jejunostomy
D. Jejuna-cystostomy
Correct answer: B

253 . A 50 years old male patient, an alcoholic and smoker presents with a 3 hour
history of severe retrosternal chest pain and increasing shortness of breath. He
started having this pain while eating, which was constant and radiated to the back
and interscapular region. He was a known hypertensive. On examination, he was
cold and clammy with a heart rate of 130/min, and a BP of 80/40 mmHg. JVP was
normal. All peripheral pulses were present and equal. Breath sounds were
decreased at the left lung base and chest x-ray showed left pleural effusion. What
is the most likely diagnosis?
A. Acute aortic dissection
B. Acute myocardial infarction
C. Rupture of the esophagus
D. Acute pulmonary embolism
Correct answer: A

276 . Most common complication of Hiatal hernia is ?


A. Esophagitis
B. Aspiration pneumonitis
C. Volvulus
D. Esophageal stricture
Correct answer: A

277 . Left sided portal hypertension is best treated by


A. Splenectomy
B. Portocaval shunt
C. Lena-renal shunt
D. Spleno-renal shunt
Correct answer: A

- 476 -
278 . Which of the following microscopic features is typical of Paget's disease -
A. Presence of large vacuolated cells
B. Epidermal hypertrophy
C. Subdermal round cell infiltration
D. All of the above
Correct answer: A

279 . Which malignancy of parotid gland spreads through the neural sheath
A. Adenoid cystic carcinoma
B. Adenocarcinoma
C. Pleomorphic adenoma with malignant change
D. Sarcoma
Correct answer: A

280 . Preferred initial treatment for Ca anal canal is -


A. Surgery
B. Chemoradiation
C. Immunotherapy
D. Observation
Correct answer: B

281 . Minimum nitrogen required for an adult with dynamic tissue turnover to
maintain a positive nitrogen balance is -
A. 3.5-4.5 g
B. 5.5-6.6 g
C. 7.5-8.5 g
D. 9.5-10.5 g
Correct answer: B

282 . In treating Cancer prostate, the following is used except -


A. Bilateral Orchidectomy
B. Testosterone
C. LHRH - Agonist
D. Estrogen
Correct answer: B

283 . In Video assisted thoracoscopic surgery for better vision the space in the
operative field is created by:
A. Self retaining retractor
B. CO2 insufflations
C. Collapse of ipsilateral lung
D. Rib spacing
Correct answer: C

284 . Most common cause of intestinal obstruction is ?


A. Ileocecal tuberculosis
B. Carcinoma colon
C. Intussusception
D. Adhesions
Correct answer: D

- 477 -
285 . Chronic paronychia is caused by
A. Mycobacterium tuberculosis
B. Improper treatment of acute paronychia
C. Fungal infection
D. Repeated trauma
Correct answer: C

286 . A 25 year old boy reports 36 hours following a RTA with splenic injury-
100/60 mm Hg and pulse-180/minute. Investigations show splenic lower pole
laceration with hemoperitoneum. The patient is resuscitated with blood
transfusion and adequate fluid to the extend BP-120/80 mm Hg and pulse-
80/minute. Next step in management would be:
A. Splenorraphy
B. Lower pole excision
C. Splenectomy
D. Observation with close monitoring and no surgery until further deterioration
Correct answer: D

287 . True about denaturation of proteins is all except -


A. Sequence of amino acids remain the same
B. Biological activity is retained
C. Unfolding occurs
D. Disruption of secondary structure occurs
Correct answer: B

288 . Obstruction due to chronic ulcer usually occurs at -


A. Cardiac end
B. Antrum
C. Fundus
D. None of the above
Correct answer: D

289 . Most common type of hypospadias is ?


A. Glandular
B. Penile
C. Scrotal
D. Perineal
Correct answer: A

290 . A pregnant female presents with pain in abdomen on examination,


tenderness is found in right lumbar region. TLC is 12000/cmm. and urine
examination is normal. For diagnosis further test done is-->
A. Chest X-ray with abdominal shield
B. Ultrasound abdomen
C. Non contrast CT abdomen
D. Laparoscopy
Correct answer: B

- 478 -
291 . Increase in resting metabolic expenditure is seen maximally following
A. Selective surgery
B. Multiple fractures
C. Peritonitis
D. Burns
Correct answer: A

292 . Line of surgical division of the liver into left and right lobes is:
A. Gall bladder fossa to IVC
B. Gall bladder fossa to the bare area
C. IVC to falciform ligament
D. Falciform ligament to diaphragm
Correct answer: A

293 . Which of the following is seen in tumor lysis syndrome?


A. Hypokalemia
B. Hyponatremia
C. Hypercalcemia
D. None
Correct answer: D

294 . False regarding hypernephroma is -


A. Radiosensitive
B. Arise from cortex usually from preexisting adenoma
C. May present with rapidly developing varicocele
D. Usually, adenocarcinoma
Correct answer: A

295 . A 58 years has h/o IHD and atherosclerosis. He presents with abdominal
pain and maroon stools: Possible diagnosis is:
A. Acute mesenteric ischemia
B. Acute intestinal obstruction
C. Appendicitis
D. Peritonitis
Correct answer: A

296 . Burgers’ disease affects


A. Small and medium-sized vein and arteries
B. Small and medium-sized vein
C. Small-sized arteries
D. Small and medium-sized arteries
Correct answer: A

297 . Ochsner-Sherren regimen is used for ?


A. Acute appendicitis
B. Apendicular lump
C. Apendicular abscess
D. Apendicular perforation
Correct answer: B

- 479 -
298 . A surgeon exercises a portion of liver to the left of the attachment of the
falciform ligament. The segments that have been resected are:
A. Segment 1 and 4b
B. Segment 1a and 4
C. Segment 1 and 3
D. Segment 2 and 3
Correct answer: D

299 . Commonest cause of death in penetrating injury of chest -


A. Tracheobronchial injury
B. Esophageal rupture
C. Pulmonary laceration
D. Chylothorax
Correct answer: A

300 . McNeal's peripheral zone in prostate gland is the seat of -


A. Benign Hypertrophy Prostate
B. Cancer
C. Prostatitis
D. Calculi
Correct answer: B
301 . Referred pain from all may be felt along the inner side of right thigh,
except:
A. Stone in pelvic ureter
B. Pelvic abscess
C. Inflamed pelvic appendix
D. Inflamed ovaries
Correct answer: B

302 . A 50 year old man known to have diabetes, presents with, poor urinary
stream, hesitancy, difficulty in micturition and failure to void completely. What
is the diagnosis?
A. BPH
B. Autonomic neuropathy
C. UTI
D. Atonic bladder
Correct answer: B

303 . What is true about carcinoma penis -


A. Metastasis is rare
B. Occurs more commonly in circumscribed male
C. Arises from corona of glans
D. Pain in frequent
Correct answer: C

304 . A patient was operated for colonic carcinoma and later a solitary nodule
was found in the liver. Treatment of choice is:
A. Chemotherapy
B. Conservative treatment

- 480 -
C. Surgery
D. Radiation
Correct answer: C

305 . All of the following are true regarding stones in the common bile duct,
except-
A. ERCP, sphincterotomy and balloon clearance is now the standard treatment
B. When removed by exploration of the common bile ducts the T-tube can be
removed after 3 days
C. Can present with charcot's triad
D. Are suggested by a bile duct diameter > 6 mm of ultrasound
Correct answer: B

306 . Drug used for intralesional injection of keloid is -


A. Prednisolone
B. Triamcinolone
C. Androgen
D. Hydrocortisone
Correct answer: B

307 . Not true about carcinoma penis is:


A. Erythroplasia of Queret is a precancerous condition
B. 40% of pts are under 40 years of age
C. Circumcision if done any time before puberty provides 100% protection against
carcinoma penis
D. More than 50% patient have inguinal LN enlargement when they present
Correct answer: C

308 . In Nyhan's classification of hernia type 3A is ?


A. Inguinal hernia with enlarged ring
B. Recurrent hernia
C. Femoral and inguinal hernia both
D. Direct inguinal hernia
Correct answer: D

309 . Hypochloremia, hypokalemia and alkalosis are seen in:


A. Congenital hypertrophic pyloric stenosis
B. Hirschsprung's disease
C. Esophageal atresia
D. Jejunal atresia
Correct answer: A

310 . Common site for extradural hematoma is ?


A. Frontal
B. Temporo-parietal
C. Occipital
D. Brain stem
Correct answer: B

- 481 -
612 . While packing catgut suture, Preservative is-
A. Glutaraldehyde
B. Hydrogen peroxide
C. Isopropyl alcohol
D. Colloidal iodine
Correct answer: C

311. At the time of TURP, surgeon take care to dissect above the verumontanum
to prevent injury to:
A. Prostatic Utricle
B. Trigone of bladder
C. External Urethral Sphincter
D. Urethral crest
Correct answer: C

312 . Treatment of choice for Warthin’s tumor is:


A. Superficial parotidectomy
B. Enucleation
C. Radiotherapy
D. Injection of a sclerosant agent
Correct answer: A

313 . All of the following are indications for thoracotomy in blunt chest injury
except -
A. Initial drainage of 350 ml blood from the intercostal drain
B. Continuous bleeding of >200 ml / hour for 3 hours from intercostal drain
C. Continuous brisk bleeding of > 100 ml / 15 minutes from intercostal drain
D. Rupture of diaphragm
Correct answer: A

314 . What is the cell of origin of primary lymphoma of the GI tract associated
with celiac disease?
A. B cell
B. T cell
C. Histiocyte
D. Dendritic cell
Correct answer: B

315 . Which of the following statements regarding hepatoma is incorrect?


A. 80% of cases have elevated alfa-fetoprotein
B. Hepatitis-C infection can lead to hepatoma
C. Can rupture spontaneously
D. Fibrolamellar hepatoma has worse prognosis
Correct answer: D

316. A patient has small, oval multiple ulcers in oral cavity with red
erythematous margins. The diagnosis is -
A. Carcinoma
B. Aphthous ulcer

- 482 -
C. Tubercular ulcer
D. Syphilitic ulcer
Correct answer: B

317 . Normal CVP is -


A. 5-8 cm
B. 12-15 cm
C. > 20 cm
D. 0-3 cm
Correct answer: A

318 . Commonest retroperitoneal tumor is -


A. Liposarcoma
B. Teratoma
C. Lipoma
D. Neuroblastoma
Correct answer: A

319 . All are to be done in case of 20 years old female coming to causality with
right iliac fossa pain, with local guarding and tenderness, except:
A. IV glucose
B. Pethidine 100 mg.I.M.
C. Nill orally
D. None
Correct answer: D

320. Regarding testicular tumour, the following are false except


A. They are commonest malignancy in older man
B. Seminomas are radiosensitive
C. Only 25% of Stage 1 teratomas are cured by surgery alone
D. Chemotherapy rarely produces a cure in those with metastatic disease
Correct answer: B

321 . Common sites for Cushing ulcers include all of the following except:
A. Ist part of duodenum
B. Distal duodenum
C. Esophagus
D. Stomach
Correct answer: B

322 . In case of newborn the commonest cause of intestinal obstruction is -


A. Annular pancreas
B. Duodenal Atresia
C. Jejunal atresia
D. Oesophageal Atresia
Correct answer: B

323 . A neonate with a scaphoid abdomen and respiratory distress has


A. Congenital pyloric stenosis
B. Diaphragmatic hernia

- 483 -
C. Volvulus
D. Wilm's tumor
Correct answer: B

324 . Pharyngo - cutaneous Fistula is seen in all Except:


A. Chemotherapy
B. Surgery
C. No wound care
D. Radiotherapy
Correct answer: A

325 . Incidence of malignancy is maximum in -


A. Villous adenoma
B. Juvenile polyps
C. Hyperplastic polyps
D. Tubular adenoma
Correct answer: A

326 . BCC is used in treatment of


A. Bladder cancer
B. Colon cancer
C. Breast cancer
D. Thyroid cancer
Correct answer: A

327 . Empyema thoracic, treatment of choice:


A. Rib resection and drainage
B. Needle aspiration
C. Thoracic lavage
D. Antibiotic
Correct answer: A

643 . In the immediate post operative period the common cause of respiratory
insufficiency could be because of the following, except
A. Residual effect of muscle relaxant
B. Overdose of narcotic analgesic
C. Mild Hypovolemia
D. Myocardial infarction
Correct answer: C

328 . Which of the following procedure is done prior to surgery for carcinoma
rectum:
A. Rigid proctoscopy
B. MRI
C. Rectal endosonographic
D. Barium enema examination
Correct answer: C

- 484 -
329 . All of the following are true regarding Warthin’s tumour except:
A. More common in females
B. Commonly involve the parotid glands
C. They arise from the epithelial and the lymphoid cells
D. 10% are bilateral
Correct answer: A

330 . Mangled Extremity Severity Score (MESS) includes A/E-


A. Ischemia
B. Shock
C. Energy of injury
D. Neurogenic injury
Correct answer: D

331 . A 48 years man presents with progressive cervical lymph nodes


enlargement, since 3 months; Which of the following is the most diagnostic
investigation?
A. X-ray soft tissue
B. FNAC
C. Lymph node biopsy
D. None of the above
Correct answer: C

332 . A 45 years lady patient of post-cholecystectomy biliary stricture has


undergone an ERCP four days ago. Following this she has developed acute
cholangitis. The most possible organism is:
A. Streptococcus viridans
B. Pseudomonas aerogenosa
C. Escherichia colli
D. Bacillus fragilis
Correct answer: C

333 . Best treatment modality for common bile duct stone is -


A. Endoscopic sphincterotomy
B. Observation
C. Chenodeoxycholic acid
D. Percutaneous removal
Correct answer: A

334 . A patient undergoing surgery suddenly develops hypotension. The monitor


shows that the end tidal CO 2 has decreased abruptly by 155 mm Hg. What is the
probable diagnosis?
A. Hypothermia
B. Pulmonary embolism
C. Massive fluid deficit
D. Myocardial depression due to anaesthetic agent
Correct answer: B

- 485 -
335 . Post-Dural puncture headache is typically:
A. A result of leakage of blood into the epidural space
B. Worse when lying down than in sitting position
C. Bifrontal or occipital
D. Seen within 4 hours of dural puncture
Correct answer: C

336 . Ankle-brachial pressure index (ABM) value suggestive of critical ischemia


is
A. 1.0
B. 0.9
C. 0.5
D. 0.3
Correct answer: D

337 . Which of the following is NOT a component of Saint's triad?


A. Oesophageal diverticula
B. Hiatus hernia
C. Gall stones
D. Colonic diverticula
Correct answer: A

338 . Anchovy sauce pus is a feature of:


A. Splenic abscess
B. Pancreatic abscess
C. Amoebic liver abscess
D. Lung abscess
Correct answer: C

339 . Most common site of Curling's ulcer -


A. Stomach
B. Ileum
C. Esophagus
D. Duodenum
Correct answer: D

340 . The following statement about Pilonidal Sinus is true:


A. More common in females
B. Mostly congenital
C. Prognosis after surgery is poor
D. Treatment of choice is surgical excision of sinus tract
Correct answer: D

341 . Poland's syndrome of the congenital absence of the sternal head of the
pestoralis major muscle is associated with -
A. Mastalgia
B. Amazia
C. Polymazia
D. Gynecomastia
Correct answer: B

- 486 -
342 . In abdominal surgery lambert sutures refer to -
A. Single layer suturing
B. Sero muscular sutures
C. All coat intestinal suturing
D. Skin suturing
Correct answer: B

343 . A lady presented with VVF due to prolong labour one year back, on
examination ectopic opening is found on the interuretric bar close to the opening
of right ureter, Treatment of choice is:
A. Transvaginal repair of VVF
B. Transvesical VVF repair
C. Transperitoneal trans vesical fistula repair with reimplantation of right ureter
D. All of the above
Correct answer: C

344 . Most common cause of cholangitis -


A. Viral infection
B. CBD stone
C. Surgery
D. Amoebic infection
Correct answer: B

345. Commonest stone in case of UTI -


A. Phosphate
B. Urate
C. Cysteine
D. Calcium oxalate
Correct answer: A

346 . All the following are true about Meckel's diverticulum except
A. Bleeding
B. Intussusception
C. Arises at the mesenteric border
D. Located 60 cm from the cecal valve
Correct answer: C

347 . A 55 year male, working as a hotel cook, has four dependent family
members. He has been diagnosed with an early-stage squamous cell cancer of
anal canal. He has more than 60% chances of cure. Which of the following is the
best treatment option?
A. Combined surgery and radiotherapy
B. Abdomino-perineal resection
C. Chemotherapy alone
D. Combined chemotherapy and radiotherapy
Correct answer: D

- 487 -
348 . Which of the following malignant disease of children has the best
prognosis?
A. Wilm's tumor
B. Neuroblastoma
C. Rhabdomyosarcoma
D. Primitive neuroectodermal tumor
Correct answer: A

349 . Type of fibro adenosis most likely to undergo malignant change is -


A. Adenosis
B. Epitheliosis
C. Sclerosing adenosis
D. Cystic
Correct answer: B

350 . The drug of choice for schistosomiasis is -


A. Albendazole
B. Metronidazole
C. Praziquantel
D. Triclabendazole
Correct answer: C

351 . Most common site of Morgagni hernia -


A. Left anterior
B. Left posterior
C. Right anterior
D. Right posterior
Correct answer: C

352 . Which of the following Abdominal structure will be responsible for sharp
pain while doing Abdominal surgery:
A. Parietal peritoneum
B. Liver parenchyma
C. Small intestine
D. Colon
Correct answer: A

353 . Which Ca of Parotid, spreads through neural sheath -


A. Adenoid cystic carcinoma
B. Adeno carcinoma
C. Pleomorphic adenoma with malignant change
D. Sarcoma
Correct answer: A

354 . Screening for thrombophilia is indicated in


A. Recurrent venous thrombosis
B. Venous thromboembolism at less than 45 years age
C. Family history of thrombophilia
D. All of the above
Correct answer: D

- 488 -
355 . In case of Ca lung which among the following will be C/I for surgical
resection:
A. Malignant pleural effusion
B. Hilar lymphadenopathy
C. Consolidation of one lobe
D. Involvement of visceral pleura
Correct answer: A

356 . Which of the following statement is true about external hemorrhoids?


A. Painful, below the dentate line
B. Painless, above the dentate line
C. Rubber band ligation is the treatment of choice
D. May be associated with a skin tag
Correct answer: A

357 . All about radiological evaluation of a patient with Cushing's syndrome are
true except-
A. MRI of the adrenals may distinguish adrenal adenoma from carcinoma
B. Adrenal CT scan distinguished adrenal cortical hyperplasia from an adrenal tumor
C. MRI of the sella turcica will identify a pituitary cause for Cushing's syndrome
D. Petrosal sinus sampling is the best way to distinguish a pituitary tumor from an
ectopic ACTH producing tumor
Correct answer: C

358. 65-year-old men presents with Abdominal pain and distension of abdomen.
His stools were maroon colored, and he gives a past history of cerebrovascular
accident and myocardial infarction. What will be the probable diagnosis?
A. Ulcerative colitis
B. Acute mesenteric ischemia
C. Irritable bowel syndrome
D. Chron's disease
Correct answer: B

359. Imatinib mesylate used in the treatment of GIST is a inhibitor of -


A. IL-12
B. TNF-Alpha
C. Tyrosine kinase
D. VEGF
Correct answer: C

360 . Components of Glasgow Coma Scale are A/E-


A. Motor response
B. Eye opening
C. Verbal response
D. Pupil size
Correct answer: D

- 489 -
361 . Treatment of choice of Warthin’s tumour is:
A. Radiotherapy
B. Chemotherapy
C. Superficial parotidectomy
D. Excision
Correct answer: C

362 . Lung sequestration occurs most commonly in which lobe -


A. Apical
B. Left posterior basal
C. Left posterosuperior
D. Right lateral basal
Correct answer: B

363 . Pringle's maneuver is mainly used to control bleeding from ?


A. IVC
B. Hepatic artery
C. Portal vein
D. Hepatic vein
Correct answer: C

364 . In which of the following, Skip lesions are seen ?


A. Crohn’s disease
B. Ulcerative colitis
C. Tuberculosis
D. Typhoid
Correct answer: A

365 . The preferred initial treatment for carcinoma of the anal canal is -
A. Abdominoperineal resection
B. Abdominoperineal resection followed by radiotherapy
C. Radiotherapy alone
D. Chemoradiation
Correct answer: D

366 . Organism causing infection in patients following splenectomy:


A. H. Influenzae
B. Pseudomonas
C. Strep pneumoniae
D. Staphylococcos aureus
Correct answer: C

367 . Dumping syndrome is due to:


A. Presence of hypertonic content in small intestine
B. Diarrhea
C. Reduced gastric capacity
D. Vagotomy
Correct answer: A

- 490 -
368. A new born presented with bloated abdomen shortly after birth with
passing of less meconium. A full-thickness biopsy of the rectum was carried out.
Rectal biopsy findings is most likely to be present-
A. Thickened muscularis propria.
B. Lack of ganglion cells.
C. Fibrosis of submucosa.
D. Hyalinization of the muscular coat.
Correct answer: B

369. Sentinel lymph node biopsy was first done in ?


A. Carcinoma breast
B. Carcinoma colon
C. Carcinoma penis
D. Melanoma
Correct answer: C

370. A 30 year male patient presented with left sided abdominal pain to the
emergency room, 6 hours after an RTA. He was hemodynamically stable and
FAST positive. Contrast Enhanced CT(CECT)scan showed grade III splenic
laceration. The most appropriate treatment is-
A. Splenic Artery Embolization
B. Conservative Management
C. Splenectomy
D. Splenorrhaphy
Correct answer: B

371 . Alpha feta-protein is increased in which of the following?


A. Seminoma
B. Renal cell carcinoma
C. Hepatoblastoma
D. Neuroblastoma
Correct answer: C

372 . Sitaram a 40-year-old man, met with an accident and comes to emergency
department with engorged neck veins, pallor, rapid pulse and chest pain.
Diagnosis is:
A. Pulmonary Laceration
B. Cardiac tamponade
C. Hemothorax
D. Splenic rupture
Correct answer: B

373 . A 50 year old male Raju present with occasional dysphagia for solids,
regurgitation of food and foul smelling breath. Probable diagnosis is-
A. Achalasia cardia
B. Zenkers diverticulum
C. CA esophagus
D. Diabetic gastroparesis
Correct answer: B

- 491 -
374 . Features of dequervan's disease are A/E
A. Tends to regress spontaneously
B. Painful and associated with enlargements of thyroid
C. Autoimmune in etiology
D. Increase ESR
Correct answer: C

375 . CSF otorrhoea is caused by -


A. Fracture of cribriform plate
B. Fracture of parietal bone
C. Fracture of petrous temporal bone
D. Fracture of tympanic membrane
Correct answer: C

376. Treatment of extragonadal germ cell tumour is:


A. Surgery
B. Immunotherapy
C. Chemotherapy
D. Radiotherapy
Correct answer: C

377. Presence of nephroblastomata is in a biopsy specimen from Wilm's tumor of


left kidney indicates high probability of -
A. Mutation in insulin like growth factor
B. Denys-Drash syndrome
C. Lymph node metastasis
D. Increased risk of tumor in right kidney
Correct answer: D

378 . Ischemia of which of the vessels would cause least damage -


A. Renal artery
B. SMA
C. IMA
D. Celiac trunk
Correct answer: C

379. Charcot's triad includes all of the following except


A. Pain
B. Fever
C. Jaundice
D. Vomiting
Correct answer: D

380 . A post-op patient presents with peritonitis and massive contamination


because of duodenal leak. Management of choice is:
A. Four quadrant peritoneal lavage
B. Duodenostomy + feeding jejunostomy + Peritoneal lavage
C. Total parenteral nutrition
D. Duodenojejunostomy

- 492 -
Correct answer: C

381 . Meconium ileus is associated with:


A. Cirrhosis of liver
B. Malnutrition
C. Fibrocystic disease of pancreas
D. Liver aplasia
Correct answer: C

382 . Familial polyposis coli is due to -


A. Abnormality of chromosomes five
B. Abnormality of chromosome
C. Intestine tuberculosis
D. Intussusception
Correct answer: A

383 . Man presents with recurrent attacks of cholelithiasis, U/S examination


shows a dilated CBD of 1 cm. The next line of management is:
A. ERCP
B. PTC
C. Cholecystostomy
D. Intravenous cholangiogram
Correct answer: A

384 . Management of epidural abscess is ?


A. Immediate surgical evacuation
B. Conservative management
C. Antibiotics
D. Aggressive debridement
Correct answer: A

385 . True about acute epididymitis is -


A. Associated with urinary infection
B. Painless
C. Scrotum size is reduced
D. Does not mimic with torsion of testes
Correct answer: A

386. Least chances of complication like dumping syndromes, and diarrhea is seen
in:
A. Highly selective vagotomy
B. Truncal vagotomy and Gastrojejunostomy
C. Antrectomy with truncal vagotomy
D. Vagotomy with pyloroplasty
Correct answer: A

387 . Not true regarding Dandy Walker cyst -


A. Cerebellar Vermis hypoplasia
B. Hydrocephalus
C. Arachnoid Cyst

- 493 -
D. Posterior fossa cyst
Correct answer: C

388 . Which statement regarding inguinal hernia is true?


A. Neck of the sac lies below and lateral to pubic tubercle
B. Direct inguinal hernia bulges medial to inferior epigastric vessels
C. The indirect inguinal hernia is more likely to get strangulated than indirect
inguinal hernia
D. Direct inguinal hernia is in front of conjoin tendons
Correct answer: B

389 . Tumor marker for primary hepatocellular carcinoma are all except -
A. Alpha 2 macro globin
B. Alpha feto protein
C. Neurotension
D. PIVKA-2
Correct answer: A

390 . Which of the following dye is used in chromoendoscopy for detection of


cancer?
A. Hemotoxiline and eosine
B. Methylene blue
C. Gentian violet
D. Toluidine blue
Correct answer: B

391 . Which is not elevated in a child presenting with jaundice, icterus, pruritus
and clay-colored stools.
A. 5' - nucleotidase
B. Glutamate dehydrogenase
C. Gamma glutamyl transpeptidase
D. Alkaline phosphatase
Correct answer: B

392 . Pseudo obstruction of intestine is also known as ?


A. Hartmann's syndrome
B. Ozili's syndrome
C. Ogilive's syndrome
D. Merizzi's syndrome
Correct answer: C

393 . Treatment of pleomorphic adenoma of parotid is


A. Deep lobe removal
B. Total removal
C. Superficial parotidectomy
D. Radical removal
Correct answer: C

- 494 -
394 . After cholecystectomy tensile strength of incision is improved by -
A. Granulation tissue formation
B. Cross linking of protocollagen
C. Macrophage invasion
D. Replacement of type iii collagen
Correct answer: B

395. Which of the following statement is true regarding subclavian steal


syndrome -
A. Reversal of blood flow in the ipsilateral vertebral artery
B. Reversal of blood flow in the contralateral carotid artery
C. Reversal of blood flow in the contralateral vertebral artery
D. Bilateral reversal of blood flow in the vertebral arteries
Correct answer: A

396 . Which of the following is C/I for Bag and mask ventilation?
A. Septicemia
B. Tracheoesophageal fistula
C. Meconium aspiration
D. Diaphragmatic hernia
Correct answer: D

397 . Which of the following is the most common cause of acute mesenteric
ischemia ?
A. Venous thrombosis
B. Arterial thrombosis
C. Non occlusive disease
D. Embolism
Correct answer: D

398 . Not true about polycystic kidney disease is -


A. Autosomal dominant
B. Proteinuria<2 gm/day
C. Leads to CRF
D. Decompression of cyst leads to normal renal function
Correct answer: D

399 . Hyoid bone is closely associated with:


A. Cystic hygroma
B. Branchiogenic cyst
C. Branchial cyst
D. Thyroglossal cyst/fistula
Correct answer: D

400 . Regarding adhesive intestinal obstruction, true is -


A. Avoid surgery for initial 48-72 hours
B. Never operate
C. Operate after minimum 10 days of conservative treatment
D. Immediate operation
Correct answer: A

- 495 -
401 . Margolin’s ulcer is characterized by all EXCEPT
A. Development in a scar
B. Slow growth
C. Painless nature
D. Spreads to lymph nodes
Correct answer: D

402 . 40 years old male, chronic smoker presents with claudication and a medial
leg ulcer. For the past one month he is having rest pain. Which of the following
procedures would not relieve his rest pain:
A. Lumber Sympathectomy
B. Omentoplasty
C. Conservative Amputation
D. Femoropopliteal bypass
Correct answer: D

403 . Most common content in 'Hernia on glissade' is


A. Omentum
B. Urinary bladder
C. Caecum
D. Sigmoid colon
Correct answer: D

404 . For thyroglossal cyst MOST COMMON site is -


A. Suprahyoid
B. Sub hyoid
C. Foramen caecum
D. Anterior border of sternomastoid
Correct answer: B

405 . Blood stained nipple discharge suggests which diagnosis -


A. Ductal papilloma
B. Plasmacytoma
C. Fibro adenosis
D. Fibroadenoma
Correct answer: A

406 . Bishop Koop operating done for -


A. Meconium ileus
B. Hirschsprung’s disease
C. Duodenal atresia
D. Meckel's diverticulum
Correct answer: A

407 . Which is not a preferred site for planning vascular access for maintenance
hemodialysis?
A. Radio cephalic AV fistula
B. Saphenofemoral fistula
C. Nondominant extremity

- 496 -
D. Upper limb
Correct answer: B

408 . Budd-Chiari syndrome is due to:


A. Thrombosis of hepatic veins
B. Obstruction due to hepatic valves
C. Web in the hepatic veins
D. Thrombosis of the inferior vena cava
Correct answer: A

409 . Most common cause of biliary stricture is -


A. CBD stone
B. Trauma
C. Asiatic cholangitis
D. Congenital
Correct answer: B

410 . Which does not predispose to cholangiocarcinoma -


A. Ulcerative colitis
B. Clonorchis sinensis
C. Choledochal cyst
D. Chronic pancreatitis
Correct answer: D

411 . The prognosis of rhabdomyosarcoma is poor in which site?


A. Para testicular
B. Orbit
C. Urinary bladder
D. Extremity
Correct answer: D

412 . Not true about Bockdaleck hernia -


A. Seen in Rt site
B. Associated with hypoplasia of lung
C. Associated with hiatus hernia
D. Pericardial cyst is a differential diagnosis
Correct answer: A

413 . All of the following are features of frontal lobe abscess, except -
A. Personality change
B. Urinary incontinence
C. Anosmia
D. Memory is intact
Correct answer: D

414 . True about medulloblastoma is -


A. Highly radiosensitive
B. Surgery is the only treatment
C. Occurs in adult age group
D. Chemotherapy is useful

- 497 -
Correct answer: A

415 . At the time of reconstruction of an amputated limb, the first step is-
A. Venous repair
B. Arterial repair
C. Nerve anastomoses
D. Fixation of the bone
Correct answer: D

416 . Pringle maneuver is a means to stop bleeding from -


A. Celiac axis
B. Liver and cystic artery
C. Cystic artery
D. Duodenal ulcer
Correct answer: B

417 . The rectum measures -


A. 18 cm
B. 22 cm
C. 17 cm
D. 15 cm
Correct answer: D

418 . Clinically a saphena varix is likely to be confused with


A. Baker's cyst
B. Femoral hernia
C. Spermatocele
D. Varicocele
Correct answer: B

419. Most common testicular tumor in prepubertal adults is -


A. Seminoma
B. Teratoma
C. Yolk sac tumor
D. Embryonal cell Ca
Correct answer: C

420 . A 35 year male with stab injury to anterior abdomen presents with a tag of
omentum protruding through the abdominal wall near the umbilicus. On
evaluation he is hemodynamically stable and shows no signs of peritonitis. Initial
management of patient should be-
A. Local Wound Exploration and Suturing
B. CECT Abdomen
C. FAST
D. Exploratory Laparotomy
Correct answer: B

- 498 -
421 . For Ca of Anal canal which one is the treatment of choice -
A. Radiotherapy + APR
B. Only radiotherapy
C. Chemoradiation
D. Radiotherapy + chemotherapy
Correct answer: C

422 . Surgery for undescended testes should be done before which of the
following age?
A. 12 months
B. 6 months
C. 36 months
D. 24 months
Correct answer: A

423 . Madura foot is


A. An infection caused by a type of actinomycetes
B. An infection caused by a type of candida
C. A swelling due to an infected perforating ulcer
D. The same as a trench foot
Correct answer: A

424 . False about testicular tumor is-


A. Seminoma can be treated by orchidectomy with radiotherapy
B. Seminoma spreads commonly through lymphatics
C. Pure seminoma is more aggressive than nonseminoma
D. Seminoma is radiosensitive
Correct answer: C

425 . A-75 year-old frail elderly man underwent TURP. The biopsy revealed
adenocarcinoma. What is the next line of management
A. Radiotherapy
B. Surgery followed by Hormonal replacement therapy
C. Constructive treatment
D. Surgery followed by radiotherapy
Correct answer: C

426 . True regarding cystic hygroma is -


A. Non transilluminate
B. Lined by columnar epithelium
C. Lined by stratified squamous epithelium
D. Develops from jugular lymphatic sequestration
Correct answer: D

427 . Sheela, a middle-aged diabetic female presented with flank pain and fever.
On U/S the kidney was irregular and showed fat density lesion with calculi. The
diagnosis is most probably:
A. TB kidney
B. Xanthogranulomatous kidney

- 499 -
C. Chronic pyelonephritis
D. Renal abscess
Correct answer: B

428 . The commonest type of trachea-esophageal fistula is:


A. Proximal end blind and distal end communicating with trachea
B. Distal end blind and proximal end communicating with trachea
C. Both ends blind
D. Both ends open
Correct answer: A

429 . A 63 years male patient of coronary artery disease was on Aspirin for 2
years. He now complains of black stools. Abdominal examination is normal. The
most likely diagnosis is-
A. Esophageal varices
B. Duodenal ulcer
C. Ileocecal TB
D. Ca colon
Correct answer: B

430 . Regular drinking of which of the following fruit juices can prevent UTI -
A. Raspberry
B. Grape
C. Cranberry
D. Orange
Correct answer: C

431 . Disparity of the bowel ends during end to end anastomosis is corrected by -
A. Cheatle's manoeuvre
B. Connell suture
C. Lembert suture
D. Czerny technique
Correct answer: A

432 . Which of the following is the diagnostic method of choice for Hirschsprung
disease?
A. Contrast enema
B. MRI
C. Rectal Biopsy
D. Manometry
Correct answer: C

433 . A 6 years boy presents with recurrent abdominal pain and bilious vomiting.
Condition was diagnosed by barium follow through. Surgery was done, - mesenteric
widening, appendicectomy, cutting the Ladd's band. The diagnosis is-
A. Mal rotation
B. Recurrent Caecal volvulus
C. Stricture TB
D. Recurrent appendicitis
Correct answer: A

- 500 -
434 . All are true regarding Kaposi sarcoma except one -
A. Predominant in male
B. Multicentric origin
C. Chemotherapy is treatment of choice
D. Occurs in AIDS patient only
Correct answer: D

435 . Regarding bile duct injury following Cholecystectomy which of the


following statement is false -
A. The incidence following open cholecystectomy is in the range of 0.2-0.3%
B. The incidence rate following Laparoscopic cholecystectomy is three times higher
than the rates following open cholecystectomy
C. Untreated cases may develop secondary biliary cirrhosis
D. Routine use of 'open' technique laparoscopic port insertion has resulted in a
decline in the incidence of post laparoscopic cholecystectomy bile duct injuries
Correct answer: D

836 . A 70-year-old patient with thigh and calf claudication, and ulcers of lower
extremity, treatment of choice
A. Streptokinase
B. Thrombolysis
C. Aorto femoral bypass
D. Embolectomy
Correct answer: C

437 . Treatment of Desmoid tumor is -


A. Conservative
B. Radiotherapy
C. Surgery
D. Radiotherapy following chemotherapy
Correct answer: C

438 . Abbey estender flap is based on -


A. Facial artery
B. Lingual artery
C. Internal maxillary artery
D. Labial artery
Correct answer: D

439 . All are true regarding primary CNS lymphoma except -


A. Radiotherapy and chemotherapy is of no value
B. Occurs in AIDS patient
C. Commonly occurs in immuno-compromised persons
D. EBV may be cause
Correct answer: A

- 501 -
440 . A case of spontaneous pneumothorax comes to you what will be earliest the
treatment of choice -
A. IPPV
B. Needle aspiration
C. ICD
D. Wait and watch
Correct answer: B

441 . Mayo's operation done in-


A. Epigastric hernia
B. Umbilical hernia
C. Incisional hernia
D. Lumbar hernia
Correct answer: B

442 . Hypercatabolic state is marked by increase in metabolic rate and protein


catabolism of more than
A. 25%
B. 50%
C. 75%
D. 100%
Correct answer: A

443 . Which is not true about Sturge weber syndrome -


A. Portwine Stain
B. Calcification in brain
C. Cortical atrophy
D. Intra cranial hamartoma
Correct answer: D

444 . A patient of peptic ulcer disease. When investigated endoscopically showed


chronic antral gastritis. Which of the following dye will be able to stain the
specimen:
A. PAS
B. Zeil-Neilson stain
C. Gram stain
D. Warthin-Starry stain
Correct answer: D

445 . Excessive bleeding during hemothorax is caused by usually -


A. Vena cava
B. Internal mammary artery
C. Heart
D. Major artery
Correct answer: D

446 . A 21 days male baby presenting with vomiting and failure to thrive is found
to have pyloric stenosis. The next step of management is-
A. Fluid resuscitation may be delayed
B. It is emergency so do pyloromyotomy immediately

- 502 -
C. Cardiopulmonary resuscitation
D. Correction of electrolyte disturbances
Correct answer: D

447 . A 65 years male was diagnosed as carcinoma right lung. On CECT chest
there was a tumor of 5x5 cm in upper lobe and another 2x2 cm size tumor nodule
in middle lobe. Which of the following is the primary modality of treatment ?
A. Chemotherapy
B. Radiotherapy
C. Supportive treatment
D. Surgery
Correct answer: D

448 . Which of the following is not included in Charcot's triad?


A. Jaundice
B. Gallbladder lump
C. Fever
D. Pain
Correct answer: B

449 . Testicular teratoma has all marker except -


A. AFP
B. HCG
C. CEA
D. LDH
Correct answer: C

450 . In case of a long standing goitre, ca most probable to occur is -


A. Follicular
B. Papillary
C. Anaplastic
D. Medullary
Correct answer: A

451 . Ileal resection for intusussception in adults would be done in which?


A. Lymphoma
B. Carcinoid Tumor
C. Soft tissue
D. Villous adenoma
Correct answer: D

452 . The commonest worm implicated in intestinal obstruction is


A. Ankylostoma
B. Ascaris
C. Enterobius
D. Taenia saginata
Correct answer: B

- 503 -
453 . In a survey, many children are examined and were found to have urogenital
abnormalities. Which congenital anomaly is associated with increased risk of
bladder carcinoma ?
A. Bladder exstrophy
B. Medullary sponge kidney
C. Double ureter
D. Unilateral renal agenesis
Correct answer: A

454 . All are true about Gleason score, except:


A. Scores range from 1-10
B. Used for grading prostate cancer
C. Helps in planning management
D. Higher the score, poorer the prognosis
Correct answer: A

455 . Which of the following causes least irritation of the peritoneal cavity:
A. Blood
B. Bile
C. Pancreatic enzyme
D. Gastric enzyme
Correct answer: A

456 . All of the following drugs are used in treatment of Hirsutism except -
A. Cyproterone acetate
B. Spironolactone
C. Flutamide
D. Mifepristone
Correct answer: D

457 . Meckel's diverticulum arises from anti-mesenteric border of


A. Duodenum
B. Ileum
C. Jejunum
D. Transverse colon
Correct answer: B

458 . The causal mechanism in carcinoid syndrome is due to -


A. Erythropoietin and immunoglobulin
B. Serotonin and bradykinin
C. Insulin and insulin like substances
D. ACTH and ACTH like substances
Correct answer: B

459 . What is the Meaning of Triage -


A. Sorting out of cases on availability of medical resources and severity of pt
condition
B. patient is divided in three groups
C. Severely injured patient is taken first
D. Used in military hospital

- 504 -
Correct answer: A

460 . Inspissated bile syndrome is usually seen in -


A. Choldochocoele
B. Excessive hemolysis in infancy
C. Cholecystitis
D. Cholangitis
Correct answer: B

461 . Apart from Escherichia coli, which of the bellow is the most common
organism implicated in acute suppurative bacterial peritonitis?
A. Pepto streptococcus
B. Pseudomonas aeruginosa
C. Bacteroides
D. Klebsiella
Correct answer: C

462 . Which among the following parotid tumor spreads through neural sheath -
A. Mixed parotid tumor
B. Adenocystic Ca
C. Sq. cell Ca
D. Oxyphilic lymphoma
Correct answer: B

463 . Acute bacterial sialadenitis is managed by -


A. Immediate sialography
B. Fluid intake restriction
C. Culture of the FNAC from the involved gland
D. Use of blood spectrum antibiotics
Correct answer: D

464 . Hernia with highest rate of strangulation is ?


A. Direct inguinal hernia
B. Indirect inguinal hernia
C. Femoral Hernia
D. Incisional hernia
Correct answer: C

465 . A 23 year male with azoospermia found to have normal FSH and
testosterone levels and normal size testes. Possible cause is -
A. Undescended testis
B. Klinefelter’s syndrome
C. Vas obstruction
D. Kalman syndrome
Correct answer: C

466 . Tumors associated with infective etiology are A/E-


A. Hepatocellular CA
B. Nasopharyngeal CA
C. Gastric CA

- 505 -
D. Non - small cell CA lung
Correct answer: D

467 . A 49-year-old man suffering from carcinoma of prostate was x-rayed. He


showed areas of sclerosis and collapse of T10 and T11 vertebrae in x-ray. The
spread of this cancer to the above vertebrae was through
A. Sacral canal
B. Lymphatic vessels
C. Internal vertebral plexus of veins
D. Superior rectal vein
Correct answer: C

468 . Cause of Haemobilia All/Except-


A. Trauma to Abdomen
B. Malignancy
C. Rupture of hepatic artery aneurysm
D. Hepatitis
Correct answer: D

469 . Which malignancy would occur in prolonged multinodular goitre:


A. Papillary Ca
B. Follicular Ca
C. Anaplastic Ca
D. Medullary Ca
Correct answer: B

470 . Hypotension in acute spinal injury is due to -


A. Orthostatic hypotension
B. Vasovagal attack
C. Loss of sympathetic tone
D. Loss of parasympathetic tone
Correct answer: C

471 . Best indication for Testicular biopsy in a male is


A. Necrospermia
B. Azoospermia
C. Polysperamia
D. Oligospermia
Correct answer: B

472 . Gangrene of intestine is seen in all the following conditions, except:


A. Mesenteric Artery Thrombosis
B. Volvulus
C. Tricuspid Valve Endocarditis
D. Shock
Correct answer: C

473 . Persistent fetal lobulation of adult kidney is due to -


A. Intrauterine infections and scar
B. Is a normal variant

- 506 -
C. Congenital renal defect
D. Obstructive uropathy
Correct answer: B

474 . A 65 years man develops testicular tumor; most possible to be:


A. Teratocarcinoma
B. Lymphoma
C. Germ cell tumor
D. Sertoli cell tumor
Correct answer: B

475. The most troublesome source of bleeding during a radical retropubic


prostatectomy is-
A. Superior vesical pedicle.
B. Seminal vesicular artery.
C. Dorsal venous complex.
D. Inferior vesical pedicle.
Correct answer: C

476 . A 17 years old girl presents with history of generalized abdominal pain,
fever, recurrent vomiting. On examination, she has a temperature of 103 degree
F, Right Iliac fossa tenderness; and WBC are 19600/mm3:
A. Ruptured Ectopic Pregnancy
B. Intussusception
C. Ruptured Appendicular Abscess
D. Twisted ovarian cyst
Correct answer: C

477 . Laparoscopic cholecystectomy done in a patient with cholelithiasis.


Pathology report shows adenocarcinoma with invasion of muscular layer. CT
was normal. Further treatment is-
A. Wedge hepatic resection with lymph node dissection
B. Wait and regular follow up
C. Radiotherapy
D. Excise all port sites
Correct answer: A

478 . Thyrotoxic myopathy affects -


A. Muscles of the neck
B. Proximal limb muscles
C. Distal limb muscles
D. Muscles of the tongue
Correct answer: B

479 . Who said these words 'To study phenomenon of disease without books is to
sail an uncharted sea, while to study without patients is not to go to sea at all '
A. Hamilton Bailey
B. Sir Robert Hutchison
C. Sir William Osler

- 507 -
D. JB Murphy
Correct answer: C

480 . Which of the following will give Medusa head colony on x-ray:
A. Hook worm infestation
B. Round worm infestation
C. Tenia solium infestation
D. a + b
Correct answer: B

481 . Most common cause of amoebic lung abscess is -


A. Direct extension from liver
B. Hematogenous spread
C. Lymphatic spread
D. By inhalation
Correct answer: A

482 . Pancreatic pseudocyst the best management is:


A. Cysto jejunostomy
B. Cysto gastrostomy
C. Excision of the cyst
D. External drainage
Correct answer: A

483 . A 4-year-old child presents with ballooning of prepuce after micturition.


Examination of penis reveals preputial adhesions. The best treatment is-
A. Circumcision
B. Adhesolysis and dilatation
C. Conservative management
D. Dorsal slit
Correct answer: A

484 . Lipodermato sclerosis is most commonly seen at ?


A. Anterior aspect of leg
B. Medial aspect of leg
C. Anterior aspect of thing
D. Posterior aspect of thing
Correct answer: B

485 . Most common complication after splenectomy is -


A. Chest infection
B. Hematemesis
C. Subphrenic collection
D. Acute dilation of stomach
Correct answer: A

486 . Laugier's hernia is a type of -


A. Inguinal hernia
B. Femoral hernia
C. Incisional hernia

- 508 -
D. Obstructed hernia
Correct answer: B

487 . After RTA a young male present with non pulsatile retroperitoneal
hematoma. On table IVU was done. Right kidney not visualized. Left kidney
showed immediate excretion of dye. What is the next step in management -?
A. Nephrectomy
B. Open Gerota's facia and explore proximal renal vessels
C. Performed Retrograde pyelography
D. Perform on table angiography
Correct answer: D

488 . In a post operative ICU, five patients developed post-operative wound


infection on the same wound. The best method to prevent cross infection
occurring in other patients is
A. Disinfect the ward with sodium hypochlorite
B. Practice proper hand washing
C. Give antibiotics to all other patients in the ward
D. Fumigate the ward
Correct answer: B

489. Antiphospholipid antibody syndrome is characterized by all of the following


features except -
A. Thrombocytosis
B. Arterial and venous thrombosis
C. Recurrent abortions
D. Livedo reticularis
Correct answer: A

490 . Ureter is diagnosed during operation by:


A. High arterial supply
B. Venous plexus
C. Circumference
D. Peristaltic movements
Correct answer: D

491 . Features of abdominal tuberculosis include the following except for -


A. Ascites
B. Sub-acute intestinal obstruction
C. Absence of pulmonary tuberculosis in all cases
D. Mass in right iliac fossa
Correct answer: C

492 . Ectopic ureter opening is not located in -


A. Bulbar urethra
B. Prostatic urethra
C. Seminal vesicle
D. Bladder neck
Correct answer: A

- 509 -
493 . Urine gets collected in which place in case of extra peritoneal rupture of
bladder -
A. Groin
B. Below urogenital diaphragm
C. Above urogenital diaphragm
D. Perineal space
Correct answer: C

494 . Rx of choice in postoperative lung collapse is A/E -


A. Needle drainage
B. Corticosteroids
C. Pulmonary resection
D. Endoscopic suction
Correct answer: D

495 . In pyogenic liver abscess commonest route of spread is -


A. Hematogenous through portal vein
B. Ascending infection through biliary tract
C. Hepatic artery
D. Local spread
Correct answer: B

496 . Ludwig's angina -


A. May result in glottic edema
B. Is caused by staphylococcal infection
C. Has only intraoral signs
D. Is characterized by soft swelling in the sub mandibular region
Correct answer: A

497. Dysphagia lusoria is due to:


A. Oesophageal diverticulum
B. Aneurysm of aorta
C. Oesophageal web
D. Compression by aberrant blood vessel
Correct answer: D

498 . 'Limey bile ' is -


A. Present in the CBD
B. Thin and clear
C. Like toothpaste emulsion in the gall bladder
D. Bacterial rich
Correct answer: C

499 . Which among the following will complicate as hyperchloremic acidosis -


A. Ureterosigmoidostomy
B. Diarrhea
C. Vomiting
D. Ileoplasty
Correct answer: A

- 510 -
500 . Most common site of anorectal abscess is ?
A. Perianal
B. Ischiorectal
C. Pelvirectal
D. Intersphincetric
Correct answer: A

501 . Which one is not radiosensitive -


A. Ewing's sarcoma
B. Lymphoma
C. Seminoma testis
D. Osteosarcoma
Correct answer: D

502 . Which of the following is the best prognostic factor for head injury?
A. Mode of injury
B. CT findings
C. Glasgow coma scale
D. Age
Correct answer: C

503 . The peak flow rate which indicates the degree of bladder obstruction
should be -
A. Less than 100 ml/sec
B. Less than 50 ml/sec
C. Less than 25 ml/sec
D. Less than 15 ml/sec
Correct answer: D

504 . Most common cause of death in penetrating injury to chest


A. Chylothorax
B. Pulmonary contusion
C. Tracheo bronehial injury
D. Oesophageal rupture
Correct answer: C

505 . Sialography is contraindicated in


A. Chronic parotitis.
B. Ductal calculus.
C. Recurrent sialadenitis.
D. Acute parotitis.
Correct answer: D

506 . Most common causes of acute mesenteric ischemia is -


A. Venous thrombosis
B. Arterial thrombosis
C. Non occlusive disease
D. Embolism
Correct answer: D

- 511 -
507 . In Richter's hernia the sac contains -
A. Greater omentum
B. Caecum
C. Part of the small gut wall
D. Spleen
Correct answer: C

508. The treatment of choice for symptomatic, retained common bile duct stones,
is:
A. Immediate surgery
B. Conservative treatment with antibiotics
C. Endoscopic sphincterotomy
D. Medical dissolution of the stones
Correct answer: C

509 . Rupture of which artery leads to extradural haemorrhage -


A. Maxillary artery
B. Middle meningeal artery
C. Internal carotid artery
D. Ophthalmic artery
Correct answer: B

510. Rupture of bulbar urethra is associated with all except-


A. Fractured pelvis
B. Blood per urethra
C. Perineal hematoma
D. Distended bladder
Correct answer: A

511 . Most common post operative complications of ileo anal pouch anastomosis
in ulcerative colitis is -
A. Pouchitis
B. Pelvic abscess
C. Small bowel obstruction
D. Perianal complication
Correct answer: A

512 . From the index finger infection goes to -


A. Thenar space
B. Hypothenar space
C. Mid palmar space
D. Space of parona
Correct answer: A

513 . A victim of road traffic accident came to emergency with breathlessness,


cyanosis, and external chest wall injuries. On examination, trachea is deviated to
left side. Immediate management is:
A. Arterial blood gas analysis
B. Wide-bore needle insertion on right second intercostal space
C. X-ray chest P / A view

- 512 -
D. Watchful observation
Correct answer: B

514 . Characteristics of cyst sarcoma phylloides includes all except:


A. Malignant changes is common
B. Stomal component>epithelial component in the mass
C. Is giant fibroadenoma
D. Wide local excision or simple mastectomy done
Correct answer: A

515 . Spermatic duct involvement in testicular tumor


A. T1
B. T2
C. T3
D. T4
Correct answer: C

516 . Which malignancy would occur in long standing multinodular goitre ?


A. Papillary Ca
B. Follicular Ca
C. Anaplastic Ca
D. Medullary Ca
Correct answer: B

517 . Following are the complaints of a patient with history of ischemic heart
disease, acute abdominal pain, tenderness, distension of abdomen, absent blow
sound, maroon colored blood in stool. Diagnosis is -
A. Mesenteric ischemia
B. Colon carcinoma
C. Ulcerative colitis
D. Crohn's disease
Correct answer: A

518 . Which of the following is the most preferred approach for pituitary
surgery?
A. Trans ethmoidal
B. Transcranial
C. Transcallosal
D. Trans-sphenoidal
Correct answer: D

519 . The treatment of choice for adult hypertrophic pyloric stenosis is:
A. Pyloromyotomy
B. Pyloroplasty
C. Billionth’s I anastomosis
D. Gastrectomy
Correct answer: B

- 513 -
520 . Clay colored stools are seen in
A. Hemolytic jaundice
B. Hepatic jaundice
C. Obstructive jaundice
D. Amoebic hepatitis
Correct answer: C

521 . Which of the following is not a congenital sinus ?


A. Pilonidal sinus
B. Umbilical
C. Urachal
D. Sacral
Correct answer: A

522 . A Fogarty catheter is designed to be used for


A. Parenteral nutrition
B. Ureteric catheterization
C. Removing blood clot from arteries
D. Arteriography
Correct answer: C

523 . Immediate surgery is not done in one of the following types of intestinal
obstruction:
A. Postoperative adhesion
B. Volvulus
C. Strangulated inguinal hernia
D. Paralytic ileus following appendicular perforation
Correct answer: A

524. Commonest site of accessory spleen is -


A. Lieno renal ligament
B. Hilum of spleen
C. Gastro splenic ligament
D. Around tail of pancreas
Correct answer: B

525 . After ureterosigmoidostomy which electrolyte abnormality may occur:


A. Hyperchloremic acidosis
B. Metabolic alkalosis
C. Metabolic acidosis
D. Hypochloremic acidosis
Correct answer: A

526 . Prehn's sign is used for the differential diagnosis between:


A. Acute appendicitis and Meckel's diverticulum
B. Acute testicular torsion and epididymoorchitis
C. Acute cholecystitis and perforated peptic ulcer
D. Acute pancreatitis and acute pyelonephritis
Correct answer: B

- 514 -
527. Chronic Burrowing ulcer is caused by -
A. Streptococcus viridans
B. Streptococcus pyogenes
C. Microaerophilic streptococci
D. Pepto streptococcus
Correct answer: C

528. Richter's hernia -


A. Is a strangulated hernia
B. Is a hernia in which sac contains a portion of the circumference of the intestine?
C. When appendix is in the hernial sac
D. Is a sliding hernia
Correct answer: B

529 . The initial therapy of documented DVT in a postoperative case is -


A. Subcutaneous heparin therapy
B. IV heparin therapy
C. Antithrombosis therapy with urokinase
D. Aspirin therapy
Correct answer: A

530 . Anorchia confirmed by -


A. Laparoscopy
B. Pet CT
C. USG
D. SPECT
Correct answer: A

531 . CABG is done for all of the following indications except


A. To prolong life
B. To prevent progress of native blood vessel disease
C. To reduces symptoms
D. To prevent further catastrophic
Correct answer: B

532 . 32-year-old female, from rural background presented with a h/o chronic
tobacco chewing since 14 year of age. Now she has difficulty in opening her
mouth. On oral examination, no ulcers are seen. Most probable diagnosis is:
A. Submucous oral fibrosis
B. Carcinoma of buccal mucosa
C. TM joint arthritis
D. Trigeminal nerve paralysis
Correct answer: A

533 . Which of the following is the most effective intravesical therapy of


superficial bladder cancer -
A. Mitomycin
B. Adriamycin
C. Thiotepa
D. BCG

- 515 -
Correct answer: D

534 . Right hepatic duct drains all, Except


A. Segment III
B. Segment I
C. Segment VI
D. Segment V
Correct answer: A

535 . Drug of choice for infection by Gardnerella vaginitis is -


A. Ampicillin
B. Metronidazole
C. Vancomycin
D. Cephalosporin
Correct answer: B

536 . A 45 year old gentleman has undergone truncal vagotomy and pyloroplasty
for bleeding duodenal ulcer seven years ago. Now he was intractable recurrent
symptoms of peptic ulcer. All of the following suggest the diagnosis of Zollinger
Ellison syndrome, except -
A. Ulcers in proximal jejunum and lower end of esophagus
B. Serum gastrin value of 200 pg/ml with secretin stimulation
C. Basal acid output of 15 meq/hour
D. Serum gastrin value of 500 pg/ml
Correct answer: B

537 . The operative wound after surgery for gangrenous, perforated appendicitis
is best managed by -
A. Primary suture
B. Delayed primary closure
C. Healing by secondary intention
D. Antibiotic lavage
Correct answer: B

538. In Couinaud classification, segment IV of liver is -


A. Quadrate lobe
B. Caudate lobe
C. Left lobe
D. Right lobe
Correct answer: A

539 . Most common type of Intussusception is:


A. Cola-colic
B. Caeca-colic
C. Ilia-colic
D. Ilia-ileal
Correct answer: C

- 516 -
540 . All are middle mediastinal masses except:
A. Ganglioneuroma
B. Aortic aneurysm
C. Bronchogenic cyst
D. Pleuro - pericardial cyst
Correct answer: A

541 . Duodenal blow out following Billroth gastrectomy most commonly occurs
on which day -
A. 2rd day
B. 4th day
C. 6th day
D. 12th day
Correct answer: B

542 . Intralobal sequestration of lung takes its blood supply from -


A. Internal mammary artery
B. Descending abdominal aorta
C. Pulmonary artery
D. None of the above
Correct answer: B

543 . Renal cell carcinoma -


A. Rarely presents with hematuria
B. Pyrexia is not a feature
C. Tumor thrombus in renal vein is characteristic
D. Is commonly seen in children
Correct answer: C

544 . To differentiate between stress incontinence and detrusor instability


investigation done is -
A. Cysto-urethroscopy
B. Urodynamic study
C. MCU
D. Retrograde urethroscopy
Correct answer: B

545 . A 15-year-old boy is admitted with a history and physical findings


consistent with appendicitis. Which of the following findings is most likely to be
positive?
A. Pelvic crepitus
B. Iliopsoas sign
C. Murphy's sign
D. Flank ecchymosis
Correct answer: B

546 . Metabolic abnormality seen in large colorectal villous adenoma -


A. Hypokalemic metabolic acidosis
B. Hypokalemic metabolic alkalosis
C. Chlorine resistant metabolic alkalosis

- 517 -
D. Chlorine sensitive metabolic acidosis
Correct answer: A

547 . Biliary stricture developing after laparoscopic choice-cystectomy usually


occurs at part of the common bile duct is-
A. Lower
B. All side with equal frequency
C. Upper
D. Middle
Correct answer: C

548 . Clinical feature of thromboangitis obliterans are


A. Claudication of extremities
B. Raynaud's phenomenon
C. Migratory superficial thrombophlebitis
D. Absence of popliteal pulse
Correct answer: D

549. Which muscles maintains transverse arch of foot -


A. Adductor hallucis
B. Abductor halluces brevis
C. Peroneus longus and peroneus brevis
D. Extensor digital brevis
Correct answer: A

550 . Which is not true about thoracic outlet syndrome -?


A. Radial nerve is commonly affected
B. Neurological features are most common
C. Resection of 1 st rib relieves symptom
D. Positive Adson's test
Correct answer: A

551 . All of the following are Brilliantly transilluminated swellings, except-


A. Meningocele
B. Hydrocele
C. Cystic hygroma
D. Lipoma
Correct answer: D

552 . Which of the following is not a sign of appendicitis?


A. Rovsing's sign
B. Murphy's sign
C. Obturator sign
D. Psoas sign
Correct answer: B

553 . Polyp associated with malignancy except:


A. Familial polyposis
B. Juvenile polyp

- 518 -
C. Adenomatous polyp
D. All
Correct answer: B

554 . A 70 year old patient underwent right hemicolectomy after 7 days he


presented with abdominal distension with hemorrhage from the wound. Next
step would be:
A. Look for internal hemorrhage from anastomotic leak
B. Observe for wound dehiscence
C. Barium meal for detection of intestinal obstruction
D. Do a coagulation profile urgently
Correct answer: B

555 . True about Ogilvie’s syndrome are all except -


A. It occurs after previous surgery
B. It occurs after commonly after narcotic use
C. It is caused by mechanical obstruction of the colon
D. It involves entire/part of large colon
Correct answer: C

556 . Least amenable to screening is -


A. Breast
B. Cervix
C. Oral cavity
D. Lung
Correct answer: D

557 . Which of the following is the treatment of choice for the management of
carcinoma of the anal canal?
A. Primary radiotherapy
B. Abdominoperineal resection
C. Neoadjuvant chemotherapy and local excision
D. Combined radio and chemotherapy
Correct answer: D

558 . What is true about carcinoma bladder -


A. Common in smokers
B. Commoner in females than that in males
C. Mostly adenocarcinoma
D. Pain in suprapubic region is the first symptom
Correct answer: A

559 . By mucosal resection which carcinoma can be diagnosed early -


A. Oesophageal Carcinoma
B. Anal Carcinoma
C. Colon Carcinoma
D. Pancreatic Carcinoma
Correct answer: A

- 519 -
560 . Antral obstruction with vomiting is not characterized by -
A. Hypokalemia
B. Hypochloremia
C. Acidosis
D. Hyponatremia
Correct answer: C

561 . All of the following statements about Lymphoepithelioma of the parotid


gland are true, Except
A. It is highly radiosensitive
B. It is a type of squamous cell carcinoma
C. Parotid gland is the most common site of Lymphoepithelioma in the Head and
Neck region
D. It is associated with EBV infection
Correct answer: C

562 . Most common origin of cerebral metastases is -


A. Colon
B. Kidney
C. Melanoma
D. Lung
Correct answer: D

563 . A new born child has not passed meconium for 48 hrs. What is the
diagnostic procedure of choice?
A. Contrast enema
B. USG
C. MRI
D. CT
Correct answer: A

564 . All of the following polyps are premalignant except


A. Juvenile polyposis syndrome
B. Familial polyposis syndrome
C. Juvenile polyp
D. Peutz-Jeghers syndrome
Correct answer: C

565 . At which age, Orchidopexy for cryptorchidism is done?


A. Puberty
B. Neonatal period
C. 1 to 2 years
D. 5 to 6 years
Correct answer: C

566 . A 6-week neonate presented with undescended tests on the right side and
normal testes on left side. The treatment of choice would be:
A. 5 year age before going to school
B. Puberty
C. Orchiopexy at 2 years

- 520 -
D. Immediately
Correct answer: C

567 . First human liver transplantation was performed by


A. Thomas Starz
B. Alexis Carrel
C. Christian Barnard
D. Halstead
Correct answer: A

568 . Measurements of intravascular pressure by a pulmonary artery catheter


should be done:
A. During mid inspiration
B. During mid expiration
C. At end expiration
D. At peak of inspiration
Correct answer: C

569 . Which structures are preserved in Scanlon's modified radical mastectomy?


A. Lateral pectoral nerves
B. Leve II nodes in the axilla
C. Pectoral fascia
D. Nipple and the areola
Correct answer: A

570 . 70 years old male presents with a h/o lower GI bleed for last 6 months.
Sigmoidoscopic examination shows a mass, of 4 cm about 3.5 cm above the anal
verge. The treatment of choice is:
A. Colostomy
B. Anterior resection
C. Abdominoperineal resection
D. Defunctioning anastomosis
Correct answer: C

571 . Which of the following is the most common cause of death in Crohn's
disease?
A. Electrolyte disturbance
B. Malignancy
C. Sepsis
D. Thromboembolic complication
Correct answer: B

572 . Indication of laparotomy in a case of tuberculosis of intestine-


A. Peritonitis
B. GI bleeding
C. Suspected diagnosis
D. Intestinal obstruction
Correct answer: D

- 521 -
573 . After laparotomy most common site of collection and abscess formation-in
intra-abdominal cavity.
A. Pelvic
B. Subhepatic
C. Subphrenic
D. Paracolic
Correct answer: A

574 . A 30-year-old male, chronic alcoholic presents with sudden onset of


epigastric pain that radiates to the back. All are seen except:
A. Low serum lipase
B. Increased LDH
C. Hypocalcemia
D. Increased serum amylase
Correct answer: A

575 . For screening of Medullary ca which of the following is estimated -


A. Serum HCG
B. Serum AFP
C. Serum Calcium
D. Serum calcitonin
Correct answer: D

576 . NOT true about FAP(Familial Adenomatous Polyposis)


A. Polyps develop in late adulthood
B. Epidermal cysts and osteomas may occur
C. AR inheritance
D. Screening done by sigmoidoscopy
Correct answer: C

577 . The most common cause of a pyogenic liver abscess is:


A. Biliary Tract Infection
B. Trauma
C. Appendicitis
D. Colonic Diverticulitis
Correct answer: A

578 . A 70-year-old man comes to casualty with urinary retention and back pain.
Which investigation should be performed:
A. Serum acid phosphatase
B. Serum Calcium
C. Serum alkaline phosphatase
D. Serum electrophoresis
Correct answer: A

579 . Which of the following is the most important prognostic factor for
colorectal CA ?
A. Age of patient
B. Lymph node status
C. Site of lesion

- 522 -
D. Tumour size and characteristics
Correct answer: B

580 . Which is the commonest Post splenectomy infection:


A. Streptococcus pyogenes
B. Staphylococcus aureus
C. Streptococcus Pneumoniae
D. Pseudomonas aeruginosa
Correct answer: C

581 . A sequestrum is
A. A piece of dead soft tissue
B. A dead tooth
C. A piece of dead bone
D. A piece of dead skin
Correct answer: C

582 . About Gastrointestinal Carcinoid tumors, all are true, except


A. Rectum is spared
B. Small intestine and appendix account for almost 60% of all gastrointestinal
carcinoid.
C. Appendiceal carcinoids are more common in females than males.
D. 5-year survival for carcinoid tumors is > 60%
Correct answer: A

583 . Permanent colostomy is indicated in-


A. Anterior resection
B. Abdominoperineal resection
C. Trauma to the rectum
D. All of the above
Correct answer: B

584 . First line treatment for keloid is -


A. Intralesional injection of steroid
B. Local steroid
C. Radiotherapy
D. Wide excision
Correct answer: A

585 . Bleeding from the umbilicus in an adult female during menstruation is


suggestive of
A. Bleeding diathesis
B. Vicarious menstruation
C. Persistent urachus
D. Purpura
Correct answer: B

586 . Radiological picture of corkscrew appearance is a sign of -


A. Oesophageal spasm of diffuse nature
B. Oesophageal varices

- 523 -
C. Carcinoma of esophagus
D. Plummer Vinson syndrome
Correct answer: A

587 . Lumber sympathectomy is not indicated in -


A. Healing of ulcer over grate toe
B. Claudication
C. Rest pain
D. Buerger's disease
Correct answer: B

588 . All of the following produce Osteoblastic secondaries except


A. Ca Prostate
B. Carcinoid tumors
C. Ca Breast
D. Multiple myeloma
Correct answer: D

589 . Superior vena cava syndrome is caused by -


A. Adenocarcinoma
B. Squamous cell carcinoma
C. Small cell carcinoma
D. Large cell carcinoma
Correct answer: C

590 . The most unlikely clinical feature of Hepatocellular carcinoma is


A. Hepatomegaly
B. Raised alpha-fetoprotein levels
C. Raised alkaline phosphatase
D. Jaundice
Correct answer: D

591 . A patient developed single liver metastasis of 2 cm after undergoing surgery


for CA colon. What should be the next step?
A. Acetic acid injection
B. Radio frequency ablation
C. Resection
D. Chemo radiation
Correct answer: C

592 . A newborn baby presents with absent anal orifice and meconuria. What is
the most appropriate management?
A. Posterior Sagittal Anorectoplasty
B. Perineal V-Y plasty
C. Transverse colostomy
D. Conservative
Correct answer: C

- 524 -
593 . Pancreatic pseudocyst most commonly occurs after
A. Trauma
B. Pancreatitis
C. Pancreatic surgery
D. Pancreatic malignancy
Correct answer: B

594 . A female noted an abdominal mass in left. side of her 6 month old child.
Which showed calcification near the left kidney. What will be the cause:
A. Leukemia
B. Neuroblastoma
C. Renal cell carcinoma
D. Lymphoma
Correct answer: B

595 . Heller's operation is associated with which of the following diseases?


A. Hirschsprung’s disease
B. Crohn's disease
C. Achalasia cardia
D. Rectal incontinence
Correct answer: C

596 . Which of the following not premalignant?


A. Ulcerative colitis
B. Villous adenoma
C. Familial polyposis
D. Peutz-Jegher's syndrome
Correct answer: D

597 . Head injury patient, eye opening is seen with painful stimulus, localizes the
pain and there is inappropriate verbal response. What would be the score on
GCS-
A. 9
B. 8
C. 11
D. 10
Correct answer: D

598 . Glomus tumor arises from -


A. Subcutaneous arteriovenous shunts
B. Lymphatics
C. Nerves under the nails
D. None of the above
Correct answer: A

599 . Sis-trunk operation is done for -


A. Carotid body tumor
B. Cystic hygroma
C. Thyroglossal cyst
D. Sterna-mastoid tumor

- 525 -
Correct answer: C

600 . The most common retro peritoneal sarcoma is ?


A. Liposarcoma
B. Leiomyo sarcoma
C. Fibro sarcoma
D. Neural sheath sarcoma
Correct answer: A

601. All are true except:


A. Teratoma occurs in younger age group than seminoma
B. Seminoma has better prognosis than teratoma
C. Seminoma is less radiosensitive than teratoma
D. HCG , AFP both increased in teratoma
Correct answer: C

602 . Lowest recurrence rate in duodenal ulcer treatment is seen with -


A. Highly selective vagotomy
B. Truncal vagotomy and antrectomy
C. Truncal vagotomy and pyloroplasty
D. Truncal vagotomy
Correct answer: B

603 . Which of the following is not a component of APACHE score?


A. Serum potassium
B. Serum sodium
C. Serum Calcium
D. Creatinine
Correct answer: C

604. Glomus tumor is seen in -


A. Pituitary
B. Finger
C. Liver
D. Adrenals
Correct answer: B

605 . When S1 spinal root is compressed, which of the following occurs:


A. Paresthesia of sole
B. Loss of ankle jerk
C. Pain in the calf
D. Loss of dorsiflexion of ankle
Correct answer: B

606 . An otherwise normal female presents with symptoms of flatulent dyspepsia.


She was started on proton pump inhibitors, which controlled her symptoms. The
next step in management of this condition should be:
A. Laparotomy after 1 or 2 months.
B. Immediate laparoscopic cholecystectomy
C. Endoscopic retrograde cholangiopancreatography

- 526 -
D. Wait and watch
Correct answer: D

607 . Private streak remnants give rise to ?


A. Neuroblastoma
B. Wilm's tumour
C. Sacrococcygeal teratoma
D. Hepatoblastoma
Correct answer: C

608. Which of the following is not an indication for cholecystectomy -


A. 65 year old female with large gallbladder polyp
B. 55 year old with an asymptomatic gallstone
C. 70 year old male with symptomatic gallstones
D. 20 year old male with sickle cell anemia and symptomatic gallstones
Correct answer: B

609. Which vessels is most commonly involved in a haemorrhage from duodenal


ulcers?
A. Superior mesenteric Artery
B. Inferior Pancreaticoduodenal Artery
C. Inferior Vena Cava
D. Gastroduodenal Artery
Correct answer: C

610 . Causes of retromammary abscess include:


A. Paget's disease
B. Empyema Necessitans
C. Intraductal carcinoma
D. Lobular carcinoma
Correct answer: B

611 . Which of the following is associated with cystic hygroma -?


A. Marfans syndrome
B. Turner's syndrome
C. Down's syndrome
D. Noonans's syndrome
Correct answer: B

612 . Which of the following is not true of gas gangrene?


A. It is caused by clostridium perfringens
B. Clostridium perfringens is a gram-negative spore-bearing bacillus
C. Gas gangrene is characterized by severe local pain crepitus and signs of toxemia
D. High dose penicillin and aggressive debridement of affected tissue is the treatment
of established infection
Correct answer: B

- 527 -
613. Limb salvage can be done in all except -
A. Nerve injury
B. Vascular injury
C. Bone injury
D. Muscle injury
Correct answer: B

614. Chronic Burrowing ulcer is caused by


A. Microaerophilic streptococci
B. Pepto streptococcus
C. Streptococcus viridans
D. Streptococcus pyogenes
Correct answer: A

615 . A 50-year-old man, known to have diabetes, presents with poor urinary
stream, increased frequency of micturition and hesitancy. What is the most likely
diagnosis?
A. BPH
B. Autonomic neuropathy
C. UTI
D. Atonic bladder
Correct answer: A

616 . Marker for testicular tumor is -


A. Beta HCG
B. Acid phosphatase
C. Alkaline phosphatase
D. Alpha fetoprotein
Correct answer: A

617 . A patient comes with stony hard, painless lymph node in left
supraclavicular fossa. A biopsy report states Sq cell Carcinoma. What is the
diagnosis:
A. Stomach carcinoma
B. Breast carcinoma
C. Lung carcinoma
D. Pancreas carcinoma
Correct answer: C

618 . A 45 years patient presents with recurrent duodenal ulcer of 2.5 cm size;
procedure of choice is-
A. Highly selective vagotomy
B. Laparoscopic vagotomy and gastrojejunostomy
C. Truncal vagotomy and antrectomy
D. Truncal vagotomy and gastrojejunostomy
Correct answer: C

- 528 -
619 . Most common site of rhabdomyosarcoma is ?
A. Orbit
B. Nasopharynx
C. Extremities
D. Hypopharynx
Correct answer: A

620 . A six-year-old female presents with constipation and urinary retention. On


examination a presacral mass is noted. Most probable diagnosis is -
A. Sacrococcygeal teratoma
B. Anterior sacral meningocele
C. Pelvic neuroblastoma
D. Rectal duplication cyst
Correct answer: B

621. For CA anal canal the treatment of choice is -


A. Surgery
B. Surgery + Radiotherapy
C. Chemoradiation
D. Chemotherapy
Correct answer: C

622 . Internal fistula is most common between gall bladder and -


A. Colon
B. Duodenum Ist part
C. Jejunum
D. Transverse colon
Correct answer: B

623 . Which one does not cause VII Nerve palsy -


A. ASOM
B. Herpes zostey
C. Parotid tumour
D. Pertusis
Correct answer: D

624 . A 45-year-old female presents with symptoms of acute Cholecystitis. On


USG there is a solitary gallstone of size 1.5 cm. Symptoms are controlled with
medical management. Which of the following is the next most appropriate step in
the management of this patient?
A. IV Antibiotics
B. Regular follow up
C. Open cholecystectomy immediately
D. Laparoscopy cholecystectomy immediately
Correct answer: D

625. Which of the following statement about volvulus is False:


A. Sigmoid volvulus is more common than cecal volvulus
B. More common in psychiatric patients
C. Volvulus of caecum is managed by conservative methods

- 529 -
D. Lower GI scopy is contraindicated in sigmoid volvulus
Correct answer: D

626 . Most common location of spinal tumors -


A. Intradural extramedullary
B. Intramedullary
C. Equally distributed
D. Extradural
Correct answer: D

627 . FNAC needle size -


A. 22-26
B. 18-22
C. 16-18
D. 27-29
Correct answer: A

628 . On mammogram all of the following are the features of a malignant tumor
except:
A. Spiculation
B. Microcalcification
C. Macrocalcification
D. Irregular mass
Correct answer: C

629. All of the following are true regarding solitary rectal ulcer except:
A. Mostly involve midline in posterior wall
B. History of per rectal examination in 50%
C. 20% multiple
D. Causes rectal prolapse
Correct answer: A

630. Treatment of squamous cell carcinoma of anal canal is


A. Cisplatin based chemotherapy followed by radical radiotherapy
B. Abdomino perineal resection
C. Radical radiotherapy
D. Radical radiotherapy followed by mitomycin-c based chemotherapy
Correct answer: A

631 . Paget's disease of the nipple is treated by


A. Radiotherapy
B. Radical mastectomy
C. Biopsy and simple mastectomy
D. Chemotherapy
Correct answer: C

632. Increased anticholinesterase enzyme is seen in:


A. Cardiac defect
B. Oesophageal atresia
C. Neural tube defects

- 530 -
D. Diaphragmatic hernia
Correct answer: C

633. Find needle aspiration cytology is not suitable for diagnosing:


A. Plasmacytoma
B. Aneurysmal bone cyst
C. Tubercular lymphadenitis
D. Papillary carcinoma thyroid
Correct answer: B

634 . In which bilateral parotid enlargement does not occur -


A. Sjogren's syndrome
B. Sarcoidosis
C. SLE
D. Chronic pancreatitis
Correct answer: C

635 . Extravasated urine following rupture of bulbar urethra collects in all the
following regions except -
A. Penis
B. Scrotum
C. Inguinal canal
D. Beneath the superficial fascia m the abdominal wall
Correct answer: C

636 . A patient presents to emergency ward with massive upper gastrointestinal


bleed. On examination, he has mild splenomegaly. In the absence of any other
information available. Which of the following is the most appropriate
therapeutic modality -
A. Intravenous propranolol
B. Intravenous vasopressin
C. Intravenous pantoprazole
D. Intravenous somatostatin
Correct answer: C

637 . Heller's operation is done for -


A. Achalasia cardia
B. Pyloric stenosis
C. Peptic ulcer
D. CA Esophagus
Correct answer: A

638 . A 45 year old male having a long history of cigarette smoking presents with
gangrene of left foot. An amputation of the left foot was done. Representative
sections from the specimen revealed presence of arterial thrombus with
neutrophilic infiltrate in the arterial wall. The inflammation also extended into
the neighboring veins and nerves. The most probable diagnosis is -
A. Hypersensitivity angitis
B. Thrmoboangitis obliterans
C. Takayasu arteritis

- 531 -
D. Giant cell arteritis
Correct answer: B

639 . Not true about PUJ obstruction is:


A. Retrograde pyelography is useful to locate the site of obstruction
B. Endoscopic pylotomy is contraindicated
C. Whittakar test is of clinical significance
D. Dismembered pyeloplasty is the procedure of choice
Correct answer: B

640 . Best prognostic factor for head injury is:


A. Glasgow coma scale
B. Age
C. Mode of injury
D. CT
Correct answer: A

641 . All are treatment of acute fissure in ano except one -


A. Conservative
B. Dilatation under GA
C. Lateral sphincterotomy
D. External sphincterotomy
Correct answer: D

642 . A patient with BHP underwent TURP under spinal anesthesia. An hour
later he developed altered sensorium, nausea, and vomiting. The cause is:
A. Water intoxication
B. Rupture bladder
C. Hypernatremia
D. Bupivacaine overdose
Correct answer: A

643 . What is not true regarding Bockadolech hernia:


A. Early respiratory distress leading to early diagnosis and treatment are good
prognostic sign
B. Stomach and transverse colon are commonest content to herniate
C. Diagnosed prenatally by ultrasound
D. Common on left posterior site.
Correct answer: A

644 . 72 years. old man, on examination he have a prostatic nodule. Best


investigation to evaluate him:
A. CT scan pelvis
B. IVP
C. Prostatic massage for examination of secretions
D. Transrectal U/S
Correct answer: D

- 532 -
645. Kehr's sign seen in splenic rupture is -
A. Pain over left shoulder
B. Pain over right scapula
C. Periumbilical pain
D. Pain over renal angle
Correct answer: A

646 . Spontaneous regression of malignant tumor is feature of -


A. Neuroblastoma
B. Renal cell carcinoma
C. Burkitt's lymphoma
D. Wilm's tumour
Correct answer: A

647 . Commonest site of gallstone impaction in acute cholecystitis is


A. CBD
B. Hepatic duct
C. Fundus of gallbladder
D. Neck of gallbladder and cystic duct
Correct answer: D

648. In Ranson's criteria to predict severity of acute pancreatitis -


A. A serum amylase that is 4 times normal is significant
B. Serum amylase level is not a criteria
C. Rising serum amylase level are important
D. Serum amylase equals urine amylase
Correct answer: B

649 . Which is not true regarding Insulinoma -?


A. Hypoglycemia attacks
B. Weight loss
C. Usually, Solitary tumour
D. Mostly benign tumour
Correct answer: B

650 . The cisterna chyli are situated in the


A. Pelvis
B. Thorax
C. Neck
D. Abdomen
Correct answer: B

651 . Marjolin's ulcer is:


A. Malignancy developing in a scar
B. Malignancy developing in a sinus
C. Ulcer following burn injury
D. Histologically basal cell carcinoma
Correct answer: A

- 533 -
652. Which is not an indication of radiotherapy in Pleomorphic adenoma of
parotid?
A. 2nd histologically benign recurrence
B. Involvement of deep lobe
C. Malignant transformation
D. Microscopically positive margins
Correct answer: A

653 . All are radiopaque except one


A. Oxalate
B. Uric acid
C. Cystine
D. Mixed
Correct answer: B

654 . In the treatment of femoral hernia Lockwood's operation refers to -


A. Low operation
B. High operation
C. Inguinal operation
D. Laparoscopic surgery
Correct answer: A

655 . Murphy' sign is seen in cases of


A. Acute pancreatitis
B. Acute cholecystitis
C. Acute appendicitis
D. Splenic rupture
Correct answer: B

656 .Preservation of ilioinguinal nerve is an important step in inguinal operation


while -
A. Incising the sub cutaneous tissue
B. Incising the external oblique aponeurosis
C. Incising the cremasteric fascia
D. Isolating the sac
Correct answer: B

657 . For open pneumothorax which of the following is management of choice -


A. IPPV
B. ICD with underwater seal
C. Thoracostomy and close the rent
D. Wait and watch
Correct answer: B

658 . A patient with long standing multinodular goiter develops hoarseness of


voice; also, the swelling undergoes sudden increase in size. Possible diagnosis is
A. Medullary ca
B. Anaplastic ca
C. Follicular ca
D. Papillary ca

- 534 -
Correct answer: C

659 . Immediate management of a patient with Multiple fracture and Fluid loss
includes the infusion of
A. Blood
B. Dextran
C. Normal saline
D. Ringer lactate
Correct answer: D

660 .Bisgaard treatment refers to that of


A. Venous ulcer
B. Ruptured tendo Achilles
C. An ingrowing toenail
D. An ischemic ulcer
Correct answer: A

661 .Which of the following regarding finasteride are true, except?


A. Impotence is well documented after its use.
B. It is used in the medical treatment of benign Prostatic hypertrophy (BPH).
C. It is a 5-alfa-reductase inhibitor.
D. It blocks the conversion of dihydrotestosterone to testosterone.
Correct answer: D

662 .The investigation of choice for gallstones is -


A. USG
B. ERCP
C. Oral cholecystography
D. Sulfur colloid scan
Correct answer: A

663 .Treatment of carcinoma penis involving proximal shaft without lymph node
involvement:
A. Palliative chemotherapy
B. Radiotherapy
C. Total penectomy with block dissection of ilioinguinal lymph node
D. Total penectomy with bilateral sentinel node biopsy
Correct answer: C

664 .What is the The treatment for pleomorphic adenoma -


A. Radical parotidectomy
B. Superficial parotidectomy
C. Deep parotidectomy
D. Chemotherapy
Correct answer: B

665 .The most common cause of delayed urinary tract obstructive symptoms after
TURP is-
A. Stricture of the bulb of urethra
B. Bladder neck stenosis

- 535 -
C. Stricture of the Navicular fossa
D. Stricture of the membranous urethra
Correct answer: B

666 .Saline drop test is used in:


A. Veress needle during laparoscopy
B. Pneumoperitoneum
C. Choledocho - duodenostomy
D. Histopathology
Correct answer: A

667 .Auxiliary orthotopic liver transplant is indicated for -


A. Drug included hepatic failure
B. Acute fulminant liver failure for any cause
C. Metabolic liver disease
D. As a standby procedure until finding a suitable donor
Correct answer: B

668 .Hernia with highest rate of strangulation is ?


A. Direct inguinal hernia
B. Indirect inguinal hernia
C. Femoral hernia
D. Incisional hernia
Correct answer: C

669 .Diabetic foot ideal treatment is-


A. Amputation and diabetic control
B. Wide debridement and diabetic control
C. Antidiabetic and antibiotics
D. Amputation only
Correct answer: B

670 .Treatment for Desmoid tumor is -


A. Wide excision
B. Local excision
C. Radiotherapy
D. Wide excision with radiotherapy
Correct answer: A

671 .Which of the following is the most common cause of unilateral parotid
swelling in an adult?
A. Pleomorphic adenoma
B. Warthin’s tumor
C. Hemangioma
D. Adenocarcinoma
Correct answer: A

672 .Commonest presentation of wilm's tumour is -


A. Hematuria
B. Abdominal lump

- 536 -
C. Hydronephrosis
D. Pain in abdomen
Correct answer: B

673 .First treatment in asymptomatic gall bladder stone ?


A. Immediate surgery
B. Oral dissolution
C. Endoscopic sphincter ablation
D. Wait and watch
Correct answer: D

674 .Lembert Suture is used for:


A. Intestinal anastomosis
B. Liver resection
C. Ureteric anastomosis
D. Esophageal resection and anastomosis
Correct answer: A

675 .Lumbar sympathectomy done in all except:


A. Burgers’ disease
B. Hyperhidrosis
C. Causalgia
D. Raynaud's disease
Correct answer: D

676 .A boil is
A. Any abscess of the skin
B. Same as a carbuncle
C. An acute infection of a hair follicle
D. The same as a pyogenic granuloma
Correct answer: C

677 .The connect of Littre's hernia is ?


A. Omentum
B. Bladder
C. Meckel's diverticulum
D. Part of Circumference of intestine
Correct answer: C

678 . TIPSS involves percutaneous creation a shunt between -


A. Portal vein and vena cava
B. Portal vein and hepatic vein
C. Hepatic vein and vena cava
D. Portal vein and hepatic artery
Correct answer: B

679 .All of the following are features of exstrophy of the bladder except:
A. Cloacal membrane is present
B. Epispadias
C. Umbilical and inguinal hernia

- 537 -
D. Posterior bladder wall protrudes through the defects
Correct answer: A

680 .About congenital torticollis all are true except -


A. 2/3 rd cases have palpable neck mass at birth
B. Uncorrected cases developed plagiocephaly
C. Always associated with breech extraction
D. Spontaneous resolution in most cases
Correct answer: C

681 . Congenital sensorineural deafness with abnormal iodine metabolism and


goiter in children is called -
A. Cogan's syndrome
B. Pendred syndrome
C. Gradenigo's syndrome
D. Norrie's syndrome
Correct answer: B

682 .Following are features of Zollinger-Ellison syndrome EXCEPT


A. Gastrin hyper secretion
B. Beta-cell tumor of pancreas
C. Elevated serum gastrin
D. Peptic ulcer disease
Correct answer: B

683 . Features of hypospadias are all except:


A. No-treatment required for glandular variety
B. Cryptorchidism
C. Chordae
D. Hooded prepuce
Correct answer: B

684 .Condyloma acuminata


A. Are not sexually transmitted
B. Are associated with psoriasis
C. Are caused by chlamydia infection
D. Are caused by human papilloma virus
Correct answer: D

685 .Stone which is resistant to lithotripsy -


A. Triple phosphate stone
B. Calcium oxalate
C. Uric acid stone
D. Cystine stone
Correct answer: D

686 .In extraperitoneal rupture of bladder, urine extravasates in -


A. Groin
B. Intraperitoneal region
C. Extraperitoneal region

- 538 -
D. Peri vesical space
Correct answer: D

687 .Renal calculi are commonly made up of -


A. Calcium oxalate
B. Magnesium ammonium phosphate
C. Uric acid
D. Cystine
Correct answer: A

688 .Hilton's method of abscess drainage is used for abscess in


A. Breast
B. Parotid
C. Gluteal region
D. Legs
Correct answer: B

689 .Cause(S) of intestinal paralytic ileus is/are -


A. Pancreatitis and pneumonia
B. Peritonitis or abscess
C. Pneumonia viscera
D. All of the above
Correct answer: D

690 .Carcinoembryonic antigens(CEA) is increased in all except -


A. Colon cancer
B. Osteogenic sarcoma
C. Lung cancer
D. Breast cancer
Correct answer: B

691 .ERCP is used in diagnosis for -


A. Pancreatic and biliary duct dilatation
B. Peptic ulcer
C. CA stomach
D. Secondaries liver
Correct answer: A

692 . A 50 years old male presents with a hard scrotal swelling. All of the following
can be done except:
A. Testicular biopsy
B. Chest x-ray
C. Inguinal exploration
D. CT abdomen
Correct answer: A

693 . Hippocratic facies is seen in -


A. Peritonitis
B. Pancreatitis
C. Facial nerve injury

- 539 -
D. Marginal mandibular nerve injury
Correct answer: A

694 . Peptic ulcer is associated with all except one -


A. Cirrhosis
B. Zollinger Ellisons syndrome
C. Primary hyperparathyroidism
D. Pernicious anemia
Correct answer: D

695 . Bloody discharge from nipple is seen in:


A. Duct papilloma
B. Duct ectasia
C. Duct carcinoma
D. All
Correct answer: D

696 . Which one is not true regarding Burger's disease -


A. Men are usually involved
B. Occurs below 50 years age
C. Smoking is predisposing factor
D. Veins and nerves are never involved
Correct answer: D

697 . You have seen a patient in emergency, and you are suspecting a strangulated
indirect inguinal hernia; the first thing you will do:
A. Straight X-ray of abdomen
B. Prepare the patient for emergency operation
C. Aspirate to confirm
D. Give immediate IV antibiotics
Correct answer: B

698 . At which stage, Orchidopexy for cryptorchidism is done?


A. Puberty
B. Neonatal period
C. 1 to 2 years
D. 5 to 6 years
Correct answer: C

699 . Most important screening test for acute pancreatitis is -


A. Serum Amylase
B. Serum Lipase
C. CT( abdomen )
D. ERCP
Correct answer: A

700 . Which is not true about choledochal cyst?


A. Type 2 is most common
B. Associated with anomalous junction of the pancreatic and biliary duct
C. If ruptures can cause biliary peritonitis

- 540 -
D. Surgical removal is the treatment of choice
Correct answer: A

701 . Most common site of Felon is ?


A. Index finger
B. Ring finger
C. Little finger
D. Thumb
Correct answer: D

702 .What is The treatment of pleomorphic adenoma of parotid -


A. Radial parotidectomy
B. Superficial parotidectomy
C. Deep parotidectomy
D. Chemotherapy
Correct answer: B

703. Rate of recovery following axonotmesis is


A. 1 mm per day
B. 5 mm per day
C. 1 cm per day
D. 2 cm per day
Correct answer: A

704 . In erect posture commonest site of foreign body in bronchus:


A. Right posterior basal
B. Right anterior basal
C. Lateral basal
D. Medial basal
Correct answer: A

705 . Distended abdomen in intestinal obstruction is mainly due to


A. Diffusion of gas from blood
B. Fermentation of residual food
C. Bacterial action
D. Swallowed air
Correct answer: D

706 .In the TNM staging of lung cancer N2 nodes includes all of the following
EXCEPT -
A. Paratracheal nodes
B. Subcarinal nodes
C. Nodes in the inferior pulmonary ligaments
D. Interlobar nodes
Correct answer: D

707 . Not true regarding choledochal cyst -


A. Epigastric mass
B. Jaundice
C. Pain in abdomen

- 541 -
D. Cysto - jejunostomy is the treatment of choice
Correct answer: D

708 . False about Peutz-Jegher's syndrome is ?


A. Benign
B. Familial
C. Malignant
D. Melanosis of lip
Correct answer: C

709 .The commonest cause of water intoxication in surgical wards is due to -


A. Cola-rectal wash with plain water
B. Syndrome of inappropriate secretion of ADH
C. Irrigation during transurethral resection prostate
D. Over prescription of 5% glucose
Correct answer: D

710 . All of the following are synonyms for Renal cell carcinoma except -
A. Adenocarcinoma of kidney
B. Hypernephroma
C. Grawitz tumor
D. Transitional cell tumor of the kidney
Correct answer: D

711 .Normal level of PSA in male is ?


A. < 4 ng/ml
B. 4-10 ng/ml
C. >10 ng/ml
D. PSA is not produced by normal males
Correct answer: A

712 . All are seen after splenectomy except -


A. Thrombocytopenia
B. Acute gastric dilation
C. Sub diaphragmatic abscess
D. Pulmonary complications
Correct answer: B

713 . Treatment of choice of Desmoid tumour is -


A. Surgery
B. Chemotherapy
C. Radiotherapy
D. Surgery + Chemotherapy
Correct answer: A

714 .After blunt injury in abdomen, patient was hemodynamically stable, the next
investigation is-
A. Barium Swallow
B. X-ray abdomen
C. DPL

- 542 -
D. FAST
Correct answer: D

715 .An open wound contracts by -


A. Stretching of surrounding tissues
B. Epithelial growth
C. Skin grafting
D. Fibroblast proliferation
Correct answer: D

716 .Strangulation most commonly occurs in:


A. Femoral hernia
B. Direct inguinal hernia
C. Indirect inguinal hernia
D. Umbilical hernia
Correct answer: A

717 .A 45 years male after laparoscopic cholecystectomy, specimen is sent for


histopathology which shows CA gallbladder stage T1a. Most appropriate
management is:
A. Extended cholecystectomy
B. Conservative and follow up
C. Radiotherapy
D. Excision of all port sites
Correct answer: D

718 . Peau d' orange is due to -


A. Arterial obstruction
B. Blockage of subdermal lymphatics
C. Invasion of skin with malignant cells
D. Secondary infection
Correct answer: B

719 . A young patient presents with history of dysphagia more to liquid than solids.
The first investigation you will do is:
A. Barium Swallow
B. Esophagoscopy
C. Ultrasound of the chest
D. C.T. scan of the chest
Correct answer: A

720 .Raised gastrin level without associated increase acid secretion is seen in -
A. Carcinoma stomach
B. Gastrinoma
C. Pernicious anemia
D. G cell hyperplasia
Correct answer: C

- 543 -
721 .In which of the following, Double Bubble sign is seen?
A. Duodenal atresia
B. Pyloric stenosis
C. Esophageal atresia
D. Ileal atresia
Correct answer: A

722 .Most common cause of lower GI bleed in India is ?


A. Tubercular enteritis
B. Typhoid enteritis
C. Bacillary Angiomatosis
D. Amoebic Angiomatosis
Correct answer: B

723. Composition of which of the following intravenous fluids has the closest
resemblance to plasma?
A. Isotonic saline
B. 1/5 isotonic saline
C. 4.3% dextrose
D. Ringer's lactate
Correct answer: D

724 . Nicoldoni sign is also known as -


A. Darrier sign
B. Branham sign
C. Murrary sign
D. Frei sign
Correct answer: B

725 . In a case of accidental injury of the external femoral vein with intact femoral
artery next step in management would be:
A. Repair of the vein with end-ta-end anastomosis
B. Ligation of both ends
C. Ligation of only the proximal end
D. Ligation of the vein as well as the artery
Correct answer: A

726 . Most common Tumor of Spleen is:


A. Hemangioma
B. Metastasis
C. Lymphoma
D. Sarcoma
Correct answer: C

727 . A 55 year old diabetic patient presented with impotence with history of
failure to get erection after papaverine intracavernous injection. Colour doppler
shows no abnormality of arteries but shows mild venous run-off, Treatment of
choice here would be:
A. Penile prosthetic implants
B. Intracavernous injection of papaverine

- 544 -
C. Psychotherapy
D. Vacuum constriction device
Correct answer: D

728 . Splenectomy is not done in


A. Myelofibrosis
B. Sickle cell anemia
C. Hereditary spherocytosis
D. None
Correct answer: D

729 . Regarding adhesive intestinal obstruction, true is -


A. Avoid surgery for initial 48-72 hours
B. Never operate
C. Operate after minimum 10 days of conservative treatment
D. Immediate operation
Correct answer: A

730 . What is not true about Fibrolamellar carcinoma of the liver:


A. More common in females
B. It occurs in younger individuals
C. Better prognosis than hepatocellular Ca
D. APF levels >1000
Correct answer: D

731 . Malignant potential most common in-neoplastic polyp.


A. Sessile polyp
B. Pedunculated polyp
C. Juvenile polyp
D. Hyperplastic polyp
Correct answer: A

732 . True regarding testicular tumors is-


A. Bilateral in up to 10% cases
B. Are embryonal cell carcinomas in 95% of cases
C. Usually present after 50 years of age
D. Teratomas are more common than seminomas
Correct answer: A

733 . A male executive, 50-year age is seen in casualty with hypotension and
hematemesis. There is previous history suggestive of alcohol intake of 100 ml.
daily. The blood loss is around 2 liters. Most probable diagnosis is:
A. Gastritis
B. Duodenal ulcer
C. Mallory-Weiss tear
D. Esophageal varices
Correct answer: B

- 545 -
734 . Which is MOST COMMON site for iatrogenic oesophageal performation -
A. Abdominal portion
B. Cervical portion
C. Above arch of aorta
D. Below arch of aorta
Correct answer: B

735 . Which part of sac is incised first in case of strangulated hernia -


A. Fundus
B. Neck
C. Body
D. Mouth
Correct answer: A

736 . Which polyp has maximum malignant potential -


A. Sessile
B. Pedunculated
C. Superficial spreading
D. Any of the above
Correct answer: A

737 . True about Epigastric Hernia is -


A. Located below the umbilicus and always in the midline
B. Located above the umbilicus and always in the midline
C. Located above the umbilicus and on the either side
D. Can be seen anywhere on abdomen
Correct answer: B

738 . Minimum urinary output required to excrete end products of protein


metabolism is -
A. 200 ml / 24 hours
B. 300 ml / 24 hours
C. 400 ml / 24 hours
D. 500 ml / 24 hours
Correct answer: C

739 . Early complication of ileostomy in the post-operative period -


A. Obstruction
B. Necrosis
C. Diarrhea
D. Prolapse
Correct answer: B

740 . Commonest cause of lung abscess -


A. Aspiration
B. Hematogenous spread from distant site
C. Direct contact
D. Lymphatic spread
Correct answer: A

- 546 -
741 . All of the following are seen in persisting vomiting except:
A. Hypokalemia
B. Decreased K+ in urine
C. Elevated pH of blood
D. Metabolic alkalosis
Correct answer: B

742 . Vitamin deficiency seen in short bowel syndrome is ?


A. Vitamin K
B. Vitamin B12
C. Vitamin A
D. Vitamin D
Correct answer: B

743 . Not a feature of arterial Occlusion -


A. Cyanosis
B. Pallor
C. Paralysis
D. Paraesthesia
Correct answer: A

744 . Extradural hematoma indicates rupture of which artery


A. Maxillary artery
B. Middle meningeal artery
C. Internal carotid artery
D. Ophthalmic artery
Correct answer: B

745. Risk factors for cholangiocarcinoma are A/E-


A. Parasitic infestation
B. Choledocholithiasis
C. Chronic typhoid
D. Chronic ulcerative colitis
Correct answer: B

746 . True regarding keloid is:


A. Local recurrence is common after excision
B. They do not extend into normal skin
C. They often undergo malignant changes
D. They are more common in whites than in blacks
Correct answer: A

747 . Wilm's tumor is associated with all of the following, except-


A. Hypertension
B. Bilateral polycystic kidney disease
C. Hemihypertrophy
D. Aniridia
Correct answer: B

- 547 -
748 . Triad of haemobilia includes all, except -
A. Pain
B. Fever
C. G.I. bleeding
D. Jaundice
Correct answer: B

749 . Which of the following age is best for surgery of hypospadias?


A. 6-10 months of age
B. 1-4 months of age
C. 2-4 years of age
D. 12-18 months of age
Correct answer: A

750 . The most common cause of Superficial Thrombophlebitis is


A. Varicose veins
B. Trauma
C. Intravenous catheters / infusion
D. DVT
Correct answer: C

751 . The fascia separating rectum from coccyx is called -


A. Scarpa's fascia
B. Waldeyer's fascia
C. Denovillier's fascia
D. Colle's fascia
Correct answer: B

752 . Thoracic extension of cervical goiter is usually approached through -


A. Neck
B. Chest
C. Combined cervico-thoracic route
D. Thoracoscopic
Correct answer: A

753 . One side of kidney is normal, other side kidney is contracted kidney with
scar, what is the most probable diagnosis ?
A. Chronic pyelonephritis
B. Polycystic kidney
C. Renal artery stenosis
D. Tuberculosis of kidney
Correct answer: A

754 . Sign sign in 'Large duct papilloma' is ?


A. Nipple discharge
B. Breast mass
C. Skin excoriation
D. Lymph node involvement
Correct answer: A

- 548 -
755 . All except one is removal in Whipple's operation -
A. Duodenum
B. Head of pancreas
C. Portal vein
D. Common bile duct
Correct answer: C

756 . Regarding Budd-Chiari syndrome which of the following statements is


incorrect
A. It is due to obstruction of hepatic venous outflow secondary to spontaneous
thrombosis of hepatic veins
B. Liver transplantation may be required
C. Ascites is minimal
D. Liver scan demonstrates absent function through most of the liver
Correct answer: C

757 . Most common site of intraperitoneal abscess is:


A. Subphrenic space
B. Paracolic gutter
C. Pouch of Morrison
D. Cave of Retzius
Correct answer: C

758 . Which of the following nerves is sacrificed in Patey's Mastectomy ?


A. Axillary nerve
B. Thoracodorsal nerve
C. Long thoracic nerve
D. Intercostobrachial nerve
Correct answer: D

759 . Following modalities can be used for in situ ablation of liver secondaries,
except:
A. Alcohol
B. Radia-frequency
C. Ultrasonic waves
D. Cryotherapy
Correct answer: C

760 . A 38 year lady presents with fever for 3 weeks. On examination she is
observed to have Splenomegaly. Ultrasonography reveals a hypoechoic shadow in
spleen near the hilum. Gram negative bacilli are isolated on blood culture. The
most Possible causative organism is-
A. Toxoplasmosis
B. Cytomegalovirus
C. Lymphoma virus
D. Salmonella
Correct answer: D

- 549 -
761 . All of the following are modalities of therapy for hepatocellular carcinoma
except -
A. Radiofrequency ablation
B. Trans arterial catheter embolization
C. Percutaneous acetic acid
D. Nd Yag laser ablation
Correct answer: D

762 . Odynophagia is -
A. Throat pain
B. Pain on swallowing
C. Heartburn
D. Painless dysphagia
Correct answer: B

763 . Trousseau's sign is seen in all the following except


A. Ca Lung
B. Ca Stomach
C. Ca Pancreas
D. Liposarcoma
Correct answer: D

764 . Most common malignant parotid tumor in children is:


A. Mucoepidermoid carcinoma
B. Adenocystic carcinoma
C. Adenocarcinoma
D. Warthin tumor
Correct answer: A

765 . An absolute contraindication for IVP is


A. Allergy to the drug
B. Multiple myeloma
C. Blood urea > 200 mg
D. Renal tumor
Correct answer: A

766 . Boundaries of Gastrinoma triangle are all except -


A. Cystic duct and bile duct junction
B. Junction of second and Third part of duodenum
C. Junction of hepatic ducts
D. Junction of head and neck of pancreas
Correct answer: C

761 . Which of the bellow is the earliest manifestation of increased intracranial


pressure following head injury?
A. Contralateral pupillary dilatation.
B. Ipsilateral pupillary dilatation.
C. Hemiparesis.
D. Altered mental status.
Correct answer: D

- 550 -
762 . Most Imp. technical consideration at the time of doing below knee
amputation is:
A. Post. flap should be longer than the anterior flap
B. Stump should be long
C. Stump should be short
D. Ant flap should be longer than post flap
Correct answer: A

763 . Cells from the neural crest are involved in all except
A. Hirschsprung's disease
B. Neuroblastoma
C. Primitive neuroectodermal tumour
D. Wilm's tumor
Correct answer: D

764 . The recommended treatment for preputial adhesions producing ballooning


of prepuce during micturition in a 2-year-old boy is:
A. Wait and watch policy
B. Circumcision
C. Dorsal slit
D. Preputial adhesions release and dilatation
Correct answer: B

765 . The most important consideration in a patient with borderline pulmonary


function requiring lung resection is -
A. The amount of non-functioning lung tissue to be removed
B. The amount of functioning lung tissue to be removed
C. Experience of the surgical team
D. Elevated pulmonary arterial pressure
Correct answer: B

766 . Pyoderma-gangrenosum is most commonly associated with:


A. Amoebic colitis
B. Ischemic colitis
C. Ulcerative colitis
D. Crohn’s disease
Correct answer: C

767 . Treatment of retained CBD stone is ?


A. Endoscopic Sphincterotomy
B. Laparoscopic CBD exploration
C. Percutaneous stone extraction
D. ESWL
Correct answer: A

768 . Prognosis of portocaval shunt depends on all except -


A. Serum bilirubin
B. Serum albumin
C. Refractory ascites
D. Type of shunt

- 551 -
Correct answer: D

769 . In a patient of Rheumatoid arthritis, steroid is indicated in -


A. Subcutaneous nodule
B. Erosive arthritis
C. Mononeuritis multiplex
D. Pulmonary fibrosis
Correct answer: C

770. Acalculous cholecystitis is seen in -


A. Patients recovering from major surgery
B. Trauma
C. Burns
D. All of the above
Correct answer: D

771 . Most probable malignancy that develops in a case of long-standing goiter is


A. Papillary Ca
B. Anaplastic Ca
C. Follicular Ca
D. Medullary Ca
Correct answer: C

772. 40-year-old male, chronic alcoholic, diagnosed as cirrhosis, presents with a


lump in the right lobe of liver. Serum AFP level is normal. Most probable diagnosis
is:
A. Fibro hyperplasia
B. Hepatocellular carcinoma
C. Secondaries
D. Hepatocellular adenoma
Correct answer: B

773 . OPSI is related to ?


A. Kidney
B. Brain
C. Lung
D. Spleen
Correct answer: D

774 . What does 'Take in' means of skin grafting -


A. Revascularization of the graft
B. Return of the sensation
C. When the graft becomes adherent to recipient site
D. Non adherent graft is shed off
Correct answer: A

775 . Early postoperative complication of ileostomy -


A. Diarrhea
B. Necrosis
C. Obstruction

- 552 -
D. Prolapse
Correct answer: C

776 . A 73 years male patient with benign prostatic hyperplasia underwent


transurethral resection of prostate under spinal anesthesia. One hour later, he
developed vomiting and altered sensorium. The most possible cause is:
A. Hyperkalemia
B. Water intoxication
C. Over dosage of spinal anaesthetic agent
D. Rupture of bladder
Correct answer: B

777 . Disc prolapse between L4-L5. The nerve root(s)compressed is(are):


A. L4
B. L5
C. L4 and S1
D. L4 and L5
Correct answer: B

778 . Most common site of splenuoles is ?


A. Omentum
B. Spleno - colic ligament
C. Gastro - coli ligament
D. Hilum of spleen
Correct answer: D

779 . All are false except one in case of hypertrophic pyloric stenosis
A. Symptomatic within one week
B. Lump is always clinically palpable
C. The treatment of choice is Finney's pyloroplasty
D. Ultrasonography is diagnostic test
Correct answer: D

780 . Rupture of urethra above the deep perineal pouch causes urine retention in
which of the following region?
A. Scrotum
B. Medial aspect of thigh
C. Anterior abdominal wall
D. True pelvis only
Correct answer: D

781 . Preferred incision for abdominal exploration in Blunt injury abdomen is:
A. Always Midline incision
B. Depending upon the organ
C. Transverse incision
D. Paramedian
Correct answer: A

- 553 -
782 . Verrucous carcinoma of buccal mucosa is usually a -
A. High grade malignancy
B. Associated with early lymph noses
C. Deep invasive malignancy
D. Densely Keratinized
Correct answer: D

783 . Which of the following is the pre-malignant condition with the highest
probability of progression to malignancy?
A. Leucoplakia
B. Erythroplakia
C. Dysplasia
D. Hyperplasia
Correct answer: B

784 . Not true regarding neurofibromatosis is -


A. Nerve become malignant
B. Is encapsulated
C. Resected along nerve fibers
D. Can present with deafness
Correct answer: A

785 . A patient presents with symptoms of regurgitation of foul smelling food,


and dysphagia. He is diagnosed to have cricopharyngeal diverticula.
Management of choice is:
A. Excision of the diverticula
B. Cricopharyngeal myotomy alone
C. Myotomy with excision of the sac
D. Marsupialization of the sac
Correct answer: C

786 . Regarding achalasia cardia, which of the following statements is incorrect?


A. Is treated by pneumatic dilatation
B. Symptoms are relieved by bethanechol
C. More common in men
D. Is associated with dysphagia for liquids
Correct answer: B

787 . Most common cause of gangrene of foot of 30 years old farmer who is a
chronic smoker:
A. Raynaud's disease
B. Myocardial infarction
C. Atherosclerosis
D. Thromboangitis obliterans
Correct answer: D

788 . Cardinal features of Intestinal obstruction are as follows, except for -


A. Abdominal pain
B. Vomiting
C. Diarrhea

- 554 -
D. Distension of Abdomen
Correct answer: C

789 . Concentration of Na (meq/1) in normal saline is ?


A. 77
B. 109
C. 130
D. 154
Correct answer: D

790 . A 68 years woman, on her third day of hospitalization, who is being treated
with antibiotics for acute cholecystitis develops increased pain and tenderness in
the right upper quadrant with a palpable mass. Her temperature rises to 40 degree
C (104 degree F) her blood pressure falls to 80/60mmHg. Hematemesis, melena
and petechiae are noted. Laboratory studies reveal thrombocytopenia, prolonged
prothrombin time, and a decreased fibrinogen level. The most important step in
the correction of this patient's coagulopathy is:
A. Administration of e-Aminocaproic acid
B. Administration of fresh frozen plasma
C. Exploratory laparotomy
D. Administration of heparin
Correct answer: C

791 . Which of the following is NOT a concept of damage-control surgery?


A. Arrest haemorrhage
B. Control sepsis
C. Protect from further injury
D. Definitive repair of injury
Correct answer: D

792 . Zuska syndrome common in smokers causes?


A. Acute mastitis
B. Chronic areolar abscess
C. Fibro adenosis
D. Acute abscess formation
Correct answer: B

793 . All are resected in Whipple’s operation except -


A. Duodenum
B. Head of pancreas
C. Portal vein
D. Common bile duct
Correct answer: C

794 . Most common cause of acute intestinal obstruction is -


A. Adhesions
B. Volvulus
C. Inguinal hernia
D. Internal hernia
Correct answer: A

- 555 -
795 . True about Acute Epididymitis is
A. Associated with urinary infection
B. Painless
C. Scrotum size is reduced
D. Clinically does not mimic torsion of testes
Correct answer: A

796 . Which one of the following is used as preservative for packing catgut suture?
A. Isopropyl alcohol
B. Colloidal iodine
C. Glutaraldehyde
D. Hydrogen peroxide
Correct answer: A

797 . A 40 years old female presented with a progressively increasing lump in the
parotid region. On oral examinations, the tonsil was pushed medially. Biopsy
showed it to be pleomorphic adenoma. The appropriate treatment is:
A. Superficial parotidectomy
B. Lumpectomy
C. Conservative total parotidectomy
D. Enucleation
Correct answer: C

798 . Which of the following does not predispose to Cholangiocarcinoma?


A. Ulcerative colitis
B. Clonorchis sinensis
C. Choledochal cyst
D. Chronic pancreatitis
Correct answer: D

799 . Testicular teratoma in adult is ?


A. Benign
B. Malignant
C. Locally aggressive
D. Borderline
Correct answer: B

800 . What is the reason for following set of symptoms after prostatic Sx-
Restlessness, vomiting and change in sensorium-?
A. Electrolyte imbalance
B. Bladder neck obstruction
C. Acute pyelonephritis
D. Ureter stenosis
Correct answer: A

801 . True about femoral artery cannulation, except-


A. Single wall puncture is indicated in those with normal coagulation profile
B. Common femoral artery is cannulated
C. Seldinger technique is used both for femoral artery and vein
D. Femoral artery is catheterized at medial third of femoral head

- 556 -
Correct answer: A

802. A 27 years male presented with retroperitoneal, necrotic, heterogenous


enhancing mass on CT near the hilum of the left kidney. Most possible diagnosis
is-
A. Lymphoma
B. Metastatic transitional cell tumour
C. Metastatic germ cell tumor
D. Metastatic melanoma
Correct answer: C

803 . True about congenital torticollis, except-


A. 2/3rd cases have palpable neck mass at birth
B. Uncorrected cases develop plagiocephaly
C. Always associated with breech extraction
D. Spontaneous resolution in most cases
Correct answer: C

804 . Heller's myotomy is indicated for:


A. Congenital hypertrophic pyloric stenosis
B. Achalasia Cardia
C. Esophageal diverticula
D. Diffuse esophageal spasm
Correct answer: B

805 . Not a criteria of a solitary benign pulmonary nodule on Chest X-ray -


A. Less than 5cm size
B. Peripheral Location
C. Cavitation
D. Concentric Calcification
Correct answer: C

806 . In a blast injury, which of the following organ is least vulnerable to the blast
wave?
A. G.I. tract
B. Lungs
C. Liver
D. Ear drum
Correct answer: C

807 . Spontaneous regression is seen in :


A. Strawberry hemangioma
B. Portwine hemangioma
C. Arterial angioma
D. Cavernous hemangioma
Correct answer: A

808 . Surgical wound of gastrojejunostomy is an example for -


A. Clean wound
B. Clean contaminated wound

- 557 -
C. Contaminated wound
D. Dirty wound
Correct answer: B

809 . Which of the following is false regarding intussusceptions?


A. In X-ray multiple fluid air levels are seen
B. CT is diagnostic
C. USG is diagnostic
D. It is common cause of intestinal obstruction in children
Correct answer: C

810 . Not true regarding Hirschsprung disease is -


A. Autosomal dominant
B. Absent ganglionic cells in myenteric plexus
C. Absent ganglionic cells in submucous plexus
D. Rectal biopsy is diagnostic
Correct answer: A

811 . Thoracic outlet syndrome is primarily diagnosed by:


A. MRI
B. Angiography
C. Clinical Evaluation
D. CT scan
Correct answer: C

812 . Staghorn renal calculus is made up of -


A. Oxalate
B. Phosphate
C. Uric acid
D. Cystine
Correct answer: B

813 . In which condition, medical the treatment of gall stone is indicated -


A. Stone is<15 mm size
B. Radio opaque stone
C. Calcium bilirubinate stone
D. Nonfunctioning gall bladder
Correct answer: A

814 . Ligation of which nerve will lead to parasthesia and pain on the dorsum of
foot during venesections of great saphenous vein
A. Sural nerve
B. Geniculate
C. Saphenous nerve
D. Deep peroneal nerve
Correct answer: C

815 . Ideal age for orchiopexy for cryptorchidism is -


A. At puberty
B. <1 year of age

- 558 -
C. At 6-10 years
D. At 1-2 years
Correct answer: B

816 . A 60 year old male presents with hematuria at onset of micturition-cause is:
A. Urethral stone
B. Bladder tumor
C. Ureteric stone
D. Prostatitis
Correct answer: A

817 . Tumor with perineural invasion:


A. Squamous cell Ca
B. Adenoid cystic Ca
C. Adenocarcinoma
D. Basal cell Ca
Correct answer: B

818 . One of the following will always present with Bilious vomiting
A. Pyloric stenosis
B. Esophageal atresia
C. Atresia of the 3rd part of the duodenum
D. Malrotation of the gut
Correct answer: C

819 . Alvarado’s score is done for ?


A. Pancreatitis
B. Appendicitis
C. Cholecystitis
D. Cholangitis
Correct answer: B

820 . Commonest part of subclavian artery to be affected by stenosis is:


A. Third part
B. Equally affected
C. First part
D. Second part
Correct answer: C

821. In which of the bellow, Peutz-Jeghers polyps commonly seen-


A. Esophagus
B. Jejunum
C. Rectum
D. Colon
Correct answer: B

822 . Meckel's diverticulum is remnant of -


A. Thyroglossal tract
B. Midgut
C. Urachus

- 559 -
D. Vitelline duct
Correct answer: D

823 . A 34 year old patient is brought to the emergency as a case of head injury,
following a head on collision road traffic accident. His BP is 90/60 mmHg.
Tachycardia is presents. Most possible diagnosis is:
A. Intracranial hemorrhage
B. Intraabdominal bleed
C. EDH
D. SDH
Correct answer: B

824 . All are component of saint's triad except -


A. Renal stones
B. Hiatus hernia
C. Diverticulosis coli
D. Gall stones
Correct answer: A

825 . Which of the following is not a complication of surgery for thoracic outlet
syndrome -
A. Brachial plexus injury
B. Pneumothorax
C. Long thoracic nerve injury
D. Lymphocutaneous fistula
Correct answer: D

826 . Cystosarcoma phylloides is treated by -


A. Simple mastectomy
B. Radical mastectomy
C. Modified radical mastectomy
D. Antibiotic with conservative treatment
Correct answer: A

827 . Multiple strictures in intestine are found in:


A. Ulcerative colitis
B. Gastric erosion
C. Radiation enteritis
D. Duodenal ulcer
Correct answer: C

828 . Ideally which gas is used for laparoscopy ?


A. NO
B. N2O
C. CO2O
D. AIR
Correct answer: C

- 560 -
829 . Zollinger-Ellison syndrome is characterized by all of the following except
A. Post bulbar ulcer
B. Recurrent duodenal ulcer
C. Severe diarrhea
D. Massive HCL in response to histamine injection
Correct answer: D

830 . A 55 years male presents with h/o abdominal pain. He is found to have a
fusiform dilatation of the descending aorta. Possible cause is-
A. Atherosclerosis
B. Trauma
C. Syphilitic aortitis
D. Right ventricular failure
Correct answer: A

831. In case of carcinoid tumor which site is having least potential for malignancy
A. Lung
B. Stomach
C. Small Intestine
D. Appendix
Correct answer: A

832 . Not a posterior mediastinal tumor -


A. Neurofibroma
B. Lymphoma
C. Thymoma
D. Gastroenteric cyst
Correct answer: C

833 . A patient with ITP has a platelet count of 50,000 and is being planned for
splenectomy. What is the best time for platelet infusion in this patient?
A. At the time of skin incision
B. 2 hours before surgery
C. Immediately after removal of spleen
D. After ligating the splenic artery
Correct answer: D

834 . Most common site of gall stone impactation is


A. Proximal to ilio - caecal junction
B. Duodenojejunal junction
C. Colon
D. Distal to iliocaecal junction
Correct answer: A

835 . True about choledochal cyst is -


A. Always extrahepatic
B. The treatment is cystojejunostomy
C. Excision is ideal the treatment
D. Drainage is the treatment of choice
Correct answer: C

- 561 -
836 . 'Spider leg' appearance of calyces on intravenous urography is seen in
A. Congenital cystic kidneys
B. Hypernephroma
C. Horse shoe kidney
D. Hydronephrosis
Correct answer: A

837 . A 12 year old female swimmer comes with history of abdominal pain clinical
examination reveals rigidity, guarding and generalized tenderness over abdomen
with masking of liver dullness. What is Diagnosis:
A. Primary peritonitis
B. Intussusception
C. Enteric perforation
D. Biliary tree obstruction with peritonitis
Correct answer: C

838 . Nissens fundoplication is a first line of treatment for ?


A. GERD
B. Hiatus hernia
C. Esophageal atresia
D. Congenital hypertrophic pyloric stenosis
Correct answer: B

839 . Treatment of choice in case of chronic ulcerative colitis is -


A. Colectomy with ileostomy
B. Colectomy + manual proctectomy + ileoanal pouch anastomosis
C. Proctocolectomy with ileoanal anastomosis
D. Ileorectal anastomosis
Correct answer: C

840 . Best treatment modality for Warthin's tumor is


A. Superficial parotidectomy
B. Radical parotidectomy
C. Radiotherapy
D. Curettage
Correct answer: A

841 . Which fruit juice helps in preventing UTI -


A. Raspberry
B. Grape
C. Orange
D. Cranberry
Correct answer: D

842 . 'SEPS' is a procedure used for:


A. Lymphatics
B. AV fistula
C. Veins
D. Arteries

- 562 -
Correct answer: C

843 . Due to advances in cancer treatment the progressive of which of the


following has become better?
A. Glioblastoma Multiforme
B. Esophageal Carcinoma
C. ALL in children
D. Cholangiocarcinoma
Correct answer: C

844 . All of the following are known predisposing factors for cholangiocarcinoma
except:
A. Ulcerative colitis
B. Primary sclerosing cholangitis
C. CBD stones
D. Clonorchis sinensis
Correct answer: C

845 . The floor of a tuberculous ulcer will be seen to contain


A. Apple jelly granulations
B. A wash-leather slough
C. A pyogenic granuloma
D. Fat
Correct answer: A

846 . Zollinger-Ellison syndrome is due to tumor of the


A. Alpha cells
B. Beta cells
C. PP cells
D. G-cells
Correct answer: D

847 . The following is a maker of Paget's disease of the mammary gland?


A. S-100
B. HMB 45
C. CEA
D. Neuron specific enolase
Correct answer: C

848 . Treatment of choice in a case of flail chest with respiratory distress:


A. IPPV
B. Endotracheal intubation
C. Strapping
D. Repair of chest segment
Correct answer: A

849 . Surgery of choice for chronic duodenal ulcer is -


A. Vagotomy + antrectomy
B. Total gastrectomy
C. Truncal vagotomy and pyloroplasty

- 563 -
D. Highly selective vagotomy
Correct answer: D

850 . The most commonly practiced operative procedure for a perforated


duodenal ulcer is -
A. Vagotomy and pyloroplasty
B. Vagotomy and antrectomy
C. Vagotomy and perforation closure
D. Graham's omental patch repair
Correct answer: C

851 . Which of the following is false about hepatic duct?


A. Caudate lobe drains only left hepatic duct
B. Left hepatic duct formed in umbilical fissure
C. Left hepatic duct crosses IV segment
D. Right hepatic duct formed by V and VIII segments
Correct answer: A

852 . Commonest malignancy in HIV patient:


A. Kaposi sarcoma
B. Adenoma of stomach
C. Astrocytoma
D. CNS lymphoma
Correct answer: A

853 . A 65 years male has foul breath; He regurgitates food that is eaten 2 days
ago: Possible diagnosis is:
A. Scleroderma
B. Achalasia cardia
C. Zenkers diverticulum
D. Meckel’s diverticulum
Correct answer: C

854 . The commonest complication after splenectomy is -


A. OPSI
B. Injury to tail of Pancreas
C. Left lung lower lobe atelectasis
D. Perforation of the stomach
Correct answer: C

855 . Regarding angiomyolipoma of kidney, what is incorrect -


A. Pain in the loin
B. Presents with hypertension
C. Bleeding in self limited
D. Nephrectomy is the treatment of choice
Correct answer: D

856 . Which of the following is not a component of Saint's triad ?


A. Renal stone
B. Gall stone

- 564 -
C. Hiatus hernia
D. Diverticulosis of colon
Correct answer: A

857 . A case of acute ruptured appendicitis presents with an abscess following


laparotomy, The most common organism responsible is:
A. Bacteroides
B. E. coli
C. S. fecalis
D. S. pneumoniae
Correct answer: A

858 . The most common site of enlargement of the lymph nodes in Hodgkin's
lymphoma is
A. Mediastinal
B. Axillary
C. Cervical
D. Abdominal
Correct answer: C

859 . True about atrial myxoma is ?


A. Most common in left atrium
B. Reoccurs after excision
C. Distant metastases are seen
D. More common in males
Correct answer: A

860 . A man with blunt injury abdomen after road side accident has a blood
pressure of 100/80 mm Hg and a pulse rate of 120 bpm. Airway has been
established and respiration has been stabilized. Next best step in management is.
A. Blood for cross matching and IV fluids
B. Immediate Blood Transfusion
C. Rush the patient to the OT
D. Ventilate the patient
Correct answer: A

861 . The initial investigation of choice for a post cholecystectomy biliary


stricture is -
A. Ultrasound scan of the abdomen
B. Endoscopic cholangiography
C. Computed tomography
D. Magnetic resonance cholangiography
Correct answer: C

862 . Burgers’ disease involved ?


A. Arteries only
B. Veins only
C. Arteries and veins
D. Artery, vein and nerve
Correct answer: D

- 565 -
863 . The treatment of cholecysto enteric fistula following ulceration of gallstones
through the gall bladder into the duodenum -
A. Excision of track and closure of fistula
B. Diverted the bile of another anastomosis
C. Cholecystectomy and primary repair of the bowel
D. Radical excision of all related structures and reconstruction
Correct answer: C

864 . Most definite investigation for localization of upper G.I. bleeding:


A. Endoscopy
B. Angiography
C. USG
D. Barium study
Correct answer: A

865 . Which of the following is not included in Ranson's score for acute
pancreatitis ?
A. Age
B. WBC count
C. Blood glucose
D. Serum amylase
Correct answer: D

866 . Investigation of choice to diagnosis Hirschsprung’s disease is:


A. Rectal manometry
B. Barium enema
C. Rectal biopsy
D. Laparotomy
Correct answer: C

867 . Renal calculus is seen in massive bowel resection due to -


A. Reduced renal calcium excretion
B. More calcium absorption in gut
C. More oxalate absorption in gut
D. None of the above
Correct answer: C

868 . All of the following about benign prostatic hypertrophy (BPH) are true
EXCEPT -
A. Decreased force and caliber of urinary stream is the presenting feature
B. Increased prostate specific antigen level (PSA) is one of the diagnostic features
C. Dynamic uroflowmetry is one of the diagnostic modalities
D. Incidence is age related
Correct answer: B

869 . On which of the following, BRAC 1 gene is located ?


A. Chromosome 11
B. Chromosome 13
C. Chromosome 22

- 566 -
D. Chromosome 17
Correct answer: D

870 . Fogarty's catheter is used for which of the bellow?


A. Removal of blood clots from the arteries
B. Urethral catheterization
C. TPN
D. Bladder drainage
Correct answer: A

871 . Serum calcitonin is a marker for


A. Anaplastic carcinoma
B. Papillary carcinoma
C. Medullary carcinoma
D. Follicular carcinoma
Correct answer: C

872 . All of the following statements about Gynecomastia are true except:
A. Subcutaneous mastectomy is the initial treatment of choice
B. Seen in liver disease
C. There may be estrogen/testosterone imbalance
D. Can be drug induced.
Correct answer: A

873 . An elective surgery is to be done in a patient taking heavy doses of Aspirin.


Management consists of:
A. Stopping aspirin for 7 days and then do surgery
B. Proceed with surgery
C. Intra operative Platelet transfusion
D. Preoperative platelet transfusion
Correct answer: A

874 . Membranous urethra rupture causes collection of blood in -


A. Ischiorectal fossa
B. Deep perineal pouch
C. Superficial inguinal region
D. Pelvic diaphragm
Correct answer: B

875 . After removal of the parotid gland, patient is having sweating on cheeks
while eating. In this complication seen after parotidectomy, the auriculotemporal
nerve which contains parasympathetic secretomotor fibers to parotid gland is
fused with which nerve ?
A. Greater petrosal nerve
B. Facial nerve
C. Great auricular nerve
D. Buccal nerve
Correct answer: C

- 567 -
876 . Due to decelerations, aorta can be ruptured at place where it is fixed except -
A. At ligament arteriosum
B. Behind the esophagus
C. Behind the crura of diaphragm
D. Aortic valve
Correct answer: B

877 . All are true about hypospadias, except -


A. Circumcision in infancy is contraindicated
B. Avoid surgery till puberty
C. No treatment required in glandular variety
D. If associated chordee is present, 2 stage operation is done
Correct answer: B

878 . Acalculous cholecystitis can be seen in all of the following conditions except -
A. Enteric fever
B. Dengue hemorrhagic fever
C. Leptospirosis
D. Malaria
Correct answer: D

879 . USG shows diffuse wall thickening of gall bladder with hyperechoic nodules
at neck and comet tail artifacts. The most possible diagnosis will be:
A. Xanthogranulomatous cholecystitis
B. Cholesterol crystals
C. Adenomyomas
D. Adenocarcinoma of gall bladder
Correct answer: C

880. The intra-abdominal pressure during laparoscopy should be set between:


A. 5-8 mm of Hg
B. 10-15 mm of Hg
C. 20-25 mm of Hg
D. 30-35 mm of Hg
Correct answer: B

881 . In which of the following surgery is treatment of choice:


A. Massive sliding hiatal hernia
B. Paraoesophageal hernia
C. Pharyngoesophageal traction diverticula
D. Pharyngeal web
Correct answer: B

882 . Endoscope wash with a dirty water is associated with ?


A. M.tuberculosis
B. M.bovis
C. M.chelonae
D. M.ulcerans
Correct answer: C

- 568 -
883 . Which of the following is not associated with left sided portal hypertension?
A. Secondary to pancreatic inflammation
B. Normal superior mesenteric and portal venous pressure
C. Easily reversed by splenectomy
D. Isolated esophageal varices present
Correct answer: D

884 . Smoking may be associated with A/E-


A. Ca Larynx
B. Ca Esophagus
C. Ca Bladder
D. Ca Nasopharynx
Correct answer: D

885 . In a case of retrocecal appendicitis which movement aggravates pain -


A. Extension
B. Flexion
C. Lateral rotation
D. Medial rotation
Correct answer: A

886 . One of the following is characterized by RIM sign -


A. Hydronephrosis (Gross)
B. Hypernephroma
C. Chronic pyelonephritis
D. Polycystic kidney
Correct answer: A

887 . Most common complication of splenectomy is -


A. Hematemesis
B. Left lower lobe atelectasis
C. Peritoneal effusion
D. Acute dilation of stomach
Correct answer: B

888 . Claudication is -
A. Pain at rest
B. Pain relieved by rest
C. Pain at first step
D. Pain not relived by rest
Correct answer: B

889 . In which case cytometric study is indicated -


A. Neurogenic bladder
B. Stress incontinence
C. Fistula
D. Urge incontinence
Correct answer: A

- 569 -
890 . Type III category of basal fracture of skull are -
A. Those that run in the coronal plane from lateral end of one petrous through sella
turcica to lateral end of contralateral petrous ridge
B. Run from side to side in the coronal plane but do not pass through the Sella turcica
C. RUn from to the contralateral back passing through the sella turcica
D. Run from front to back involving all cranial fossa and sella turcica
Correct answer: B

891 . Ormond's disease is -


A. Idiopathic lymphadenopathy
B. Idiopathic pericarditis
C. Idiopathic retro-peritoneal fibrosis
D. Idiopathic mediastinitis
Correct answer: C

892 . After renal transplant the commonest malignancy is -


A. Lymphoma
B. Renal cell Ca
C. Skin cancer
D. Adrenal cancer
Correct answer: C

893 . Treatment for squamous cell carcinoma stage T3 N0 M0 is ?


A. Maxillectomy
B. Radiotherapy
C. Maxillectomy and Radiotherapy
D. Maxillectomy and Chemotherapy
Correct answer: C

894 . Allens test for integrity of palmar arch tests which of the following ?
A. Radial artery
B. Ulnar artery
C. Both
D. None
Correct answer: C

895 . Following catheter material is most suited for long-term use-


A. Silicone.
B. Latex .
C. Polyurethane.
D. Rubber.
Correct answer: A

896 . Complication of porcelain gallbladder:


A. Malignancy
B. Gangrene
C. Stone
D. Perforation
Correct answer: A

- 570 -
897 . Which is not a premalignant condition?
A. Bowen's disease
B. Leukoplakia
C. Acanthosis nigricans
D. Solar keratosis
Correct answer: C

898 . Most common site of hypertrophic keloid is -


A. Face
B. Leg
C. Presternal area
D. Arm
Correct answer: C

899 . Bowel can get strangulated in all of the following space except:
A. Rectouterine pouch
B. Ileocolic recess
C. Para duodenal recess
D. Omental bursa
Correct answer: A

900 . Hamartoma which is incorrect-


A. Desmoid tumor
B. Neurofibroma
C. Hemangioma
D. Glomus tumor
Correct answer: A

901 . Ape thumb deformity is found in paralysis of:


A. Ulnar nerve
B. Radial nerve
C. Median nerve
D. Posterior interosseous nerve
Correct answer: C

902 . A 45 year male patient has a surgical cause of obstructive jaundice. USG
can tell A/E-
A. Peritoneal deposits
B. Level of obstruction
C. Ascites
D. Gall bladder stones
Correct answer: A

903 . Orthoboric oxygen is used in:


A. Anerobic infection
B. Gangrene
C. CO poisoning
D. Ventilation failure
Correct answer: C

- 571 -
904 . In case of clearing airway one of the following is not included -
A. Neck tilt
B. Mouth gag
C. Chin lift
D. Head lift
Correct answer: D

905 . Minimum nitrogen required for an adult with dynamic tissue turnover to
maintain a positive nitrogen balance is -
A. 3.5-4.5 g
B. 5.5-6.6 g
C. 7.5-8.5 g
D. 9.5-10.5 g
Correct answer: B

906 . Contraindications to major hepatic resection for metastatic disease include


all of the following except -
A. Total Hepatic involvement
B. Advanced cirrhosis
C. Extrahepatic tumor involvement
D. Jaundice from extrinsic ductal obstruction
Correct answer: C

907 . An adult patient with leg pain gangrene of toe. His ankle to brachial
arterial arterial pressure ratio would be less than?
A. 1
B. 0.3
C. 0.5
D. 0.8
Correct answer: B

908 . The treatment used for lower ureteric stone is -


A. Endoscopic removal
B. Diuretics
C. Drug dissolution
D. Laser
Correct answer: A

909 . All of the following favor postoperative wound dehiscence except -


A. Malignancy
B. Vitamin B complex deficiency
C. Hypoproteinemia
D. Jaundice
Correct answer: B

910 . Who said 'skin' is the best dressing?


A. Joseph lister
B. John Hunter
C. James Paget
D. McNeill Love

- 572 -
Correct answer: A

911 . Risk factors for malignant change in any asymptomatic patient with a gall
bladder polyp on USG include A/E-
A. Rapid increase in size of polyp
B. Age > 60 yr.
C. Associated Gall stones
D. Size of polyp > 5mm
Correct answer: D

912 . Alvarado score is used in the diagnosis of -


A. Diverticulitis
B. Liver failure
C. Chronic hepatitis
D. Acute appendicitis
Correct answer: D

913 . True about thyroglossal cyst are A/E-


A. The cyst is located within 2 cm of the midline
B. Frequent cause of anterior midline neck masses in the first decade of life
C. The swelling moves upwards on protrusion of tongue
D. Incision and drainage is the treatment of choice
Correct answer: D

914 . What will be the diagnosis of the child pulsatile swelling on medial side of the
nose -
A. Teratoma
B. Meningocele
C. Dermoid cyst
D. Carcinoma of ethmoid bone
Correct answer: B

915 . Which is not an investigation of syphilis


A. VDRL
B. FTA-ABS
C. Weil-Felix
D. TPI
Correct answer: C

916 . Which of the following is the most common cause of acquired arteriovenous
fistula?
A. Blunt trauma
B. Penetrating trauma
C. Bacterial infection
D. Fungal infection
Correct answer: B

917 . All the following statements about Pancreatic Carcinoma is true, except-
A. Hereditary Pancreatitis significantly increases the risk
B. Mutation in P53 gene is associated in 75% of cases

- 573 -
C. Five year survival after curative pancreaticoduodenectomy is 15-20%
D. Median survival in locality advanced (stage III) disease is 3-6 months
Correct answer: D

918 . Charcot's triad includes all the following except -


A. Pain
B. Vomiting
C. Jaundice
D. Fever
Correct answer: B

919 . During bilateral adrenalectomy for Cushing's disease, intraoperative dose of


hydrocortisone should be given after:
A. On opening the abdomen
B. Ligation of left adrenal vein
C. Ligation of right adrenal vein
D. Excision of both adrenal glands
Correct answer: D

920 . The following statements are true about Pyrone’s disease except
A. Patient presents with complaints of painful erection
B. Condition affects adolescent males
C. The condition can be associated with Dupuytren's contracture of the tendon of the
D. Spontaneous regression occurs in 50% of the cases
Correct answer: B

921 . True about Burgers disease are A/E-


A. Neural involvement present
B. Ulnar artery and peroneal arteries involved
C. Phlebitis migrans
D. Small acral vessels of limb involved
Correct answer: D

922 . Which of the following may present as pulsatile swelling on medial side of
nose:
A. Meningoencephalocele
B. Dermoid cyst
C. Cystic hygroma
D. Supra orbital artery aneurysm
Correct answer: A

923 . Which of the following is false about Meckel's diverticulum?


A. It is present in 2% of population
B. It is up to 2 inches long
C. It does not possess all the 3 coats of intestinal wall
D. It contains hetero-topic epithelium in 20%
Correct answer: C

- 574 -
924 . Commonest presentation of Hodgkin's lymphoma is
A. Fever
B. Leukocytosis
C. Painless enlargement of lymph node
D. Pruritis
Correct answer: C

925 . A man comes to emergency with stab injury to left flank. He has stable vitals.
What would be the next step in management -?
A. Laparotomy
B. Laparoscopy
C. CECT
D. Diagnostic peritoneal lavage
Correct answer: C

926 . Klatskin tumor involves -


A. Intrahepatic bile duct
B. Hepatic duct confluence
C. Lowest 1/3 bile duct
D. Periampullary area
Correct answer: B

927 . Difference between thyrotoxicosis and malignant hyperthermia is:


A. Hyperthermia
B. Tachycardia
C. Muscle rigidity
D. Elevated serum CPK level
Correct answer: D

928 . A patient presents with Abdominal pain, Jaundice and Malena. The diagnosis
is:
A. Carcinoma gall bladder
B. Acute pancreatitis
C. Haemobilia
D. Acute cholangitis
Correct answer: C

929 . Which of the following is not an Absolute Indication of Splenectomy?


A. Fibrosarcoma
B. Autoimmune Hemolytic Anemia
C. Splenic abscess
D. Hereditary Spherocytosis
Correct answer: B

930 . The most common facial abnormality seen in Gardener's syndrome is -


A. Ectodermal dysplasia
B. Odontomes
C. Multile osteomas
D. Dental cysts
Correct answer: C

- 575 -
931 . In current scenario which is the most common cause of septicemia?
A. Gram negative bacteria
B. Gram positive bacteria
C. Fungus
D. Parasite
Correct answer: A

932 . Which of the following is the commonest cause of an obliterative stricture of


the membranous urethra?
A. Road-traffic accident with fracture pelvis and rupture urethra
B. Fall- astride injury
C. Gonococcal infection
D. Prolonged catheterization
Correct answer: A

933 . The 3 years old boy presents with poor urinary stream. Most likely cause is:
A. Stricture urethra
B. Neurogenic bladder
C. Urethral calculus
D. Posterior urethral valve
Correct answer: D

934 . After 3 weeks of duration pancreatic pseudocyst 5 cm in size should be


management by which method:
A. Cystogastrostomy
B. Needle aspiration
C. External drainage
D. USG and follow up
Correct answer: D

935 . Biochemical changes associated with urinary diversion include all of the
following EXCEPT
A. Hypokalemia
B. Uremia
C. Acidosis
D. Hypochloremia
Correct answer: D

936 . All are used in preoperative preparation of thyrotoxicosis except


A. Lugol's odine
B. Propranolol
C. Wysolone
D. Carbimazole
Correct answer: C

937 . All can be prevented by orchiopexy in cryptorchidism except:


A. Testicular tumor
B. Epididymo orchitis
C. Torsion of testis
D. Sexual ambiguity

- 576 -
Correct answer: A

938 . Urinary excretion of urobilinogen by a patient suffering from obstructive


jaundice will be
A. Below normal
B. Normal
C. Excessive
D. Nil
Correct answer: D

939 . Popliteal Artery Pulsations are difficult to feel because


A. It does not cross prominent bone
B. It is not superficial
C. Its pulsations are weak
D. It is not superficial and does not cross prominent bone
Correct answer: D

940 . Maximum chance of anastomotic ulcer is after:


A. Gastrojejunostomy
B. Partial gastrectomy with gastrojejunostomy
C. Highly selective vagotomy
D. Truncal vagotomy and antrectomy
Correct answer: A

941 . A 55 years lady presents with 2 year history of recurrent abdominal pain
with radiation to her back. Pain is severe in intensity and refractory to simple
analgesics. USG and Contrast Enhanced CT scan (CECT)confirmed the diagnosis
and showed a dilated pancreatic duct. The likely recommended surgical procedure
is-
A. Whipple's Procedure
B. Longitudinal Pancreatico jejunostomy
C. Vagotomy with Antrectomy
D. Vagotomy with Gastrojejunostomy
Correct answer: B

942 . In thymoma, all are seen except:


A. Hypo gamma albuminemia
B. Hyperalbuminemia
C. Red cell aplasia
D. Myasthenia Gravis
Correct answer: B

943 . Varicose vein with local gigantism is seen in:


A. Congenital A-V fistula
B. DVT
C. Tumor
D. Saphenofemoral incompetence
Correct answer: A

- 577 -
944 . Most common infection after splenectomy is -
A. Anaerobic
B. Staphylococcal
C. Streptococcal
D. Pneumococcal
Correct answer: D

945 . Correct regarding AV fistula except:


A. Proximal compression causes increase in heart rate
B. Arterialization of the veins
C. Causes LV enlargement and LVF
D. Overgrowth of a limb
Correct answer: A

946 . Most common organism causing appendicitis is ?


A. Bacteroides
B. E.coli
C. Staphylococcus
D. Streptococcus
Correct answer: A

947 . Features of hyperplastic tuberculosis of gastrointestinal tract are A/E-


A. Most common site is ileocecal junction
B. ATT is the treatment of choice
C. Presents with a mass in RIF
D. Barium meal shows 'pulled up caecum'
Correct answer: B

948 . Concomitant chemoradiotherapy is indicated in all of the following Except:


A. T2 No Mo Anal Cancer
B. Stage III B Ca Cervix
C. T1 N2 Mo Nasopharyngeal Cancer
D. T2 No Mo Glottic Cancer
Correct answer: D

949 . A patient has acute abdominal pain with blood and mucus in stool with
palpable mass per abdomen is due ta-
A. Meckel’s diverticulum
B. Volvulus
C. Intussusception
D. Hypertrophic pyloric stenosis
Correct answer: C

950 . Hirschsprung's disease is best diagnosed by -


A. Full thickness rectal biopsy
B. Partial thickness rectal biopsy
C. Pressure studies
D. Barium enema
Correct answer: A

- 578 -
951 . One is not the feature of obstructive jaundice -
A. Pruritis
B. Elevated level of S. bilirubin
C. Raised alkaline phosphatase
D. Raised Urinary urobilinogen
Correct answer: D

952 . Meconium ileus is usually associated with all of the following EXCEPT
A. Absence of air-fluid levels in X-ray
B. High sodium chloride concentration in sweat
C. Grossly distended large intestine
D. Surgically treated by Bishop-Koop operation
Correct answer: C

953 . Management of pancreatic abscess is -


A. Needle aspiration
B. Gastro - cystostomy
C. External drainage
D. Jejuno - cystostomy
Correct answer: C

954 . The most common site of intestinal obstruction in gallstone ileus is -


A. Duodenum
B. Jejunum
C. Ileum
D. Sigmoid colon
Correct answer: C

955 . True about hydatid cyst of lung is:


A. Calcification is common
B. More common in lower lobes
C. Always associated with cyst in liver
D. Never ruptures
Correct answer: B

956 . Which of these does not change or remains same throughout life:
A. Salmon patch
B. Strawberry angiomas
C. Portwine stain
D. Capillary hemangioma
Correct answer: C

957 . Treatment of symptomatic retained GB stone is ?


A. Sphincterotomy
B. ESWL
C. Laparoscopic removal
D. All of the above
Correct answer: A

- 579 -
958 . True regarding mesothelioma, except-
A. Histopathology shows biphasic pattern
B. Occurs in late middle age
C. Bilaterally symmetrical
D. Associated with asbestos exposure
Correct answer: C

959- Which of the following statements are true and which are false?
1 . The surface area of the peritoneal membrane is nearly equal to that of the skin.
Correct answer T
2 . The parietal peritoneum is poorly innervated.
Correct answer F
3. The peritoneum has the capacity to absorb large volumes of fluid.
Correct answer T
4. The peritoneum has the ability to produce fibrinolytic activity.
Correct answer T
5. When injured, the peritoneum produces an inflammatory exudate.
Correct answer T
6. Peritonitis in perforated duodenal ulcer is initially sterile.
Correct answer T
7. Immunocompromised patients may present with opportunistic peritoneal infection.
Correct answer T
8. Bacteroides are sensitive to penicillin.
Correct answer F
9. In perforated duodenal ulcer there may be signs of peritonitis in the right iliac fossa.
Correct answer T
10. Children can localise infection effectively.
Correct answer F
11. The tumour marker of HCC is alpha fetoprotein.
Correct answer T
12. The blood supply of HCC is the hepatic artery rather than the portal vein.
Correct answer T
13. Venous drainage of the testis is systemic haematogenous spread of testicular cancer goes
to the lungs rather than the liver.
Correct answer T
14. In Glasgow coma scale, Eye opening scale is 1 to 4, Best verbal response scale is to 5.

- 580 -
Best motor response scale is 1 to 6. Thus, the lowest scale is 3, while the highest scale (fully
conscious is 15).
Correct answer T
15. The oesophagus is 30 cm long.
Correct answer F
16. The oesophagus has three natural constrictions.
Correct answer T
17. It is lined throughout by columnar epithelium.
Correct answer F
18. The lower oesophageal sphincter (LOS) is a zone of high pressure.
Correct answer T
19. The pressure at the LOS is 50 mmHg.
Correct answer F
20. Difficulty on swallowing (dysphagia) is a cardinal symptom of oesophageal carcinoma.
Correct answer T
21. Retrosternal pain on swallowing (odynophagia) is always of cardiac origin.
Correct answer F
22. Heartburn is a common symptom of gastro-oesophageal reflux disease (GORD).
Correct answer T
23. Dysphagia in the oral or pharyngeal (voluntary) phase, when patients say they cannot
swallow, is usually from neurological or muscular diseases.
Correct answer T
24. Regurgitation and reflux are the same and are caused by obstruction to the esophagus.
Correct answer F
25. Barium swallow is the investigation of choice in GORD.
Correct answer F
26. Flexible oesophago - gastroduodenoscopy (OGD) is the initial investigation of choice in
suspected carcinoma.
Correct answer T
27. Endosonography (EUS) should be carried out when a carcinoma is seen in the esophagus.
Correct answer T
28. Oesophageal manometry should be done when motility disorder is suspected.
Correct answer T
29. 24 h pH recording is an accurate method of evaluating GORD.
- 581 -
Correct answer T
30. In a suspected foreign body (FB) in the oesophagus, water-soluble contrast examination
should be carried out.
Correct answer F
31. When a food bolus is stuck in the oesophagus, always suspect an underlying disease.
Correct answer T
32. Most iatrogenic perforations of the oesophagus can be treated conservatively.
Correct answer T
33. Most spontaneous perforations of the oesophagus (Boerhaave’s syndrome) require an
operation.
Correct answer T
34. In Mallory–Weiss syndrome the tear is usually in the lower end of the oesophagus.
Correct answer F
35. Pharyngeal (Zenker’s) diverticulum is precancerous for esophagus.
Correct answer T
36. GORD is precancerous for esophagus.
Correct answer T
37. Achalasia is precancerous for esophagus.
Correct answer T
38. Corrosive stricture is precancerous for esophagus.
Correct answer T
39. Barrett’s oesophagus is precancerous for esophagus.
Correct answer T
40. Whipple’s operation is a curative operation for carcinoma of the oesophagus.
Correct answer F
41. Anderson–Hynes operation Is a curative operation for carcinoma of the oesophagus.
Correct answer F
42. Heller’s operation Is a curative operation for carcinoma of the oesophagus.
Correct answer F
43. Ivor–Lewis operation Is a curative operation for carcinoma of the oesophagus.
Correct answer T
44. Hartmann’s operation Is a curative operation for carcinoma of the oesophagus .
Correct answer F
45. The right gastric artery is a branch of the coeliac artery.
- 582 -
Correct answer F
46. Vagal fibers to the stomach are afferent.
Correct answer F
47. The parietal cells are in the body of the stomach and are the acid-secreting cells.
Correct answer T
48. The venous drainage of the stomach ends in the inferior vena cava (IVC).
Correct answer F
49.The chief cells secrete pepsinogen.
Correct answer T
50. Gastric acid is a predisposing factors for gastric ulcer.
Correct answer F
51. H. pylori infection is a predisposing factor for gastric ulcer.
Correct answer T
52. NSAIDs is a predisposing factor for gastric ulcer.
Correct answer T
53. Smoking is a predisposing factor for gastric ulcer.
Correct answer T
54. High socioeconomic groups. is a predisposing factors for gastric ulcer.
Correct answer F
55. From the clinical features of peptic ulcers: The pain never radiates to the back and this
differentiates this from biliary colic.
Correct answer F
56. From the clinical features of peptic ulcers: Vomiting is a notable feature.
Correct answer F
57. From the clinical features of peptic ulcers: Bleeding is rare.
Correct answer F
58. From clinical features of peptic ulcers: They may cause gastric outlet obstruction.
Correct answer T
59. From the clinical features of peptic ulcers: Weight loss is a typical symptom.
Correct answer F
60. H. pylori infection is important in the causation of gastric cancer?
Correct answer T
61. Gastric atrophy is important in the causation of gastric cancer?
Correct answer T
- 583 -
62. Pernicious anemia is important in the causation of gastric cancer?
Correct answer T
63. Previous gastric surgery is important in the causation of gastric cancer?
Correct answer T
64. Smoking. is important in the causation of gastric cancer?
Correct answer T
65. The clinical features of gastric cancer may be non-specific in the early stages.
Correct answer T
66. Anaemia can be a presenting symptom of gastric cancer.
Correct answer T
67. Troiser’s sign refers to a palpable ‘Virchow’s node’ in the right supraclavicular fossa.
Correct answer F
68. Gastric cancer is a rare cause of GOA.
Correct answer F
69. Trousseau’s sign is diagnostic of gastric cancer.
Correct answer F
70. Mutation or loss of heterozygosity in APC gene is associated with the molecular
pathology of gastric cancer?
Correct answer T
71. Mutation in gene coding for beta-catenin is associated with the molecular pathology of
gastric cancer?
Correct answer T
72. Mutations in gene coding for E-cadherin is associated with the molecular pathology of
gastric cancer?
Correct answer T
73. Inactivation of p53 is associated with the molecular pathology of gastric cancer?
Correct answer T
74. Microsatellite instability. is associated with molecular pathology of gastric cancer?
Correct answer T
75. Gastric cancer reaching the serosa usually indicates incurability.
Correct answer T
76. In gastric cancer blood-borne metastases commonly occur in the absence of lymph node
spread.
Correct answer F
- 584 -
77. In gastric cancer Krukenberg’s tumors are always associated with other areas of trans
coelomic spread.
Correct answer F
78. In gastric cancer Sister Joseph’s nodule is diagnostic of gastric cancer.
Correct answer F
79. In gastric cancer The lymphatic vessels related to the cardia have no relation to the
oesophageal lymphatics.
Correct answer F
80. Hematogenous metastases is an unequivocal evidence of incurability in gastric cancer.
Correct answer T
81. Involvement of distant peritoneum is an unequivocal evidence of incurability in gastric
cancer.
Correct answer T
82. N3 nodal disease is an unequivocal evidence of incurability in gastric cancer.
Correct answer F
83. Involvement of adjacent organs is an unequivocal evidence of incurability in gastric
cancer
Correct answer F
84. Gastric outlet obstruction is an unequivocal evidence of incurability in gastric cancer.
Correct answer F
85. D1 resection is superior to a D2 resection in the treatment of gastric cancer.
Correct answer F
86. The 5-year survival of gastric cancer in the UK is between 50 and 75 per cent.
Correct answer F
87. In the treatment of gastric cancer there is a definite role for neoadjuvant chemotherapy.
Correct answer T
88. Gastrointestinal continuity is established after total gastrectomy by a Roux loop.
Correct answer T
89. In the treatment of gastric cancer Radiotherapy to the gastric bed is a useful adjunct.
Correct answer F
90. The liver is fixed in its place by peritoneal reflections called ligaments.
Correct answer T
91. The major part of the blood supply of the liver is derived from the hepatic artery.
Correct answer F
- 585 -
92. The portal vein lies posterior to the hepatic vein and common bile duct.
Correct answer T
93. The bile duct, portal vein and hepatic artery are contained in the lesser omentum.
Correct answer T
94. The portal vein is formed by the union of the superior mesenteric and left gastric veins.
Correct answer F
95. The left hepatic duct has a longer extrahepatic course than the right.
Correct answer T
96. The major venous drainage is by three veins – right, middle and left hepatic veins
draining directly into the inferior vena cava (IVC).
Correct answer T
97. All the three major veins join the IVC within the liver parenchyma.
Correct answer F
98. The functional lobes of the liver are divided by the falciform ligament.
Correct answer F
99. The liver is divided into eight functional segments – I to IV in the left hemi-liver and V–
VIII in the right hemi-liver.
Correct answer T
100. The spleen lies in front of the left 10th, 11th and 12th ribs.
Correct answer F
101. The splenic artery arises from the coeliac axis.
Correct answer T
102. The inferior mesenteric vein empties into the splenic vein.
Correct answer T
103. The tail of the pancreas lies in the lienorenal ligament.
Correct answer T
104. The inner surface of the spleen has two impressions – gastric and colic.
Correct answer F
105. Partial splenectomy can result in splenic regeneration.
Correct answer T
106. During splenectomy the tail of the pancreas can be damaged.
Correct answer T
107. A gastric or pancreatic fistula can occur as a post-splenectomy complication.
Correct answer T
- 586 -
108. In a left hemicolectomy the spleen can be in danger.
Correct answer T
109. The risk of opportunist post-splenectomy infection (OPSI) is greatest after the first 6
months of splenectomy.
Correct answer F
110. Splenuli are present in approximately 10–30 per cent of the population.
Correct answer T
111. Splenic artery aneurysm can occur as a complication of acute pancreatitis.
Correct answer T
112. Plain abdominal X-ray is the ideal imaging modality.
Correct answer F
113. A massively enlarged spleen is prone to infarction.
Correct answer T
114. The splenic hilum is a common site for the development of a pseudocyst of the pancreas.
Correct answer T
115. Patients with asymptomatic gallstones should routinely be advised to have a
cholecystectomy.
Correct answer F
116. Ninety per cent of patients with acute cholecystitis respond to conservative treatment.
Correct answer T
117. Antibiotics are not required in the management of acute cholecystitis in the absence of
jaundice.
Correct answer F
118. Urgent laparoscopic cholecystectomy in a patient with acute cholecystitis is associated
with a five times greater conversion rate compared with elective surgery.
Correct answer T
119. Acalculous cholecystitis has a mild clinical course.
Correct answer F
120. The incidence of symptomatic bile duct stones varies from 5 to 8 per cent.
Correct answer T
121. Charcot’s triad consists of pain, stones and jaundice.
Correct answer F
122. Primary sclerosing cholangitis (PSC) is associated with hypergammaglobulinemia and
elevated smooth muscle antibodies.
- 587 -
Correct answer T
123. Clonorchiasis can predispose to bile duct carcinoma.
Correct answer T
124. Bismuth type 3 biliary stricture is a hilar stricture.
Correct answer T
125. Gall bladder carcinoma is a rare disease.
Correct answer T
126. The majority of Gall bladder carcinoma are adenocarcinomas.
Correct answer T
127. In Gall bladder carcinoma the CA19–9 is elevated in 80 per cent of the cases.
Correct answer T
128. In Gall bladder carcinoma A palpable mass is an early sign.
Correct answer F
129. Prognosis of Gall bladder carcinoma is generally good.
Correct answer F
130. The pancreas weighs 200 g.
Correct answer F
131. The uncinate process lies behind the superior mesenteric vessels.
Correct answer T
132. The vast majority of pancreatic tissue is composed of exocrine acinar tissue.
Correct answer T
133. Of the endocrine cells, 75 per cent are B cells, 20 per cent are A cells and the remainder
are D cells.
Correct answer T
134. The accessory pancreatic duct drains the head and uncinate process.
Correct answer T
135. Ultrasonography (US) is the initial investigation of choice in the jaundiced patient.
Correct answer T
136. When doing a computed tomography (CT) scan, initially an unenhanced scan must be
done followed by a scan after intravenous contrast injection (CECT).
Correct answer T
137. While doing a magnetic resonance cholangiopancreatography (MRCP), intravenous
secretin injection helps to determine any obstruction to the pancreatic duct.
Correct answer T
- 588 -
138. An increase in serum amylase is diagnostic of acute pancreatitis.
Correct answer F
139. Endoscopic retrograde cholangiopancreatography (ERCP) should always be preceded
by a plain radiograph.
Correct answer T
140. Pancreatic injury is common following blunt abdominal trauma.
Correct answer F
141. Pancreatic injury is often accompanied by damage to the liver, spleen and duodenum.
Correct answer T
142. The serum amylase is raised in most cases of pancreatic injury.
Correct answer T
143. In Pancreatic injury A CECT scan will delineate the damage.
Correct answer T
144. In doubtful cases of Pancreatic injury, urgent ERCP is helpful.
Correct answer T
145. All patients with pancreatic trauma should undergo an exploratory laparotomy.
Correct answer F
146. Pancreatic duct disruption requires surgical exploration.
Correct answer T
147. Severe injury to the duodenum and the head of the pancreas requires a
pancreatoduodenectomy.
Correct answer T
148. After conservative management for pancreatic injury, duct stricture and pseudocyst may
occur as complications.
Correct answer T
149. During splenectomy, iatrogenic injury to the pancreatic tail can occur.
Correct answer T
150. Acute pancreatitis accounts for 3 per cent of hospital admissions in the UK for
abdominal pain.
Correct answer T
151. Acute pancreatitis is classified into mild and severe.
Correct answer T
152. 80 per cent of Acute pancreatitis are mild acute pancreatitis, with a mortality rate of 1
per cent.
- 589 -
Correct answer T
153. 20 per cent of acute pancreatitis are severe acute pancreatitis, with a mortality of 20–50
per cent.
Correct answer T
154. In all cases of acute pancreatitis, there is a marked rise in serum amylase.
Correct answer F
155. The small bowel is approximately 10 meters long.
Correct answer F
156. The colon is approximately 3 meters long.
Correct answer F
157. The most fixed part of the small bowel is the duodenum.
Correct answer T
158. The jejunum is wider, thicker and more vascular than the ileum.
Correct answer T
159. Peyer’s patches are contained in the ileum.
Correct answer T
160. Duodenal diverticulum may result from a long-standing duodenal ulcer.
Correct answer T
161. Jejunal diverticula may give rise to malabsorption problems.
Correct answer T
162. A Meckel’s diverticulum can cause severe lower gastrointestinal haemorrhage.
Correct answer T
163. A suspected Meckel’s diverticulum is best imaged by a barium meal and follow
through.
Correct answer F
164. Pain originating in a Meckel’s diverticulum is located around the umbilicus.
Correct answer T
165. In the Western world, 60 per cent of the population over the age of 60 have diverticular
disease of colon.
Correct answer T
166. A low-fibre diet causes Diverticular disease of colon.
Correct answer T
167. Diverticular disease of colon consist of mucosa, muscle and serosa.
Correct answer F
- 590 -
168. Diverticular disease of colon with perforation have a 10 times higher mortality than
those with an inflammatory mass.
Correct answer T
169. In diverticular disease of colon Sepsis is the principal cause of morbidity.
Correct answer T
170. From the complication of diverticular disease of the colon : Paracolic abscess.
Correct answer T
171. From the complication of diverticular disease of the colon : Fistulae.
Correct answer T
172. From the complication of diverticular disease of the colon : Lower gastrointestinal
haemorrhage.
Correct answer T
173. From the complication of diverticular disease of the colon : Carcinoma.
Correct answer F
174. From the complication of diverticular disease of the colon : Stricture.
Correct answer T
175. In complicated diverticular disease : Urinary symptoms may be the predominant
presentation at times.
Correct answer T
176. In complicated diverticular disease : Profuse colonic haemorrhage may occur in 17 per
cent.
Correct answer T
177. In complicated diverticular disease: Fistulae occur in 5 per cent of cases.
Correct answer T
178. In complicated diverticular disease: The commonest fistula is coloenteric.
Correct answer F
179. In acute diverticulitis, CT scan is the ‘gold standard’ for imaging.
Correct answer T
180. In the surgical treatment of diverticular disease: Colonoscopy must be carried out in all
elective cases.
Correct answer T
181. In the surgical treatment of diverticular disease : Barium enema is essential prior to
elective operation.
Correct answer T
- 591 -
182. In the surgical treatment of diverticular disease : Primary resection and end-ta-end
anastomosis mustt be carried out in all cases.
Correct answer F
183. In the surgical treatment of diverticular disease : Hartmann’s operation is the procedure
of choice in perforated diverticulitis.
Correct answer T
184. In the surgical treatment of diverticular disease with vesicocolic fistula, a one-stage
operation can usually be done.
Correct answer T
185. In 95 per cent of cases of ulcerative colitis, the disease starts in the rectum and
spreadsproximally.
Correct answer T
186. Ulcerative colitis is a diffuse disease affecting all the layers of the large bowel.
Correct answer F
187. In ulcerative colitis Granulomas are a typical microscopic feature.
Correct answer F
188. The transverse colon is affected in toxic megacolon.
Correct answer T
189. Patients with ulcerative colitis may present as an emergency with fulminating colitis in
5–10 per cent.
Correct answer T
190. Complication of ulcerative colitis include: Carcinoma.
Correct answer T
191. Complication of ulcerative colitis include: Primary sclerosing cholangitis.
Correct answer T
192. Complication of ulcerative colitis include: Internal fistulae.
Correct answer F
193. Complication of ulcerative colitis include: Ankylosing spondylitis.
Correct answer T
194. Complication of ulcerative colitis include: Perforation.
Correct answer T
195. Barium enema finding in UC: Loss of haustrations.
Correct answer T
196. Barium enema finding in UC: Narrow contracted colon.
- 592 -
Correct answer T
197. Barium enema finding in UC: Increase in the presacral space.
Correct answer T
198. Barium enema finding in UC: Cobble stone appearance.
Correct answer F
199. Barium enema finding in UC: Backwash ileitis.
Correct answer T
200. The indications for surgery in UC: Severe fulminating disease not responding to
vigorous medical treatment.
Correct answer T
201. The indications for surgery in UC: Severe dysplastic change or cancer on biopsy.
Correct answer T
202. The indications for surgery in UC: Non-compliance of medical treatment.
Correct answer F
203. The indications for surgery in UC: Chronic steroid-dependent disease requiring large
doses.
Correct answer T
204. The indications for surgery in UC: Extraintestinal disease.
Correct answer T
205. Regarding surgery in UC: In the emergency situation, total abdominal colectomy and
ileostomy should be the procedure of choice.
Correct answer T
206. Regarding surgery in UC: Proctocolectomy and ileostomy are associated with the
lowest complication rate.
Correct answer T
207. Regarding surgery in UC: Restorative proctocolectomy with an ileoanal pouch should
be considered in all patients.
Correct answer T
208. Regarding surgery in UC: Colectomy with ileorectal anastomosis is the most favored
procedure.
Correct answer F
209. Regarding surgery in UC: Ileostomy with a continent intraabdominal pouch is not often
done.
Correct answer T
- 593 -
210. About Crohn’s disease (CD):The ileum is affected in 60 per cent of cases.
Correct answer T
211. About Crohn’s disease (CD): It affects the entire thickness of the bowel.
Correct answer T
212. About Crohn’s disease (CD): Non-caseating granulomas are found in only 60 per cent
of patients.
Correct answer T
213. About Crohn’s disease (CD): One in 10 patients have a first-degree relative with the
disease.
Correct answer T
214. About Crohn’s disease (CD): A patient can be cured of CD once the diseased small or
large bowel is removed.
Correct answer F
215. About imaging in Crohn’s disease (CD): Small-bowel enema is the imaging of choice
in small-bowel disease.
Correct answer T
216. About imaging in Crohn’s disease (CD): Barium enema and colonoscopy should be
done for large-bowel disease.
Correct answer T
217. About imaging in Crohn’s disease (CD): MRI is the ‘gold standard’ for perianal fistulae.
Correct answer T
218. About imaging in Crohn’s disease (CD): CT scan is used for suspected intraabdominal
abscess and internal fistulae.
Correct answer T
219. About inflammatory bowel disease (UC and CD) : Patients must be managed jointly by
the physician and surgeon.
Correct answer T
220. About inflammatory bowel disease (UC and CD) : Surgery, when indicated, must be as
radical as possible.
Correct answer F
221. About inflammatory bowel disease (UC and CD) : Patients must be given a good trial
of optimum medical treatment prior to surgery.
Correct answer T

- 594 -
222. About inflammatory bowel disease (UC and CD) : There is more chance of a cure after
surgery in UC than in CD.
Correct answer T
223. About inflammatory bowel disease (UC and CD) : In emergency presentation, patients
must be vigorously resuscitated prior to operation and managed in the ITU postoperatively.
Correct answer T
224. In Familial Adenomatous Polyposis: 20 per cent of FAP arise as a result of new
mutations in the adenomatous polyposis coli (APC) gene.
Correct answer T
225. In Familial Adenomatous Polyposis: Large-bowel cancer occurs 15–20 years after the
onset of the disease.
Correct answer T
226. In Familial Adenomatous Polyposis: On surveillance, if there are no polyps by the age
of 30 years, FAP is unlikely.
Correct answer T
227. In Familial Adenomatous Polyposis: Polyps do not develop anywhere else other than
the colon.
Correct answer F
228. In Familial Adenomatous Polyposis: Colectomy with ileorectal anastomosis may result
in rectal cancer later.
Correct answer T
229. Right colonic cancers present with features of anemia.
Correct answer T
230. Left colonic cancers present with rectal bleeding and obstructive symptoms.
Correct answer T
231. In Colon cancer: Even for an experienced colonoscopies, the failure rate to visualize
the caecum is 10 per cent.
Correct answer T
232. In Colon cancer: Intravenous urography (IVU) should be routinely done.
Correct answer F
233. Synchronous colon cancers occur in 5 per cent.
Correct answer T
234. In large bowel cancer: Thorough preoperative assessment and staging should be done
with colonoscopy, US and spiral CT.
- 595 -
Correct answer T
235. In large bowel cancer: Resection is not done if the patient has liver metastases.
Correct answer F
236. In large bowel cancer: If, at operation, hepatic metastases are found, biopsy should be
done.
Correct answer F
237. In large bowel cancer: Hepatic resection for metastases should be considered as a staged
procedure.
Correct answer T
238. In large bowel cancer: Over 95 per cent of colonic carcinomas can be resected.
Correct answer T
239. Causes of adynamic intestinal obstruction include: Paralytic ileus.
Correct answer T
240. Causes of adynamic intestinal obstruction include: Hernia.
Correct answer F
241. Causes of adynamic intestinal obstruction include: Mesenteric vascular obstruction.
Correct answer T
242. Causes of adynamic intestinal obstruction include: Pseudo-obstruction.
Correct answer T
243. Causes of adynamic intestinal obstruction include: Adhesions.
Correct answer F
244. Increased postoperative adhesions occurs with: Ischemic areas.
Correct answer T
245. Increased postoperative adhesions occurs with: Good surgical technique.
Correct answer F
246. Increased postoperative adhesions occurs with: Foreign material.
Correct answer T
247. Increased postoperative adhesions occurs with: Covering anastomosis and raw areas.
Correct answer F
248. Increased postoperative adhesions occurs with: Crohn’s disease.
Correct answer T
249. Regarding intussusception: It is most common in children with a peak incidence
between 5 and 10 months of age.
Correct answer T
- 596 -
250. Regarding intussusception: About 10 per cent of infantile intussusceptions are
idiopathic.
Correct answer F
251. Regarding intussusception: It causes obstruction but not strangulation.
Correct answer F
252. Regarding intussusception: Meckel’s diverticulum can be a cause in older children.
Correct answer T
253. About sigmoid volvulus: This is the most common site of volvulus in adults.
Correct answer T
254. About sigmoid volvulus: The predisposing factors include constipation, long pelvic
mesocolon and narrow attachment of the mesocolon.
Correct answer T
255. About sigmoid volvulus: The rotation is usually in a clockwise direction.
Correct answer F
256. About sigmoid volvulus: Flatus tube decompression is associated with a low rate of
recurrence.
Correct answer F
257. About sigmoid volvulus: There is no role for emergency surgery.
Correct answer F
258. Muscles of the leg are enclosed in a tight sleeve that is formed of the deep fascia.
Fascial septa divide these muscles into two compartments.
Correct answer F
259. Rigid fixation of a fracture is needed for healing of an arterial repair, otherwise it will
be disrupted or contused and thrombosed.
Correct answer T
260. This patient is suspected to have injury of the bulbous urethra. If the patient voids,
urine will spread in the subcutaneous tissues, deep to the membranous part of the superficial
fascia.
Correct answer T
261. The most important factor that affects wound healing is vascularity of its edges.
Correct answer T
262. A hernia must be repaired with absorbable material, so as to resist stresses for life.
Correct answer F

- 597 -
263. Delayed primary closure of the skin means skin closure when pus stops to drain from
the wound.
Correct answer F
264. Incised wounds treatment is by hemostasis and leaving the wound open to drain.
Correct answer F
265. The extent of tissue damage by a missile depended upon its weight.
Correct answer T
266. The extent of tissue damage by a missile is proportionate to its velocity.
Correct answer T
267. Stab wound usually fracture bones and thus add to tissue damage.
Correct answer F
268. Stab wounds are inflicted by knives and may injure viscera.
Correct answer T
269. In the presence of contamination, delayed primary closure of an incised wound reduces
the possibility of infection.
Correct answer T
270. In healing by secondary intention wound contraction and granulation tissue does not
play major roles.
Correct answer F
271. Shoch with bradycardia is Hypovolemic shock.
Correct answer F
272. Disseminated intravascular coagulation (DIC) is a complication of shock particularly
the hypovolemic type.
Correct answer F
273. Metabolic acidosis is commonly seen in cases of shock though the pH is normal a low
bicarbonate level indicates metabolic acidosis
Correct answer T
274. Adult respiratory distress syndrome (ARDS) is one example of the multiple organ
failure syndrome which occurs with shock.
Correct answer T
275. Shock particularly the septic type, major burns, and severe pancreatitis can all trigger
the release of cytokines that lead to systemic inflammatory response syndrome (SIRS) the
to multiple organ failure (MOF).
Correct answer T
- 598 -
276. Renal failure that is caused by shock is irreversible.
Correct answer F
277. In cardiogenic shock, the neck veins are collapsed.
Correct answer F
278. Treatment of cyclic mastalgia can be avoidance of breast support.
Correct answer F
279. A Fibroadenoma is a mixed epithelial and mesenchymal benign tumour. The peak age
of appearance is late teens and early twenties.
Correct answer T
280. Regarding breast cysts, Multiplicity of cysts raises suspicion of malignancy.
Correct answer F
281. The commonest histological type of breast cancer is Ductal carcinoma in situ.
Correct answer F
282. In breast cancer, T stands for tumour size. A tumour between 2-5 cm is T2.
Correct answer T
283. Mammography indicated for regular screening of women who are at high-risk of
development of breast cancer.
Correct answer F
284. Breast cancer is better for tumors in the medial half than the lateral one of the breast
Correct answer F
285. Paget’s disease of the nipple is lobular carcinoma arising at the opening of a lactiferous
duct on the nipple.
Correct answer F
286. In a young female patient, a Lanz incision is preferred as this is more in keeping with
langer's lines.
Correct answer T
287. A Kocher's incision is a subcostal incision used for open cholecystectomy.
Correct answer T
288. A Collar incision is made for submandibular gland excision and block dissection.
Correct answer F
289. Esophageal carcinoma is commoner in smokers, in people with increased alcohol
intake oesophageal strictures or certain dietary deficiencies.
Correct answer T

- 599 -
290. A pharyngeal pouch occurs in a weakness in the superior constrictor muscle. It is also
known as killian's dehiscence.
Correct answer F
291. Achalasia is Treated by injection of botulinum toxin.
Correct answer T
292. In venous type ulcer the duplex show good arterial flow, so compression bandaging
would be suitable treatment.
Correct answer T
293. In an arterial ulcer with stigmata of chronic arterial insufficiency, Revascularization
should be attempted, either endovascularly or surgically.
Correct answer T
294. Inter-sphincteric abscesses can be incised, drained, and laid open. Trans-sphincteric
abscesses are usually laid open to reduce the risk of incontinence following damage to both
internal and external sphincter mechanisms in this case seton drainage.
Correct answer T
295. Sub-phrenic abscess is a known complication of diverticular disease, usually occurring
approximately 1-3 days following the initial episode.
Correct answer F
296. Amylase of 1000 Somogyi U/dl is characteristic of acute pancreatitis.
Correct answer T
297. Epstein-Barr virus is associated with Burkitt's lymphoma in the presence of malarial
infection.
Correct answer T
298. Aspergillus flavus releases aflatoxin which occur as a dietary contaminant and are
associated with hepatocellular regeneration.
Correct answer F
299. Primary hydrocele is most common at young age.
Correct answer F.
300. Fournier's gangrene is necrotizing fasciitis of the scrotum.
Correct answer T.
301. This man has a Glasgow coma scale (GCS) of 3. He is unable to protect his own
airway; securing the airway is therefore paramount.
Correct answer T

- 600 -
302. Hyperplasia is increased tissue or organ size due to increased cell size in response to
increased functional demand.
Correct answer F
303. Atrophy is the decrease in size of an organ or tissue by reduction in cell size and /or
reduction in cell numbers.
Correct answer T
304. Agenesis is failure of development of an organ or structure. This differs from atresia
which is failure of development of a lumen in a normally tubular structure.
Correct answer T
305. Transitional cell carcinomas of the renal pelvis resemble those affecting the bladder
but are much less common they tend to invade the renal parenchyma and have a tendency to
distal spread.
Correct answer T
306. hemorrhagic SHOCK class 1 is loss of 0-15% of circulating volume; there are no
obvious changes apart from the patient perhaps feeling uncomfortable and restless.
Correct answer T
307. Painless jaundice together with a history of weight loss is strongly suggestive of a
malignancy.
Correct answer T
308. Sharing of needles among intravenous drug misusers is a serious risk factor for
hepatitis B and C both of which predispose to the development of hepatocellular cancer in
later years Alpha-fetoprotein is a marker for this cancer although it is also raised in
hepatoma.
Correct answer T
309. Cola-colic intussusception typically presents as in this case and the tumor acts as the
apex. Half of all adult intussusceptions are due to carcinoma.
Correct answer T
310. Hyperparathyroidism presents with the symptoms of hypocalcemia including
paraesthesia, cramps, tetany, circum-oral tingling, and convulsions.
Correct answer F
311. Secondary hyperparathyroidism is caused by chronic renal failure in response to a low
calcium level, and is characterized by a low serum and a raised PTH level.
Correct answer T

- 601 -
312. A recent general anesthetic and opioid-based analgesia causing constipation are two
factors which predispose to developing urinary retention post-operatively.
Correct answer T
313. Umbilical hernias in young children should be treated surgically.
Correct answer F
314. Laparoscopic repair of inguinal hernias is recommended for the management of
bilateral inguinal hernias unless there are any contraindications present.
Correct answer T
315. Squamous cell carcinoma is most common in sun-exposed areas frequently bleeds and
is characterized by an undermined edge.
Correct answer T
316. Squamous cell characterized by a rolled pearly edge. It does not tend to bleed as often
as melanoma.
Correct answer F
317. Full-thickness skin grafting can be used on exposed tendon, but is not good for exposed
bone or cartilage.
Correct answer T
318. Local flaps provide their own blood supply and can transfer skin, fascia, muscle, and
bone.
Correct answer T
319. Free flaps can not provides their own blood supply, and can be used with local flap.
Correct answer F
320. Hemorrhoidal bleeding is painless and not mixed with stools it is bright red as its origin
from the arteriovenous cushions of the anal canal.
Correct answer T
321. Rubber band ligation is most useful in stopping bleeding from first- and second-degree
hemorrhoids for third degree hemorrhoids, band ligation is enough to control the prolapse.
Correct answer F
322. Hemorrhoidectomy causes considerable postoperative pain. This may result in reflex
retention of urine, rather than incontinence.
Correct answer T
323. Fecal incontinence can occur if the whole anal sphincter mechanism is divided Anal
stenosis may follow hemorrhoidectomy if the surgeon leaves intact muco-cutaneous bridges
between the raw areas.
- 602 -
Correct answer F
324. The majority of abscesses start as infection of an anal gland. These glands are present
in the intersphincteric space and have their ducts open at the dentate line.
Correct answer T
325. Anal fistulotomy or fistulectomy wounds should be sutured.
Correct answer F
326. Surgery for a high fistula requires division of the entire anal sphincter mechanism and
would produce incontinence if done in one stage.
Correct answer T
327. The majority of anal canal cancers are of the squamous carcinoma type.
Correct answer T
328. The caudate lobe is segment I. it lies between the porta hepatic and the inferior vena
cava (IVC) posteriorly.
Correct answer T
329. The enzyme alkaline phosphatase is elevated in cases of bill stasis and with bone
diseases particularly bone resorption and destruction.
Correct answer T
330. Being a retroperitoneal organ an isolated injury of the kidney is likely to stop bleeding
spontaneously because of the tamponade effect of the posterior peritoneum on the
hematoma.
Correct answer T
331. The main danger of liver injury is bleeding However bleeding can stop spontaneously.
Correct answer F
332. Acute appendicitis always causes a liver abscess if it happens infection gets to the liver
through lymphatics.
Correct answer F
333. An amoebic liver abscess characteristically contains clear fluid.
Correct answer F
334. The normal portal pressure is 8-12cm water.
Correct answer T
335. Schistosomiasis carries a worse prognosis than post-hepatitis cirrhosis.
Correct answer F
336. Viral hepatitis A resolves spontaneously without ill effects. One the hand types B and
C can cause liver cirrhosis and predispose to the development of hepatocellular carcinoma.
- 603 -
Correct answer T
337. Hemangiomas of the liver are common findings on imaging studies (CT or
ultrasound)that are done for other indications.
Correct answer T
338. carcinoembryonic antigen is elevated in patients with GIT and breast cancer.
Correct answer T

‫تم بحمد للا‬

- 604 -

View publication stats

You might also like